Download as docx, pdf, or txt
Download as docx, pdf, or txt
You are on page 1of 159

FAMILY MEDICINE EOR: Cardio 15%, Pulm 12%, GI 11%, EENT 8%, OBGYN 8%, MSK 8%, Neuro

6%, Derm 5%, Endo 5%, Psych 5%, GU 5%, Heme 4%, ID 4%, Urgent Care 4%

CARDIO 15%
CARDIO About Clinical Presentation Diagnostics Treatment
Myocarditis Inflammation of heart muscle due to 1. Viral prodrome: fever, myalgia, •CXR: cardiomegaly •supportive symptomatic therapy
myocellular damage  necrosis and malaise  HF symptoms •beta blockers
dysfunction  heart failure •EKG: dysrhythmias, sinus tachy •IVIG
2. Heart Failure:
MCC: Viral infection dyspnea at rest •Labs: (+) cardiac enzymes, ESR
exercise intolerance
Etiologies: syncope •TTE Echo: ventricular dysfunction
•Infectious: Enterovirus (MC), tachypnea, tachycardia
coxsackie B, adenovirus, parovirus, impaired systolic function (S3/S4) •Endomyocardial biopsy: infiltration
HHV-6, EBV, HIV, VZV megacolon of lymphocytes with myocardial tissue
•Bacterial: Lyme, RMSF, Chagas +/- concurrent pericarditis necrosis
•Toxic: diphtheria, scorpion *GOLD STANDARD!
•Autoimmune: SLE, rheumatic, RA, 3. Pericarditis:
•Systemic: Uremia fever
•Medications: Clozapine, chest pain
Methyldopa, ABX, Isonizid, friction rub
Cyclophoshamide… effusion

Dilated MC CARDIOMYOPATHY Heart Failure symptoms •CXR: cardiomegaly, edema, effusion •Treat underlying source
Cardio- -systolic failure •EKG: arrhythmias, tachycardia •CHF management: ACEI, diuretics, beta
myopathy Systolic dysfunction, LVEF <40% -left ventricular dilation •Echo: LV dilation blockers, Na+ restriction
 ventricular dilation  dilated, -decreased ejection fraction •Prevention of sudden cardiac arrest
weak heart *MC in Men -arrhythmias

Causes: Exam: S3 gallop, laterally displaced


Idiopathic (MC) PMI, MR or TR
Viral: Enterovirus, Coxsackie, PB19,
Chagas, HIV, Lyme
Toxic: Alcohol, cocaine,
Doxorubicin
Other: pregnancy, metabolic

Hypertrophic Inherited genetic disorder (autosomal •Dyspnea (MC complaint) Murmur: •Avoid volume depletion and extreme
Cardio- dominant)  mutation of sarcomeres •Angina, Syncope, Arrythmias •Harsh, mid-systolic crescendo- exercise
myopathy •LV or RV hypertrophy *esp septal •Sudden cardiac death decrescendo murmur, at 3rd/4th ICS
•Best heard at left sternal border •Beta-blockers (1st line), Verapamil
Pathophysiology: •louder with valsalva, standing (CCB), Disopyramide
•subaortic outflow obstruction  •quiet w/ squat, supine -Negative inotropes that increase
hypertrophied septum, systolic ventricular diastolic filling time
anterior motion of MV and papillary •EKG: LVH (V5, V6, aVL)
muscle displacement •Echo: DIAGNOSTIC •Septal myetomy/alcohol septal ablation
assymetrical wall thickeness *inject alcohol-kill off part
•diastolic dysfunction: impaired systolic anterior motion of mitral valve
relatxation and filling
CARDIO About Clinical Presentation Diagnostics Treatment
Restrictive •Impaired diastolic function with •Right sided HF sx •CXR: enlarged atria •Treat underlying cause if
Cardio- relatively preserved contractility •Pulmonary HTN •EKG: low voltage known
myopathy •Rigidity impairs filling (stiff) •Normal ejection fraction
•Echo: non-dilated ventricles with normal wall
Cause: Amyloidosis (MCC), Exam: Kussmaul sign (JVP increases with thickness  dilation of both atria, diastolic
sarcoidosis, hemochromatosis, inspiration) dysfunction
scleroderma, fibrosis, cancer, chemo

Stress- •Can cause an ACS or STEMI • psycho or physiological stress EKG: ST elevations, (+) enzymes •Recover in a few weeks
Induced •event causing catecholamine surge  Systolic dysfunction of apex Echo/LV angiography: LV apical ballooning •Beta Blockers for one year
(Takotsubo) *MC in postmenopausal women and/or mid segments

CARDIO About Symptoms Management


Sinus Arrhythmia •Cyclic increase in normal heart rate with reflex changes in vagal influence on the normal No treatment required
inspiration and decrease with expiration pacemaker and disappears with breath holding or *not pathologic
increase in heart rate

Sinus Bradycardia Heart rate <60bpm •Increased vagal influence on the normal pacemaker •1st line-permanent pacemaker
Rate increases with exercise, atropine or organic disease of sinus node
- Fear, bearing down, SSS Acute setting: transcutaneous pacing, temporary
Causes: transvenous pacing, Atropine (ACLS protocol)
•Increased ICP: hemorrhage •Severe <45bpm-sinus node pathology
•Anterior MI: Inf. wall MI -> RCA supply SA
•OSA: decreased Hr <30bpm in apnea SX:
•Other: hypothermia, hypothyroidism •weakness, dizziness, confusion, syncope

Sick Sinus Syndrome •Commonly have recurrent supraventricular “When they go into AFIB its really fast then when Symptomatic  Pacemaker (PPM)
arrhythmias and bradycardia “tachy-brady” they go out of afib they go into a really slow sinus
brady”
•MC in elderly •Cause: medications

Sinus Tachycardia •HR >100bpm Symptoms: palpitations, lightheadedness Physiologic: *usually compensatory
•Onset & termination usually GRADUAL *usually related to the cause
Symptomatic: Beta-blocker (Metoprolol), CCB
Causes: MC-exercise, anger/stress •2 types: inappropriate, POTS

CARDIO About Causes Signs and Symptoms Diagnose/Management


Premature ectopic focus in the atria that fires •increased frequency with age Palpitations •Not required most of the time
Atrial before the next sinus node impulse •structural changes •Beta Blockers if significant symptoms
Contractions *diff P wave morphology •precuroseor to afib, aflutter •Class IC antiarrhythmis is second-line

Premature Premature depolarization originates *frequently in normal heart •Palpitations MC complaint Usually no treatment needed
Ventricular from the ventricles •caffeine, stress, alcohol
Contractions - wide QRS complex w/ pause •electrolyte abnormalities EKG: T wave usually opposite QRS, Symptomatic: BB (Metoprolol)
occurring earlier than expected •thyroid QRS is wide 2nd line: Class Ic or III AAD
Significant burden: cath ablation

CARDIO About Causes Signs and Symptoms Diagnose/Management


AV Block First degree: PR >0.20 with all atrial First Degree and Mobitz Type I: First Degree Block: Diagnostic Studies:
impulses conducted, P for every QRS •may occur in normal individuals -diagnosed by EKG alone •Review medications & HX
*AV node with heightened vagal tone *REVIEW FIRST!
dysfunction Second degree: intermittent blocked •drug effect Mobitz type I: •12 lead EKG
beats, not a P for every QRS •electrolyte abnormalities -most commonly asymptomatic •telemetry monitoring (inpt)
•organic disease: ischemia, infarction, -may note palpitations, DOE, •echo to r/o structural disease
Mobitz type I (Wenckebach) AV block: inflammatory, fibrosis, calcification, dizziness •S/S ischemia -> cath
AV conduction time (PR) progressively infiltration Labs: CBC, CMP, TSH
lengthens with the RR interval Mobitz type II:
shortening before the blocked beat Mobitz Type I: irregular rhythm Management:
*abnormal conduction AV node •Causes: inferior wall MI, AV node -may be asympatomatic •First Degree:
blocking agents (BB, CCB, Digoxin), -palpitations, DOW, weakness, dizzy -avoid medications that may prolong PR
Mobitx type II AV block: there are Lyme, hyperkalemia, surgery interval and slow AV conduction
intermittently non-conducted atrial beats Third-Degree block: -asymptomatic: no treatment
*block in HIS bundle Mobitz Type II: -sx vary; worse w/ exertion -symptomatic: Atropine, Epi
•PR prolonged, all the same length •Causes: RARE in structural heart -palpitations, DOE, weakness, near
disease; AV node blocking agents, syncope, syncope, and/or HF •Mobitz type I:
Third degree: complete heart block, in ablation, surgery -avoid meds that slow AV conduction
which no supraventricular impulses are -symptomatic: Atropine
conducted to the ventricles Third degree:
•regular P-P intervals and regular R- •ORGANIC DISEASE involving the •Mobitz type II/Third-Degree:
R, but are not related to each other infranodal conduction system -unstable, usually require pacemaker
•transient or permanent
•AV dissociation
•inferior wall MI, AV nodal blocking
agents, endocarditis, myocarditis

CARDIO About Causes Symptoms Management


Atrial Results from an ectopic atrial focus firing at a rate •severe COPD -palpitations •treat underlying conditions
Tachycardia faster than the sinus rate -> becomes pacemaker •structural heart disease -heart racing sensation
•may occur in normal hearts -SOB •CCB (Verapamil) 1st line option
•Onset and termination occur abruptly *not commonly -dizziness *especially if have COPD
•Atrial rate about 100-160bpm •digoxin toxicity -near syncope
•Unifocal or multifocal (3+ different morphologies) •refractory: class IC or III

AV AV node becomes the dominant pacemaker •sinus disease EKG:


Junctional •CAD •regular rhythm
Dysrhythmia Junctional: rate 40-60 •Digoxin •inverted P waves or not seen
s Accelerated: 60-100 •myocarditis •narrow QRS
Junctional tachycardia: >100
CARDIO About Causes Signs & Symptoms//Diagnosis Management
Paroxysmal •Common cause of paroxysmal •MC mechanism for •Awareness of rapid heart beat Acute Management: STABLE PT
Supra- tachycardia paroxysmal supraventricular •Chest pain, SOB 1. Mechanical measures *done by pt
ventricular tachycardia is reentry •Dizziness, syncope •valsalva*
Tachycardia Two common types of reentry: - typically initiated by •Abrupt onset and termination, last •stretching the arms and body, lowering the head
•AVNRT (AV node reentrant a PAC or PVC seconds to several hours or longer between knees*
tachy): one normal and one •coughing, breath holding*
accessory pathway within the node EKG: •cold water
*within the node •rate ~140-240 •carotid sinus massage-> PROVIDER
*Most Common
•Orthodromic (95%): regular, 2. Drug Therapy (IV therapy): “ABCD”
•AVRT (AV reciprocating tachy): narrow complex tachy with no •Adenosine**, CCB (Diltiazem), BB (Metoprolol)
one normal and one accessory discernable P waves •wide complex: Amiodarone
OUTSIDE the node “if you cant tell if the bump is a P
 WPW: accessory path (bundle or a T, then it must be SVT” 3. Cardioversion (shock) *sedate if awake
of kent) preexcites ventricles •FIRST LINE if hemodynamically unstable
 LGL •Antidromic: regular, wide •synchronized electrical cardioversion at 100J
*accessory path outside AV node complex tachy (mimics v-tach)
4. Definitive: ablation
WPW EKG:
W: wave  delta wave (slurred WPW:
QRS upstroke) •Procainamide**, Amiodarone
P: PR interval short •avoid AV blocking agents ABCD (Adenosine, BB,
W: wide QRS CCB, Digoxin)
•DEFINITIVE: Ablation

Prevention: BB & CCB are 1st LINE


CARDIO Phases Risks/Causes/Complications Symptoms Management
Atrial •Multiple irritable atrial foci fire at Risks: Symptoms: Three-Fold:
Fibrillation fast rates •CHF* & HTN* •may be asymptomatic 1. Rate control
•MC chronic arrhythmia •Old age •palpitations, heart racing 2. Rhythm control
•CAD •chest pain, SOB, fatigue, CHF 3. Thromboembolic event prevention (ALL)
Paroxysmal: in and out on own •valvular disease
Physical Exam: Rate Control:
Persistent: >7 days Causes: •irregularly irregular rhythm •CCB (Verapamil, Diltiazem) *NOT in HF!
•alcohol •signs of CHF •Beta Blockers (Metoprolol, Atenolol,
Long Standing Persistent: 1 year •thyroid disease, lung, heart •distal pulses may be difficult Esmolol)
•fhx •Digoxin
Permanent (Chronic): staying in •sleep apnea EKG:
•pericarditis •irregular irregular ~250bpm Rhythm Control: dependent on comorbidities
•no P waves
Complications: HF, thromboemboli •Ashman’s phenomenon
(wide QRS after short R-R)

Rhythm Control Anticoagulation Maintenance/Education


Hemodynamically unstable: CHA2DS2SASc Score Long-Term: Maintenance:
Cardioversion *calculate on EVERY Afib pt •Warfarin (INR goal 2-3) •CBC and BMP q6months
•Score 0: no therapy •outpatient ambulatory monitoring
*most successful within 7 days of •Score 1: consider oral anticoag or •Dabigatran (Pradaxa): reduce if *if on Amiodarone, then make sure they are
onset  echo prior to make sure there antiplat (ASA 81) CrCl 15-30 getting routine imaging and labs
is no thromboemboli •Score 2: oral anticoagulant
•Rivaroxiban (Xarelto): reduce if Patient Education:
AF >48 hours: anticoagulation CrCl 15-50 •avoid alcohol ANYTIME
needed for 3 weeks before •Valvular (mitral stenosis or •control underlying risk factors
cardioversion, continue for 4 weeks regurg) heart disease: Coumadin •Apixaban (Eliquis: reduce if 2+: •monitor for signs of bleeding if anticoagulated
after cardioversion 80+, wt </=60kg, Cr 1.5+

•Edoxaban (Savaysa): reduce if


CrCl 15-50

Atrial Causes/Risks/Presentation Treatment


Atrial Flutter •One atrial focus firing at a fast rate Stable: vagal maneuvers Unstable: Higher rate of cure with catheter-based
(~300bpm) •Rate: BB, CCB (Diltiazem, •synchronized cardioversion radiofrequency ablatation
•Presentation, risk factors, causes are cVerapamil) - DEFINITIVE
the same as afib •Anticoagulation: CHADSVASc
CARDIO Definition/ Causes Symptoms Management
Ventricular •Three or more consecutive PVCs Patient may be asymptomatic, Acute sustained VT Long-Term therapy
Tachycardia -nonsustained: less than 30 seconds, especially with nonsustained Unstable: immediate synchronized direct (prevention/recurrence):
-sustained: greater than 30 seconds current cardioversion •ICD
•usual rate is 120-300bpm Majority have symptoms: *unsynchronized if no pulse •BB
•palpitations, heart racing •Amiodarone, Sotalol (class III)
Causes: •near syncope, syncope Stable: •catheter ablation
•ischemic heart disease (MC) •confusion, fatigue •IV Amiodarone likely will convert to sinus
•structural heart defects •chest pain, SOB •IV Lidocaine *MI or still in VT with Amio Non-sustained VT: Beta
•prolonged QT interval •Add IV Magnesium *Torsades Blockers
•electrolyte abnormalities (low Mg, K, Ca) EKG: regular, wide complex

Torsades De •Variant polymorphic ventricular tachy •palpitations EKG: polymorphic ventricular tachycardia IV Magnesium Sulfate
Pointes •dizziness (cyclic alteration of the QRS amplitude) (suppresses early afterdepol)
•Prolonged repolarization and early •fatigue *sinusoidal waveform
afterdepolorization, and triggered activity •dyspnea •discontinue all QT prolongating
•chest pain agents
Causes:
•prolonged QT
•electrolytes (low Mg, K, Ca)
•meds: Class IA, Digoxin, Macrolides,
antipsychotics, antidepressants, antiemetics
Arrhythmias Definition
Ventricular •Leading cause of sudden death •unresponsive, pulseless,syncope IMMEDIATE UNSYNCHORNIZED
Fibrillation •MCC is ischemic heart disease DEFIBRILLATION & CPR

Arrhythmia About Mechanisms Causes/Management


Accelerated Regular wide complex rhythm with a rate Two possible mechanisms: 2. escape rhythm due to suppression of higher No treatment unless UNSTABLE
Idioventricular of 60-120bpm 1. Slow ventricular tachycardia pacemakers resulting from sinoatrial and AV
Rhythm due to increased automaticity block or depressed sinus node

CARDIO About Causes Evaluation Management


Left Bundle •MC in pts with underlying heart Causes: Evaluation: Asymptomatic and isolated LBBB:
Branch Block disease; Can also be seen in -structural heart disease •chest pain/ACS sx -no specific therapy
(LBBB) structurally normal heart -functional LBBB (rate-related) -treat underlying cause (treat like MI)
Diagnostics:
*Left anterior descending artery •echo SX pts with LBBB and low EF: CRT
•stress test or LHC
Right Bundle Receives blood supply from septal Conduction can be compromised by: Generally asymptomatic Do not require further dx or tx
Branch Block branches of the left anterior •Structural heart disease
(RBBB) descending •Functional factors

Pulseless PEA: organized rhythm seen on a •CPR & epinephrine


Electrical monitor, but patient has no pulse
Activity/ •Check for shockable rhythm every 2
Asystole Asystole: no rhythm minutes

Class I Drug About Drug Side Effects/CI/PK/Caution


Class IA Quinidine •Potent anticholinergic properties that affect SA and AV •Increased ventricular rates with afib or aflutter
•decreases conduction velocity nodes -> increase SA nodal discharge rate and AV nodal  PROLONG QT INTERVAL -> torsades
•prolongs repolarization conduction  Add BBB, CCB, or Digoxin to prevent
•prolongs refractory period •SE: GI, N/V/D
•prolongs action potential *don’t use because of QT prolongation, and rhythm issues •PK: CYP3A4 inducer or inhibitor
•increase excitation threshold
Procainamide •No anticholinergic property •Prolongs QT -> Increases torsades
moderate depression of phase 0 •LUPUS (SLE) clinical syndrome *MC
WORKS ON FAST SODIUM •N/V/D, fever

Disopryamide •Potent anticholinergic and negative inotropic effects •Prolongs QT -> Increases torsades
(Norpace)  Use more for HOCM •Precipitation of CHF
•Dry mouth, urinary retention, constipation, blurred vision

•CI: reduced LV EF (<40%)

Class IB Lidocaine •Selective to ischemic tissue •SE: CNS, dizziness, disorientation, tremor, agitation, seizure,
•Active fast sodium channel in the bundle of HIS, purkinje respiratory arrest
•decreases conduction velocity *IV form fibers, and ventricular myocardium
•shortens repolarization  Little effect on non-ischemis tissue, atrial •Caution: liver failure *stop if Lidocaine level is over 4
•shortens actions potential myocardium, SA node
•best for ventricular dysrhythmias with acute MI •PK: hepatic metabolism

*do not effect QRS complex Mexiletine •Similar to lidocaine but oral *not used alone •SE: GI, N/V highly, neurologic effects
*can use in combo with class IA and III drugs for refractory - Dizziness, confusion, ataxia, speech disturbances
*oral form ventricular dysrhythmias

Class IC Flecainide •Slows conduction velocity in the Purkinje fibers and the AV •SE: rapid VT resistant to resuscitation
(Tambacor) node  Lengthen PR interval and QRS duration
•decreases conduction velocity •Used for afib/aflutter - Blurred vision, dizziness, HA, tremor, N/V
•NO effect on action potential
•CI: CAD, LV dysfunction, LVH, valvular disease

NORMAL repolarization Propafenone •Slows conduction velocity of purkinje fibers and AV node; •SE: same as Flecainide, metallic taste
LONGER depolarization (Rythmol) mild non-selective BB effect  May lengthen PR interval and QRS
*similar to Flecainide
•Used for afib/aflutter •CI: CAD, LV dysfunction, LVH, valvular disease

Class II Drugs About Indication


Beta Blockers Decreased automaticity (lowers resting threshold, takes longer to fire off), Useful in suppressing ventricular dysrhythmias and
-Metoprolol (cardioselevtive) prolong AV conduction, prolong refractoriness supraventricular dysrhythmias
-Esmolol (IV)  (-) chonotripic and inotropic effects
-Atenolol, Carvedilol, Labetlol *beta blockers A  M are cardio selective, N and lower are NOT •CI: bradycardia, 2nd 3rd degree AV block

Class III Drugs About SE/CI/DDI


*use these pretty routinely Amiodarone •Possesses characteristics of all 4 classes SE: *cumulative effects after years, not a one-time dose
(Pacerone) •K+ channel blocks, but also blocks sodium channels, •metabolite accumulation of IODINE in numerous organs
Block potassium channels and non-selective BB activity, weak CCB propreties •Pulmonary Toxicity *pulmonary fibrosis
prolong repolarization, widening •Works on all cardiac cells  Stop immediately if occurs
the QRS and prolonging the QT •Minimal to no negative inotropic effects •Thyroid abnormalities (hypo or hyper) due to iodine
interval *can use in LV dysfunction •Ocular complications (corneal and lens) *rarely effects vision
 Decrease automaticity and  Photophobia, halos, blurred
conduction and prolong •GI: Nausea (acute), vomit, anorexia, abdominal pain *common
refractoriness MOST •Neurologic toxicities frequently (warn about)
 Cant fire neighboring cell COMMON  Tremors, ataxia, peripheral neuropathy, fatigue, insomnia
as easily, takes longer DRUG USE! •Derm: photosensitivity, blue/gray skin IN SUN (warn about)

*avoid w/ QT prolongation drugs Monitor:


 Anti-psychotic drugs •CXR and dilated eye exams yearly
 Anti-fungals •TSH & LFTs every 6 months
 -Fluoroquinolones •wear sunscreen

DDI: CYP 3A4 inhibitor-Warfarin


*can double serum digoxin concentration

Sotalol •Has non-selective B-adrenergic blocking properties CI: LV dysfunction


(Betapace) •PUT IN HOSPITAL TO START AND MONITOR
 Decrease cardiac contractility SE: QT prolongation; DC if QT >550ms
 Prolongs atrial and ventricular refractoriness *most due to BB properties; do NOT combine with other BB

Dofetilide •Results in prolonged action potential and an increased SE: development of torsades
(Tikosyn) QT interval *pt in hospital on telemetry with q12h EKGs for first 3 days
 Effects atria more than ventricles
•No negative inotropic effects *Safe in LV dyfcuntion DDI: cimetidine, ketoconazole, megestrol, prochlorperazine, Bactrim,
verapamil
*must have board of pharmacy certification in order
to prescribe because of QT prolongation PK: Renal excretion (CI if CrCl <20) *regular BMP for CrCl

Dronedarone •Exhibits AA properties of all 4 classes CI: symptomatic CHF, permanent AF, hepatic
(Multaq)  However, does not work as well
SE: CHF exacerbation, QT prolongation, bradycardia, hepatotoxicity,
“safe cousin of Amiodarone” pulmonary toxicity, N/D, pruritis, dyspepsia
*don’t use often & more expensive
Monitor: BUN/CR, EKG q3mo for 1st 6mo tx

Ibutilide •Similar to sotalol, but no beta-blocking activity SE: torsades (QT prolongation)
(Corvert) •Close to a pure potassium channel blocking agent *monitor on telemetry for several hours following IV infusion
•Only in IV form and indicated for afib/flutter
cardioversionto sinus rhythm CI: LV dysfunction and electrolyte abnormalities (low K & Mg)
 Monitor in ICU

Class IV Drugs About SE/CI/DDI


Calcium Channel Verapamil •Decreases automaticity and AV conduction CI: LV dysfunction; heart failure (due to negative inaotropic)
Blockers Diltiazem •Potent negative inotropic effects
*Non-dihydropuridines SE: peripheral edema, bradycardia, AV blocks

Other Drugs Drugs About


Digoxin •Predominant AA effects on AV node EKG changes: PR prolongation and ST segment depression
-inhibits calcium currents in AV node and activated ACH-
mediated K+ currents in the atrium PK:
-slows conduction through the AV node and prolongs AV •IV or oral; DOSING IN MICROGRAMS
refractory period •Tablets incompletely bioavailable
-mainly used for slowing ventricular rate in afib/aflutter and •Slow distribution to effector sides
termination reentrant arrhythmias involving AV node *requires loading dose of 0.6-1mg over 24 hours then reduce to maintenance dose
•t ½ of 36 hours
•renal elimination (reduce dose in renal insufficiency)

DDI: amiodarone, quinidine, verapamil, diltiazem, itraconazole, propafenone, flecainide


*decrease clearance, decrease digoxin with these meds

Digoxin •Can be precipitated by: S/S: visual disturbances (blurry/tunnel vision), dizziness, weakness, N/V/D, anorexia, any
Toxicity -declining renal function dysrhythmia
-electrolyte abnormalities
-hypoxia or drug interactions Treatment:
•Manage arrhythmias
•Narrow therapeutic index •IV hydration and electrolyte correction
*can check therapeutic levels •Digoxin immune Fab (immunoglobin fragment)-> high affinity for digoxin molecules

Adenosine •Activates potassium channels and by increasing the outward •Used for converting SVT  sinus rhythm *essentially sinus arrest (stop the heart)
potassium current hyperpolarizes the membrane potential (doesn’t let • t ½ short <10 seconds, so sinus arrest does not last long
*SVT anything in to block it), decreasing spontaneous SA nodal
depolarization
 Inhibits automaticity and conduction in the SA/AV nodes 
SLOWS AV NODE SE: chest discomfort, dyspnea, flushing, HA
*doesn’t really work for atrial fibrillation/atrial flutter *causes vasodilation with a stress test

Atropine •Parasympathetic that enhances both sinus nodal automaticity and •Used in emergent setting of symptomatic bradycardia
AV nodal conduction through direct vagus action •Blocks ACH at parasympathetic neuroeffector sites
 Someone that has a vagal response
SE: tachycardia, paradoxical slowing of HR in patients with Mobitz type II AVB and third
INCREASES HEART RATE degree AVB *never use this drug unless you have a crash cart ready

*Class I and III primarily used for rhythm, class II and IV primarily used for rate control

Goals of dysrhythmia management: prevent sudden death, reduce symptoms, improve QOL, reduce hospitalizations,
- consider costs and risk of therapies, overall patient condition

CARDIO About Clinical Presentation Diagnostics Management


Angina Substernal chest pain usually History: ALL: EKG and CXR Revascularization
Pectoris brought on my exertion •Chest pain: poorly localized, EKG: WITHIN 10 MINUTES •PCI: 1 or 2 vessel disease not of L main
*complication of coronary substernal •ST elevation •ST depression *classic findings •CABG: L main or 3 vessel of EF <40%
artery disease causing SX •Radiation: arm, teeth, jaw •+/- T wave inversion, poor R wave progression
•Duration: Short Medical Management
Pathophysiology: (1-5 minutes, <30 minutes) Stress Test *most useful non-invasive tool •NTG
•inadequate tissue perfusion due •Alleviating: rest, NTG •exercise stress test: ST depression, -increase supply, O2, decreases demand,
to decreased blood supply & •Aggravating: exertion, anxiety hypotension/HTN, arrhythmias, sx -SE: headache, tachyphylaxis, flushing
increased demand
Other SX: diaphoresis, dyspnea, •myocardial perfusion (exercise, pharm) •Beta-Blockers
Classes fatigue, numbness, nausea -thallim-201 or 00m-technetium sestamibi -increases blood supply by prolonging coronary
•Class I: strenuous activity -indication: baseline EKG abnormalities artery filling times
•Class II: more prolonged Exam: usually normal -pharm agents: Adenosine -reduces demand by reducing HR
rigorous activity -CI: asthmatics
•Class III: daily activity •CCB (Diltiazem, Verapamil)
•Class IV: at rest Low Dose ASA Prevention: •stress echo: assess global and regional wall -prolongs diastolic filling increasing supply
•considered for primary motion abnormalities -decreased contractility reducing demand
prevention in patients 40-75 -Dobutamine  increases O2 demand
who are not at increased •Aspirin: does not work with supply/demand,
bleeding risk •cardiac MRI: dobutamine infusion, assess but prevents progression of by preventing
•NOT for ages >70 for primary perfusion and wall motion abnormalities platelet aggregation
•NOT for anyone with increased
risk of bleeding Coronary Angiography (GOLD) “Cath” Outpatient: ASA*, BB*, statin, NTG

CAD/ Causes: Pathophysiology: RV infarction triad: elevated JVP, clear lungs, •PCI (w/n 90 minutes) or TPA (30 minutes)
Acute •MCC: atherosclerosis 1. fatty streak formation: lipid (+) Kussmaul sign •MONA: morphine (if NTG fails to relieve
Coronary (hardening and narrowing of deposits in the WBC *no sx *inferior wall MI may be associated with heart pain), oxygen, NTG, ASA*avoid NTG and
Syndrome the coronary arteries) blocks or bradycardia morphine in posterior and inferior wall MIs
•Coronary artery vasospasm 2. early plaque formation:
•Aortic stenosis/regurg LDL enters endothelium in the STEMI: (+) enzymes, ST elevation >1mm Normal EKG:
•Pulmonary HTN fatty streak and LDL is oxidixed *total occlusion MONA + serial enzymes and EKG
•HTN NEW LBBB IS STEMI EQUIVALENT
•HOCM 3. foam cells: macrophages UA or NSTEMI: MONA, Hepatin, BB
ingest LDL NSTEMI: (+) enzymes, ST depressed •TIMI or HEART assessment
Risks: *subtotal occlusion
•DM (biggest risk!) 4. mature fibrous plaque: STEMI: MONA, Heparin, BB, reperfusion
•smoking •HLD proliferating smooth muscle and Unstable: (-) enzymes, ST depressed •ACEI for long term therapy
•HTN •males connective tissue *subtotal occlusion
•age (>45 men, >55 women) Location:
•fhx Symptoms: Enzymes: Anterior: V1-V4 (LAD), V1-V2 (p LAD)
CP: retrosternal, pain at rest, no CK/CK-MB: appears in 4-6hr, peak 12-24h Lateral: V5-6, I, aVL (Circumflex)
relief with NTG, >30min Troponin I: appears 4-8h, peak 12-24h Inferior: II, III, aVF (RCA)
*false elevations in renal failure, HF, PE, CVA Posterior: ST dep in V1-V2 (RCA, CFX)

Vasospastic Triggers: cold weather, exercise, Chest Pain: •EKG: transient ST elevations, resolves with •1st line: CCB (Diltiazem, Verapamil,
(Variant, cocaine, hyperventilation •rest (midnight, mid-morning) symptom resolution Amlodpine, Nicardipine) nightly
Prinzmetal •not exertional •Angiography: r/o CAD, may rule in •2nd line: NTG
) Angina Risks: female, >50, smoker •not relieved by rest vasospasm esp with Ergonovine AVOID BETA BLOCKERS

CARDIO About Pathophys/Clinical Presentation Diagnostics Treatment


Congestiv Inability of heart to pump enough blood to meet Pathophysiology: GOAL: confirm sx & Lifestyle modifications:
e Heart the demands of the body (MCC: CAD) Increased preload and afterload cause of HF •Wt loss, sodium <2g/d, fluids <2L/d
Failure and/or reduced contractility •Daily weight monitoring, smoking cessation
•Acute: began last few days to weeks Initial Testing
*systolic  HTN, MI, papillary muscle tear Compensations EKG CI: No-Dihydro CCB( Diltiazem, Verapamil)
1. increased sympathetic CXR: kerley B, effusion,
•Chronic: sx present for months 2. myocyte hypertrophy/remodel congestion, cardiomegaly, Pharmacotherapy: **loop, ACEI, BB
*typically cardiomyopathy or valve disease 3. RAAS activation  fluid butterfly pattern Loop Diuretics (Furosemide, Torsemide)
overload, ventricular remodeling MOA: inhibits water transport across loop
•High: heart can’t meet demands Initial Labs: SE: hypOkalemia, hypOcalcemia, hypOnatremia
(supply<demand) Left: Increased pulmonary venous •CBC, CMP, coagulation
-thyrotoxicosis, anemia; pagets, wet pressure  fluid backing to lungs studies, glucose, lipids Class I: *reduce mortality
beriberi, AV shunting, decreased CO -SOB •BNP *ACEI (“-pril”)
-Rales (>100 likely, >500 very) -reduce preload and afterload
•Low: insufficiency forward output (low EF) -Chronic, non-productive cough -released by ventricles -reduce aldosterone production (RAAS);
with pink frothy sputum •cardiac enzymes vasodilates
•Systolic: reduced EF (HFrEF) *MC type, S3 *MCC transudative effusions -reduces ventricular remodeling
-MI, dilated cardiomyopathy, myocarditis -increased sympathetic (adrenergic): Echo: GOLD SE: hypotension, hyperkalemia, cough,
-thin ventricles, dilated chamber sweating, dusky/pale skin, Normal EF: 55-60% angioedema, renal insuffieiency (CrCl <30)
tachycardia, cool EF <35% high mortality CI: hypotension, pregnant
•Diastolic: preserved EF (HFpEF), S4
-normal or increased EF  forced contraction Right:Increased venous pressure  NYHA Class: *Entresto (Sabucitril + Valsartan)
into a stiff ventricle systemic fluid retention I: normal activity, no sx
-HTN, LVH, elderly , constrictive pericarditis, -edema II: stairs, mild sx *BB (Carvedilol, Metoprolol, Bisprolol)
restrictive cardiomyopathy -JVD III: walking, marked -reduce mortality (increase EF, reduce ventricular
-thick ventricle, small chamber -ascites, HSM, RUQ tenderness limitation size)
IV: conversation/rest SE: dizziness, hypotension
•Left: DOE, PND, orthopnea, fatigue Symptoms: -stop during decompensated HF because EF
-MCC CAD, HTN •Chest pain, HTN, Flu-like sx reduces initially
-caution: 1st AVB, asthma, bradycardia
•Right: JVD, ascities, edema Physical Exam:
-MCC LHF, pulmonary, mitral stenosis General: tachy, diaphoresis *Aldosterone Antagonists (Spironolactone)
Narrow pulse pressure: <25 -SE: hyperkalemia, gynecomastia
Pulsus Alternans: LV HF -CI: K+ >5 or GFR <30
Precordial: lateral displaced PMI
Heart Sounds: S3 & S4 *Hydralazine/Nitrate (if black + ACEI/BB)
*safe in pregnancy
MOA: reduces preload and afterload
SE: dizziness, HA, tachyphylaxis

Acute: LMNOP: Lasix, Morphine, O2, Nitrate,


position

CARDIO About Clinical Presentation Diagnostics Management


HTN Risks: Causes: 2+ elevated readings on 2+ different •healthy >60yo-150/90
OSA, smoking, alcohol, obesity, 1. Sympathetic NS & RAAS visits >140/90 •<60yo, CKD, DM-140/90
metabolic syndrome, NSAIDs, 2. Pressure/natriuretic •UA, BMP, EKG, lipids
high sodium in diet (low K+) 3. Renal/cardio development •1st line-ACE/ARB
4. Increase in Na & Ca levels Complications: •AA 1st line-CCB/thiazide, HF/MI-BB
Pre-HTN: 120-139/80-89 Cardio: CAD, HF, MI, LVH, dissection,
Stage 1: 140-159/90-99 Fundoscope:, arterial narrowing AAA, PVD Diet: restrict sodium <2.4g/day
Sage II: >160/100  AV nicking  hemorrhages Neuro: TIA, CVA, aneurysm,
and exudates  papilledema encephalopathy
*isolated systolic is at (malignant HTN) Neph: renal stenosis, CKD
GREATER risk for Optic: retinal hemorrhage, retinopathy
cardiovascular disease then
isolated diastolic

Secondary HTN due to an underlying, Causes: Other: coarc, sleep apnea, ETOH, OCPs, “
HTN identifiable cause Renal: MCC of secondary COX-2 inhibitors
Endo: hyperaldosteronism,
*suspect is refractory to pheo, cushings
medication
HTN Hypertensive Urgency •Urgency: poorly controlled •CBC, CMP, UA; EKG, CXR Hypertensive Urgency: adjust medications
Urgency v. >220/125 WITHOUT end- •CT if neuro sx in emergency Decrease BP over 24-48h with ORAL agents
Emergency organ damage •Emergency: identify organ -hospital: Clonidine, Captopril, Nifedipine
failure  brain, kidney, heart Complications: -oral: labetolol, captopril, NTG, clonidine
Hypertensive Emergency •Renal-nephrosclerosis
>220/125 WITH end-organ •Eye-retinopathy, cotton wool Hypertensive Emergency: 1st line: BB, CCB
damage •Vascular-atherosclerosis, aneurysm Nicardipine, Labetalol, Esmolol
•CVA (SPB); S4, heave -ischemic CVA: keep BP 180-200
-hemorrhagic CVA: <130
-dissection: 100-110
HTN PHARM Mechanism of Action Side Effects CI
ACE Inhibitors Decreases synthesis of AG II/Aldosterone Hypotension CI:
“-pril” Cardio & renoprotective Cough (high bradykinin) Pregnancy
synergestic effect when used with thiazides Renal Insufficiency AKI
Reduce preload & afterload Hyperkalemia
Angioedema (high bradykinin)

ARB Blocks the effects of angiotensin II Hypotension CI


“-sartan” *consider if cant tolerate ACEI or BB Renal Insufficiency Pregnancy
Hyperkalemia
Calcium Channel Dihydropyridines: potent vasodilators HA CI:
Blockers -little or no effect on contractility or conduction Dizziness, lightheaded CHF *especially non-dihydro
Dihydropyridine *commonly used for HTN Flushing AVB
Nifedipine Edema
Amlodipine Non-dihydro: affect cardiac contractility and conduction
Non-dihydro *commonly used in HTN and A-Fib Verapamil: constipation
Verapamil
Diltiazem
Beta Blockers Catecholamine inhibitor Fatigue AVB
“-olol” Blocks adrenergic release of renin Depression Decompensated HF
Impotence Asthma/COPD
Alpha-1 blockers Blocks alpha-1 Syncope
Prazosin (Minipress) Increases HDL and loswers LDL Dizziness
Terazosin (Mytrin) Improves insulin sensitivity headache
Doxazosin (Cardura)
*good for HTN with BPH
Thiazide Diuretics Affect BP by reducing blood volume Hyponatremia
HCTZ Prevents kidney Na and water reabsorption in DISTAL TUBULE Hypokalemia
Chlorthalidone HyPERcalcemia
Metolazone *caution with gout and DM HyPERuricemia
HyPERglycemia

Loop Diuretics Inhibits water transport across loop of Henle Volume depletion Sulfa allergy
Furosemide Increases excretion of water, Na, Cl, K Hypokalemia, natremia, calcemia
Torsemide Hyperglycemia
Bumetanide Metabolic alkalosis
Ototoxicity

K+ Sparing Diuretics Inhibits aldosterone-mediated Na+/H2O absorption Hyperkalemia Renal Failure


Spironolactone Gynecomastia *spirolnolacton Hyponatremia
Amiloride
Eplerenone

**choices for gout: CCB or Losartan (only ARB that does not cause hyperuricemia)

CARDIO About Clinical Presentation Diagnostics Management


Hyper- Causes: •Xanthomas (fatty growths Screening: Lifestyle: weight, exercise, dietary changes
lipidemia •Hypercholesterolemia: underneath skin-joints) Ages 20-79 every 4-6 years
hypothyroidism, pregnancy, High risk: screening 20-25yo M, 30-35yo F Pharmacotherapy:
kidney failure •Xanthelasama (lipid plaques on Low risk: 35yo M, 45yo F •LDL: Statin, bile acid sequestrants
eyelids) •Triglycerides: Fibrates, Niacin
•Hypertriglyceridemia: DM, Who to Treat: •increase HDL: Niacin, Fibrate
ETOH, obese, steroids, estrogen Pancreatitis may be seen in 1. DM between 40-75 years old
hypertriglycerides 2. 40-75yo with risk 7.5% and higher High intensity:
Linkage between LIPIDS and 3. Age 21+ and LDL 190+ Atorvastatin 40-80 & Rosuvastatin 20-40
INFLAMMATION 4. Clinical ASCVD •Age 40-75yo with DM
•Clinical CAD
•LDL 190+,
•Age 40-75 10 year risk >7.5%

Goals: LDL <100, Cholesterol <200, HDL >60

HLD PHARM Mechanism of Action Side Effects CI


HMGcoA Reductase Inhibits the rate limiting step in hepatic cholesterol synthesis Myositis/myalgia/rhabdo Best given at bedtime when cholesterol
Inhibitors (Statins) Removes LDL from the blood Hepatitis synthesis is maximal
BEST DRUG TO REDUCE LDL *SE rate increased with tetracyclines

Fibrates Inhibits peripheral lipolysis and reduced hepatic triglyceride Myositis, mylagias CI: hepatobiliary disease or severe renal
Gemfibrozil production (by decreasing hepatic extraction of fatty acids) Hepatitis disease
Fenofibrate Gallstones (blie acide lithogenicity)
BEST DRUG TO REDUCE TRIGLYCERIDES

Niacin (B3) Increased HDL, reduces plasma fibrinogen levels, decreased hepatic Flushing *decrease with ASA before PUD
production of LDL and VLDL Headache, Warm sensation High glucose
Pruritis Hepatotoxicity, N/V/D
Hyperuricemia, Hyperglycemia Dry skin

Bile Acid Binds bile acid in intestine blocking liver reabsorption of bile acids GI: N/V/ bloating, cramping
Sequestrates Hepatitis
Cholestyramine ONLY MEDICATION SAFE IN PREGNANCY Increased TGs
Colestipol *can also be used for pruritis with biliary obstruction
Colesevelam
Ezetimide (Zetia) Inhibits intestinal cholesterol absorption  lowers LDL
*may be used with statins

CARDIO About Clinical Presentation Diagnostics Treatment


Cardiogenic Primary cardiac/myocardial Causes: •Oxygen
Shock dysfunction  inadequate perfusion Cardiac disease: MI, myocarditis, •Vasoconstriction •Isotonic fluids (NOT AGGRESSIVE)
 low CO and increased systemic valve, congenital, arrhythmia •Hypotension *only shock where large amounts of fluids
vascular resistance (SVR) & •Low CO and increased pulmonary are not given
pulmonary wedge pressure (>15) capillary wedge pressure
Inotropic support: increase contractility
and CO
•Dobutamine, Epinephrine

Orthostatic Fall in SBP by 20mmHg and/or Cerebral hypoperfusion •blood pressure Conservative:
Hypotension DBP by 10mmHG •dizziness, lightheadedness •tilt table: BP reduction at 60 degrees •increase salt and fluid intake
•palpitations •labs •gradual position changes
Causes: •blurred vision •compression stocking
•impaired autonomic function •darkening of visual fields •discontinue offending meds
•medications: anti-HTN, diuretics, •syncope
narcotics, antipsych, antidep, alcohol Medication
•neuro: parkinsons, DM neuropathy •Fludrocortisone *if fail conservative
•hypovolemia

Vasovagal Systemic hypotension associated with Prodromal: dizziness, Triggers:


Syncope bradycardia and/or peripheral lightheadedness, epigastric pain, •blood
vaso/venodilation palpitations, blurred vision •phobias
•emotiona stress/fear
MCC of syncope! Postdromal: syncope •trauma

CARDIO About Clinical Presentation Diagnostics Treatment


Endocarditis Cause: ORAL (dental, candy, brush) MC: 3 blood cultures, 1 hour apart Acute Endocarditis: Nafcillin +
-others: IVDU, EGD, TURP, cath, •fever (90%), chills, night sweats EKG, TTE  TEE echo Gentamicin x4-6 weeks
•MSK pain (back) Increased ESR and CRP
•Localization of the infection is •murmurs (80%) Subacute Endocarditis: PCN or
determined by turbulent blood flow •CHF (2/3 cases) DUKE CRITERIA Ampicillin + Gentamicin
Major criteria
•Infection of endothelium/valves due Peripheral Manifestations: • (+) blood culture (2 are +) Prosthetic valve: Vancomycin +
to colonization •petechia *strep or staph •endocardial involved on echo (TEE) Gentamicin + Rifampin
•splinter hemorrhages *strep, staph •new regurg murmur
•MC bug: S. aureus •osler nodes *strep *PAIN (finger) Fungal: Amphotericin B
•MC on Left side, TV MC in IVDU •Janeway *PAINLESS (palms) Minor Criteria:
•Roth Spots (eyes on funduscopic) •heart condition or IVDU Prophylaxis: Amoxi 2g 30-60min
•fever (>100.4F) -prosthetic heart valves
•Acute: infection of normal valves •vascular and embolic PNA -prior endocarditis
•Subacute: abnormal valve •immunologic phenomena -cyanotic congenital heart disease
•IVDU: MRSA, Pseudomonas •microbiologic evidence -cardiac transplantation
•Prosthetic: Staph epidermis *1 normal, or 2+ abnormal

Interpretation:
•Definitive: 2 major or
1 major + 3 minor, or 5 minor

CARDIO About Clinical Presentation Diagnostics Treatment


Pericarditis Inflammation in pericardium •PLEURITIC CHEST PAIN •Viral titers •FIRST LINE-NSAIDS x7-14d
-worse with inspiration & supine •Cardiac enzymes -Ibuprofen 600-800mg TID or
Causes: -sharp pain, persistent •Echo: assess complications *nml Indomethacin TID for 1-2 weeks
•Idiopathic *MC -relieved with sitting up and leaning •CBC, BMP, ESR/CRP -ASA post MI
•Viral (entero, coxsackie, echo) forward
*MC! -radiate to back, neck, shoulder EKG: diffuse ST segment elevation 2nd line: Colchicine
•Systemic: thyroid, lupus, RA •Fever  pseudonormalization  T wave -use for recurrence
•Neoplasms: lung & breast CA inversion  normal
•Drugs: procainamide, INH, Auscultation: -smiley face & diffuse Steroids if sx >48h and refractory
hydralazine pericardial friction rub *washing -concave V1-V6, PR depressions in
•Myocardial Injury -best at end of expiration while leads with ST elevation
•Pericardial Injury upright and leaning forward *opposite in AVR

Pericardial •Fluid in pericardial space Distant (muffled) heart sounds CXR: cardiomegaly Small/no tamponade: observe and
Effusion EKG: low QRS voltage treat underlying cause
•MCC: pericarditis Echo: increased pericardial fluid
•Others: infection, radiation, cancer,
dialysis, CT disease

Cardiac Effusion causing significant pressure BECKS TRIAD: Echo: effusion + diastolic collapse of Pericardiocentesis
Tamponade on the heart  restricted filling  •JVD cardiac chambers Pericardial window if recurrent
decreased CO •Muffled heart sounds
•hypotension EKG: electric alternans (alternate small
and big QRS complexes)
Exam: Kussmaul & Pulses paradoxus
>10mmHg decrease in SBP with
inspiration
-increased filling of R heart with
inspiration  decreased L filling

Constrictive Thick, fibrotic and calcified heart •Dyspnea (MC sx) Echo: thickening of pericardium, Diuretics for sx improvement
Pericarditis  diastolic dysfunction •RHF: JVD, edema, N/V, ascites
 increased venous pressure and Kussmaul sign: increased JVD with CXR: pericardial calcificiation Definitive: pericardiectomy
decreased stroke volume inspiration -if unresponsive to diuretics
Cardiac CT/MRI: thickening
MCC=TB *underdeveloped Murmur: pericardial knock
-high pitched 3rd heart sound due to Cardiac Cath: *CONFIRMATORY
MC is radiation, surgery, and viral sudden cessation of ventricular filling “square root sign”
pericarditis *developed from thickened pericardium

VALVE DISORDERS
•Increase in venous return increases the intensity of ALL murmurs EXCEPT: HOCM and MVP click
“when the MVP with the hypertrophied arms squatted, the sound of the crowd got lower in anticipation”
“when the MVP with the hypertrophied arms stood up, the sound of the crowd got louder”
•Inspiration increases the intensity of right sided murmurs & decreases intensity of left sided
•Radiation: axilla (MR), carotid (AS)

CARDIO General Features Clinical Presentation Diagnostic Studies Management


Aortic Stenosis Congenital v. acquired Most asymptomatic for years EKG: may demonstrate LVH •1st line-AV Replacement with Sternotomy
-Congenital: bi or quad Cardinal SX: angina, syncope, CHF -symptomatic patients, or no sx & severe
Echo *diagnostic study of choice!
-Degenerative: * elderly Murmur: -severe: <1 cm 2 -critical: <0.7 cm 2 •2nd line-Transcutaneous Aortic Valve
•rheumatic fever -Harsh, Mid-systolic at R 2nd ICS Implantation/Replacement (TAVI/R)
•degenerative (Ca2+) -Crescendo-decrescendo •Cardiac Catherization -poor candidates for open heart
•Risks: HTN, HLD, smoke -louder with leaning forward, confirms severe AS and any CAD -multiple comorbidities, high pre-op risk
squatting, supine -higher risk for stroke immediately after
Pathophysiology: -decreased with valslava
thickening & calcification -Radiates to CAROTIDS
 narrowing  LVH  -Thrill, S4 gallop, displaced PMI •Balloon Valvuloplasty
systolic dysfunction, -congenital AS; not used for degenerative
increased afterload Exam: weak, delayed carotid pulse
*no medical tx-manage symptoms of HF
*avoid nitrates, CCB, BB because patients
are dependent on preload

Chronic Causes: *Asymptomatic  CHF, angina Signs of widened pulse pressure: •Valve Replacement (AVR): those with
Aortic •Aortic leaflets: Water Hammer Pulse: swift upstroke symptomatic severe AR or with LV changes
Regurgitation rheumatic fever, congenital Murmur: and rapid fall of radial pulse accentuated
abnormalities, endocarditis •Early diastolic, radiate to apex with wrist elevation •AR prior to LC dilation greater than 50mm or
•Decrescendo, blowing reduction in EF to less than 50%
•Aortic root: •High pitch, 2nd and 4th L spaces Corrigans pulse: same but carotid
dissection or dilation, •Widened pulse pressure  •Vasodilator therapy-unload the ventricle
Marfan bounding pulses Hills sign: popliteal artery systolic  Arterodilation
•S3&S4 gallop pressure > brachial artery by 60mmHg  Reduce afterload  less pressure to
*LVH due to dilation from •Austin Flint Murmur: mid late (MOST SENSISTIVE) push back into the ventricle
regurg volume diastolic rumble at apex due to
retrograde regurg De Mussets sign: head bobbing w/ beat

Echo*diagnostic of choice
Acute Aortic Causes: •S/S of cardiogenic shock Echo *diagnostic study of choice SURGICAL EMERGENCY
Regurgitation •acute MI •Pale, cool extremities -performed quickly and bedside
•infective endocarditis •Weak, rapid pulse Vasodilator and diuretic if BP is stable
EMERGENCY •trauma rupture of leaflet *LV in unable to accommodate the -Inotropic agents & vasopressors
•aortic root dilation Murmur: increased diastolic volume -> increased
•acute dysfunction of valve •low pitch, early diastolic pressures and pulmonary congestion **Treatment of choice: Urgent AVR

CARDIO General Features Clinical Presentation Diagnostic Studies Management


Mitral Stenosis •2/3 are women •Dyspnea, orthopnea EKG: LAE, RVH, A-fib Mild to moderate: CONTROL HR
•Don’t develop sx until 20-40 years •Atrial fibrillation *enlarged atria
following disease ~30-40yo •Blood tinged sputum *pulmonary edema Echo: *diagnostic of choice Moderate to Severe (NYHA III-IV):
•mitral facies: ruddy (flushed) cheeks with •Rhematic deformitity-doming •Percutaneous balloon valvuloplasty
Causes: facial pallor (hockey stick) *not common or definitive treatment
•Rheumatic fever is MCC! •Large LA compared to LV
•congenital abnormalities, Additional findings: -normal MV area: 4-6cm 2 Surgery is definitive treatment with
connective tissue disorders, left PE, edema, Ortner Syndrome (L recurrent bioprosthetic or valve replacement
-critical: <1cm 2
atrial tumors, aggressive surgical laryngeal nerve compression due to RAE)
repair of MR •smoke in left atrium
Murmur:
•Diastolic, Low-pitch, rumbling Cardiac cath: measure CO
•Heard at apex in LLD position w/ bel
•Opening snap following S2

Mitral Valve •1-3% population; MC in women Nonspecific sx (MVP syndrome): DX: Echo •NO TREATMENT if mild prolapse
Prolapse •superior displacement in systole •autonomic dysfunction: atypical CP, *diagnostic study of choice and asymptomatic
anxiety, panic attacks palpitations
Causes: •Beta Blockers in patients with
-connective tissue disorder Murmur: autonomic dysfunction
-Marfans or SLE •Mid-systolic click, usually followed by
-Autosomal dominant late-systolic murmur
-Myxomatous degeneration •Heard at the apex
•Squatting: delays click
•Standing: earlier click
*gallops won’t change with position!

Chronic Abnormalities of mitral leaflets, •fatigue EKG: Reduce Afterload:


Mitral annulus, chordae, papillary muscles •dyspnea on exertion •LA enlargement, LVH, A-fib •vasoduilators (ACEI, hydralazine)
Regurgitation -MVP (MCC!) •peripheral edema •diuretics
-LV dilation, posterior MI Echo: *Diagnostic of choice
-rheumatic fever Murmur: Surgery: annuloplasty *DEFINITIVE
-endocarditis •blowing holosytolic murmur Cardiac Cath: •should be performed before
•heard at apex & radiates to axilla/back •Assess function & filling pressures irreversible myocyte damage and left
•Regurgitating blood flow from LV •mid-systolic click centricular remodeling occur
to LA in systole •best heard in LLD position
•widely split S2 and displaced PMI Monitoring: annual echo
Acute Mitral •LA does not dilate to Signs of cardiogenic shock SURGICAL EMERGENCY
Regurgitation accommodate regurg volume -
increase in LA and pulmonary Murmur: Urgent valve replacement
venous pressure  congestion •Soft, low-pitched sound in early systole
EMERGENCY *have high index of suspicion
Causes:
•acute MI, trauma, endocarditis
•tachyarrhythmia w/ chronic MR
•MVP

CARDIO Causes/ S&S/Physical Exam/Murmur Diagnostics Treatment/Management


Tricuspid *MC in women •Uncommon •Liver congestion •Echo-significant: <1cm 2 Diuretics
Stenosis •Usually associated with AS or MS •Varices, Ascites, JVD, LE edema •Loop diuretics are best
•Palpable presystolic pulsation with •EKG: RA enlargement •ascites  ADD aldosterone
Causes atrial contraction over liver antagonists (Spironolactone)
•rheumatic heart disease MCC! *always palpate liver with ascites •CXR: cardiomegaly from
•carcinoid, congenital valve enlarged R atrium Surgery:
abnormalities, tumors Murmur: •TV replacement if symptomatic
•soft, high-pitch, mid-diastolic, L 4th ICS
•Reduced RA emptying  Increased •Accentuated with inspiration
venous congestion and reduced RV •Opening snap at left sternal border
CO  fluid overload

Tricuspid •Regurgitating blood flow from RV •Well tolerated without pulmonary HTN CXR: •Treatment of underlying cause
Regurgitation to RA •As it progresses, may have RV failure:
-fatigue, abd bloat & peripheral edema •Repair TV with annuloplasty
•Secondary to dilation of the RV and *persistent symptoms
tricuspid annulus due to RV failure Murmur:
(pulm HTN or L failure) •High pitch, pansystolic, tricuspid post •TV replacement
•accentuated with inspiration or leg raise Physical Exam: *pts with underlying primary leaflet
•Increased RA pressure -> increased •Carvallos sign: increased intensity with •JVD, hepatic congestion and pathologic condition
venous congestion & reduced RV inspiration palpable systolic pulsation
output -> volume overload

CARDIO Causes S/S Murmur Management/Treatment


Pulmonic CONGENITAL (peds pt)**** Critical at birth: Murmur: Mild: (pressure <30mmHg)
Stenosis •Rheumatic disease is RARE •Central cyanosis at birth •systolic at pulmonic post •asymptomatic and no intervention
•Noonan, Trisomy 13 •otherwise asymptomatic until adolescence •increases with inspiration
•Isolated congenital lesion or young adulthood •RV lift on palpation of Moderate-Severe: (>50mmHg)
precordium •Balloon valvuloplasty or surgical valve
fusion of pulmonary leaflets -> Moderate-Severe: •S1 followed by opening click replacement
pressure overload -> RVH •fatigue, dyspnea  RV dysfunction  that’s louder with expiration
RV failure

Pulmonic •Result of dilation of the PV annulus •Sx due to primary disease and secondary Murmur: TREAT PULMONARY HTN!
Regurgitation secondary to pulm HTN to RV failure •High pitched, blowing
•usually congenital •Heard at 2nd L ICS •PV replacement is rare because its being
cause by pulmonary HTN, so it will
come back if that is still present
Graham-Steell murmur:
brief decrescendo early diastolic
murmur at LUSB with full
inspiration
•increased w/ squat, inspiration
•decreased with valsalva

CARDIO About/Risks Clinical Presentation Diagnostics/Screening Management


Abdomina •Dilation of the infrarenal aorta Most are asymptomatic! DX: Repair for:
l Aortic is a normal part of aging ~2cm CBC, BMP, PT/INR, PTT -aortic aneurysm larger than 5.5cm
Aneurysm -AAA when >3cm Symptomatic: *more concerning Abdominal US- study of choice -rapid expansion (>0.5cm in 6mo)
(AAA) •UNRUPTURED PAIN *especially if unstable -symptomatic pain & tenderness
Risks: -Mild to severe mid-abdominal
•white male, smoke, age >60 discomfort often radiating to lower back •CT scan w/ IV contrast provide a more >4.5cm: refer to vascular surgery
•Atherosclerosis (MC) -Constant or intermittent reliable assessment of diameter 4-4.5cm: monitor US q6mo
-Exacerbated by gentle pressure -perform when >5cm & surgery 3-4cm: monitor US qyear
Pathophysiology: -Distal embolization is rare -test of choice for THORACIC
Proteolytic degeneration of -pulsatile abdominal mass Beta blockers help reduce shearing
aortic wall, connective tissue, and •Angiography: GOLD forces and decrease expansion
inflammation •RUPTURE PAIN *lethal
-Sudden blood into retroperitoneal space Screening:
Two major groups: -Severe pain, palpable mass, hypotension •One-time screening for:
Fusiform: circumferential -Free rupture into peritoneal cavity is -men 65-75 and have smoked 100
Saccular: outpouching LETHAL *bruising on the back (Flank) cigarettes in a lifetime
*higher risk of rupture
Thoracic Risks: •Most asymptomatic •CXR: widened mediastinum •Repair when 6cm or larger
Aortic •Most due to atherosclerosis *easiest initial tool, do 1st Treatment: endovascular grafting
Aneurysm •Symptoms depend on size and position
•CT disorders: Ehlers-Danlos, -Substernal back or neck pain •CT scan: modality of choice Involvement of proximal aortic arch:
Marfan Syndrome -Hoarseness due to L recurrent laryngeal -open surgery *substantial risk
-aortic regurg due to dilation -may need replacement/repair of AV
•Bicuspid aortic valve -performed by CT surgeon

Aortic •Tear in the innermost part of the S/S: •EKG: LVH Medical control:
Dissection aorta (intima) •Chest pain: SUDDEN, severe, ripping, •CXR: widened mediastinum *initial *descending with no complications
•Locations: Ascending (MC), radiating to back or neck •Multiplanar CT of chest/ abdomen:
descending, aortic arch •hypertensive immediate diagnostic study BP Control:
*ascending has high mortality •N/V, sweating •CTA, MRA, TEE: GOLD •lower SBP 100-120 & pulse pressure
•MC in men over 50 •syncope, hemiplegia, paralysis of LE •BB first line! (Lebetolol, Esmolol)
•ischemia (MI) Debakey (Stanford) Classes: -can add CCB or Nitroprusside
Risks: •unequal blood pressure in both arms •Type I (A): ascending aorta  aortic
•aging, atherosclerosis, HTN -variation >20mmHg b/w R & L arm arch and possibly beyond Pain: Morphine (pain & vasodilation)
•blunt trauma, Marfans, aortic •diastolic murmur •Type II (A): confined to ascending
valve defect, aortic coarctation, •acute new-onset aortic regurg with •Type III (B): descending aorta Surgical: acute proximal or acute distal
preexisting, prego ascending dissection with complications

CARDIO About Clinical Presentation Diagnostics Treatment


PAD Stenosis or occlusion in artery due to •Atypical leg pain •Pulses: decreased or absent •Lifestyle: smoking cessation
atherosclerosis (MC in LE) -rest pain means advanced disease •decreased capillary refill •Exercise therapy
•Blood pressure, carotid bruits
•Coronary artery: angina •Intermittent Claudication (MC!) •Exam legs and feet
•Carotid artery: stroke, TIA -aching, pain, tightness in LE •Skin: COLD, pale, atrophic changes, Pharm: Aspirin or Plavix
•Renovascular: HTN, renal -reproducible thin/shiny hair, hair loss •Cilostazol (Pletal)
•PAD: claudication, limb ischemia -brought on by EXERCISE -ulcers: LATERAL MALLEOLUS -supress cAMP degredation
-relieved with rest within 10 min -reversibly inhibits platelete
•Distal aorta & proximal iliac: smokers -sx are distal to lesion Leg Lift Test: 60 degrees x1min aggregation
-buttock, hip, groin pain (+) if feet turn white when lower down -SE: edema, GI, HA, bleed
-Leriche syndrome: claudication, impotence, •Functional Impairement -CI: HF
decreased femoral pulses -do not have claudication but have Dependent Rubor: seated to supine,
•Femoral & popliteal: 60+, minorities rest pain or ulceration assess blood flow; longer red  severe Others:
-thigh or upper calf •ACEI, statin, glycemic control
•Tibial artery: Diabetics •Limb ischemia: ulcer, gangrene ABI: BEST TOOL (normal 1-1.2)
-lower calf, ankle, foot Abnormal is < 0.90, < 0.50 is severe
-ABI of 0.85 needed to heal ulcer
MC in femoral or popliteal artery -use TBI if non-compressible (>1.40)

Arteriography: GOLD STANDARD

Acute Causes: 5 Ps: •MEDICAL EMERGENCY! Immediate revascularization


Arterial •thrombus: stable atheroma with fibrous cap •pallor 1. vascular surgery consult
Occlusion  plaque rupture  acute occlusion •pain/ paresthesias •Doppler: little to now flow 2. begin IV Heparin bolus and
of a Limb (numbness/tingling) •EKG: determine if Afib continuous infusion
•embolus  AFIB MC cause •pulseless •Labs: CBC, PT/INR, PTT
•paralysis (muscles w/ no perfusion) •Echo: done LATER if embolic source is •Once stable: Warfarin for at least
MEDICAL EMERGENCY! •polar (cold) suspected (TEE w/ bubble) 3 months+, goal INR 2-3

Thrombo- NOT ATHEROSCLEROSIS TRIAD: •Aortography: occlusive lesions of TOBACCO CESSATION!


angiitis •Segmental, inflammatory, thrombotic 1. Superficial migratory small/medium vessels with corkscrew
Obliterans processes in small/medium distal arteries thrombophlebitis: large, red, tender collaterals *GOLD -revascularization
(Buerger superficial veins  nodules -amputation frequently required
Disease) •Closely linked to tobacco use •Allen test: abnormal -wound care
•young males <40, smokers 2. Distal ischemia (MC): -assess patency of radial and ulnar -NSAIDS or opioids for pain
claudication, toes to finger ischemia arteries
Pathophysiology: Raynauds: CCB (Nifedipine,
•Vasodysfunction and microthrombi 3. Raynauds Nicardipine, Amlodipine)

•Presents with distal ischemic rest


pain or ischemic ulceration

Atrial •MC cardiac cancer •Dyspnea Diagnostics: Surgical removal


Myxoma •MC in the left atrium •Weight loss •TEE: pedunculated mass with “ball-
•Syncope *due to mitral stenosis valve” obstruction of the mitral valve
•Can cause an obstruction of the mitral orifice •Flu-like symptoms orifice
mimicking mitral stenosis
Exam: similar to mitral stenosis

CARDIO About Clinical Presentation Diagnostics Treatment


Deep Vein MC in the calf Unilateral swelling of LE (>3cm), tender D-Dimer: highly sensitive but Anticoagulant x3 months-First Line
Thrombosis not specific -LMWH + Warfarin *prego, cancer
(DVT) Pathophysiology: Exam: *negative d-dimer can eliminate -Heparin + Dabigatran (Pradaxa) or
Virchows Triad: •WARM skin & Dusky cyanosis Heparin + Edoxaban (Savaysa)*1st line
1. Venous stasis •Palpable cord, normal pulses Venous duplex US: 1st line -Rivaroxaban (Xarelto) or Apixaban
2. Endothelial damage •Homans sign: calf pain with -noncompressible echogenicity (Eliquis) *1st line
3. Hypercoaguability dorsiflexion (unreliable)
Venography: GOLD Heparin (antithrombin III): PTT
Wells Criteria: Warfarin (Vitamin K antagonist)
Consequences: PE -extrinsic pathway, monitor PT/INR

IVC Filter: recurrent despite anticoag,


anticoagulation is CI, or RV dysfunction

Varicose Veins •Varicose veins develop in LE •Dull, aching heaviness or feeling of •No diagnostic evaluation •stockings (20-30mmHg), leg elevation
•Dilated, tortuous superficial veins fatigue in legs brought on by periods of
Increased intraluminal pressure  standing, relieved with elevation •Imaging needed for surgical •Surgical:
reverse venous flow intervention-duplex US -sclerotherapy: inject sclerosing agent
•Itching , palpable -laser therapy: wavelength
Hallmark: venous reflux & HTN -endovenous ablation -vein stripping

CARDIO About Clinical Presentation Diagnostics Treatment


Chronic Venous •Severe manifestation of venous •Burning, aching, heavy leg pain Compression stockings, leg elevation
Insufficiency hypertension -worse with standing/sitting Regular exercise
-better with walking & elevation lipodermato
•Valve leaflets that do not coapt •Edema, hyperpigmentation, Ulcer: UNNA boot, wet to dry dressings
because thickened and scarred or
in a dilated vein so impaired •Venous ulcers: medial malleolus
function atrophic blanche
 legs develop venous •Lipodermatosclerosis
HTN and high -pigmented, swelling, red, “bowling pin”
hydrostatic force
•Atrophie Blanche corona
MCC-prior deep venous -star shaped ivory-white depressed
thrombophlebitis atrophic plaque; red dots within scar Trendelenburg test: elevate one
leg at 90 degrees, occlude great
•Corona Phlebectatica saphenous vein, have patient
-dilated veins around the ankle stand for 20 sec  slow ankle
fillings suggests competency
Superficial •Inflammation of a vein due to Trousseau’s sign: migratory •CBC: WBC may be elevated 1st line: supportive therapy
Venous clot in the superficial vein thrombophlebitis with malignancy -elevate, warm compression, compression
Thrombophlebiti -great saphenous vein •Venous Doppler US: non- stocking, NSAIDs
s compressible vein with clot
Causes: IV catheter, trauma, Local phlebitis: tender, pain, edema, Septic state:
pregnancy, varicose veins erythema, induration •PCN + Aminoglycoside
•+/- palpable cord •Fever (if septic) •Vanc + Rocephin
•MC pathogen-Staph Aureus
PULM 12%-tobacco
PULM About Presentation Diagnostics Treatment
COPD Progressive irreversible airflow obstruction •cough, dyspnea with exertion Screening: annual screen with Category A (Gold 1-2):
- Loss of elastic recoil of alveoli •excess sputum production CT in adults 55-80yo w/ 30yr SABA or SAMA
- Increased airway resistance pack hx and currently smoke or +/- LABA or LAMA
smoked within past 15yr
Risks Category B:
•SMOKING (>15 pack years) CXR: SABA or SAMA + LABA or LAMA
•secondhand smoke, occupational/environmental •hyperinflation, flat
•alpha-1-antitrypsin deficiency diaphragm Category C:
*only genetic disease linked to COPD in age <40 •increased radiolucency LAMA* (reduce exacerbations)
•narrowed heart shadow + LABA, LABA/ICS
•increased retrosternal space
Category D:
Chronic •Productive cough x3mo for 2 consecutive years •Productive chronic cough Pulmonary Function Tests: LABA/LAMA,
Bronchitis •dyspnea Spirometry *Gold Standard consider triple therapy (Trelegy)
•Chronic airway inflammation  mucus gland •cyanotic, edema, RHF, cor pulm •decreased FVC
*Blue hyperplasia and goblet cell mucus production, •respiratory acidosis •decreased FEV1 > FVC
Bloater airway narrowing, increased airway resistance  •V/Q mismatch (normal in •decreased FEV1/FVC ratio
obstruction *infiltration of neutrophils and CD8 emphysema) •increased TLC (air trapping) Other Therapies:
Pulmonary rehab: Categories B-D
*Increased susceptibility to S. pnuemo and H. flu Exam: rales, rhonchi, wheeze (Outpatient program)

Emphysema •Irreversible enlargement of air spaces distal to •dyspnea with exertion Labs: increased Hgb and Hct and Oxygen therapy:
terminal bronchioles  destruction of alveolar (HALLMARK) hypercapnia due to hypoxia PaO2 ≤55 or pulse ox ≤88%
*Pink Puffer capillaries and wall destruction •pink complexion (C. BRONCHITIS)
•thin, barrel chest, no edema Reduce Mortality:
•loss of elastic recoil in acinus, collapse, and •respiratory alkalosis Pulse Ox: (94-99%) •smoking cessation
increased compliance  trapping & obstruction •pursed lips to prevent airway • >90% mild, <90%-needs O2 •O2 (paO2 ≤55 or 88%, cor pulm)
collapse • <88%–supp O2 by medicare •PNA and Influenza vaccines
Areas:
•centrilobar: proximal acinar-bronchioles Exam: Arterial blood gas Acute Exacerbation (C. Bronchitis)
(MC with smoking) •decreased breath sounds •mild: low O2, nml CO2 •Macrolides (Azithromycin)
•panacinar: diffuse-bronchioles and alveoli •increase A-P diameter (barrel) •mod/severe: low O2, high CO2 •Cephalosporins (Cefuroxime)
(MC with A1AT deficiency) •hyperresonance to percussion •Augmentin
•paraseptal: distal-alveoli only •Fluoroquinalones
Tidal Volume: volume of air moved into or out of the lungs during quiet breathing
Residual Volume: air in lungs after maximal expiration
Expiratory Reserve Volume: volume of air that can be further exhaled
Inspiratory Reserve Volume: volume of air that can be further inhalaed
Vital Capacity: max volume that can be exhaled following max inspiration
Total Lung Capacity: volume in lungs at max inspiration
Functional Residual Capacity: volume of gas in lungs as normal TV (air which gas
exchange takes place)
Forced Expiratory Volume (FEV1): air exhaled after 1 second forced expiration
Forced Vital Capacity (FVC): volume of air expelled from max inflated lung with
patient breathing as hard and fast as possible
COPD Treatment

- GOLD Guidelines:
o GOLD1: mild  FEV1 80%+
o GOLD 2: moderate  FEV1 50-70%
o GOLD 3: Severe  FEV1 30-49%
o GOLD 4: Very Severe  FEV1 <30%

- Gold Stages
o GOLD A: mMRC 0-1, CAT <10, 0-1 exacerbation
o GOLD B: mMRC 2+, CAT 10+, 0-1 exacerbation
o GOLD C: mMRC 0-1, CAT <10, 2+exacerbation or 1+ hospital
o GOLD D: mMRC 2+, CAT 10+, 2+ exacerbation or 1+ hospital
Class Drugs
SABA Albuterol, Levalbuterol

MOA: interacts with B-2 receptors (peripheral) allowing for dilation of


the smooth muscles in the bronchioles
SE: Tachycardia, Arrhythmias, tremors, CNS stimulation, low K+
-metered dose inhaler: 90mcg two puffs q4-6h prn
-dry powder inhaler: 200mcg, 1 inhalation prn; max 4/day
SAMA Ipratropium

MOA: Central bronchodilator  Inhibits vagal mediated


bronchoconstriction and nasal mucosal secretions
SE: Dry mouth, thirst, Blurred vision, glaucoma, Urinary retention, BPH,
dysphagia
-nasal: 0.03% solution, two sprays per nostril q8-12h
-inhaler:
-metered dose inhaler: 2 puffs q6h -nebulizer: 0.5mg every 6-8h
ICS Pulmicort, Flovent, Azmacort, Qvar, Beclomethasone
MOA: reduces inflammation; SE: thrush

LABA Salmeterol, Formoterol, Arformoterol, Indacaterol, Olodaterol

BBW: asthma-related death as a monotherapy (w/o ICS)


-dry powder: one inhalation BID

LAMA Tiotropium (Spiriva)


-dry powder inhalaer: 1 capsule (18mcg) inhaled daily
-soft-mist inhalaer: two inhalations (5mcg) once daily
Aclindinium (Tudorza)
-dry powder inhalation: one inhalation (400mcg) twice daily
Umeclidinium (Incruse)
-dry powder inhalation: one inhalation (62.5mcg) once daily

LABA-ICS -Budesonide/Formoterol (Sumbicort)


Combo -Fluticasone/Salmeterol (Advair)
-Momentasone/Formeterol (Dulera)
LABA- Tiotropium-Olodaterol (Stiolto Respimat)
LAMA -Soft mist inhaler: 2 inhalations once daily
Combo Umeclidinium-Vilanterol (Anoro Ellipta)
-Dry powder inhaler: 1 inhalation once daily
Glycopyrrolate-Formoterol (Bevespi)
-Metered dose inhaler: 2 inahlations twice daily

Triple Trelegy -dry powder inhaler: 1 inhalation once daily

PULM About Presentation Diagnostics Treatment


Asthma Reversible; airway inflammation & •Dyspnea Pulmonary Function Test Mild Intermittent:
bronchoconstriction in response to •Wheezing Spirometry *GOLD •SX ≤2d/wk, night awake ≤2x/mo, FEV1 >80%
trigger •Cough (worse at night) •low FEV1 & low FEV1/FVC
•Chest tightness Mild Persistent:
Pathophys: •Fatigue Bronchoprovocation: •SX >2d/wk, night 3-4x/mo, FEV1 >80%
1. airway hyperactivity: IgE binds Methacholine challenge (≥ 20 decrease
to mast cells Physical Exam: on FEV1) followed by bronchodilator
•Prolonged expiration challenge (FEV1 increases by 12% and Moderate Persistent:
2. bronchoconstriction: airway •SX daily, night awake >1x/wk, FEV1 60-80%
narrowing due to smooth muscle •Hyperresonance to percussion 200ml)
contraction, edema, mucus, •Decreased breath sounds
•Tachycardia, tachypnea Peak Expiratory Flow (PEFR) Severe Persistent:
hypertrophy  air trapping •Accessory muscles *assesses asthma exacerbation severity •SX throughout day, awake nightly, FEV1 <60%
3. inflammation and response
Severe: •Normal is 400-600 Treatment
Risks: Atopy, FHX, air pollution, •silent chest (no air movement) •discharge criteria is >70% predicted or 1. SABA (all)
obesity, second hand smoke, males •altered mental status >15% initial attempt 2. +ICS
•pulsus paradoxus 3. +ICS/LABA or med ICS
Culture: curschmann’s spirals, 4. +medium dose ICS/LABA
Atopic Triad: asthma, atopic
Charcot-Leyden crystals 5. +high dose ICS/LABA
dermatitis, allergic rhinitis
6. +high dose ICS/LABA or oral
ABG: respiratory alkalosis (due to
Samters Triad: asthma, ASA
tachypnea) Admit if: PEFR <50%, ER within past 3 days, status
allergy, nasal polyps
asthmaticus

PULM Drugs Side Effects Routes


SABA Albuterol, Levalbuterol SE: Tachycardia, -metered dose: 90mcg two puffs q4-6h
MOA: interacts with B-2 receptors (peripheral) allowing for dilation of the smooth arrhythmias, tremors, low K+ -dry powder inhaler: 200mcg, 1 inhalation
muscles in the bronchioles prn; max 4/day

ICS Pulmicort, Flovent, Azmacort, Qvar, Beclomethasone


MOA: reduces inflammation; SE: thrush

LABA Salmeterol, Formoterol, Arformoterol, Indacaterol, Olodaterol BBW: asthma-related death -dry powder: one inhalation BID
as a monotherapy (w/o ICS)

LABA-ICS -Budesonide/Formoterol (Sumbicort) -Fluticasone/Salmeterol (Advair)


-Momentasone/Formeterol (Dulera)

Mast Cell Cromolyn, Nedocromil SE: throat irritation


MOA: Inhibits mass cell and leukotriene-mediated degranulation

Leukotriene Montelukast (Singulair), Zafrilukast (Accolate), Zileuton (Zyflo) SE: increased LFTs, HA, GI
Modifiers MOA: Blocks leukotriene-mediated neutrophil migration, capillary permeability, mylagias
smooth muscle contraction

Theophylline MOA: bronchodilator that improves respiratory muscle endurance, SE: nervous, tachycardia,
*narrow TI phosphodiesterance inhibitor which inhibits leukotreiene syntheseis and inflammation N/V, anorexia, HA

PULM
Acute •Inflammation of the bronchi •non-productive cough CLINICAL *Reassurance and education
Bronchitis •MC in the winter *HALLMARK •rest and hydration, NSAIDs
•Wheezing CXR: usually normal
MCC: Viral (Adenovirus, parainfluenza, •Rhonchi (clear with cough) AVOID ABX unless indicated!
influenza, coronavirus, coxsackie, •Pharyngitis •Atypicals: Macrolide, FQs
rhinovirus, RSV) •Fatigue/Malaise
Bacterial: S. pneumo, M. cat, H. flu, •Fever (low-grade)
•hemoptysis
Pathophys: infection  inflamed  *acute bronchitis and
exudate  spasm bronchiectasis are the MCC of
hemoptysis
Two phases:
1. direct inoculation of tracheobronchial
epithelium
2. Hypersensitivity of airway receptors
(persistent 1-3wks)  increased sputum

PULM
Pneumonia •Influenza •Fever, chills, HA, sore throat •NP swab O2, rest, fluids, antipyretics,
(Viral) •adenovirus, parainfluenza, RSV •nonproductive cough •CXR analgesics, IVF
•rhinorrhea, myalgia

Pneumonia •Unicellular, Fungal pneumonia •Fever Labs: Elevated LDH & B-D-glucan •BACTRIM x 21 days
(HIV/Fungal) •dry nonproductive cough •add Prednisone if hypoxic (PaO2
P. jivoreci Transmission: inhalation airborne spread •dyspnea on exertion CXR: diffuse, bilat interstitial infiltrates <70)
•respiratory failure CT scan: ground glass opacitis
Risks: immunocompromised, HIV/AIDS G6PD Deficiency:
(CD4 ≤ 200) Exam: Lavage with fluorescent antibody staining •Atovaquone
•O2 desaturation with ambulation •trophic: wright-giemsa stain •AVOID Dapsome or Primaquine
•cyst: methenamine silver and toluidine blue
stains

Biopsy: DEFINITIVE

Pneumococcal Vaccines
 Pneumovax (PCV) 13: Indications: Administer in patients 6 weeks old – 5 years old; 4 dose series: 2, 4, 6, 12-15 months
 Pneumococcal (PPSV) 23: Indications: ALL adults 65 years and older, younger patients with increased risk of pneumococcal disease
o If given prior to 65 years old, then should be revaccinated at 65 year old unless it was given within 10 years then revaccinate 10 years after vaccines
 No prior vaccines: PCV 13  wait 8 weeks  PPSV 23 If received PPSV 23 in past: PCV 13 on year after PPSY vaccine

PULM
Pneumonia CAP: Typical CXR (PA & Lateral): GOLD CAP Out:
(Bacterial) within 48 hours of hospital or outpatient •productive cough •infiltrates No comorbid/ABX:
-S. pneumo, H. flu, M. pneumo •fever •HAP: new infiltrate •Azithromycin or Clarithromycin
•dyspnea •Doxycycline
HAP/VAP/HCAP: •pleuritic CP CT: if unclear
develops 48hours AFTER •chills (rigors)
Comborbid/ABX:
-Pseudomonas, Klebsillea, S. aureus •tachypnea Labs:
•accessory muscles •hospital: cx, NP, CBC, CMP •Levofloxacin
•Azithroycin + Rocephin
Ventilator: Pseudomonas, Acinetobacter
Typical Exam Admission Criteria:
•Dullness to percussion PSI: >90=hospital CAP Inpatient:
Aspiration: anaerobes *MC R lower lobe •Increased TF Non-ICU:
• (+) egophony CURB-65 (2+=hospital): •Levofloxacin or Moxifloxacin
S. pneumo *gram + diplocci •tachypnea, tachycardia •confusion •Rocephin + Azithromycin
•commonly see chills and rigors •urea (BUN) >19 (>7mmol)
•MCC of CAP Aspiration PNA •Respiratory rate 30+ ICU:
•foul smelling sputum (rotten egg) •BP low <90/60 •Rocephin + Azithromycin
H. Flu *gram – rods •pulmonary abscess or empyema •age >65 •Rocephin + Levofloxacin
•extremes of ages
•imunocompromised, pulmonary disease HAP (low risk drug-resistant):
Sputum Colors
•Levofloxacin, Zosyn, Cefepime
rust: S. pneumo (MC)  RLL
Klebsiella *gram - rods
green: H, flu, pseudomonas
•alcoholics HAP (high): Vancomycin +
red currant jelly:
•cavitary lesions on CXR 1. Cefepime or Ceftazadime
Klebsiella  upper lung
2. Imipenem or Carbapenem
Foul smell/bad taste: anaerobic
S. aureus *gram + cocci in clusters 3. Zosyn + Ciprofloxacin
•after viral infection, HAP 4. Gent or Amikacin
•bilateral multilobar infiltrates on CXR
Aspiration:
•ampicillin-sulbactam or Augmentin

Atypical Bugs: Atypical: CXR: reticulonodular pattern Antibiotics


Pneumonia Mycoplasma (MCC): •non-productive cough •diffuse, patchy infiltrates •Macrolides (Azithromycin)
•young and healthy (schools, college, •low-grade fever •Doxycycline
military) •extrapulmonary symptoms PCR: TEST OF CHOICE
•late summer, early fall •pharyngitis •cold agglutinins Resistant to B-lactams due to LACKING A
•URI prodrome CELL WALL
Other bugs: Complications:
•Chlamydia pneumonia Exam: usually normal •SJS, TEN
•Legionella pneumophila •cold autoimmune hemolytic
anemia (IgM)
•bullous myringitis

Legionella •Anaerobic, gram (-) bacteria •fever, chills Labs: Antibiotics


Pneumophilia Transmission: contaminated water sources •dyspnea •hyponatremia •Macrolides (Azithromycin)
•GI: diarrhea (watery, non-bloody), •increased LFTs •Respiratory Fluoroquinalones (Levofloxacin,
Risks: immunosuppressed, elderly, nausea, vomiting Moxifloxacin, Gemifloxacin)
smokers, lung disease •Neuro: HA, AMS, confusion DX: PCR, urine antigen

PULM
Tuberculosis Mycobacterium tuberculosis (Mtb) Pulmonary: productive cough, CP, Sputum acid-fast stain RIPE x2 months (6 months total)
(Atypical) hemoptysis •x3 on 3 consecutive days •Rifampin: orange fluids, low platelets
Transmission: airborne droplets •Izoniazid: hepatitis, neuropathy
Constitutional: night sweats, fever, Quanterferon Gold: blood test, *prevent neuropathy w/ Pyridoxine (B6)
Pathophysiology: chills, fatigue, anorexia, weight loss not effected by BCG vaccine •Pyrazinamide: hepatitis, hyperuricemia
Inhalation  alveoli  incorporated into *caution with gout and liver
macrophages  disseminates Extra-pulmonary: Mantoux TB skin test •Ethambutol: optic neuritis (E=eye),
•vertebral (Potts Disease) (+) 5mm+ peripheral neuropathy
Risks: close contact, immigrants, crowded •cervical lymph nodes (scrofula) •HIV/immunosuppressed *substitution is Streptomycin
conditions, healthcare, immunosupressed •pericarditis •(+) CXR (aminoglycoside)
•adrenal gland and GU •close contact
Types of TB (+) 10mm+ then Rifampin and Isoniazid for 4 months
Primary TB: *CONTAGIOUS Latent TB Criteria: •high risk populations: IVDU,
•initial infection, contagious NEED ALL 3 homeless, age <5yo, immigrant,
•(+) PPD *2-4wk after infection recent travel Latent:
Chronic (latent): *NOT CONTAGIOUS •no symptoms (+) 15mm+ everyone •Isoniazid + Pyridoxine (B6)
•caseating granuloma formation (central •no imaging findings •Rifampin x 4 months
necrosis, acidic, low O2  hard for Mtb) CXR: apical cavities *not contagious in latent period
-reactivation: apical, upper lobes *no sx, (+) PPD, (-) CXR
Secondary (reactivate): *CONTAGIOUS -primary: middle/lower lobe
•reactivation of latent TB -miliary: millet seeds
•apex of upper lobes *increased O2

PULM About Presentation Diagnostics Treatment


Hypoventilatio •involuntary cessation of breathing during sleep •snoring, interrupted sleep Polysomnography (sleep study) •weight loss
n Syndrome Risks: obesity, age, males •daytime sleepiness, fatigue, -15 or more events/hour •avoid alcohol and sedatives
(sleep apnea) •wake up with breath holding, •sleep position
Types: gasping, choking •Alternative: home sleep apnea
Central: reduced CNS respiratory drive •HTN, CAD, DM, mood testing (HSAT) Mild/moderate: CPAP
Obstructive: physical airway obstruction
Exam: Labs: polycythemia Severe: CPAP, surgery
1. pharyngeal wall collapse repetitively •large neck circumference
2. failure of upper airway dilator muscle •crowded oropharynx
3. sleep-related obstruction and breath cessation •micrognathia

PULM About Clinical Presentation Diagnostics Treatment


Carcinoid •Neuroendocrine tumor •Asymptomatic (25-40%), wheezing, cough, Bronchoscope: pink-purple tumor •Ocreotide may reduce SX
Tumor •slow growth, low mets recurrent PNA, hemoptysis, SIADH, Cushings *well vascularized, central
Definitive management: surgery
MC sites: GI tract, lungs Carcinoid syndrome: diarrhea due to serotonin •Secrete serotonin, ACTH, ADH,
•Increased bradykinin and histamine  flushing, melanocyte stimulating hormone MC complication: bleeding
tachycardia, bronchoconstriction, acidosis •Biopsy is DEFINITIVE

Solitary Nodule: <3cm Increased Risk of Malignancy *large >2cm CXR: initial test Low (<5%): watch
Pulmonary Mass: >3cm •spiculated, irregular border CT Chest: IMAGE OF CHOICE
Nodule •asymmetric calcification • >40yo, smoker Intermediate (5-60%): biopsy
Causes: infectious Screening: annual low dose CT •bronchoscopy if central
granuloma (mycobacteria Decreased Risk: *small <1cm 55-80 years old with no sx AND •transthoracic needle aspiration if
and fungi) •well circumcised, smooth borders 30 year pack history who currently smoke peripheral
•dense diffuse calcifications “popcorn” or quite within 15 years
Risks: Thymomas (MC) • <30yo, non-smoker *stop after 15 years of quitting High (>60%): resect with biopsy

Bronchogenic MCC cause of cancer SCLC: fast growing, starts in bronchi (central) Superior Vena Cava (SVC) Syndrome: NSCLC: Surgical Resection
Carcinoma related deaths in US •paraneoplastic syndromes: SVC syndrome, •partial of complete obstruction of flow treatment of choice
SIADH (hyponatremia), Cushing, Lambert-Eaton •SX: face or neck swelling, facial
Risks: cigarettes, asbestosis •lambert-eaton (weakness improves WITH use) plethora, headache, dilated neck veins SCLC: Chemo treatment of choice
•histology: sheets of small dark blue cells with •CXR: right hilar mass, CT: better
Two Types: rosette formation SVC Syndrome: supportive
•NSCLC: adenocarcinoma,
large cell, squamous cell Lambert-Eaton Syndrome:
*MC type is Adenocarcinoma (MC!!): slow, peripheral •antibodies against presynaptic voltage Lambert-Eaton:
adenocarcinoma •lepidic pattern: rare, low-grade subtype with gated Ca+ channels prevent Ach release Pyridostigmine
voluminous sputum and interstitial lung pattern on •SX: proximal muscle weakness, treat underlying
•SCLC CXR improved with repetitive use, ED, dry
•histology: gland formation, mucin production mouth, orthostasis, Pancoast Tumor:
•DX: voltage gated Ca antibody assay •chemo  resection
Squamous: central (bronchial)
•CCCP: central, cavitation, hypercalcemia, Superior Sulcus (Pancoast) Tumor:
pancoast-shoulder pain, horners, hand atrophy •tumor in superior sulcus (apex)
•histology: keratinization, desmosome compression lower brachial plexus
•SX: shoulder/arm pain,
Large: rapid, periphery, aggressive weakness/atrophy of hand, ulnar
neuropathy, horners (ipsilateral ptosis,
miosis, anhidrosis)
•DX: CXR  MRI  biopsy

Mesothelioma Tumor in the pleura (MC), Pleural: CXR: unilateral pleural thickening, Chemo and resection
peritoneum, tunica •pleuritic CP, dyspnea bloody pleural effusions
vaginalis, or pericardium •fever, night sweats, weight loss Non-surgical: chemo or radiation
•hemoptysis Pleural Biopsy: closed or VATS

GI 11%
GI Types Five Groups Associated Sx Invasive v. Noninvasive Treatment
Acute *more than 90% 1) Travelers diarrhea (Giardia) N/V Noninvasive 1. Fluid Repletion: oral* or IV
Diarrhea diarrhea due to •S. aureus, B. cerus *small bowel •sports drinks, broth, IV saline
infectious agent 2) Consumers of certain foods *begins 1-6h after *enterotoxins, no cell •fluids with glucose, Na, K, Cl
Acute •Chicken-salmonela, campylobacter, shigella •Noro & rotavirus destruction or invasion
<4wks Most acquired by: •Undercooked hamburger-E. coli *begins 24-48h •watery, volume diarrhea 2. Diet-BRAT
Fecal-oral transmission •Fried rice or reheated-Bacillus cereus after •nausea/vomiting •bowel rest (no fiber, milk, alc, caff)
Persistent Ingestion of food/water •Mayo or creams-Staph or Salmonella •no blood, WBC, mucus
2-4wks Disturbances of flora by •Eggs-Salmonella Volume depletion 3. Anti-Motility Agents
antibiotics  C. diff •Undercooked food or soft cheese-Listeria •Non-inflammatory Invasive: DO NOT GIVE •Indications: patients <65 with
•Vibrio cholera ANTIMOTILITY DRUG mod/severe volume depletion
When to admit: 3) Immunodeficient *large bowel •CI: INVASIVE diarrhea (bloody,
Severe dehydration Immunocomp. *cytokines cause mucosal high fever, systemic toxicity)
organ failure 4) Daycare-shigella, giardia, rotavirus, hep •CMV invasion and damage
altered mental status •Protozoans •vomiting 4. Anti-Emetics: Ondansetron,
hemolytic-uremic syn. 5) institutionalized •fever Promethazine, Metolopramide
(AKI, *MC nosocomial infection •fecal blood, WBC, mucus
thrombocytopenia, ABX-Cipro, Levo; Doxy, Bactrim
hemolytic anemia)

GI About Associated Sx Diagnostics Treatment


Giardia Lamblia •protozoan parasite associated with sporadic or •frothy, greasy, foul-smell diarrhea •antigen assay •rehydration
*Protozoan epidemic diarrheal illness *due to steatorrhea
•no blood or pus Stool: ABX: Metronidazole, Tinidazole,
Sources: contaminated water from streams/wells •abdominal cramps, bloating •trophozoites, cysts Albendazole, Quinacrine
(Beaver’s fever, backpackers diarrhea)

Diarrhea Drugs MOA CI Adverse Effects


Loperamide (Imodium) •Inhibits peristalsis and gut transit •Bloody or C. diff diarrhea; Pts <2yo •Constipation, abd cramp, dizzy, CNS
Diphenoxylate (Lomotil) •Opioid receptor agonists (mu receptor gut) •Loperamide: Acute dysentery or •Serious: paralytic ileus, toxic megacolon
Schedule V; 1st line colitis  toxic megacolon
Bismuth •Reduces secretions, antimicrobial •Children with viral illness •Black stool, black tongue
Pepto-Bismol, Maalox, Kaopectate *can be used in dysentery (fever, bloody) •Constipation, Tinnitus
Ocreotide (Sandostatin) Inhibits intestinal fluid secretion and stimulates Caution: DM, thyoid, pancreas, kidney, •Cholelthiasis/cholecystitis/biliary tract
intestinal absorption*chronic secretory diarrhea liver, arrhythmia disease
•Edema, Constipation
Antispasmodics/ Anticholinergics •Relaxes intestinal smooth muscle, inhibits •Toxic megacolon •Ileus, delirium, nervous, palpitations,
Hyoscyamine (Levisin) spasms and contraction •Inflammatory bowel disease constipation, xerostomia, mydriasis
Dicyclomine (Bentyl) •anticholingeric  inhibits Ach
Phenobarbital, Atropine, Scopalamine *diarrhea associated with IBS (Bentyl)
*can use for bladder spasms (Levsin)

GI
Acute Causes: •Epigastric pain that radiates into the back •Serum amylase & lipase (BEST) Assessment of Severity:
Pancreatiti •gallstone (MC) *because retroperitoneal •CBC: leukocytosis Ransons “LAAWG”
s •heavy alcohol -sudden, steady •CMP: high glucose, ALP, LFT, 3+ indicates severe course
•medications: thiazides, protease -worse w/ activity & supine TG, bilirubin •LDH >350
inhibitors, Exenatide, valproic acid, -improve w/ leaning forward •hypocalcemia: necrotic fat binds •AST >250
estrogens, didanosine •N/V, fever to calcium lowering the levels •Age >55
•Weakness, sweating, anxiety •WBC >16,000
Pathophysiology: X-ray: •Glucose >200
•Edema/obstruction at ampulla of Vater PE: *can dehydrate quickly •Sentinel loop (localized ileus)
•acinar cell injury  premature or over •Epigastric tenderness •colon cut off (abrupt collapse of Treatment:
activation of enzymes  self-digestion •Distended abdomen colon near the pancreas) **Supportive:
•Absent BS if with ileus •IVF high volume (LR)
•Tachycardia, hypotension CT: diagnostic study of choice •NPO
•Cullens (periumbilical) & Grey Turners •pain (Demerol)
(flank) ecchymosis Criteria: 2/3
•acute persistent, severe pain ABX: broad spectrum if severe
•elevated of lipase or amylase 3x *not routine
•characteristic imaging finsings

Chronic Chronic inflammation causing •Chronic, steady/intermittent epigastric pain Labs: amylase and lipase normal Oral pancreatic enzyme
Pancreatiti parenchymal destruction  loss of *often radiates to back replacements
s exocrine function, sometimes endocrine •Anorexia, weight loss, N/V, constipation X-ray: calcifications in pancreas
Endoscope US: honeycomb •alcohol abstinence
Causes: Pancreatic insufficiency •low fat diet
•Alcoholism –MC! exocrine: malabsorption & steatorrhea CT (best): pancreatic enlargement, •vitamin supplementation
•Idiopathic endocrine: glucose high or low pseduocysts, calcifications
•hypocalcemia, HLD
•islet cell tumors TRIAD: calcifications, steatorrhea, DM Pancreatic Function: fecal elastase
•familial, traumatic, iatrogenic *MOST SENSITIVE & SPECIFIC

Pancreatic Cystic collection of tissue, fluid, and •Abdominal pain •Study of choice: CT Scan Persists >4-6 weeks
Pseudocyst necrotic debris surrounding the pancreas •Abdominal mass •Ultrasound -percutaneous drainage
-surgical decompression
Associated with: *NO true epithelial lining in the capsule Complications: (pancreaticogastrostomy)
•acute or chronic pancreatitis •Peritonitis, infection -drain into stomach or bowel
•trauma to chest

GI Etiology/Pathophysiology Clinical Presentation Labs/Diagnostics Treatment


Carcinoma •75% adenocarcinoma (ductal) •painless jaundice *classic Labs: Surgical resection if localized disease:
of Pancreas •70% in pancreatic head •weight loss, anorexia, fatigue •Amylase/lipase nml to high •Whipple Procedure *head or duodenal
*poor prognosis with body and •Vague epigastric and back pain •Glycosuria, Hyperglycemia *radical pancreaticoduodenal resection
tail •pruritis (bile salts) •Elevated LFTs if CBD obstruction -precede w/ biliary stenting if CBD block
-often followed with chemotherapy
Risks: Exam: Tumor marker: CA 19-9 *staging laparoscopy performed prior
•Advanced age (>55) •Courvoiser’s sign: enlarged, *metastasis in nodes-will not do
•tobacco use palpable gallbladder, nontender Diagnostics:
•heavy alcohol use •1st line-CT w/ FNA biopsy Distal Pancreatectomy
•obese, chronic pancreatitis, DM •Endoscopic U/S, ERCP •Resection of tumor in body and tail
•fhx, males Non-resectable: ERCP w/ biliary stent if
Staging: TNM symptoms of obstruction, chemo, palliative

GI About/Causes Clinical Presentation Diagnostics Treatment


Alcoholic Liver •Excessive alcohol intake •Fatty liver: •Fatty liver: *usually incidental Abstinence from Alcohol-most important!
Disease •Men >2drink/day *>5% fatty deposits -mild AST/ALT elevations -especially if fatty liver or alcoholic hepatitis
•Women >1drink/day -aymptomatic, reversible -anemia •nutritional support
*risk increases if consumed -hepatomegaly •folic acid, zinc, thiamine supplementation
daily for 10yrs •Alcoholic Hepatitis:
*risk most in men •Alcoholic Hepatitis: -Greater AST/ALT elevation Pharmacotherapy:
-symptomatic, reversible -AST not > 300 •Methylprednisolone x1 months
-fatigue, nausea, anorexia, -AST >ALT is greater 2:1 *alcoholic hepatitis
Risk of cirrhosis increases if recurrent infections -ALP elevations *see improvement in bilirubin in ~7days
other associated liver -bouts of jaundice -Elevated total bilirubin
disease present or obesity -Leukopenia, thrombocytopenia •Pentoxifylline 400mg TIX x1 month
•Cirrhosis: *severe alcoholic hepatitis, especially if with
-symptomatic, NOT reversible Alcoholic Cirrhosis: hepatorenal syndrome or hepatic
-end stage liver failure -Elevated total bilirubin >10 encephalopathy (elevated ammonia)
-ascites, abd pain, -Hypoalbuminemia
-splenomegaly, fever -Prolonged PT/PTT Prognosis:
-Anemia •Fatty liver: good IF stop drinking
-Last disease -> electrolytes, •Alcoholic Hepatitis:
elevated BUN/CR -unfavorable if: prolonged PT/PTT
-bilirubin >10, hepatic encephalopathy,
Imaging: CT scan azotemia (renal)
Definitive Dx: liver biopsy Cirrhosis: poor prognosis

Non-Alcoholic NAFL: benign, no fibrosis •Asymptomatic or mild, vague •Mildly elevated AST/ALT •Lifestyle modifications: weight loss, exercise,
Steatohepatitis NASH: inflammation and symptoms -AST/ALT ratio <1 dietary fat restriction, glucose control
(NASH) fibrosis *cirrhosis potential •RUQ discomfort *differentiate from alcoholic
•Hepatomegaly steatohepatitis •Hepatic vaccines
Causes: •Signs of chronic liver disease
•Obesity, DM, metabolic uncommon •CT scan (do this 1st!) •Medications:
•TGs, HLD •Definitive DX: liver biopsy vitamin E, thiazolidinediones (Actos, Avandia) and
•Medications-steroids microvesicular fatty deposits Metformin, Pentoxifylline, wt loss agents
•Cushings, PCOS, Genetics

GI About/Causes Clinical Presentation Diagnostics Treatment/Prevention


Hepatitis Acute viral infection of liver •Most are asymptomatic •Hep A Antibody Treatment: self-resolving
A •Spiking fever -Anti-HAV IgM peak in first week *active •Symptomatic  rest & fluids
Incubation: ~30 days •Malaise, N/V, anorexia, abd pain -IgG rises for months  years.
*seen in feces 2wks prior •Aversion to smoking (no craving) -past exposure: (+) IgG, (-) IgM Post-Exposure Prophylaxis: *2 wk
•healthy 1-40years old: vaccine
Route: Fecal-oral route Exam: Labs: High ALT, AST, bilirubin •healthy >40yo: vaccine +/- Ig
•Crowding, poor sanitation •RUQ abdominal pain, jaundice •immunocompromised or liver:
•Contaminated food •Mild hepatomegaly, liver tenderness Blood smear: Atypical large lymphocytes vaccine + immunoglobulin (Ig)
•International Travel
Acute •Double stranded DNA •Asymptomatic Window: (+) anti-HBc IgM •Recovery ~3-6 months
Hepatitis •Inner core protein and outer •Distaste for smoking Acute: (+) HBsAg, (+) anti-HBc IgM •Rest, increase fluids, nutrition
B surface coat Chronic: (+) HBsAg, (+) anti-HBc IgG •Avoid activity, alcohol, hepatotoxic
Sub-clinical: constitutional (malaise, Immunization: (+) anti-HBs, rest negative
Incubation: 12-14 weeks myalgia, fatigue, URI, N/V, abd pain) Recovery: •Hep B Immunoglobulin *within 7d
(+) anti-HBs, (-) HBsAG, (+) anti-Hbc IgG
Transmission: percutaneous, Icteric: jaundice (30%) Persistant/Severe Symptoms:
sex, parenternal, perinatal •HBsAG (surface antigen)-infective! *1st marker •Entecavir, Tenofovir
Fulminant: acute hepatic failure (+) means acute or chronic
Causes: (encephalopathy, jaundice, coag) •Anti-HBs (surface antibody)-recovery or vaccine Cleared Infection:
•Bloodborne, Sexual (+): vaccine (rest -) or recovery (will have + IgG) •Two consecutive tests 4wks apart
•Mother  baby Exam: •Anti-HBC (antibody to core antigen)-IgM or IgG
•Mild RUQ pain and hepatomegaly •BEeAG (envelope antigen)-rapid replication Vaccine: *CI if bakers yeast allergy
Risk populations: •HBV DNA: best way to test viral replication Infant: birth, 1-2mo, 6-18mo
•IVDU, inmates, healthcare •Elevated LFTs, bilirubin, ALP, PT Adult: 3 doses 0, 1, 6 months

Hepatitis Incubation: 6 weeks •no sx  Fatigue, nausea, RUQ pain HCV antibodies (Anti-HCV)=INFECTION! Treatment: *all equal efficacy
C *signifies HCV as causes of hepatitis •Ledipasvir-Sofosbuvir
Transmission: Exam: •Direct Acting Antiviral (DAA)
•Parenternal (blood), sex •Jaundice wax and wane •HCV RNA: indicates current infection •Interferon
-MCC: IVDU •dark urine, clay colored stools -CONFIRMATORY (most sensitive)
Response to therapy determined by
Risk populations: IV drugs, Complications:Hep C, Non-Hodgkins, •ALT/AST rise and decline PCR-RNA load at 12-24 weeks
inmates, healthcare workers DM, glomerulonephritis, pulmonary
fibrosis, thyroiditis

Hepatitis Pathophysiology: •asymptomatic •anti-HDV •no FDA approved treatments


D •Hep D uses BHsAg as its •fatigue, myalgia, malaise •Interferon alpha
envelope protein •RUQ pain, jaundice Confirmation: immunochemical staining of liver
Hep B co- •dark urine, clay colored stools biopsies for HDAg or RT-PCR assay of HDV RNA Prevention: Hep B vaccination
infection Transmission: blood

Hepatitis •RNA virus, Waterborne •malaise, anorexia, N/V, abd pain Labs: elevated LFTs and bilirubin •Self-limited
E •Asia, M. East, N. Africa
Extrahepatic: “PANT” •IgM anti-HEV in serum •Oral Ribavirin x3 months
Transmission: fecal-oral Pancreatitis, Arthritis, Neuro (Guillan- •Recombinant vaccines
•blood, mother to child Barre), Thrombrocytopenia *highest mortality due to fulminant
hepatitis in pregnancy
Spread: swine, pet, orangs Exam: jaundice, hepatomegaly
GI About/Causes Clinical Presentation Diagnostics Treatment
Autoimmune Idiopathic chronic inflammation of •Insidious onset  sudden •AST/ ALT >3x nml (ALT >1000) Prednisone
Hepatitis liver due to circulating antibodies •Vague abdominal pain, nasuea •Elevated total bilirubin -improves sx and inflammation
•Spider Nevi & Hepatomegaly •Positive ANA -Adding Azathioprine lowers steroid dose
•MC in young women •pANCA, anti-SLA (antibodies)
•Usually follows virus Extrahepatic: arthritis, failed therapy  liver transplant
thyroiditis, associated Definitive dx: LIVER BIOPSY *can progress to cirrhosis
autoimmune conditions
Complications: cirrhosis, pericarditis,
myocarditis, glomerulonephritis, uveitis

Drug/Toxin Causes: “Herbals” •Elevated ALT >AST •Can progress to liver failure
Induced Acetaminophen *alcohol •Hepatomegaly
Hepatitis Isoniazid *Rifampin •Jaundice •Stop offending agent
ABX-Tetracyclines •Fatigue, N/V •May require transplant if liver failure
develops
Fulminant Acute hepatic failure in patients •Encephalopathy: vomiting, •combination of symptoms Supportive:
Hepatitis with hepatitis coma, AMS, asterixis (flapping •IVF
tremor of hand) Labs: •electrolyte repletion
Causes: •hypoglycemia •Mannical (ICP elevation)
•Acetaminophen toxicity •Coagulopathy: increased •increased ammonia •blood products or cryo
•viral hepatitis, autoimmine PT/INR/PTT
•Reye syndrome (NSAIDS) Definitive: liver transplant
•GI: hepatomegaly, jaundice

•Reye: rash, vomiting, liver


damage, encephalopathy, dilated
pupils, multi-organ failure

GI About/Causes Clinical Presentation Diagnostics Treatment


Cirrhosis Irreversible liver fibrosis with S/S: Insidious onset •AST/ALT & ALP elevated •No cure w/o liver transplant
“liver failure” nodular regeneration  replaces •Weakness, fatigue, sleep changes •Bilirubin elevated •Treat complications!
functional hepatocytes w/ nodules •N/V, abdominal pain, ascites •Hypoalbuminemia
•Anemia •Avoid alcohol & liver toxic medications
Pathophysiology: •Portal HTN: esophageal varices, •WBC fluctuations •Strict dietary management: protein, low carb and
•nodules cause increased portal splenomegaly •Thrombocytopenia sodium; reduce protein if encephalopathy
pressure •Skin: spider angioma, palmar •Prolonged PT/PTT •Nutritional supplements
erythema, telangiectasias, caput •Vaccinations: HBV, HAV, flu, pneumococcal
Causes: medusa Diagnostics:
•Alcoholic hepatitis •Menstrual abn, ED, gynecomastia •US: assess size and ascites Pruritis: Cholestyramine
•Chronic hepatitis C (MCC) •Vitamin & nutrient deficiencies: •CT: assess liver nodules Encephalopathy: Lactulose, Rifamxin
•NASH anemia, glossitis, cheilosis •Biopsy: definitive DX Ascites: Spironolocatone, Furosemide
•Drug toxicity •EGD: varices *cirrhosis
•Autoimmune hepatitis Encephalopathy: confusion, Staging: Child-Pugh Classification-total bilirubin,
•Hemochromatosis lethargy, asterixis albumin, PT INR, ascites, encephalopathy
•Genetic, metabolic *elevated ammonia levels Class A: 5-6 points
Class B: 7-9 points
Class C: 10+ points
Cirrhosis Complications
Ascites/Edema •Sodium restriction •Transjugular Intrahepatic Portosystemic Shunt (TIPS)
•Diuretics (Spironolocatone, Furosemide) -primary indication for variceal bleeding
•Paracentesis -shunts blood out of portal system -> reduces portal HTN ->
-if unresponsive to diuretics, can’t tolerate diuretics, resp sx decreased occurrence of ascites
-can use with albumin to move fluid
Problems: bleeding, shunt occlusion, hepatic encephalopathy

Spontaneous Bacterial Peritonitis •Infection of asitic fluid without perforation of the bowel •TX: Cefotaxmine or Ceftriaxone
*MCC- E. Coli •SX: fever, chills, abdominal pain, diarrhea, fluid wave
•DX: paracentesis and culture (cell count 250 +) •Prophylaxis: Bactrim of oral fluoroquinolone

Hepatorenal Syndrome •Azotemia in the absence of intrinsic renal disease Labs: increase BUN/Cr, hyponatremia, oliguria

Occurs in 10% of patients with •Two types: •TX:


advanced cirrhosis and ascites -type I: sudden doubling of Cr >2.5 Stop diuretics & Increase blood flow to kidneys
-type II: slowly progressive IV albumin, Dialysis; TIPS, Liver transplant

Hepatic Encephalopathy •Stages: mild confusion -> drowsiness, stupor, coma TX:
•Causes: CNS drugs, TIPS •reduce protein intake
•lactulose (breaks down ammonia and neutralizes it)
•DX: clinical (elevated ammonia w/ sx), no imaging •ABX (Rifaximin, Metronidazole) lower ammonia producing bacteria

Anemia and Coagulopathy •Iron deficiency: ferrous sulfate •Transfusion if due to severe blood loss from bleeding varices
•Vitamin K for severe coagulopathies w/ FFP
•Folate deficiency: folic acid *most common in alcoholics •Oral thrombocytopoetin (maybe for thrombocytopenia)

Esophageal Varices •DX: EGD *all cirrhosis patients should undergo a EGD •Procedures: banding, sclerotherapy, balloon tamponade
50% patients with cirrhosis *balloon tamponade has complications, only do if others don’t work
•TX: IV fluids, PRBCs, FFP, Ocretotide, EGD when stable, ABX
•Prevention: band ligationg AND BB (Propranolol), TIPS, transplant

GI
Irritable •Idiopathic, with NO organic cause •abdominal pain with altered •Diagnosis of exclusion 1st line: lifestyle and diet changes
Bowel defecation/bowel habits •low fat, high fiber
Syndrome Onset: late teens, early 20s *MC in women •pain relieved with defecation Rome IV Criteria: •unprocessed food diet
(IBS) Recurrent crampy, abdominal •avoid sorbitol or fructose
Pathophysiology: ALARM SX: pain 1d/wk for 3 MONTHS •avoid gas producing (beans, veggies)
•abnormal motility: chemical imbalance in •GI bleeding, anemia AND 2/3: •sleep, smoking cessation, exercise
intestine (serotonin & Ach) •anorexia, weight loss -relieved with defecation!
•visceral hypersensitivity •fever -Change in stool frequency Diarrhea:
•psychosocial and altered CNS •nocturnal sx -change in stool form •Loperamide
•family history of GI cancer •Dicyclomine (anticholinergic)
•IBD or celiac •Eluxadolin, Rifamixin
•diarrhea  dehydration
•severe constipation or impaction Constipation:
•prokinetics: fiber, psyllium
•polyethylene glycol can be added
GI
Acute Inflammation/irritation of mucosa •MC asymptomatic H. pylori: urea breath test, stool H. pylori: triple therapy
Gastritis •Epigastric pain
Gastropathy: mucosal injury without •N/V EGD with biopsy NSAID:
evidence of inflammation •anorexia *MOST ACCURATE •PPI (Omeprazole)
•biopsy CONFIRMS MOA:block H+/K+ ATP-ase of
Erosive: NSAIDs, alcohol, stress parietal cells, reducing acids
Non-erosive: H. pylori (MCC), systemic SE: B12 deficiency
•Misoprostol, Sucralfate

Peptic Ulcer MCC OF UGI BLEED •Epigastric Pain (dyspepsia) worse at night Upper Endoscopy with Biopsy: 1. Triple Therapy x10-14d
Disease *HALMARK SIGN GOLD STANDARD CAP: Clarithromycin,
Two Types: •Dull, aching, “hunger” Amoxicillin, PPI
•Duodenal (MC)-H. Pylori •Food relief (duodenum) *returns in 2-4h H. Pylori tests:
•Gastric-NSAIDS •Food worsens (gastric) •endoscopy with biopsy (GOLD) 2. Quadruple Therapy x10-14d
•urea breath test; stool (HpSA) PPI + Bismuth (pepto) +
Pathophysiology: increased aggressive, •Nausea, anorexia •antibodies Tetracycline + Metronidazole
decreased protective *erosion >0.5cm •GI bleed (melena, coffee ground emesis)
*may be asymp until bleed Complications: H.Pylori Negative:
Causes: Bleeding, perforation, •PPI: Omeprazole
•H. pylori infection *MC in duodenum Physical Exam: tenderness over epigastric obstruction, malignancy •H2 Blocker: Ranitidine
•NSAIDS *MC in stomach MOA: reducing acid/pepsin
•Zollinger-Ellison syndrome SE: B12 deficiency, high LFTs
•ETOH, smoking, stress, cancer, male, old
Refractory: parietal vagotomy
Gastric Adenocarcinoma MCC worldwide Asymptomatic until advanced Iron deficiency anemia Early local: edoscopic resection
Adeno- Other: lymphoma (4%) •Dyspepsia, vague epigastric pain
carcinoma •weight loss, persistent abdominal pain Diagnostic: EGD & biopsy Surgical resection: gastrectomy
Risks: *risk reduced with chronic NSAID •Anorexia, wt loss, early satiety
•H. pylori •Chemotherapy and/or radiation
•high salt or nitrate foods PE: *palpable mass (20%) •Palliative measures
•low fruits and vegetable diets •left supraclavicular lymph (virchows)
•Non-Hodgkin lymphoma (MC site of •umbilical node (sister mary joseph)
extranodal NHL) •rigid rectal shift (Blumer shelf)
•ovarian mets (Krukenburg tumor)
•L axillary node (Irish sign)
GI
GERD •Reflux of gastric contents into the •Heartburn (pyrosis) HALLMARK •clinical diagonsis 1st: lifestyle modifications
esophagus due to incompetent lower ~30-60min after meals •24hr pH monitoring •elevate head
esophageal sphincter •Reflux of sour/bitter contents (regurg) GOLD TO CONFIRM •avoid lying down x3hr
•Dysphagia (1/3) •avoid acidic food and alcohol
•Transient relaxation of lower esophageal •Asthma, cough, laryngitis, chest pain Upper endoscopy: 1st line if •smoking cessation
sphincter (LES) persistent or complication •weight loss
ALARM: dysphagia, odynophagia,
weight loss, bleeding Complications: < 2 episodes/ week
•Esophagitis, Stricture •H2 receptor agonists (Ranitidine) prn
•Adenocarcinoma
•Barrett’s Esopagus  2+ episodes per week: PPI (Omeprazol)
squamous to columnar

Barretts Esophageal squamous epithelium Upper endoscopy with Barrets, no dysplasia: PPIs, rescope q3-5yrs
Esophagus replaced by metastatic columnar cells biopsy Low-grade: PPI, rescope 6-12 months
from the cardia of the stomach High-grade dysplasia: Ablation

Esophageal Squamous cell: •Progressive dysphagia: solid to fluids Upper endoscopy w/ biopsy: Curable: surgery and chemo
Cancer •MC type worldwide •Odynophagia STUDY OF CHOICE
•MC in mid-upper 1/3 esophagus •weigt loss, anorexia Incurable: chemo, palliative
•risk: smoking, alcohol •Tracheo-esophageal fistula Early lesions:
•people: African Americans •horners syndrome •superficial plaques
•Chest or back pain (mediastinal •nodules, ulcerations
Adenocarcinoma: estension)
•MC type in US; MC in distal/junction Late lesions: strictures, ulcer
•risk: GERD, Barretts, smoke, high BMI
•people: Caucasian males, younger Endoscopic US: Staging
Esophagitis Causes: 3 Classics: Upper endoscopy Treat the underlying
•GERD (MCC) •odynophagia (HALLMARK) Candidal: Fluconazole, Voriconazole,
•Infectious: Candida, CMV, HSV •dysphagia Caspofungin
•Eosinophilic: allergic reaction •retrosternal chest pain CMV: Gangciclovir, Valgangciclovir,
•Pill-induced: bisphosphonates, BB, Foscarnet
CCB, NSAIDs Candida: linear yellow-white plaques HSV: Acyclovir, Foscarnet
•Caustic (corrosive): acidic or basic CMV: large superficial shallow ulcers
HSV: small, deep ulcers
Pill- Esophagitis due to prolonged pill contact •odynophagia Endoscopy: small, well- •Take pills with at least 4 ounces of water
Induced •dysphagia defined ulcers •avoid recumbency at least 30-60min after
Esophagitis Meds: bisphosphonates, BB, CCB,
NSAIDs, KCl, iron, vitamin C

Hiatal Protrusion of the abdominal cavity •asymptomatic, incidental findings Chest x-ray •Type I: same at GERD  PPI, weight loss
Hernia through the esophageal hiatus of the
diaphragm Possible: •Type II: surgical repair
•epigastric or substernal pain
•Type I: sliding hernia (MC) •postprandial fullness
•Type II: rolling hernia  fundus •retching of nausea
protrudes, GE junction remain in location
Disease About Clinical Manifestation Physical Exam Diagnosis and Treatment
Colon Polyps •Fleshly lesions protruding into intestinal lumen usually asymptomatic Diagnostics: •remove
(Adenomatous) •Growth arises due to DNA changes in the lining •Colonoscopy with polypectomy that
of the colon *cause 95% adenocarcinomas Adenomatous Polyps: is sent to pathology Screening: 50-75 YEARS OLD
MC NEOPLASTIC Colonoscopy (BEST)
Pseudopolyps: due to inflammatory bowel disease •Tubular adenoma (MC) Risk factor for high grade dysplasia: •10 years if average risk or 1st
(UC, Crohn) *not cancerous -nonpedunculated *high-grade dysplasia/CA degree relative 60+
•Tubulovillous •polyps >1cm •q5 years if 1st degree relative <60
Hyperplastic: low risk for malignancy •Villous •villous histology
*MC non-neoplastic polyp -highest risk of cancer •number of polyps FOBT
-most likely sessile •flat polyps •yearly at age 50 if avg risk
Hamartomatous •yearly at 50 if 1st degree relative
•Juvenile Larger Polyp SX
•Junvenile polyposis syndrome: autosomal •bleeding Flexible Sigmoidoscopy every 5
dominant  increased risk of colorectal and •change in bowel habits years + FOBT every 3 years
gastric cancer •obstruction
•Peutz-Jeghers: STK11 mutation, mucocutaneous •abdominal discomfort Prevention: Diet, weight loss
hyperpigmentation (lipis, oral, hands)

Colon Cancer *3rd MC cancer among men and women •iron deficiency anemia Lynch Screening: Colonoscopy q1-2 Staging and Management: TNM
*2nd leading cause of CA death •rectal bleeding years at age 20-25 system
•Adenomatous (MC) •abdominal pain *tumor, node, metastasis
•change in bowel habits Familial Screening: Sigmoidoscopy
Genetics: yearly at age 10-12 years old Treatment: *good prognosis if
•Familial adenomatous polyposis MCC OF LARGE caught early
•Turcot Syndrome BOWEL OBSTRUCTION Labs: •Surgical resection  post-op
•Lynch Syndrome: autosomal dominant, due to IN ADULTS •iron deficiency anemia chemo
loss of DNA function •CEA tumor marker (not for •Chemotherapy/radiation
-Type I R side, Type II increased risk Proximal (R side) screening-not specific) -stage II-III (node/tissue
of endometrial, ovary, SI, brain, skin •chronic occult bleed involvement)
•Peutz-Jehgers: risk of breast & pancreatic CA •diarrhea Diagnostics: -stage IV (metastatic ds)
•barium enema: apple core lesion
Risk factors: Distal colon •Colonoscopy: DEFINITIVE
•>50yo, African Americans •Obstruction *CT/MRI used for staging
•family history of colon cancer •changes in stool diameter
•diet (red meat, fat, low fiber) •Hematochezia
•smoking, alcohol •Urgency/tenesmus (rectal)
•obesity •Strep bovis endocarditis
•IBD (UC >Crohns)

Protective: activity, NSAIDS

Familial •Inherited mutation (APC gene) Soft tissue tumors in skin Diagnosis confirmed with genetic Prophylactic colectomy
Adenomatous •Development of 100s and adenomatous polyps and osteomas testing recommended!
Polyposis *stomach and duodenum
GI
Inflammatory Ulcerative Colitis and Crohns Extra-Intestinal Manifestations CROHNS DISEASE CROHNS DISEASE
Bowel *Crohns disease or UC Labs: (+) ASCA (anti-saccharomyces *NOT curative!
Disease (IBD) Risks: •MSK pain, arthritis, ankylosing cerevisiae antibodies) •Diet
•Ashkenazi jews, white spondylitis, osteoporosis •Stop smoking
•age 15-35 onset •erthema nodosum, pyoderma Initial Test: upper GI series (barium study) •Symptomatic medications
•males (UC), females (crohns) gangrenosum •String Sign (barium through narrow area)
•genetics •conjunctivitis, anterior uveitis/iritis Limited Ileocolonic:
•smoking (Crohns), may be (ocular pain, HA, blurred vision) Endoscopy: segmental skip areas, 1st line:
protective in UC •fatty liver, primary sclerosing cobblestoning, apthous ulcers, strictures •oral mesalamine (5-ASA)
•western style diet cholangitis •Glucocorticoids
•infections •B12 and Iron deficiency (Crohns!!) GOLD=Colonoscopy w/ biopsy
•meds: NSAID, OCP, hormones  skip lesions with cobblestone Ileal and proximal colon:
appearance, noncaseating granulomas •glucocorticoids
Crohns •Autoimmune inflammatory disease Ileocolitis (MC)
Disease •Crampy RLQ abdominal pain Complications:
•Transmural inflammation and skip •Diarrhea (intermittent, non-bloody) •fistuals, fissures
lesion that affects any part of GI tract •weight loss, fatigue & fever •perianal disease
*MC-terminal ileum and cecum •abscesses
Jejunoileitis:
•malabsorption (iron & B12 deficiency),
•steatorrhea
•nutritional & electrolyte deficiencies
ULCERATIVE COLITIS ULCERATIVE COLITIS
Colitis and perianal: diarrhea, abscess, Labs: high ESR/CRP, anemia, (+) P-ANCA *CURATIVE
fistula, fissure, obstruction
Upper GI Series: “stovepipe” sign due to Mild-Moderate Distal:
extra-intestinal manifestation: loss of hausta •topical 5-ASA, can add oral
•arthralgia, arthritis, iritis, uveitis
•apthous ulcers Gold Standard-COLONOSCOPY Mild-Moderate Pancolitis
•skin: Pyoderma gangrenosum  crypt abscess, basal plasmacytosis, •topical + oral ASA, steroids
erythema nodosum uniform inflammation, ulceration, polyps *same for severe but high
-CI IN ACUTE COLITIS dose

Ulcerative •Relapsing and remitting episodes of •Crampy lower LLQ abdominal pain Fulminant:
Colitis inflammation, spreads proximally •BLOODY diarrhea, pus/mucus •steroids + fluids + ABX
*mucosal layer of colon ONLY, •Fecal urgency and tenesmus
*can mimic rectum is always involved •tenesmus
diverticulitis *smoking decreases risk for UC
Complications:
mild: <4 BMs/day •Severe bleed
moderate: >4 BM/day •fulminant colitis (>10BMs/day)
Severe: 6 BM/d + •toxic megacolon, perf

GI What is it Causes Symptoms Treatment


Constipation •infrequent bowel •motor disorders (CA, DM, hypothyroid) straining See below
movements <2x/wk •drugs (Verapamil, opioids) •hard stool
•Hirschsprung distease •feeling of incomplete evacuation

IBD Pharmacotherapy Indication MOA Side Effects/Adverse Events/ BBW Notes


Aminosalicylates Induction and maintenance Inhibits prostaglandin •N/V, HA, hypersensitivity *must exert effect directly to
(5-ASA) therapy of UC and CD production, producing anti- •CI: sulfa or ASA allergy colon
Sulfasalazine** inflammatory effects
Mesalamine
Corticosteroids Most effective to induce *Can use Budesonide (Entrocort)
(Prednisone remission in severe flare for maint. crohns disease for ~3mo
40-60mg daily) *acute flare -> CD and UC
Immunomodulators/ •Steroid dependent CD and UC Leukopenia, thrompoenia, anemia
Immunosuppressants Remission maintenance in mild- Infection, N/V/D
Azathioprine (Imuran) severe disease Malaise, arthralgia
6-Meracaptopurine Adverse: lymphoma, severe
(Pruinethol) (6-MP)
BBW mutagenic potential, rapid growth, CA

Immunomodulators/ Methotrexate: Cyclosporine: Cyclosprine SE: *must put on folic acid if taking
Immunosuppressants •Mild-moderate active C and •Severe UC/CD refractory to •Multiple serious adverse effects to Methotrexate
Methotrexate maintenance *NOT IN CD steroids
Cycosporine
Antitumor Necrosis TREATMENT OF CHOICE for Inhibits TNF (which •Fever, rigors, N/V, myalgia, urticarial,
Factor Antibodies someone with crohns disease promotes inflammation) hypotension
Infliximab (Remicade) and a fistula
Adalimumab (Humira) •Mod-severe active CD and UC BBW severe infection/sepsis, malignancy
Certolizumab (Cimzia) maintenance

GI
Acute GI bleed •Ligament of Treitz is Hematesmesis: bleed proximal to ligament •type and crossmatch Upper GI Bleed
defining location -coffee-ground: mild, frank blood: severe •CBC ALL GET ENDOSCOPY
seperating UGI from LGI -normocytic: acute
bleed Hemathochezia: maroon/bright red  LGI -microcytic: chronic Stable: consult GI/surgery
•CMP: high BUN: CR >30-UGI Unstable: IVF, PRBC: FFP (4:1)
Medications: Melena: black, tarry stool (altered hgb from •PT/INR: endoscopy can not be
•NSAIDS, anticoags, digestive enzymes )  UGI performed until INR <2.5 Lower GI
antiplatelets, •consult GI/general surgery
•Iron (melena) signs of significant bleeding: hypotension, •EKG & cardiac enzymes •consider endoscopy to r/o UGI
•Beets (hematochezia) tachycardia, low pulse pressure, tachypnea •discuss colonoscopy v angiography
•sign of shock: cool, pale, low cap refill NG Tube (suspected UGI)
Social: •visual inspection of gastric content Others:
•Alcohol  gastris, liver •hyperactive BS: UGI bleed  MOST SENSITIVE PPI: acid suppression
•tenderness: inflame/infectious •Pantoprazole 80mg IV bolus
hx similar sx: 60% UGI •non-tender: vascular •indicated for bleeding from PUD
from same lesions •ascites/hepatosplenomegaly: liver Somatostatin analong
Rectal exam: if LGI suspected  guaiac test •Ocreotide 50mcg bolus
•use for variceal bleeding
Constipation Drugs MOA CI Adverse Effects
Fiber/Bulk Forming Laxatives •Fiber promotes intestinal motility by increasing “bulk” of stool •GI obstruction •GI obstruction
Metamucil Citrucil, •Draws water into stool (retains water) •Abdominal
FiberCon Benefiber *problem if not drinking water with fiber then can cause constipation cramps/distention
•Flatulence
Stool Softeners/Surfactants •Emollient that covers stool and softens it •Do NOT use for
Ducosate (Colace) •Allows it to pass through colon easier SEVERE
Mineral Oil don’t typically see mineral oil because can interfere with absorption

Osmotic Laxatives •Increases secretion of water into the intestine •Abdominal bloating/cramps
Milk of Magnesia •Softens stool and promotes defecation •Flatulence
Miralax •Usually works within 24h *not immediate relief •Diarrhea
Enulose *use for opioid induced and chronic constipation

Bowel Cleansers •Osmotic laxatice


Polyethylene glycol (GoLYTELY) •More rapid, COMPLETE bowel cleanse
Magensium citrate
Sodium phosphate (Fleets) *used prior to colonoscopy, bowel surgeries

Stimulant Laxatives •Stimulate fluid secretion and colonic contraction -> “irritants” •NOT for long term use •N/V/D, abdominal cramps
Bisacodyl (Ducolax) •Irritate intestinal wall, causing fluid accumulations and increased contractions •Electrolyte abnormalities
Senna (Senekot) of intestines -> increases motility *can become dependent
Cascara *works within 6-12 hours; can use about 3-4x per week “rescue” agent

Enemas •Commonly used as adjunct to bowel cleanse (osmotic laxative) prior to


Tap water surgical procedures/colonoscopy
Sodium Phosphate (Fleets)
Mineral Oil (ok to use here) *common in hospital setting
GI
Anal Fissure •Linear tears/ulcerations around anus •Tearing pain with defecation Inspection >80% resolve spontaneously
*MC posterior midline •Bleeding •longitudinal tear in anoderm •1st line: toileting, Sitz bath, Fiber
*small amount with bright red -extends proximally than the
•Due to trauma to anal canal during blood (streaking on TP) dentate line Chronic fissures:
defecation  constipation/hard, straining •skin tags seen as chronic •Topical NTG, Botulinum injection
•Surgery- sphincterotomy
Causes: low fiber, large & hard stools,
constipation, anal trauma

Perianal ½ perianal abscesses are caused by fistulas Abscess: Clinical rectal exam Perianal Abscess:
Abscess and MC in posterior rectal wall •anorectal swelling •I & D  WASH (warm water,
Fistula Bugs: S. aureus, E.coli, Bacteroids, Strep •rectal pain wrose with sitting, Fistula DX: US/CT cleansing, analgesics, sitz baths, high
coughing, defacation fiber diet)
Abscess: infection of anal ducts or glands •worse with sitting, cough, poo •Antibiotics; sitz baths, high fiber

Fistula: open tract b/w two epithelium areas Fistula: anal discharge and pain Fistula treatment: Fistulectomy

Diverticulosis/ Infection/inflammation of diverticuli Diverticulosis: Labs: Diverticulosis:


Diverticulitis •asymptomatic •Diverticulitis: leukocytosis •bleeding will stop spontaneously
Diverticula: outpouching due to herniation of **Diverticulosis is the MCC of •high fiber diet
the mucosa into the colon wall lower GI bleed Imaging:
*MC in RIGHT colon •Diverticulosis: colonoscopy Diverticulitis: Clear liquid
Diverticulitis •Diverticulitis: CT Abd/pelvis •Augmentin 875mg BID x7-10d
Diverticulitis: perforation of a diverticulum •Acute LLQ abdominal pain ***BEST TEST •Metronidazole + Ciprofloxacin or
leading to inflammation & necrosis •Fever, N/V, bowel changes Levofloxacin x7-10d
*MC in SIGMOID colon Complications: peroration, bleed, •Metronidazole + Bactrim
Physical exam: abscess, stricture. fistula,
Risks: low fiber, constipation, obesity •LLQ tenderness, palpable mass obstruction Admit: complication, high risk

Hemorrhoids •Engorged rectal & hemorrhoidal veins Internal (superior vein, proximal) PE: inspect, DRE Conservation
•Proximal (above) the dentate line *less pain •anoscopic evaluation •proper toileting, high fiber, sitz bath
•Intermittent bleeding •topical analgesics (lidocaine)
Causes: constipation, low fiber, straining, •Prolapse Staging: •Preparation H, Anusol, Proctofoam
pregnancy, obesity, cirrhosis (increased •mucoid discharge •I: confined in anal canal, no
venous pressure) prolapse Surgical
External (inferior vein, distal) •II: prolapse with straining •band ligation (MC)
Complications: •perianal pain and bleeding •III: require manual reduction •sclerotherapy
•Thrombosed external hemorrhoids *painful •possible tender, palpable mass •IV: always out, irreducible •infrared coagulation•
•sitz bath, topical ointments, clot evacuation •hemorrhoidectomy (stage IV ext.)
GI
Cholelithiasis Gallstones in the biliary tract Asymptomatic •Ultrasound Symptomatic (Biliary Colic):
WITHOUT inflammation *procedure of choice •Laparoscopic cholecystectomy
Symptomatic (biliary colic)
Gallstone form from: •RUQ pain (hallmark) •CT No surgery:
•Ratio of cholesterol too high -follow fatty meals Ursodeoxycholic acid
•Ratio of bilirubin too high -may radiate to back and
•Gallbladder not emptying bile right shoulder blade! Complications:
choledocholithiasis, acute
Types: Cholesterol gallstones (MC!) *don’t typically see N/V, fever, chills cholangitis, acute cholecystitis
Risks: 5 F: fat, female, fertile, fair, forty with biliary colic

Choledocolithiasi Choledocolithiasis: gallstones in Choledocolithiasis Choledocolithiasis •IV fluids, pain control, NPO
s and Cholangitis common bile duct (CBD); symptomatic •Intermittent RUQ pain, prolonged Labs: elevated LFTs, ALP, GGT •ERCP w/ stone extraction &
-passes are “uncomplicated” •Jaundice sphincterotomy
-typically symptomatic •Intermittent N/V Cholangitis
PE: normal Labs: •ABX with acute cholangitis
Cholangitis: gallstone lodged and •Leukocytosis -mild-moderate: Cipro + Flagyl
obstruction in CBD  infection Cholangitis •high alk phos, GGT, bilirubin -severe: Zosyn (Pip/Tazo) + Flagyl
-Bacterial infection or hepatic injury •Charcot Triad
*when become a problem -RUQ pain, Fever, jaundice Diagnostics: Complications:
•Reynolds Pentad: •US/CT (FIRST!) •liver damage
Bugs: E. coli, Klebsiella, Enterobacter, -Charcot + low BP/shock •septic shock with acute
B. fragilis + mental status changes •ERCP-diagnostic test of choice cholangitis
PE: RUQ pain & hepatomegaly GOLD STANDARD

Cholecystitis Inflammation and infection of the •RUQ Pain Labs: Management: NPO, IVF, ABX
gallbladder due to obstruction of the -steady, sharp pain, continuous •Leukocytosis •Ceftriaxone +/- Metronidazole
cystic duct by gallstones -precipitated by meal •increased bilirubin, ALP, LFTS •IV Morphine or Demerol for pain
•N/V & Fever •Lap cholecystectomy in 24-72hr
Bugs: E. coli (MC), Klebsiella, •precipitated by fatty or large meals Imaging:
Enterococci •Ultrasound-1st line Complications:
Exam: -thick gallbladder, sludge, stones •Gangrene
Chronic: • (+) Murphy Sign •Chronic Cholecystitis
•fibrosis and thickening due to chronic • (+) Boas: referred pain to R shoulder •HIDA scan: MOST ACCURATE
inflammatory cell infiltration  irritation of the phrenic nerve (+): no visualization of gallbladder

Acute Acalculous •Necroinflammatory disease of the •fever Labs: leukocytosis Supportive Care
Cholecystitis gallbladder NOT due to gallstones •jaundice •IV fluids
•sepsis Ultrasound: INTIAL TEST •Bowel rest
Pathophysiology: •vague abdominal discomfort •distended gallbladder •Pain control
•gallbladder stasis and ischemia  •without calcifications •Correct electrolytes
inflammation reaction  concentration •Broad spectrum ABX
of bile salts, distention, perforation CT: if dx uncertain after US
HIDA: is uncertain after CT
Risks: hospitalization, critically ill
EENT 8%

EENT About Presentation Diagnostics Treatment


Bacterial Bugs: •purulent discharge •clinical diagnosis Mild-moderate: Topical Sulfonamide
Conjunctivitis •S. aureus (Adults) •lid crusting •culture and gram stain for discharge •Polytrim (Polymyxin B, Trimethoprim)
•S. pneumoniae “eye stuck shut” •Erythromycin ointment
•H flu •conjunctival erythema •fluorescein stain for keratits or corneal
•M. cat •NO visual changes abrasions Contacts: Topical Vigamox, Oflaxacin, Cipro
•Psuedomonas
•Gonococcal (Day 2-5) Gonococcal:
•Chlamydial (Day 5-7) Infection: IM or IV Ceftriaxone
Prophylaxis: Erythromycin ointment 0.5%
Transmission: direct contact *prophylaxis not sufficient for chlamydia

Chlamydial: 1g Azithromycin ORAL

Viral Bugs: Adenovirus •Copious WATERY discharge Slit-lamp: punctate staining •Supportive
Conjunctivitis •foreign body sensation
Transmission: direct contact •erythema, itching, bilateral •COLD compress to reduce discomfort
*SWIMMING POOL •artificial tears
Exam: preauricular LAD •antihistamine (Olopatadine) for itch, redness

Allergic •Atopic asthma •itching, tearing, redness Mild-moderate: topical antihistamines


Conjunctivitis •atopic dermatitis •pruritis (HALLMARK) •Olopatadine
•allergic rhinitis •stringy discharge •Pheniramine-Naphazoline
•Ketotifen (OTC), Patanol, Bepotastine
Pathophysiology: Exam: •topical NSAIDs: Ketorolac
•mast cell degranulation and •COBBLESTONE papillae
release of histamine •watery, mucoid discharge Severe: *don’t use if h/o or suspected HSV
•chemosis (conjunctival rednes) •topical corticosteroids-Lotepredenol

Opthalmia •neonatal conjunctival Chemical: due to silver nitrate Clinical diagnosis Chemical: artificial tears
Neonatorum infection during delivery
(Neonatal Gonococcal: Gonococcal:
Conjunctivits) •Day 1: chemical •purulent conjunctivitis with Prevention: •IM or IV Ceftraixone
•Day 2-5: Gonococcal exudate and sewlling of lids •prophylaxis: topical Erythromycin •topical Erythromycin prophylaxis
•Day 5-7: Chlamydia *not effective in preventing Chlamydia
Chlamydia: discharge Chlamydia: ORAL Erythromycin

Disease Clinical Presentation Treatment


Papilledem •optic disc (nerve) swelling due to •headache Fundoscope: •Acetazolamide  decreased CSF
a elevated intracranial pressure •nausea •swollen disc & blurry margins production & aqueous humor
•vomiting •flame-shaped hemorrhage
Causes: •refer to ophthalmology
•idiopathic intracranial hypertension 1. MRI or CT first to r/o mass •treat underlying disease
•space-occupying lesions 2. lumbar puncture
•increased CSF *DO NOT PERFORM LP
•cerebral edema

Disease Bugs/Causes Symptoms Treatment


Pterygium •slow growing thickening of bulbar conjunctiva •Fleshy, elevated triangular growing fibrovascular •Artificial tears
mass medially (nasal side) of cornea •NSAIDS or corticosteroid
Risks: wind, sun (UV), sand, and dust exposure •surgery is severe vision impairment

Disease About Clinical Presentation Diagnostics Treatment


Hyphema Causes: trauma, spontaneous pain, photophobia, blurred vision; GOAL: prevent further hemorrhage
N/V-possible IOP •keep supine with head slightly elevated
•hard eye shields
•NO NSAIDS

Disease Bugs/Causes Symptoms Treatment


Bacterial •Inflammation or •ocular pain Slit-lamp: * REFERRAL TO OPHTHALMOLOGIST
Keratitis ulceration of cornea •photophobia •increased fluorescein uptake
(Corneal •eye redness ABX: Topical Fluoroquinolone:
Ulcer) Risks: •vision changes Diagnose: •Oflaxacin 0.3%
•contact lens wear •ocular discharge (hypopyon) and tearing •scrap ulcer for gram stain and •Cipro 0.5%
•dry ocular (Bells Palsy) •difficulty keeping eye open culture •Moxifloxacin
•topical steroids or *DO NOT PATCH
immunosuprresion Exam:
•ciliary injection (limbal flush)
Bugs: •hazy cornea (corneal opacification and
•S. aureus, Streptococci ulceration)
•Pseudomonas (contacts)

Disease Clinical Presentation Treatment


Central Retinal •Retinal artery thrombus or emboli •sudden, painless, monocular Fundoscope: •supine
Artery Occlusion MC 50-80 years old loss of vision • pale, retina •ocular massage
•loss of visual fields • “Cherry-red spot” macula •high concentration of oxygen 100%
Risks: atherosclerosis •amaurosis fugax •afferent pupillary defect (RAPD) •IV acetazolamide or mannitol
•loss of partial visual field •boxcar appearance of retinal vessels •vasodilator
Causes:
•emboli from carotid artery Labs: Giant Cell Arteritis:
•cardiogenic emboli •sed rate and CRP (r/o giant cell) •high dose corticosteroids
•vasculitis

Central Retinal Thrombus in central retinal vein  fluid •first noticed upon awakening Fundoscope: CONSULTATION
Vein Occlusion back up in retina •retinal hemorrhage (blood and •no definitive treatment
•sudden, painless, monocular thunder appearance)
Risks: loss of vision •retinal venous dilation and tortuosity
•HTN, DM, smoking, glaucoma •optic disc swelling
•hypercoaguable states
•multple myeloma
Disease About Symptoms Treatment
Ectropion Eyelid and lashes are turned outward (everted) •Irritation, Ocular dryness, tearing •Lubricating eye drops and moisture shields
due to relaxation of the orbicularis oculi muscle •sagging eyelid, increased sensitivity •Surgical correction

Risks: elderly, congenital, infectious, CN VII

Entropion Eyelid and lashes are turned inward due to •Corneal abrasion from eyelashes •Lubricating eye drops and moisture shields
relaxation of the orbicularis oculi muscle •Ulcerations, erythema, tearing, sensitivity •Surgical correction

Risks: elderly, orbicularis oculi spasms

Dacryoadenitis Inflammation of infection within the lacrimal •Swelling, pain, redness at lacrimal gland Bacterial: ABX +/- I&D
gland; temporal aspect of upper eyelid
Viral: supportive care
Causes: Sjogrens, mumps (viral), bacterial

Dacryocystitis Inflammation of the lacrimal sac/duct due to Acute: Acute: ABX: Clindamycin, Vancomycin,
obstruction of nasolacrimal duct •infection: pain, swelling, tenderness, redness Ceftriaxone and warm compress
*inner corner of eye near nose •warmth to medial canthal (nasal side) of lower lid
•purulent material possibly; tearing Chronic:
bugs: S. aureus/epidermis, GABHS, pseudomonas •surgery (Dacryocystorhinostomy)
Chronic: tearing and discharge

Anterior Inflammation of eyelid, skin, base of lashes •red-rimmed, burning, erythema of lid •WARM compress, eyelid scrubbing, lid
Blepharitis Risks: downs, atopic derm, rosacea, seb derm •scales or granulations seen in lashes washing with baby shampoo, artificial tears

Types: Severe/refractory: topical ABX (Azithromycin,


•infectious (ulcerative): S. aureus or S, epidermis Erythromycin, Bacitracin solution or ointment)
•seborrheic

Posterior Inflammation of the Meibomian glands •lid margins are hyperemic with telangiectasia Inflammation of conjunctiva and cornea:
Blepharitis •MC type of blepharirits (improper drainage) •long term ABX: Tetracycline 250mg,
•inflamed meibomian glands Doxycycline 100mg, Minocycline 50-100mg
Bacterial: staphylococci •tears may be frothy or greasy • corticosteroids: Prednisolone 0.125%

Hordeolum (Stye) •Localized abscess of eyelid margin External: localized, red, swollen, tender warm pustule •WARM compression (1st line)

Bugs: Staph aureus Internal: meibomian gland abscess ABX: Topical Bacitracin or Erythromycin
Risks: Seb derm, rosacea *can progress into cellulitis
•I&D if no resolution in 48h
External: infection of eyelash follicle or external
sebaceous gland (glan of Mall/Zeis)

Internal: inflammation/infection of Meibomian


Disease About Clinical Presentation Diagnostics Treatment
Macular MCC permanent legal •bilateral, progressive, CENTRAL Fundoscope REFER
Degeneration blindness and vision loss vision loss (detailed & color) Dry: Drusen bodies  small, round, Dry:
•central scrotomas yellow-white spots on outer retina •zinc and antioxidant vitamins (C & E)
Two types: •metamorphopsia •amsler grid at home
•Dry (atrophic): MC, (straight lines are bent) Wet (neovascular):
progressive over decades •micropsia new, abnormal vessels Wet:
•Wet (neovascular/exudative): •intravirtreal VEGF inhibitors:
more aggressive  months Bevacizumab *decrease new vessels

Retinal Separation of the retina from •photopsia (lights)  floaters Fundoscope: EMERGENCY
Detachment underlying retinal pigment •”curtain coming down” •detached tissue “flapping” in the •keep supine
epithelium vitreous humor •head turned TOWARD side of
•UNILATERAL PERIPHERAL loss detachment
Risks: myopia, cataract surgery, •no ocular pain or redness (+) Shafers sign: clumping of brown- *do not use miotic drops
old age, trauma colored pigment vitreious cells in the
anterior vitereous humor “dust”

Diabetic MCC of new, permanent Non-proliferative: Proliferative Nonproliferative


Retinopathy vision loss in 20-74 years old •microaneurysms •neovascularization  vitreous •strict glucose control
•cotton wool spots (soft exudates) hemorrhage •laster
3 Types: •hard exudates: yellow dots with sharp
•nonproliferative margins often circinate (lipids) Maculopathy: Proliferative: VGEF inhibitors
•proliferative •flame-shaped hemorrhages (nerve) •macular edema or exudates
•maculopathy •blurred or decreased central vision *ANNUAL EYE EXAM IN DM

Hypertensive •damage to retinal blood vessels Mild: Moderate: SEVERE IS AN EMERGENCY


Retinopathy from longstanding high BP •narrowing •hemorrhages (flame or dot)
•AV nicking (venous compression at •cotton wool spots
the arterial-venous junction) •hard exudates

Severe: ALL + papilledema

EENT Presentation Diagnostics Treatment


Corneal •Eye discomfort Check visual acuity first! Antibiotic Drops:
Abrasion •Tearing, red Non-contact: Erythromycin, Bactrim, Sulfacetamide
•Foreign Body sensation Dx: Fluorescein Stain Contacts: Ciprofloxacin or Ofloxacin (Pseudomonas)
•Photophobia, HA •”ice rink”/inear abrasion seen
•+/- ciliary flush *make sure to evert the lid to •NSAID eye drop-Diclofenac
•+/- changes in vision •Topical anesthetic prior to stain •Tetanus prophylaxis
Proparacain, Tetracain •Follow up if symptomatic in 24-48hr
Disease Causes Clinical Presentation Examination Treatment
Narrow (Closed) •increase in intraocular pressure  •SUDDEN severe, unilateral PAIN Fundoscope: TRUE OPHTHALMOLGIC
Angle Glaucoma damaging optic nerve •loss of PERIPHERAL vision cupping of optic disk EMERGENCY
(“tunnel vision” •prevent or reversal
EMERGENCY Risks: •halos around lights Tonometry: increased IOP •control intraocular pressure
*MC FORM •previous narrow angle or large lens •headache, N/ *recheck 30-60m
•age >60 Gonioscopy:
•females & Asians Signs: GOLD STANDARD •PO or IV Acetazolamide
•conjunctival redness •Timolol 0.5% (topical BB)
Pathophysiology: •corneal edema or cloudiness •Apraclonidine 1% (alpha-2 agonist)
•decreased drainage of aqueous humor via “red, steamy cornea” •Pilocarpine (miotic, cholinergic)
trabecular meshwork and canal of Schlemm •shallow anterior chamber Screening: •Prednisolone Acetate 1%
•mid-dilated pupil (4-6mm) that •40yo+ no risk: q 3-5yrs
Precipitants: poorly, or doesn’t, react to light •40yo+ risks: q 1-2yrs Definitive: Iridotomy (laser or surgical)
Mydriasis (dilation)-dim lights, •60yo+: every 1-2 weeks
anticholingerics, sympathomemetics Exam Order:
•visual acuity & visual field test
Primary: Lens far forward + rests against •pupil evaluation
iris  angle closes •intraocular pressure
•slit lamp exam of anterior segment
Secondary: Anterior angle occluded by •undilated fundus exam
PUSH of ciliary body or PULL of iris

Open (Chronic) •Angle between the cornea and iris is open •initially asymptomatic Cupping of optic disk Pharm:
Angle Glaucoma which leads to degeneration and blockage •painless •Prostaglandins-Latanoprost
of trabecular meshwork Diagnosis: Tonometry •Timilol, Apraclonidine, Pilocarpine
•Increased IOP (>40mmHg) IOP >40=emergent •Carbonic Anhydrase Inhibitors
Risks: •gradual peripheral visual field loss IOP 30-40=urgent (24h) Acetazolamide
•African Americans, age >40, fhx, DM leading to central vision loss IOP 25-29= w/n 1 week
IOP 23-24=repeat/refer Surgery-Laser Peripheral Iridotomy

Glaucoma Drug s MOA SE CI


Prostaglandinds Selective agonist of prostaglandin receptor; increase the outflow of •Bonchospam •raynauds
1st LINE *expensive aqueous humor -> dropping IOP •irritation, increase lashes, change in pigment •asthma •COPD

Beta-Blockers Reduce IOP by interfering with cyclic adenosine monophosphate •bradycardia, hypotension •bronchoscpasm •raynauds
(cAMP) which is used to produce aqueous humor •burning, stinging on application •asthma •COPD

Alpha-2 Adrenergic Causes iris to dilate (mydriasis) causing decreased congestion in the •allergic conjunctivitis •MOAIs •tricyclics
Agonists vessels of conjunctiva -> decreased IOP by reducing production of •hyperemia •CNS depressants, alcohol,
aqueous humor •ocular pruritis BB, cardiac glycosides,
hypertensives
Cholinergic •pupil constriction -> contracts trabecular meshwork -> opens schlemm’s •cramping, diarrhea, watery mouth, sweat
Agonists canal -> increase outflow of aqueous humor -> decreases IOP •fixed, small pupils; visual

Carbonic Slows the action of the enzyme carbonic anhydrase -> decreased •allergy to sulfa
Anhydrase production of aqueous humor
Inhibitors
EENT Causes S/S Diagnostics Treatment
Otitis Externa Inflammation of external •Itching, purulent discharge •clinical Protect ear from moisture + remove
“Diffuse- auditory canal •conductive hearing loss debri/cerumen + topical ABX
Swimmers Ear” •fullness/pressure otoscopy:
Causes: •Severe pain * w/ pulling tragus •edema of external auditory ABX:
Psuedomonas •water, moisture *rise in pH canal •Ofloxacin, Cipro-dexamethasone
Staph epi •trauma (Q-tips) Exam: •Cortisporin Otic Solution
Staph aureus •Pain with palpation (Neomycin, Poly B, Hydrocortisone)
Bugs: •Swollen, red canal *DO NOT USE IF TM PERFORATION
•Pseudomonas (MC!) •Moist debris in canal *AMINOGLYCOSIDE OTOTOXIC!
•Staph epidermis, Staph aureus •TM difficult to visualize •Ear wick

Chronic Otitis Invasive infection of external ear •Severe, deep seated otalgia (pain) out Otoscopy: •Aggressive glycemic control
Externa- canal and skull base (temporal of proportion to exam •Granulation tissue at bony
Malignant/ bone, soft tissue, and cartilage) •Purulent otorrhea, FOUL SMELL cartilaginous junction of ear ABX: IV 6-8wks
Necrotizing canal *hallmark of spreading •IV Ciprofloxacin
Bug: Pseudomonas Complications: •Piperacillin
Psuedomonas •spread to skull base  osteomyelitis CT or MRI: confirm •Cefepime
Risks: •immunocompromised •CN palsies or spread to meninges
•Elderly DM (MC!) biopsy: MOST ACCRATE

Disease Causes Clinical Findings Diagnosis Treatment


Acute Otitis •infection of the middle ear Serous: no infection, just fluid bulging, erythema, +/- fever Effusion but NO inflammation:
Media •usually occurs with viral URI  block AND *serous otitis media
Eustachian tube  negative pressure  Acute: Middle ear effusion •observation
allowing bacteria and viruses to colonize •Ear pain (MC complaint) •Tylenol or Ibuprofen
•Fever, irritability, HA, anorexia, Pneumatic Otoscope:
Risks: N/V/D MOST SENSITIVE Effusion and inflammation:
•children (short, narrow, horizontal ET) •infants pulling at ears •loss of mobility TX FOR 10 DAYS
•day care •Bulging of TM •Amoxicillin-1st LINE!
•pacifier or bottle use, not being breast fed A=normal •Omnicef, Ceftin
•second hand smoke TM EXAM: C.O.M.P.L.E.T.E.S. B=effusion, AOM •Zithromax (not for H. flu)
Color, other conditions, mobility C=Eustachian tube dysfunction •Augmentin (if recent abx)
Bacteria: (decreased), position, lighting, entire D=tubes, perforation •Cephalosporin (if recent abx)
•S. Pneumo surface, translucence, external
•M. cat auditory canal and auricle, seal PCN allergy: Azithromycin
•H. flu
•Group A Strep Exam: Severe: myringotomy + tubes
•bulging and erythematous TM
Virus: RSV, picornaviru), coronavirus, •middle ear effusion Referral for tubes:
influenza, adenovirus •loss of landmarks •3+ AOM within 6mo
•decreased mobility with otoscope •4+ infections within 1yr
Chornic: •unresponsive to ABX
•recurrent or persistan infection WITH TM Chronic:
perforation for 6 WEEKS •perforated TM Chronic: debride + ABX
•bug: Pseudomonas •recurrent purulent otorrhea •topical ABX (Ofloxacin)

Disease Causes Clinical Findings Diagnosis Treatment


Cholesteatoma •Abnormal keratinized collection of •painless otorrhea  Otoscope Refer for surgery for excision
desquamated squamous epithelium in the yellow/brown with a strong odor •WHITE MASS behind TM and ossicle reconstruction
middle ear  possible bony erosion of mastoid •vertigo, tinnitus, dizziness •focal granulation, cellular debri
•ear drainage >2 weeks despite tx
Causes: •hearing loss (conductive)
•Complication of AOM
•Prolonged Eustachian tube dysfunction
(chronic negative pressure)

TM Perforation •rupture of the tympanic membrane •acute ear pain Otoscope: •spontaneous healing
•conductive hearing loss •perforated TM
Causes: •sudden pain relief with bloody *do NOT perform pneumatic ABX: Topical Ofloxacin
•penetrating or noise trauma (pars tensa) otorrhea
•otitis media •tinnitus and vertigo *NO AMINOGLYCOSIDES

Disease Causes Clinical Findings Diagnosis Treatment


Acoustic •Benign tumor of schwann •Unilateral sensorineural hearing loss Audiometry: unilateral sensorineural •surgery or focused radiation
Neuroma cells (myelin forming cells) *acoustic neuroma until proven otherwise hearing loss
(Vestibular •Tinnitus, vertigo, ataxia “Example: woke up this
Schwannoma) •arises in cerebellopontine •Dizziness and balance problems Diagnostics: MRI (BEST) morning and could not hear
angle  CN VIII, VII, V •Facial numbness (CN V) or paresis (CN VII) anything from one side”

Labryinthitis/ Vestibular neuronitis: •Sudden, persistent, continuous vertigo •Clinical DX •Corticosteroids: 1ST LINE
Vestibular inflammation of vestibular •N/V; Gait sways toward affected side •May need MRI/CT brain to r/o
Inflammation division of CN VIII •Horizontal nystagmus-beats AWAY central etiology Symptomatic
•Meclizine (antihistamine)
Labryinthitis: inflammation of Labrynithisis ONLY: •Head impulse test normal in •Benzodiazepines (Diazepam)
vestibular and cochlear •above + unilateral hearing loss and tinnitus central lesion etiology •Ondansetron (Zofran) for N/V
portion of CN VIII

Causes: idiopathic, viral

BPPV MCC of peripheral vertigo •Recurrent, brief episodes of vertigo <1minute Dix-Hallpike Maneuver Reposition Otiliths
Benign Paroxysmal •occurs w/n seconds of changing head position *otilith movement  fatigable •Epley Maneuver
Positional Vertigo Causes: •possible N/V
•displaced otolith particles •posterior canal: upward, rotary •Decondition Exercises:
(calcium crystals) within the NOT WITH: tinnitus, hearing loss, or ataxia *clockwise •Brandt-Daroff
semicircular canals of inner •anterior canal: downward, rotary •Sermont Maneuver
ear (canalithiasis) *counterclockwise
*MC is posterior canal •horizontal Canal: towards floor)

Disease About Clinical Presentation Treatment


Nasal Polyps Causes: •Pale, edematous, boggy mucosally covered masses •Topical nasal steroid for 1-3 months
•Allergic rhinitis (MCC) •Chronic nasal obstruction and decreased smell *surgery is medication unsuccessful
•cystic fibrosis *avoid aspirin in asthma pt
Disease Etiology/Pathogenesis Presentation Evaluation/Diagnosis Treatment
Meniere’s •idiopathic distention of CLASSIC 4 SX: Audiometry: Lifestyle: 1st line
Disease endolymphatic compartment of •Episodic vertigo (min-hours) hearing loss during acute attacks •avoid salt, caffeine, alcohol, choclalte
(Idiopathic the inner ear due to excess •Sensorineural hearing impairment
Endolymphatic fluid (endolymph) *usually unilateral Caloric Testing: Medication: 2nd line
Hydrops) •Tinnitus (low tone & “blowing”) Reduced or absent nystagmus on •antihistamines: Meclizine
Syndrome: unidentifiable cause •Ear fullness affected side •Diazepam
Disease: idiopathic •Promethazine
•Episodic N/V with vertigo •Brain Imaging if central lesion •anticholinergics: Scopalamine
*women 20-40yo •Hearing improves between attack •diuretics: HCTZ
•Chronic, progressive, remitting
Refractory: surgical decompression,
labyrinthectomy, intraaural Gentamicin

EENT About Clinical Presentation Treatment


Epistaxis Posterior: Anterior: Anterior: Posterior:
•sphenopalentine artery •bleeding site •Pressure for 5-15 min (1st LINE) seated, lean forward •ENT consult
branches, Woodruff’s plexus visualized +/- topical vasoconstrictors (Oxymetazoline, lidocaine •Packing
with epinephrine, 4% cocain) •Oxygen
Posterior Risks: Posterior •2nd line: electrocautery or silver nitrate •Narcotic analgesic
•HTN, old, nasal neoplasm •anterior not visualized •3rd line: nasal packing •Ligation, Endovascular embolization
•bleeding from both •Balloon catheter (MC)
Anterior: Kesselbachs MC nares ABX: Augmentin or Cephalexin (Keflex)
•blood into pharynx After control:
Anterior Causes: *Follow up in 48-72 hours •Avoid vigorous exercise, spicy foods, trauma
•nasal trauma, FB, rhinitis •Lubrication w/ pretolatum or Bacitracin oint
•low humidity, hot env. •Increase home humidity
•alcohol, cocaine

Disease Clinical Findings/Causes/Bugs Diagnosis Treatment


Acute Acute inflammation of mucosa of larynx •Hoarseness (hallmark) •clinical diagnosis supportive care: hydration, vocal rest,
Laryngitis •aphonia humidification, warm saline gargles
Noninfectious causes: •dry, scratchy throat Referral: •ENT follow-up
Vocal strain/cyst, singing, polyp, reflux •Laryngitis (hoarseness) lasting >2
•Exudative Tonsillopharyngitis weeks with NO URI sx or in a patient Bacterial: PCN or Erythromycin
Bugs: with fever and anterior cervical that uses tobacco or alcohol
•Virus: rhinovirus, influenza, parainfluenza, LAN-strong bacterial *risk for laryngeal cancer Polyp: surgery
adenovirus, coxsackievirus, coronavirus, RSV •Significant edema
•Bacterial: M. cat, M. pneumo GERD: PPI, ENT referral
•Candidiasis: immunosuppressed
Disease Transmission/Causes Clinical Presentation/Physical Exam Diagnostics Treatment
Acute Bacterial Bugs: Signs/Symptoms: •clinical diagnosis ABX: if sx >10-14 days
Rhinosinusitis •Viral (MC!) •nasal congestion and obstruction •No risks: Augmentin 875/125mg BID
(rhinovirus, influenza, •purulent nasal discharge Imaging: *clavuanate improved H. flu coverage
parainfluenza) •facial pain/pressure worse with leaning forward CT is study of choice if •Risks: Augmentin 2g/125mg BID
•bacterial for bending over needed
(S. pneumo, M. cat, H. flu) •fever, cough, hyposomnia or ansomnia, ear PEN ALLERGY:
pressure and fullness, HA, halitosis (bad breath) Definitive: Biopsy •Doxy 100 BID or 200 QD
Causes: •Levofloxacin 500mg po qd
•viral URI Clinical Findings: •Moxifloxacin 400mg po qd
•dental infections •Maxillary-cheek or upper teeth (MC!)
•NG tubes •Ethmoid-between eyes or retro orbital Complications:
•smoking, allergies •Frontal-above eyebrow •Orbital cellulitis-Naficillin and Rocephin
•cystic fibrosis •Sphenoid-upper half or retro orbital w/ radiation •Frontal Subperiostela abscess (Potts Puffy
to occiput (back of head) Tumor)-IV ABX
Acute: 1-4 weeks •Intracranial complications
Subacute: 4-12 weeks
Chronic: >12 weeks

Invasive Fungal Bugs: •Rhinocerebral mucormycosis-black eschar on Biopsy: •Surgical debridement + IV Amphotericin B
Sinusitis •Aspergillus middle turbinate, palate, or face •non-septate broad
(Mucormycosis) •Mucor, Rhizopus, Asidia •Spreads to orbit and cavernous sinus hyphae with irregular Mycetoma: surgery +/- antifungal
•Cunninghamella •acute sinusitis symptoms right angle branching

Risks: Mycetoma (fungal ball):


•DM (DKA) •Long-standing unilateral symptoms,
•immunocompromised •pacification of single sinus

Chronic Bugs: •same symptoms as acute bacterial rhinosinusitis •CT scan •Refer to ENT
Bacterial •S. aureus but for > 12 weeks •ABX-culture guided
Sinusitis •Aspergillus Biopsy: •Intranasal corticosteroids (Flonase)
•Mucormycosis TEST OF CHOICE •Nasal saline irrigation
EENT About Clinical Presentation Diagnostics Treatment
Upper Respiratory Bugs: Rhinovirus SX peak day 2-3, last 10-14 days •clinical •Self-limiting
Tract Infection •nasal saline
(URI) Transmission: Clinical Features: Complications: •oral decongestants (Psuedofed)
“Common Cold” •Hand contact-2h •clear, watery rhinorrhea; nasal congestion •Secondary bacterial •nasal decongestants *few days only
“Viral rhinosinusitis” •Droplet transmission •ears, eyes, noe, throat may ne involved infections:
•Contaminated fomites rhinosinusitis, OM,
(surfaces) * hours Physical Exam: pneumonia
•Nasal mucosal swelling and discharge
•Pharyngeal erythema, Conjunctival injection

Allergic Rhinitis •IgE mediated •Episodic rhinorrhea, sneezing, nasal itching •clinical •1st line-Antihistamine (allegra) and nasal
•Nasal obstruction, post nasal drip, cough•Pruritus •eosinophils in nasal glucocorticoid (Fluticasone, Mometasone)
Allergen exposure of conjunctiva, nasal mucosa, oropharynx secretions, serum IgE
•seasonal elevated Montelukast *prophylaxis before allergy season
•perennial (chronic) Exam: •skin testing •MOA: Inflammatory mediators produced by
•Allergic shiners (dark circle under eyes  edema) leukcytes that are accompanied by production of
Atopic •Allergic salute and crease (rub nose upward) histamines and prostaglandins
•fhx AND •Accentuated lines of the eyes (Dennie-Morgan) SE: dreams, insomnia, anxiey, depression, SI
personal hx •Allergic face: high arched palate, mouth breath,
dental malocclusion •1st Gen antihistamine: Benadryl, Hydroxyzine,
•Nasal mucosa pale, bluish hue and boggy Chlorephenirmaine, Bropheniramine
•Conjunctiva congested and edematous •2nd Gen: Zyrtec, Claritin
•Pharynx cobblestoning •2nd gen metabolite: Allegra *best
•Nasal polyps •Nasal antihistamine: Azelastine, Olopatadine

Disease Clinical Findings/Causes/Bugs Diagnosis Treatment


Acute VIRUS is the MCC: Respiratory Virus •clinical •supportive
Pharyngitis Adenovirus, Rhinovirus, •sore throat, pain •rapid strep to r/o bacterial
Enterovirus, EBV, RSV, •Coryzal sx, Fever (RARE) HSV: Acyclovir, Valcyclovir
Influenza, HZV •NO adenopathy or exud

EBV: white, purple exudate


HSV: vesicles, shallow ulcer

Acute •GABHS •Fever, sore throat Centor criteria: FIRST LINE: Pen VK QID
Bacteirla •Rare in children <3 years old •Malaise, N/V •Fever •Pen G Benzathine-IM
Pahryngitis •Tender anterior cervical •Cervical LAN (anterior) •Amoxicillin (good alternative)
(Strep throat) Complications: 2-3 weeks lymphadenopathy •Lack of cough •Cephalexin
•Rhematic fever-antibodies •Pharynx, soft palate, tonsil •Pharyngotonsillar exudate
•Peritonsillar abscess erythematous and edematous PCN Allergy:, Azithromycin,
•Poststreptococcal 4/4=strong suggestion *just treat Clindamycin, Cephalosporins
glomerulonephritis Scarlantiniform rash-scarlet 3/4 =intermediate 1=unlikely
fever sandpaper rash from *ask if recent ABX in 3 MONTHS
exotoxins of GABHS Labs: Antistreptolysin O (ASO)

Tests: Rapid strep for GABHS *intial


•Throat culture *DEFINITIVE
Disease Clinical Findings Diagnosis Treatment
Peritonsillar •abscess between palentine tonsil and •Severe sore throat-unilateral Labs: CBC, CMP, blood & throat cx •Airway, hospital admission
Abscess (Quinsy) pharyngeal muscle •Fever Trismus, Drooling, •Consult ENT
•MC in adolescents & young adults odynophagia Imaging: •Needle aspiration or I&D
“quinsy abscess” *MC deep neck infection in children and •Neck swelling and pain Differentiates from other deep neck
adolescents •Muffled voice “hot potato” space infections ABX: IV Unasyn or Clinda
•Referred ear pain, Fatigue, •CT w/ IV contrast Add Vanc if no response promptly
Bugs: *often polymicrobial irritable, decreased oral intake •Lateral neck x-rays
•Strep pyogenes (Group A) *not epiglotiss or Retro abscess •Switch to 14d of oral:
•Staph aureus Exam: Augmentin or Clindamycin
•Respiratory anaerobes •swollen, fluctuant tonsil
•Uvula deviation to opposite

Retropharyngeal •deep neck space infection posterior to •Neck swelling/mass, pain lateral neck x-ray: increased •Airway first
Abscess pharyngeal wall •torticollis (wont move neck) prevertebral space >50% width
•MC in children 2-4 years old •Spiking fever ABX: IVAmpicillin-Sulbactam
*complication of •Odynophagia, Dysphagia CT neck with contrast: BEST (Unasyn) or Clindamycin
peritonsillar Bugs: *often polymicrobial •ring enhancing lesion
abscess •Strep pyogenes (Group A) Exam: •surgical incision and drainage if
•Staph aureus •midline or unilat posterior large and mature
•Respiratory anaerobes pharyngeal wall edema
•anterior cervical LAD

Disease About Clinical Findings Diagnosis Treatment


Sialolithiasis •stones within the salivary gland •sudden onset of salivary gland clinical Conservative: Sialagogues 1st line
(Salivary Gland •MC in Whartons duct (submandibular pain and swelling with eating or in *increase salivary flow
Stones) gland) & Stensens duct (parotid gland) anticipation of eating •tart hard candies, lemon drops
•increase fluids, gland massage
Risks: decreased salivation (dehydrated,
anticholinergics, diuretics) Surgery: sialoendoscopy, laser,
sialodenectomy

Acute Bacterial •bacterial infection of the parotid or •very firm and tender gland swelling CT scan ABX: Dicloxacillin or Naficillin,
Sialadenitis submandibular glands •purulent discharge if massaged Clindamycin + sialogogues
(Suppurative •dysphagia
Sialadenitis) Bugs: S. aureus (MC), S. pneumo, •trismus (reduced opening of jaw)
S. virdians, H. flu, Bacteroides •fever, chills

Risks: obstruction (stone), dehydration,


chronic illness

Apthous Ulcers •maybe association w/ HHV-6 •small, painful round/oval shallow ulcer Clinical •topical oral glucocorticoids:
(Canker Sore, (yellow, white or grey with central exudate) Clobetasol gel, Dexamethasone elixir,
Ulcerative Risks: •erythematous halo Triamcinolone
Stomatitis) •IBD (Crohns)
•HIV, celiac disease, lupus MC on buccal and labial mucosa •topical nalgesics: 2% viscous
•methotrexate, neutropenia lidocaine

OBGYN 8%

Cystocele: posterior bladder herniating into anterior vagina


Enterocele: puch of Douglas-small bowel herniating into upper vagina
Rectocele: sigmoid colon or rectum herniating into posterior distal vagina

Breast CA About Presentation Diagnostics Treatment


Pagets •Eczematoid eruption •pain, itching, burning •Full-thickness biopsy of lesion •MASTECTOMY
Disease and ulceration •erosion, ulceration •Insitu: no lymph biopsy
•may see bloody nipple discharge •Invasive: biopsy lymph •May try excision of nipple,
•Arises from NIPPLE •retracted nipple areola, local mass
can spread to areola
•palpable mass: 50% (9% invasive cancer)

•no palpable mass: noninvasive cancer or ductal carcinoma

Inflamamtory •Aggressive but rare •Diffuse, brawny edema of skin with erysipeloid border •Suspected mastitis does not rapidly •Chemo  surgery and
Carcinoma •blocked dermal lymphatics by tumor emboli response (1-2wk to ABX  biopsy radiation
•“Peau d orange” (orange peel) skin
•usually NO palpable mass

Medication and Usage MOA Side Effects DDI


SERM •Tamoxifen (Nolvadex) •binds to estrogen receptors Common •Not for use with other hormone-
•Raloxifene (Evista) •block estrogen activity in some hot flashes, nausea, muscle aches and modulating anti-CA therapy
•Toremifene (Fareston) cramps, hair thinning, headache,
•tamoxifen - blocks in breasts; paresthesias •SSRIs, cimetidine can reduce efficacy
•Used for treatment of breast CA and mimics in uterus, bone
chemoprevention of breast CA in •toremifene - blocks in breasts; Benefits: improve bone & lpids •Avoid with QT-prolonging agents
some high-risk women mimics in uterus, bone
•raloxifene - blocks in breasts, Risks: thrombosis, fatty liver,
uterus; mimics in bone endometrial cancer, false thyroid

Aromatase •anastrazole (Arimidex) •inhibit aromatase (enzyme that Common: •Caution when using with, or do not use
Inhibitors •exemestane (Aromasin) blocks conversion of testosterone hot flashes, GI upset, muscle with, other hormone-modulating anti-CA
•letrozole (Femara) *induce ovulation to estrogen) weakness, joint pain, headache, therapy
worsened ischemic heart disease
•Used for treatment of breast CA •May increase serum concentration of
Risks: methadone or L-methadone
•May be used alone, in combination •Hypercholesterolemia
with GnRH blockers, or •Insomnia, impaired cognition, •Do not use with estrogen or
before/following SERMs fatigue, mood changes, Thinning hair immunomodulating drugs

CI: pregnancy
Fulvestrant •used for metastatic breast cancer •GnRH agonists/antagonists •Used to reduce release of GnRH and
(Faslodex) •attaches to and causes destruction of FSH/LH
estrogen receptors
•does not mimic effects of estrogen

Breast CA About Clinical Presentation Diagnostics Management


Breast CA •MCC of non-skin cancer *most arise from intermediate ducts and are Mammography: women >40 Hormone Receptor Sites
in women invasive *can detect cancer as early as 2 years •ER (+) tumor: slightly lower
•microcalcifications & speculated liklihood of early recurrence
•2nd MCC cancer death in Types: •Most diagnosed after abnormal test •Er (-): mets to liver, lung, brain
women (after lung CA) •infiltrative ductal carcinoma (MC) *higher chance of recurrence
-lymphatic metastases Ultrasound: women <40
Risks: •infiltrative lobular carcinoma (+) ER:
•personal history •pagest disease MRI: rapid uptake of contrast •Anastrozole (postmenopausal)
•(+) FHX: •inflammatory •Tamoxifen (premenopausal)
-1st degree relative: 2x risk Biopsy: *prevention in high risk family history
-2nd degree relative: 3x risk Clinical Presentation •FNA: removes the least amount of tissue, •Trastuzuma (anti-HER2)
•Genes: BRCA1 & BRCA 2 •painless breast mass *>5cm suspicious but will not allow for receptor testing
-BRCA 1: higher risk •hard, fixed, irregular margins, nonmibile Radiation: usually done after
-BRCA 2: lower risk •upper outer quadrant •Core Biopsy: takes more tissue, but lumpectomy or mastectomy
•Nulliparity *due to more tissue in that area allows for receptor testing
•First full-term pregnancy Surgery:
>30 Exam: •Open Biopsy: MOST ACCURATE Radical mastectomy:
•Early menarche (before 12) •skin changes: erythema, discoloration, •removal of breast, pectoral muscles,
•Late menopause (after 50) ulceration, skin retraction (Coopers Genetic Testing axillary lymph nodes
•increasing age ligament), bloody discharge •considered for members of high risk fhx
•Postmenopasual HRT •genetic counseling needed before and after Modified radial mastectomy:
•Hx uterine CA •Palpation of regional lymph nodes: axillary, •removal of breast and underlying
pectoral, supraclavicular, infraclavicular, Screening: pectoralis major fascia
subscapular, epitrochlear and lateral chain Mammogram every 1-2yrs women 50-69
•axillary lymphadenopathy •ACS: Breast Conservation: excise tumor
CBE q3yrs 20-39, mammo + CBE age 40 with negative margins, lymph biopsy
•mets: bone, lungs, liver, brain •ACOG: •stage I & II and some III
CBE + mammo q1-2yr 40-50, yearly 50+
•USPSTF: Follow-Up:
mammo +/- CBE q1-2 yrs, age 50-74 •Close follow-up needed due to high
risk of recurrences
•exam Q 4 mo x 2 yrs, then Q 6 mo x
3 yrs, then yearly
•Mammogram 6 mo after radiation
then becomes yearly
OBGYN About Clinical Presentation Diagnostics Treatment
Fibrocystic •MCC cyclic breast pain in •Mass with pain or tenderness •US or mammogram •reassurance of benign findings
breast reproductive age women •pain secondary to proliferation of *no mammogram if •avoidance of trauma, well-fitting, supportive bra
Chnges •MC age 30-50 years normal glandular tissue <30yo •abstaining from caffeine, coffe, chocolate
(Glandular *once cyclic change stops then you *can increase and decrease size •OCPs can reduce symptoms
Hyperplasia) wont see this anymore •usually present or worse FNA:
(stop after menopause unless HRT during premenstrual phase (prior to •straw colored or •CAM: evening primrose oil, vit E 400U
menstruation) green fluid •OTC analgesics: Tyneol, Ibuprofen
Pathophysiology: (no blood) •severe pain: Danazol, Tamoxifen
•hormonal imbalance that may Exam: •Refractory: surgery
produce asymptomatic breast lumps •multiple, nodular, mobile, smooth
•estrogen considered factor round lumps in both brests Prognosis: exacerbation may occur until menopause

•possible increase if drink alcohol •estrogen stimulates ducts,


•caffeine can worsen pain progesterone stimulates stroma

Fibro- •Common, benign solid tumor •Round, firm, discrete, mobile, non- •clinical •May be confused with Phyllodes tumor: a
adenoma •focal abnormality of breast lobule tender mass fibroepithelial tumor that clinically resemble
•may enlarge with pregnancy US: solid, well- fibroadenomas and has a small chance of malignancy
•Usually in young women (within •does not change throughout cycle circumcised,  surgical excision
20 yrs of puberty) avascular mass
Unclear diagnosis or rapid growth: surgery
•more frequent and earlier age of Core Biopsy: •excision with margin of normal tissue
onset in black women DEFINITIVE
Asymptomatic: monitor, core needle biopsy to confirm
Pathophysiology: OR repeat US and breast exam in 3-6 months
•suspected possible hormonal link
•increased in size during pregnany
and with estrogen therapy
•decrease in size after menopause

OBGYN About Clinical Presentation Diagnostics Management


Cervical Types: •asymptomatic in early stages •colposcopy with biopsy Carcinoma in situ (stage 0)
Cancer •squamous cell (MC!) •excision (LEEP, cold knife)
•adenocarcinoma •post-coital bleeding or spotting CIS or pap abnormal: conization •ablation (cryotherapy, laser)
•clear cell (linked with DES) •irregular, heavy vaginal bleeding or watery •total abdominal hysterectomy +
discharge bilateral salpingo-oophorectomy
Spread: •weakness, weight loss, anemia, pelic pain
•paracervical nodes (MC) *late symtoms Stage 1A: total hysterectomy or radical
•parametrial, obturator,
hypogastric, external iliac, sacral Stage 1A2, 1B, IIA:
Exam: cervical discharge or ulceration •radiaction with brachytherapy
Risks: •early: cervix normal •radical hysterectomy with bilateral
•HPV (16, 18, 31,33) •late: enlarged, irregular, firm pelvic lymphadenectomy
•early sexual activity, STIs
•lots of sexual partners Cervix Locally advanced (IIB, III, IIVA):
•smoking, DES expsore Endophytic: barrel-shaped •radiation + chemo
•cervical intraepithelial neoplasia Exophytic: friable, bleeding, cauliflower
•immunosuppression Advanced: radiacation, chemo

OBGYN Risks HPV Dysplasia Cancerous


Cervical Risk Factors: HPV & Cervical Dysplasia: CIN I: mild cervical dysplasia Bethesda System
Dysplasia/ Sexual Activity Factors: •HPV present in >80% of all CIN •LOWER 1/3 of epithelial lining
Cancer •multiple sexual partners •HPV 16: 50-70% HPV 18: 7-20% Atypical Squamous Cell (ASC-US/H)
•early onset of sexual activity *cigarettes have a synergistic effect CIN II: moderate cervical dysplasia •undetermined significance, cannot
•high-risk sexual partner with HPV  cancer •LOWER 2/3 of epithelial lining exclude high grade lesion

Infection Factors: High Risk: 16, 18, 31, 33, 35, 39, 45, CIN III: severe cervical dysplasia Low-Grade Squamous
•HPV infection (BIG ONE!) 51, 52, 56, 58, 59, 68 •over 2/3 of epithelial lining; FULL Intraepithelial Lesion (LGSIL/LSIL)
•History of sexually transmitted thickness  corresponds to CIN-1
infection •High-risk HPV test performed after
•Immunosuppression (HIV) abnormal Pap High-Grade Squamous
•Most + do NOT develop CIN or CA ALWAYS treat CIN II and III except: Intraepithelial Lesion (HGSIL/HSIL)
Others: •pregnant woman (wait till postpartum)  corresponds to CIN II and III
•multiparity Vaccines: Gardasil 9 •CIN II in adolescents (high chance of  excision (LEEP) or ablation
•long term OCP use (6, 11, 16, 18, 31, 33, 45, 52, 58) spontaneous regression)
•female: age 11-26, male: age 11-21 Atypical Glandular Cells (AGC)
•<15yo: 2 doses, 6 months apart Glandular cells: normal components of
•>15yo: 0, 2, 6 months the endocervix  secrete mucus
*min interval b/w 1st and 2nd 4
weeks, b/w 2nd and 3rd is 12 weeks Atypical: dont match normal glandular
cells but are not definitely cancer

Pap Smears Atypical Screening Colposcopy After Colposcopy


•21-29yo: every 3 years 3 options: Illuminated low-power magnification CIN I: expectant
1. repeat serial cytology: every 6 -3-5% aqueous acetic acid solution •high chance of regression
•30+: HPV + pap q5y or pap only q3y months until 2 consecutives normal -directed biopsies of abnormal areas •2 paps every 2 months or…
-second abnormal  colposcopy -curette or brush of endocervical canal Pap + HPV at 6 months
•30+/+HPV: pap&HPV 1yr or genotype 2. High-risk HPV: colposcopy if + •repeat if abnormal cytology or HPV+
3. Immediate referral to colposcopy Indications: •if all normal, then routine screening
•>65: stop if no hx dysplasia or CA, 3 (-) •abnormal cytology or HPV test
paps or 2 (-) pap + HPV in a row in last Before repeat, treat the underlying: •clinically abnormal cervix Surgery:
10 years •hormones if atrophic vaginitis •unexplained intermenstrual or •CIN II/III
•antimicrobials for infections postcoital bleed •Invasive Cancer
LSIL, HSIL, ASC-H, AGC  •vulvar or vaginal neoplasia •Otherwise abnormal or unsatisfactory
Colposcopy •history of in utero DES exposure colposcopy
Natural Methods Periodic Abstinence
Contraceptive Coitus Interruptus (pull out): Periodic Abstinence: Calendar Method: Temperature Method
Methods •withdrawal of penis before ejaculation •avoid sex during time when ovum •ovulation after recording •more efficacious than calendar method
(Natural) and sperm could meet menstrual pattern for several mo •records basal body temp
Post-coital Douche: *2-3d before  2-3d after •ovulation normally 14d before •take in AM before activity
•water, vinegar, commercial product •methods: calendar, temperature, 1st day of next menstrual period •slight drop in temp 24-36h after ovulation,
•theory: flush semen out of vagina combined temperature/calendar, •fertile interval: min 2d before temp rises 0.3-0.4C and stays there
cervical mucus (billings), ovulation to min 2 day after •third day after onset of elevated
Lactation Amenorrhea: symptothermal method •regular menstrual cycle temperature-end of fertile period
•suckling  reduced GnRH, LH, FSH •MC method of period
•anovulatory menses for 1st 6mo if most effective determinant: LH abstinence and least reliable Cervical Mucus Method “Billings”
exclusively breastfeeding (not 100%) •possible increased incidence of •observing changes in cervical mucus
*not effective if supplemental feeding congenital anomalies among Ex. Cycle Beads •several days before to just after ovulation
•need amenorrhea to be effective children resulting from unplanned •white: stop having sex becomes thin and watery
•recommended to use contraceptive pregnancies •brown: ok to have sex •rest of menstrual cycle: thick & opaque
starting 3 months after delivery •red bead: period
Symptothermal Method (most effective)
Combined Temp/Calendar •combines cervical mucus and temp
•well-motivate & compliant •uses symptoms before ovulation
•failure rates 5/100 •bloating, tenderness, increased libido,
spotting, nausea, headache, mittelschmerz

Long-Acting About/MOA Benefits Side Effects


Depot •150mg 17-acetoxy-6-methyl progesterone IM q3mo Benefits: Major side effect: decreased bone density
Medroxyprogesterone •Take up to 10 months to return to baseline fertility •lower risk of ectopic pregnancy •osteoporosis, calcium loss
Acetate •lower risk of endometrial cancer •can cause irregular menses
“Depo Shot” MOA: Thick mucus & thin endometrium •lower risk of sickle cell crises •prolonged menstrual flow and spotting for
•may improve endometriosis first 6 months
Perfect: 0.3 pregnancies/100, Typical: 3 preg/100 women •dose not increase risk of vascular disease •often amenorrheic later in course

Implants •Single-rod implant; etonogestreol, progesterone •No major complications SE: irregular menses, weight gain
“Implanon” •Efficacious for up to 3 years; Very high efficacy
“Nexplanon”
Vaginal Ring •Flexible ring 5cm diameter and 4mm thick Failure rate: 0.65/100 women •SE similar to COCs
“Nuvaring” •less breakthrough bleeding and spotting
Combo: ethyinyl estradiol and etonogestrel
•works3 weeks/month •vaginal discomfort, leukorrhea, vaginitis,
•maintains efficacy even if removed for up to 3 hours coital problesm
•designed to be left in place during intercourse
Transdermal Patch Combo: norelgestromin and ethinlyl estradiol daily •Higher failure rate for women >198lbs •CI, SE, risks same as COCs
“OrthoEvra” •New patch wkly x 3 weeks followed by patch-free wk
“Xulane” •Attempt to reattach if comes off •slightly more breast symptoms and
Sites: buttocks, lower abdomen, upper out arm, uppter •Detached <24 hours: continue as usual dysmenorrhea (painful cramps)
torso (except breast) •Detached >24 hours: new patch, backup
contraception x1 week

About Combo New Formulation Combo


Oral OCPS: oral contraceptive pills Estrogen: ethanyl estradiol (MC), •Less hormones  less SE •21 days of active hormone,
Contraceptives mestradiol, estradiol valerate followed by 7 days of placebo
COCs: combo oral contraceptives •Monophasic: same dose of
estrogen & progestin Progestin: norethindrone, hormones daily •84 active pills, 7 days placebo
levonorgesterel, desogestrel, gestodene,
POPs: progestrin-only norgestimate •Multiphasic: different doses of •365 active pills
hormones during cycle
•Highly effective when taken as •3rd gen (Desogesterol, norgestimate) *most now trying to mimic hormone •withdrawal bleed 2-5 days after
directed and 4th gen (Drispirenone) levels in cycle stopping active pills
-good for hirsutism & acne  progesterone drops
-bad because thromboembolism

Oral Administration MOA & DDI Benefits & Disadvantages SE & Caution
Combo Oral Ideal: begin COC 1st day of cycle MOA: Advantages: CI:
Contraceptive •Suppress ovulation & implantation •improves dysmenorrhea, abnormal •pregnancy
Traditional: begin Sunday following •act on LH a&nd FSH  no follicular uterine bleeding, acne, hirsutism •undiagnosed vaginal bleeding
onset of menses •works on midcycle LH surge •migraine with aura
•Reduced cancer risk (ovarian and •women at increased risk of
Quickstart: begin any day of cycle DDI: endometrial) *decreases turnover cardiovascular sequelae
*may see more breakthrough bleeding if •Increase/Decrease effectiveness of *uncontrolled DM, HTN, lupus
not at ideal time medication •MSK: improves bone mass, protects •cigarette smoker over 35yo
-analgesics: Tylenol, opioid against osteoporosis •current or prior breast cancer
•Encourage regular routine of taking -Other: warfarin, lamotrigine, •active liver disease
pills same time daily benzos, corticosteroids, Disadvantages/Side Effects:
theophylline, metoprolol •hypercoaguability: MI, CVA Caution:
Missed pills: •Reduced OCP efficacy •cervical dysplasia/CA, breast CA •may cause/worsen HTN or HA
•Single missed dose, high-dose -ABX, anticonvulsants, sedatives •gallstones and gall stasis •may impair quality/quantity of
monophasic: conception unlikely •increased fluid retention breast milk
•increased TG
•Multiple missed or missed dose of •diabetes mellitus
lower-dose pills: double next dose & •nausea, dizzy, weight gain, decreaed
add barrier contraceptive for 7d libido, abnormal menses

•Any missed pill + coitus in past 5d:


consider emergency contraception

Progestin •Norethindrone (pill) •thickens cervical mucus Benefits: CI:


Only •thins endometrium, suppress ovulation •safe in women who cant take estrogen •unexplained bleeding
•Small quantity of progestin alone •safe during lactation •breast cancer
 does not suppress ovulation Ideal candidates: smokers >35, HTN, •reduces risk of endometrial cancer •hepatic neoplasms
migraines, breast feed, sickle cell, lupus •pregnancy
•Take at the same time every day Disadvantage: •active liver disease
•must take SAME TIME daily
•higher incidence of irregular bleeding
•higher overall pregnancy rate

Emergency About Yupze Plan B Copper IUD


Emergency •Preventive contraception after •COCs with Levonorgestrl •2 doses of 750us 12 hours apart •may inhibit implantation or
Contraception unprotected intercourse or failure to use •100ug ethinyl estradiol and 500-600ug 1st dose-recommended within 72hrs of interfere with sperm function
a contraceptive method properly levonorgestrel intercourse •insert up to 7 days from time of
•2 doses, 12 hours apart intercourse
3 methods: *ideal within 72 hours •1st dose within 72h of intercourse •may be effective as long as 5 days *good for patient that missed
•Yupze method *can do with your birth control after intercourse their 72 hours window
•Levonorgestrel (Plan B) *can leave in after and use as a
•Copper IUD (most effective) MOA: inhibits or delays ovulation •single dose of 1500ug may be as contraceptive method
effective as 2 dose
•Will not prevent contraception later in SE: nausea & vomit •most effective method of
cycle •pre-medicate with antiemetic emergency contraception

IUDs Administration/MOA Side Effects Contraindications


Copper IUD •Lifespan ~10 years Risks/SE: Contraindications:
“ParaGard” •PID, ectopic pregnancy •intrauterine contents: pregnancy or suspected pregnancy,
MOA: spermicidal, interfere with ovum •spontaneous abortion previously displaced, distorted uterine cavity (fibroids)
development or fertilization, cause inflammation •uterine perforation
of endometrium •expulsion •infections: PID, postpartum or postabortal endometriosis or septic
abortion, mucopurlent cervicitis
Failure: 0.6% per year, typical 0.8% per year Minor SE: menstrual irregular,
cramp, vaginitis •uterine or cervical cner: genital bleeding
•wilson disease
Levonorgesterl •LNG-20 Intrauterine device 52mg Risks/SE: •Liletta: similar but approved for 3 years
IUD “Mirena” •lifespan ~5 years •ectopic pregnancy
•PID •Skyla: lower dose (13.5mg), smaller device size, smaller tube size
MOA: thins endometrium, thickens cervical •spontaneous abortion for insertion  good for small uterus, low expulsion rate
mucus, decreased tubal motility •uterine perforation -duration: 3yrs, NOT approved for heavy flow, dysmenorrhea
•Irregular menses 1st 3-4 months •expulsion
•after: significant decrease in menstrual flow, •Kyleena: lower dose (19.5mg), last ~5 years, smaller size
dysmenorrhea tends to improve Minor SE: -not approved to treat heavy menses or dysmenorrhea
•irregular menses, HA
Failure: 0.1% per year, 0.7% after 5 years •acne, mastalgia Contraindications: Acute PID unless there has been a subsewuent
intrauterine pregnancy, beast cancer, acute liver disease
Barrier/Physical
Spermicides •active ingredient: nonoxynol-9 or octoxynol-9 •Must be placed high in vagina, in contact •High pregnancy rate due to inconsistent use
•physical barrier and chemical spermicidal with cervix, short before intercourse -do NOT protect from STDs
•max duration: 1 hour, avoid douching for -can cause local inflammation
at least 6 hours

Sponges •Nonoxynol-9-impregnated polyurethane disc •Efficacy unchanged by frequency of sex


-inserted up to 24 hours prior ot intercourse •More convenient but less effective than
-must remain in place for 6 hours after coitus diaphgram or condom

Male condoms •Cover for the penis during coitus •Latex-may be polyurethane or lamb ceca Advantages:
-prevents deposition of semen in vagina •lambs cecum not impermeable to most •highly effective, inexpensive, protect againt STIs, may
-MC used mechanical contraceptive organisms have spermicides

•should be recommended for ALL couples Failure: imperfections, errors, semen escape

Female condom •Thin polyurethane with 2 flexible rings Advantages: Disadvantages


-one ring: depth of vagina •under control of female partner •relatively expensive
-one ring: near the introitus •some protection against STDs •overall bulky

Diaphragm and •Circular rubber dome supported by a metal spring •Position so that the cervix, vaginal fornices, •Trial and error to find correct size
Spermicide and anterior vaginal wall are partitioned -too small: ineffective, too large: uncomfortable
•MUST USE WITH SPERMICIDE-cervical side from the ramined of vagina and penis -weight changes can change size needed
-ineffective without spermicide •place up to 6 hours before intercourse
•leave in place 6-24 hours after SE: vaginal irritation, increased UTIs
•Mechanical barrier between vagina and cervix -some protection against STIs

Cervical Cap •Small, cuplike diaphragm placed over cervix •Similar efficacy to diaphragm Disadvantages:
-held in place by suction •leave in place for 8-48 hours after sex •difficult to fit cap properly
-must fit tightly over cervix •confirm placement over cervix after each •may use for 1-2 days, but foul discharge usually
-may be used with spermicide sexual act develops after 1 days

Elective Abortion
Medical:
- Mifepristone (Mifiprex)  Misoprostol (Cytotec) 24-48h after *safe up to 10 WEEKS
o Mifeprostone: progesterone receptor antagonist (dilation and softening of cervix, placental separation)
o Misoprostol: prostaglandin E1 analog (uterine contractions)
- Methotrexate  Misoprostol 3-7days later *safe up to 7 WEEKS
o Methotrexate: folate antagonist
Intra-Amniotic: induced abortion after 1st trimester
Surgical: can be done up to 24 weeks
- Dilation and Currettage (D&C): includes usage of curette or suction *4-12 weeks gestation
- Dilation and evacuation (D&E): >12 weeks
Post-Abortion: rhoGAM to all RH (-) pts; avoid intercourse, tmapons, douches, intra-vaginal products for 2 weeks
- 2+ procedures can increase risk of mid-pregnancy loss

Menopaus Definitions Hormone/HPO Axis Change Hormones Clinical Presentation


e
Menopaus •Cessation of menses for >1 years due •loss of oocytes as ovaries age due to ANDROGRENS Menstrual: irregular, short/long
e to loss of ovarian function  low E & P ovulation and atresia •less production of androstenidone
-7 million at 20 weeks (primary androgen in women) Somatic: HA, dizzy, palpitations, breast
Risks: -1-2 million at birth •decreased levels of testosterone pain/enlargement, joint pain
•smoking (advanced age by 2 years) -300.000-500,000 by puberty *less sex hormone-binding globulin
•reproductive tract disease -400-500 ovulated  more active floating around Psych/Cognitive: sleep, depression,
•GU infections or tumor irritable, mood swings, memory
•chemo, readiation, surgical procedures •as ovaries lose oocytes, levels of ESTROGEN: *draw estradiol
•ovary blood supply, endocrine, genetic inhibin slowly decrease level Sex: vaginal dryness, low libido
(less inhibin  increase FSH) •reduced endogenous estrogen *vaginal atrophy due to low E
Stages: *inhibin inhibits FSH •greatest decrease in estradiol
Climacteric: phase when woman passes *primarily secreted from adrenal vasomotor: hot flashes, sweats
from reproductive to nonreproductive •Oocytes responsive to gonadotropins •estrone levels fall, but not as much
“climacteric sx or complaints” disappear from the ovary over time *estrone is predominant estrogen Other: incontinence, dry/itchy skin, hair
-remaining oocytes: less after menopause loss, hirsutism, wt gain, decreased bone
Menopausal Transition: climacteric responsive to FSH and LH density, decreased breast size
phase before menopause when -irregular follicle response to PROGESTERONE
menstrual cycle is irregular gonadotropins  irregular •after menopause, no functional Reproductive Changes:
(“perimenopause”) length of follicular phase  follicles  no CL  low P •vagina: thin, atrophy, flat ruggae
•lasts about 1-3 years irregular menstrual cycles •adrenal glands produce remaining •cervix: atrophy, small, less mucous
•NO clinical use in diagnosis •uterus: atrophy, shrink
Menopause: final menstrual cycle Artificial Menopause •oviducts: decrease in size
•Avg age: 51, premature: ≤40 •permanent cessation of function due to GONADOTROPINS *BEST •ovaries: atrophy, smaller, less
surgical removal of ovaries or radiation •FSH and LH RISE (FSH >LH) hormones, no ovulation
Postmenopausal: after menopause •measure FSH, LH, estradiol to help •support: loss of tone, relaxation
•most women live ½ life here diagnose menopause
*can measure FSH & LH with Urinary: estrogen maintain epithelium of
estradiol (FSH and LH increase bladder and urethra  atrophy
because no negative feedback from •atrophic cystitis: UTI sx
estrogen and progesterone) •lose urethral tone(urethral caruncle)

Menopaus About/Clinical Presentation Treatment


e
Atrophic Atrophic epithelium with flattened rugae Initial Treatment: Other options:
Vaginitis •FIRST LINE: vaginal moisturizer •Prasterone (vaginal DHEA): 6.5mg suppository
•loss of lactobacillus which converts glucose to lactic *improve symptoms but not atrophy -converts androstenedione and T  estrone and estradiol
acid  increase in vaginal pH to 5-7 -Replens, vagisil, K-Y Liquibeads
-Lubricants with sexual activity •Ospemifene (Osphena): SERM (mimics E) 60mg daily
S/S: Water-based (Astroglide, K-Y), -MC SE: hot flashes, mild endometrial thickening
•vaginal burning, soreness Silicone-based (Pjur), Oil (Elegenace)
•dyspareunia *caution with oil due to condom break •Testosterone: 1-2% cream is estrogen is CI
•dryness or thin water or serious discharge •Pelvic PT: refractory cases
-early: diffuse or patchy red, +/- petechiae, flat ruggae Mod/Severe: Topical Vaginal Estrogen •laser, vitamin, probiotics *mixed data about all of these
-late: smooth, shiny, pale *even okay if she has a uterus
-friable: bleeding w/ minimal traum •restored pH and microflora, increased Overall: VAGINAL preferred over systemic
-GU: urgency, frequency, dysuria, incontinence vaginal secretions, thickened epithelium, -cream: conjugated estrogen or estradiol
diminished overactive bladder, fewer UTIs -rings: estradiol 7.5mcg releasing x3mo
DX: clinical *may do vaginal cytology to assist •systemically absorbed -tablet: estradiol 4mcg, 10mcg
About/Clinical Presentation Treatment (HOMRONAL) Treatment (Non-hormonal)
Hot MC AND CHARACTERISTIC SX Mainstay: ESTROGENS *transdermal 1st •SSRI/SNRI: *1st line who cant/don’t want HRT
Flashes •alterations in cutaneous vasodilations, perspirations, -block sx and physiologic changes -cautions with SSRI & Tamoxifen (esp Paroxetine)
reductions in core temp, high HR -E/P combo: if can take E -increased risk of breast CA or death
-increase breast CA risk with P added
•heat or burning in face, neck, chest, back •Black Cohosh or Phytoestrogens
- HA pressure  flush  sweat •Progestin alone: women cant take E -can stimulate breast and uterine tissue
•palpitations, weak, fatigue, faint, vertigo •Tibolone: synthetic steroid with E, P, A •Gabapentin: SE: sedation
•Bioidentical hormones: $$$$ •Clonidine: more effective than placebo (vasodilation)
duration: seconds-10min (avg 4 min) -custom-made hormone •Weight loss, acupuncture, mind/body, vitamin E
frequency: 1-2/hr to 1-2x/wk •SERM + Estrogen

Benefits Risks Main Treatment Other Options


HRT Known: •endometrial CA: hyperplasia within 1yr 1st line: transdermal (decrease SE) Other Forms:
•reduced SX (vasomotor, GU) -unopposed E  proliferation, hyperplasia, •0.625mg PO conjugated estrogen •Progesterone only: *better than SNRI
*vaginal estrogen is as efficaious as neoplasia; give Progesterone to decrease •increase dose at 1 month intervals depot MPA IM or po norethindrone
oral or transderm for GU symptoms -more effective than SNRI
•breast CA: only in COMBO (due to P) •must add PROGESTIN if patient
•reduced risk of osteoporosis: -risks: early menarche, late menopause has an intact uterus (no E alone) •Tissue Selective Estrogen Complex
improves density, reduce fracture -medoxyprogesterone acetate(MPA) SERM (Duavee or BZA/BE) + Estrogen
*benefit from E •thromboembolic: 2x w/ combo, 33% E only -micronized P: lower risk of breast -agonist bone; antagonist endometrium
-lower chance with transdermal cancer and CHD -neutral on breast
Possible -lower risk breast & endometrial CA
•improved skin collagen & thickness •stroke: increased risk in E only and combo Regimens: SE: VTE, liver disease
•reduced UTI -lower incidence with transdermal v oral NOT REC. for >3-4yr
•reduced falls •PO Estrogen + Levonorgestrel IUD
•reduced cataracts, osteoarthritis, DM •gallbladder disease: greater risk with E-only Regimen One: *older method -avoid systemic effects of P but prevent
•reduced colon CA (combo therapy) •Estrogen on days 1-25 endometrial hyperplasisa and cancer
•reduced CHD (cholesterol clearance) Other Effects *menstrual cycle symptoms •progesterone days 14-25
•other: edema, bloat, mastodynia & breast •withhold both day 26-end of month •SSRI: Citalopram, Escitalopram
Lipids enlargement, PMS, HA, lots cervical mucus •light, painless period each month -Paroxetine: caution with Tamoxifen
lipids: lower LDL, higher HDL
but can increase TG CI: Regimen Two: *most common •SNRI: Venlafaxine, Desvenlafaxine
•breast & E-dependent CA *endometrial, •daily E & P together w/o stop SE: insomnia
undiagnosed abdominal vaginal bleeding •initial bleeding or spotting
•thromboembolism, liver, hypersensivity •eventually: atrophic endometrium •Anticonvulsant: Gabapentin,
•pregnancy
•Clonidine: helpful with comorbid HTN
Caution: gallbladder, cholestatic jaundice, Only use HRT for vaginal and menospausal s/s
high TG, hypothyroid, flui with cardiac/renal, atrophy and hot flashes
hypocalcemia, endometriosis, hepatic CAM: black cohosh, isflavones, Vit E,
hemangioma exercise, weight loss, relaxation

OBGYN About Causes Diagnostics Treatment


Dysfunctional •abnormal menstrual bleeding and Abnormal bleeding with relatively normal •beta-hCG Acute Hemorrhage:
Uterine bleeding due to underlying causes physical exam •CBC: H/H •IV high dose estrogen
Bleeding (Premarin)
Anovulatory: ovaries produce estrogen •mass or enlarged irregular uterus: Endometrial Biopsy: •observation, COCs
but no ovulation *no corpus luteum leiomyoma •rule out cancer in all women >35 wih •refractory: IUD
•unopposed estrogen leads to obesity, HTN, or DM with •definitive: hysterectomy
endomertrial growth & shedding •symmetrically enlarged uterus: postmenopausal bleeding
adenomyosis, endometrial cancer Postmenopausal:
•exogenous hormones
•atrophy: lubricants

Dysmenorrhe •painful menstruation that affects •recurrent, crampy midline lower abdominal Supportive:
a normal activities pain or pelvic pain 1-2 days before or at •heat compress
onset of menses •vitamin B, E
Primary: due to increased •diminished over 12-72 hours
prostaglandins NSAIDs/COCs:
•first line medical manage
Secondary: due to pelvis or uterus
pathology (endometriosis, PID, Laparoscopy: unresponsive to
adenomyosis, leiomyomas) 3 cycles of initial therapy

OBGYN About Work-Up Care and Disposition


Spontaneous Threatened: *only one potentially viable Pelvic Exam hemodynamically unstable: Complete Abortion:
Abortion •products of conception intact •caution in patients beyond 20 -emergent consult •may be discharged home if stable and
•cervical os closed weeks because risk of placentia -fluid and blood vaginal bleeding is decreasing or absent
Pregnancy previa -Rhogam if RH (-) •pelvic rest
that ends <20 Inevitable: •close follow up with OB
WEEKS •products of conception intact Labs: Types •Rhogam if indicated
gestation •bleding, cramps, os dialted •CBC Threatened Abortion:
•blood typing and crossmatch •pelvic rest Setpic abortion
Incomplete: •Rh factor •may be discharge home with •obtain CBC, blood and uterin cx, LFTs,
•some products expelled •B-hcg close follow up with OB BMP, PT/PTT
•bleeding, cramping, os dilated •Urinalysis •return to ED if significant pain, •OB/GYN consult and asmiddion
fever, vaginal bleeding, passage •Ampicilling/Sulbactam
Complete: Ultrasound of tissue •Clindamycin + Gent
•products of conception expelled transabdominal or transvaginal
•cervical os closed •may confirm fetal death by Inevitable: Incomplete/Missed:
noting evidence of fetus with no •surgical evacuation (D&C) •surgical evacuation (D&C)
Missed: fetal death <20 wks, brownish heartbeat or movement •Misoprostol •Misoprostol
discharge, no fetal tissue is passed •expectant management •expectant management
•US should sac with no heart beat

Septic Abortion MCC incomplete


evacuation
•some products of conception retained
•cervical os closed; cervical tenderness
•brown, foul discharge; fever, chills

OBGYN About Clinical Presentation Diagnostics Treatment


PID and •MC serious infection occurring Clinical Features Labs: empiric treatment for PID:
Tubo- in reproductive-aged women •lower abdominal pain •pregnancy test *r/o ectopic •sexually active women or other women at risk for
Ovarian •occurs when infection ascends •vaginal discharge & bleed, dyspareunia, •wet prep, swab for G/C STDS if they have pelvic pain + no other cause
Abscess from the lower genital tract urinary discomfort, fever, N/V •CBC, ESR, CRP for illness, plus ONE:
•peritoneal symptoms or minimal sx •pelvic US -cervical motion, uterine, or adnexal tenderness
•MCC: Neisseria gonorrhea, •RUQ pain with jaundice
Chlamydia trachomatis (MC)  Fitz-Hugh Curtis Admission criteria Inpatient Treatment options
•failure to respond to outpt tx •Cefotetan or Cefoxitin + Doxy
Risks: pelvic exam: •pregnancy and adolescents •Clindamycin + Gentamicin
•multiple partners, STDs •uterine, adnexal tenderness and/or *adolescents risky because Alternative: Augmentin + Doxycycline
•sexual abuse cervical motion tenderness transformation zone on cervix
•IUD “Chanelier sign” •toxic Oral and Outpaitent Regiments
•douching •cant tolerate oral •Ceftriaxone + Doxy +/- Flagyl
•substance abuse •cant exclude other dx •Cefoxitin + Doxy +/- Flagyl
•adolescents •3rd gen ceph + Doxy +/- Flagyl

OBGYN About Clinical Presentation Diagnostic Treatment


Candidal Bug: Candida albicans •intense vulvar pruritus •Vaginal pH: elevated 4-5 Pharm: 1-3d topical, 1d Fluconazole
Vulvovaginitis •thick, white cottage cheese discharge •topical or oral antifungals
Risks: DM, HIV, obese, prego, •minimal odor •saline prep: 1 drop discharge: 1 NS •boric acid or gentian violet
ABX, steroids, OCPs •vulvar erythema, edema -branching filaments,pseudohyphage
•burning sensation after peeing Complicated:
•KOH: 1 drop 10% KOH •4+ episodes/yr, severe symptoms, non-
-dissolves epithelial cells & debris albicans, uncontrolled DM, HIV,
steroid, pregnancy
•Culture: GOLD STANDARD -2 doses of fluconazole

Bacterial •overgrowth of abnormal flora •copious, thin, homogenous, gray-white •pH: elevated 5.5-7 (ELEVATED) •Metronidazole (Flagyl) x7d
Vaginosis •decreased lactobacillus vaginal discharge •Clindamycin (Cleocin) x3-7d
*NOT STI •itching, burning, dyspareunia •saline prep: “Clue Cells” •Tinidazoel (Tindamax) x3-7d
Bug: Gardnerella vaginilis *worse after unprotected sex •KOH: (+) “whiff test” fishy odor
•“fishy” smell, enhanced after KOH prep Prevention: probiotics, acid douche

Exam: no mucosal inflammation


Trichomonal •unicellular flagellate protozoan •profuse extremely frothy, greenish, at •pH: elevated 5-5.5 (ELEVATED) •Metronidazole or Tinidazole x1dose
Vaginitis times foul-smelling vaginal discharge -or Metronidazole 500mg BID x7d
•MC non-viral STD in US •saline prep: motile trichomonads
•perinatal complications, •erythema, petechial: strawberry cervix •nucleic acid: 45 minutes Resistant: Tinidazole 500 TID x 7d
increased HIV transmission •pap smears, culture •treat partner & screen for other STIs

Gonorrheal •MC infects glands of cervix, •80-85% of women asymptomatic •nuclei probe or culture of discharge •single IM Ceftriaxone 250mg
urethra, vulva, perineum, anus •copious mucopurulent discharge possible -gram (-) diplococci •Chlamydia TX: Azithromcyin 1g

Chlamydial •may see mucopurulent cervicitis, •culture, immunoassay, nucleic acid •Azithromcyin 1g po once
dysuria, and/or postcoital bleed •can be found on pap smear •Doxycyline 100mg po DIB x 7 days
•treat partner

OBGYN About MOA SE/DDI


Vaginal •less systemic risk; rapid relief •inhibit enzyme for membrane synthesis SE: burn, itch, swell, rash, discharge
Antifungal •weaken latex (caution) •Nystatin: increase permeability of wall
DDI: rarely Warfarin
Oral •higher systemic risk •inhibits enzyme for cell membrane synthesis SE: GI, abd pain, dizzy, HA, drowsy, allergy
Antifungal •delayed relief
(Fluconazole) •can NOT use in 1st T prego DDI: erythro, clopidogrel, warfarin, theophylline, sulfonylureas,
thiazides, rifampin, cimetidine *avoid with hepatotoxic drugs

Nitromidazole bind to and deactivates enzymes SE: dizziness, HA, false lab results •alcohol (3d), Disfulfiram (2 wks), anticaogulatns, phenytoin,
-GU: dark colored urine, irritation lithium
-GI: pain, upset, dry mouth, glossitis, altered taste
-Rare: neurotoxic, anaphylaxis, SS

Clindamycin Binds to ribosomes blocking protein SE: C. diff, local irritation, abd pain, GI, latered tast •macrolides, neuromuscular drugs, antiperistaltis drugs
synthesis Rare: blood dyscrasis, hepatotoxic, anaphylaxis

Gravida: number of times a woman have been pregnant


Parity: Number of pregnancies that led to a birth either at or after 20 weeks
- T (term)  number born at 37 weeks or older
- P (preterm)  born after 20 weeks but before 37 weeks
- A (abortion)  all pregnancy losses prior to 20 weeks
- L (living)  infant who lives beyond 30 days

Nulligravida: woman who currently is not pregnant and never has been pregnant
Primigravida: woman who currently is pregnant and has never been pregnant before
Multigravida: woman who currently is pregnant and who has been pregnant before
Nullipara: woman who has never completed a pregnancy beyond 20 weeks
Primipara: woman who has delivered a fetus or fetuses born alive or dead with an estimate length of gestation of >20weeks
Multipara: woman who completed 2+ pregnancies to 20 weeks gestation or mo
Fundal Height: 12 weeks (above pubic symphysis), 16 weeks (midway between pubis & umbilicus), 20 weeks (umbilicus), 38 weeks (2-3cm below xiphoid

Pregnancy Signs/Symptoms Diagnostic Tests


Pregnancy Amenorrhea Breast Changes Beta human chorionic gonadotropin (B-hCG) Transvaginal US
•cessation of menses in a health •tenderness & paresthesia •produced by syncytiotrophoblasts gestational sac: small anechoic
•not reliable until 10d + after •increase in breast & nipple •blood and urine about 8-9d after ovulation fluid within endometrial cavity
menses •thick yellowish fluid  •prevents involution of corpus lutem  first evidence ~4-5wk
•can have “implantation bleeding” colostrum •rare false (-): MCC is herephilic antibodies yolk sac: bright echogenic ring
for blastocyst •areola becomes deeply pigmented *urine would be negative with anechoic center
•false (+) causes: exogenous hCG, renal failure  confirms location ~5-
Vaginal/Cervical Changes Skin Changes ~16th wk with failed hCG clearance, pituitary hCG, tumors 6wk
•Chadwick: mucosa appears dark •increased pigmentation, linea *home pregnancy tests require value of 12.3 to fetal pole/embryo: ~6wk
bluish red & congested nigra detect 95% pregnancies  use 1st urination crown rump length: used up to
•Goodells sign: cervical softening •pruritic papules  steroids 12wk to predict the due date,
•chloasma  “mask” accurate within 4d
•cervical mucus: thick due to P •striae (stretch marks)

•uterine change: Fetal Movement 16-20 weeks


•Haegers sign: isthmus soften (6-8) •Primigravida ~20wk
•Ladin sign: uterus softens (6wk) •Multigravida ~16-18wk

Heart Tones: 10-12 weeks


Estimate Date of Delivery Prenatal Visit Lab Tests RhoGam
•Naegele’s Rule History First Visit MOA: suppresses immune
-LMP + 7days – 3 months •obstetrical hx: pregnancies, •CBC: WBC, Hgb, Hct, Plt response of Rh negative
-assumes pregnancy begun complications, infertility *monitor anemia & thrombocytopenia individuals to Rh (+) RBCs
2wk before ovulation •menstrual: interval between
menses, contraceptives Blood type and Rh Factor: if Rh (-) mother and Rh Dose: 0.3mg eradicates 15ml of
•Ultrasound •fhx, shx, medical hx (+) fetus then receive RhoGam ~28thwk *RhD fetal RBC
 1st trimester crown rump •antibody screening
is MOST ACCURATE Physical Exam Should be given
•uterine size *bimanual rubella serology: infection in 1st semester can -bleeding or trauma prior to 28wk
-small orange ~6wk cause abortion, malformations -postpartum if infant Rh (+)
-large orange ~8wk -invasive diagnostic prenatal test
-grapefruit ~12wk •pap smear
•cervical dilation, length, •gonococcal & chlamydia, syphilis, HIV, hepatitis Keihauer-Betke
consistency *bimanual •urinalysis •tests amount of fetal RBC in
•pelvic architecture *bimanual •urine culture & sensitivity maternal circulation

Counseling
Prenatal Vitamin Diet: additional 100-300kcal/day Seat Belt Smoking: person to person counsel
•400 micrograms of folic acid - 60-80g protein/day three point restraints with lap belt under •ask, advise, assess, assist, arrange
*4mg if hx neural tube defect abdomen and across upper thigh
Seafood Alcohol: MC in white; NO ALCOHOL
Employment •avoid fish and shellfish contain mercury Air Travel
•work until labor •High risk: shark, sword, mackerel, tile fish can fly up to 36wk, ambulate hourly and Illicit Drugs: less likely to obtain prenatal care
•no more than 6oz of white tuna wear TED hose
Weight Gain Breastfeeding
•Maternal weight gain is correlated Lead Dental: no CI including x-rays •exclusive BF preferred until 6mo
with infant weight Risk factors: •human milk contains protective
*obese women who gain <15lb have -recent immigrant Coitus (intercourse): not harmful immunological substances  IgA and GF
the lowest complication rate -living near lead source •protects against rotavirus, decreased atopic
*25-35lbs, underweight: 28-40lbs, -using lead glazed pottery Caffeine: >5c can increase abortion, dermatitis and resp infections
obese: gain around 12-20lbs -eating nonfood substances <200mg (10oz) not associated with issues
-using imported cosmetics CI: street drugs, uncontrolled alcohol,
Obesity associated risks: -remodeling home with lead Exercise galactosemia, HIV, TB, medications, breast
•gestational HTN, DM, preeclampsia -consuming lead water •do not limit; engage in moderately intense cancer treatment, active herpes on breast
•macrosomia -living w/ someone with elevated lead activity for 30min+/day; *don’t lift >25lb
•C-section •avoid: high risk falling or trauma & scuba
Routine Care Common Complaints
•Every 4 weeks until 28 weeks Nausea/Vomiting “morning sickness” Hemorrhoids Heartburn *very common
•Every 2 weeks until 36 weeks •common in 1st trimester until 16th wk •increase in pelvic venous pressure, rectal •GERD in lower esophagus due to upward
•Every week unil delivery TX: small meals, BRAT diet, ginger, vein varicositis occur displacement & compression of stomach
vitamin B6 + Doxylamine, Zofran •pain and swelling occur TX: antacids, H2 blockers, PPIs
TX: topical applied anestheics, warm soaks,
Prenatal Surveillance •Hyperemesis gravidum: vomiting severe that stool softeners Pica: craving for strange food  iron def.
dehydration, electrolyte, and acid-base
At each return visit: disturbances and starvation ketosis Varicosities Sleeping & Fatigue:
•US fetal growth (dating & anatomy) •hypokalemic, hypochloremic metabolic •femoral venous pressure in pregnant woman •efficiency declines with gestational age,
•maternal BP/weight alkalosis increased 8  24mmHg at term increase need
•U/A, protein, glucose •cosmetic blemished to severe discomfort TX: Benadryl, daytime naps
•S/S: leak, contractions, N/V, bleed Back Pain *low back pain TX: rest with leg elevation, elastic stocking
•increased with gestational age Leukorrhea: increase discharge
•Reduce By: squatting rather then bending,
avoid high heels, use a pillow back support,
belly band, PT, Tylenol prn

Prental Prenatal Diagnosis Down Syndrome & Trisomy


Prenatal Chromosomal Abnormalities: First Trimester Screening 11-14 weeks Second Trimester: 13-27 weeks Combine First & Second Trimester
Diagnosis •Risk of fetal trisomy increased with •nuchal translucency: max thickness of the •enhance aneuploidy detection rates
maternal age, especially after age subcutaneous translucent area b/w the skin Triple test •combines results from both trimesters
35 and soft tissue of spine at the back of neck, •hCG, AFP, & unconjugated estriol •highest detection of down syndrome
>3.5 is abnormal  targeted sonography, •down syndrome: •results after both completed
•Women with fhx or personal history chorionic villous sampling, aminocentesis low AFP & estriol, high hCG
of aneuploidy should be referred to •trisomy 18: all decreased Third Trimester:
genetic counseling •Biochemical screening: beta-hCG, PAPP- •spina bifida: high AFP •Gestational Diabetes: 24-28 weeks
A  1 hour glucola testing
•Downs: elevated hCG & low PAPP-A Quad Screen
•Trisomy 13 & 18: both low •above + inhibin •antibody screen in Rh (-): 28wk
•Downs: elevated inhibin *give RHOgam if negative
•ultrasound: heart tones around 10-12week
Gestational Diabetes: week 24-48 •CBC repeat: ~28wk, monitor anemia
•Chorionic Villous Sampling: 10-13 •UDS repeat if + initial: 28 week
weeks women with increased chromosomal Aminocentesis: same as chorionic •GBS: vaginal/anal culture 36-37wk
abnormalities, prior child with  IV PCN G at labor
chromosomal abnormalities, materal age *alternative: Clindaycin
>35, abnormal 1st or 2nd T maternal
screening tests, abnormal nuchal
translucency, prior pregnancy losses
Neural Tube Defects Non-Stress Testing:
Birth defects of brain: Risk Factors: Types of Spina Bifida: Baseline fetal HR 120-160bpm
•anencephaly •family history, MTHFR mutation •myelomeningocele (MC): Reactive: 2+ accelerations at rate 15+
*failure to close neural tube that •aneuploidy, diabetes, hyperthermia meninges and spinal cord herniate bpm from baseline for at 15 seconds
becomes cerebrum •medications *seizure medications through gap in vertebrae
•spina bifida Nonreactive: no fetal HR accelerations
*incompetent closure of embryonic Screening: mom serum AFP b/w 15-20 wk •occulta: no herniation; overlying or <15bpm lasting <15 seconds
neural tubule  vertebrae skin may be normal or hair; •sleeping, immature, compromised
•cephalocele AFP can be affected by: dimling or birth mark •TX: vibratory stimulation, contraction
•rare spinal fusion abnormalities •multifetal gestation stress testing
•pilonidal cyst •meningocele: only meninges
•elevated AFP in maternal and •chorioangioma of placenta herniate through gap Contraction Stress Testing:
fetal serum and amniotic fluid •placental abruption, preeclampsia Negative: no late decelerations
•AFP synthesized by fetal yolk sac •oligohydramnios, FGR Positive: repetive late decelerations
 hospitalize for fetal monitoring

MSK 8%

MSK About Presentation Diagnostics Treatment


Ankle MC lateral ligament: anterior •pain, swelling, bruising •anterior drawer test •initially: NSAIDs, RICE, protect ankle,
Sprain talofibular and calcaneofibular •report a pop •squeeze test: squeeze tib/fib midcalf brace if need
•results from inversion (rolling ankle) •pain with varus stress and  (+) pain
anterior drawer •dorsiflex and external rotation •2nd phase: when pt can bear weight
Medial liagments: deltoid injuries without increase pain or swelling (~2-
result in eversion injury X-Ray: Ottawa Rules  Ankle 4wk), use braces, start exercises
•pain along lateral malleolus
•pain along medial malleolus •3rd phase: ~4-6weeks, functional
•inability to walk >4 steps conditioning and strength, wean off brace

Achilles •inflammation of Achilles tendon •pain/stiffness 2-6cm above the •clinical •RICE, MICE, NSAIDs
Tendinitis •MC in people >30yo posterior calcaneus *burning pain •heel pads
•pain relieved with rest •exercise programs
Risks: run, jump, speed change i •steroids (last resort)
Cause: overuse

Achilles •5-7cm proximal to insertion of •sudden, severe sharp calf pain •Thompson: absensce of plantarflex •Initial: RICE, non-wt bearing, urgent
Rupture tendon on the calcaneous -“gunshot wound” “direct hit” ortho referral
•quick, stop and go sports like tennis •swelling of lower calf •calf squeeze test
or basketball (kobe!) •difficult bearing weight -if foot moves  incomplete rupture Nonsurgical: short leg plantarflex cast
•palpable defect -does NOT move foot  full rupture or functional brace with heel lift
Mechanism: mechanical overload
from eccentric contraction of •MRI evaluates severity Surgical: reconnects tendon,
gastrocsoleus complex

Risks: Fluoroquinalones, steroids

Plantar •Inflammation and tears of plantar •heel pain worse when initiating Exam: NON-SURGICAL
Fasciitis fascia due to overuse walking (most severe in 1st step in •tenderness with pressure over plantar •OTC orthotic heel pad & home
•common cause of heel pain (medial morning or after long activity) medial calcaneal tuberosity and fascia stretching
tuberosity at calcaneous) •resolve with walk but worsens -dorsiflex toes & palpate •avoid barefoot, flat shoes, activities
towards end of day •ice and NSAIDs
risks: obese, old, female flat feet
•relieved by sitting Persistent steroid injection, non-
removable cast, night splint, surgery if
no relief 6-12mo

MSK About Clinical Manifestation Diagnostics Management


Costochondriti •acute inflammation of the costal •pleuritic chest pain (worse with inspiration) Usually all normal •NSAIDs
s cartilages or costochondral junctions •worsened by inspiration, cough, movement

Causes: idiopathic, posttraumatic Exam:


•reproducible point chest wall tenderness
•MC in 3rd, 4th, 5th ICS •absence of palpable edema

Knee About Presentation Exam and Diagnostics Treatment


ACL Tear •MC knee ligament injury •sudden pain and giving away of the knee •Lachman *MOST SENSITIVE •rest, ice, crutches, PT, compression
•MC in female athletes •audible “pop” and swelling •anterior drawer *wrost •possible aspiration for effusions
•knee buckling and inability to bear weight •pivot shift test
Mechanism: •possible hemarthrosis Definitive
•noncontact pivoting •segond FX is pathologic for ACL tear x-ray: AP, lateral, tunnel views •ACL reconstruction is best for
•deceleration, change position young, active pts
•hyperextension, internal rotation Unhappy (O’Donoghue) Triad: MRI: best for confirm •older or less activity  PT
•ACL + MCL + medial meniscus

PCL Tear •strongest ligament in the knee •posterior knee pain •posterior drawer •isolated PCL treated nonsurgically
•anterior bruising *most sensitive -rest, ice, ROM exercsies
Mechanism: •effusion
•dashboard: anterior force to x-ray: AP and lateral surgical reconstruction: fail above
proximal tibia with knee flex *ACL may rupture first then PCL MRI: confirm
•direct blow/fall on flexed knee

Collateral •outside the joint and stabilize the •minor ambulatory issues (most play on) Varus and valgus stress Grade I (sprain) and II (incomplete)
Ligament Tear knee against valgus (abduction) •localized swelling, stiffness •LCL: varus stress •conserative: PT, RICE, NSAIDs
(MCL and LCL) and varus (adduction) stresses •tenderness along MCL or LCL *medial trauma •early ROM and quad strengthening
•ecchymosis and effusion in 24h •MCL: valgus stress
*lateral trauma Grades III (complete): +/- surgery

Meniscal Tears •fibrocartilaginous pads that •swelling, stiffness after activities •tender to medial/lateral joint •RICE, NSAIDs
function as shock absorbers •locking, catching, popping •postive McMurrays test
•patients can ambulate and play sports Surgery: traumatic tears in younger
Mechanism: •pain on medial/lateral side w/ twist or squat x-ray: AP/lateral, sunrise patients, older patients who do not
•degenerative or acute respond to conservative
•axial loading and rotation MRI: BEST

Hand About Presentation Diagnostics Treatment


Ganglia of •cystic structure arises from capsule of •pain, tenderness, especially with •PA, lateral and oblique xray should be Wrist ganglion:
the Wrist joint or a tendon synovial sheath movement obtained to rule out bony pathology •Typical: reassurance
and Hand •MC soft-tissue tumor of hand •atypical: excision and biopsy
•Age 15 to 40 years is MC Physical Exam •Surgery if significant sx or bothered
•smooth, round or multilobulated
Common locations structure Finger ganglion: rupture & massage
•dorsum or volar radial of the wrist •A prominent ganglion will
•base of the finger transilluminate

About Signs and Symptoms/PE Tests and Stages Diagnostics and Treatment
Rotator Cuff 4 muscles: SITS •dull ache deep in shoulder Testing: DX:
•supraspinatus *origates here •difficult sleeping on affected shoulder •suprasinpatus: “empty can test” 90% spec •xray
•inraspinatus •difficult reaching overhead, behind back, -seated, elevated UE to 30-45 degree in •MRI: better
•teres minor and even doing normal ADLs plane of scapula with internal rotation thumb
•subscapularis • “catching” or “grating/grinding” down, resist elevation: (+) pain Treatment:
•conservative tx 1st line
Risks: PE: •infraspinatous:“external rotate & lag” -rest, ice, PT, steroids
•repetitive motions *esp •ROM: active motion limited -maintain position of maximal rotation
overhead •atrophy of infraspinatus if chronic (+) is when lag or angular drop •surgery: arthroscopic, open,
•increase risk with age •cant raise the arm (only shrug) if large tendon transfer, shoulder
•tears due to direct injury •cant hold arm in elevated position •subscapularis: Gerbers test/Lift Off replacement
place their hand behind their back, with the
palm facing out  lift hand away

Labrum Tear •soft tissue that connect glenoid •grinding/catching Impinement Tests:
and humeral head •unstable sensation •Hawkins: anterior pain with int. rotation
•limited ROM •Neer test “like empty can but lift arm
•SLAP tear: superior labrum, above their head”
anterior and posterior •Drop arm: pain with slowly lowering arm
Olecranon •Olecranon Bursa is superficially •Swelling “goose egg” of the posterior •clinical Olecranon Bursitis:
Bursitis located on extensor side of elbow olecranon bursa •avoid further trauma-padding,
“student’s/bake •ROM limited depending on degree •aspiration *septic bursitis or gout
r elbow” Causes: •Possible small tender lump (recur) -WBC <500 (noninfectious) Septic: aspiration & ABX
•Trauma (fall) WBC >2000 (infectious) •Dicloxacillin, Clindamycin
•Inflammation (RA, Gout) Infection, red, tender, warm, painful
•Infection (Septic Bursitis) Admit: persistent or severe

Bicep Tendonitis: Tendonitis: •X-rays to r/o other pathology Tendonitis:


Tendinopathy/ •sports, overhead motion or •pain in anterior shoulder over bicipital •US may be more helpful •NO NSAIDs & streoids
Tendonitis v. lifting groove •MRI to r/o rotator cuff tears •ice, rest, tapping, PT/OT
Bicep Tedon •pain worse with shoulder flexion, •surgery if no improvement
Rupture Rupture: forearn supination, elbow flexion
•fall on outstretched hand Rupture:
•oversuse, heavy lifting, shoulder Rupture: sudden pain in upper arm / •Partial: nonsurgical
tears elbow  “pop” •Complete rupture: surgery
*involves proximal long head •trouble rotating arm palm down or up
• “Popeye” muscle

MSK About Clinical Manifestation Diagnostics Management


Lumbar • MC occur L4-5 or L5-S1 •radicular back pain (UNILATERAL) Xray: age-appropriate changes Acute: NSAIDs
Herniated •possible radiation down leg with -avoid prolonged standing,
Disk Causes: lift & twist *increases pressure paresthesia or numbness along dermatome MRI: CONFIRMS DX sitting, walking
•Exacerbated by sitting, walking, sx > 4 weeks, significant neuro
•Develop over time as weaker standing, coughing, sneezing deficit, or if planned surgery 2nd line: steroid injections
posterolateral portion of annulus fibrosis
develops fissures that permit the Exam: (+) SLR, crossover, limited ROM ALP: Surgery: laminectomy &
protrusion of other disk components L4: anterior thigh, weak ankle discectomy if not resolved in 6wk
dorsiflex, decreased knee jerk
L5: lateral thigh, weak big toe
extension, normal reflexes
S1: posterior leg, weak plantar
flexion, decreased ankle jerk

Vertebral Mechanism: jumping/falling from height •localized back pain with focal midline x-ray: loss of vertebral height •surgery consults
Compression tenderness at fracture level •NSAIDS, bracing
Fracture Risks: elderly (osteoporosis), •surgery: kyloplasty *severe
“burst FX” malignancy, children playing
Cauda •Distal end of spinal cord terminates at •Low back pain  Unilateral or bilateral •MRI: STUDY OF CHOICE •Neurologic emergency
Equina the L1-L2 level and below that is a sciatica •alternative: CT w/ myelography •Requires surgical
Syndrome bundle of nerve roots  cauda equina • “Saddle & perineal decompression within 12-24h
“Horses hypoesthesia/anesthesia”: decreased -imm. consult w/ surgery
Tail” •neuromuscular and urogenital sx sensation of butt, perineum, inner thigh Labs:
resulting from compression of multiple •Bowel and bladder dysfunction •CBC, glucose, BMP, BUN/Cr •NSAIDs and steroids
nerve roots below conus medullairs -urinary retention, bowel incontinence •ESR: elevated •Infection: ABX
-decreased anal sphincter tone •LP to r/o infection & •Neoplasms: Chemo/Radiation
Causes: •Reduced or absent LE reflexes inflammation
•lumbar disc degeneration
•spinal trauma, herniated risk, neoplasms, Physical Exam:
infection, spina bifida, iatrogenic •localized low back TTP
•decreased sensory and motor function
•saddle anesthesia to pin prick, vibration
•DRE: poor anal sphincter tone

Lumbar •narrowing of the spinal canal with •back pain, numbness, paresthesia MRI: TEST OF CHOICE •pain control
Spinal impingement of the nerve roots •worse with extension: standing, walking •corticosteroid infection
Stenosis •relieved with flexion: sitting, leaning
Causes: degenerative, sponylolysis forward, walking uphill, cycling

Spinal Cord •external compression of the spinal cord •SUDDDEN focal neuro deficits •MRI EMERGENCY
Compression Cause: malignancy, infection •hyerreflexia below level •steroid & surgical decompression

MSK About Clinical Manifestation Diagnostics Management


Lumbosacral •strain or tear of paraspinal muscle •back pain and muscle spasms activity related •clinical •NSAIDs
Sprain or •does NOT radiate, NO neuro sx •resume activites if preffered
Strain •MCC of low back pain Exam: decreased ROM, tenderness over muscles •brief bed rest (2 days)

Spondylolysis •pars interaticularis defect due to •asymptomatic Lateral x-ray: radiolucent defect •observation
filaure of fusion or stress fracture •lowback pain with activity in the pars •PT, activity restriction
•MC at L5-S1
Oblique x-ray: “scotty dog”,
Mechanism: hyperextension pars interarticularis break o

Spondylo- •forward slipping of vertebrae •asymptomatic x-ray: forward slipping of •PT, activity restriction
listehsis •bilateral fracture of pars interarticularis •low back pain, possible nerve compression vertebrae •severe needs surgery

Spinal •puss-filled collection *posterior MC TRIAD MRI with gadolinium: BEST •aspiration, drainage, ABX
Epidural •fever •ring enhancing lesion
Abscess Bug: S. aureus (MC), E. coli, Strep, TB •spinal pain: focal and severe ABX: Vanc + Ceftriaxone or
•neurologic deficits: radiculopathy, myelopathy Labs: Cefotaxime
Risks: old, IVDU, immunodeficiency, •high ESR and CRP
steroid, recent spine surgery, epidural •possibel high WBC
About Causes Clinical Presentation/Diagnostics Complications/Treatment
Fibromyalgia •MC in women Chronic fatigue and aching 3+ months 11/18 trigger pointa needed for Multidisciplinary approach
•Absence of objective findings •involves entire body but bony diagnosis •patient education & CBT
and lab results prominences and around neck, shoulders,
low back, and hips ACR Criteria: Pharm:
Chronic disorder: •h/o chronic widespread pain 3+ mo •Amitriptyline, Fluoxetine, Duloxetine,
•Widespread pain, tenderness •Depression, sleep disorders, fatigue •exhibit 11/18 tender points Milnacipran, Chlorpromazine,
and stiffness of muscles, Cyclobenzaprine, Pregabalin, Gabapentin
associated with CT structures

Seronegative About Clinical Presentation Diagnostics Treatment


Reactive •“Reiter’s Syndrome” TRIAD: arthritis + ocular (conjunctivitis, uveitis) + Synovial fluid: negative, inflammatory NSAIDs – 1st line
Arthritis genital (urethritis, balanitis, cervicitis) Methotrexate – 2nd line
*seronegative •Usually precipitated by a GI or X-ray: permanent or progressive joint TNF Inhibitors – 3rd line
a GU infection (STD) •Asymmetric  MC in knees, ankles (weight-bearing) disease in the SI and peripherl joints
-GI: Shigella, Salmonella, •Fever, weight loss Can treat STD at the time
Yersinia, Campylobacter and may reduce risks of
-STD: Chlamydia, Ureaplasma •Mucocutaneous lesions: balanitis, stomatitis developing
(inflammation of mucous membranes-mouth and lips)
•MC in men •keratoderma blennorrhagicum: rash on palms/soles

•Associated with HLA-B27 •extra-articular: urethritis, conjunctivitis, uveitis,


mucocutaneous lesions

Disease About Clinical Presentation


Systemic •Inflammatory autoimmune disorder TRIAD: joint pain + fever + malar Discoid Lesions: Ocular:
Lupus characterized by autoantibodies to “butterfly” rash •annular red scaly patches •conjunctivitis, photophobia,
Erythematosu nuclear antigens (ANA) •localized or widespread transient or permanent monocular
s (SLE) General: •affects cheeks, nose, ears blindness and blurring of vision
•Strong familiar connection/genetic •fever, anorexia, malaise, wt loss
component  mother/daughter •raynaud phenomenon. (20%) Lung:
-Associated with HLA markers •pleurisy, pleural effusion,
Skin/cutaneous: bronchopneumonia, pneumonitis
•Suspect in patients having a •“butterfly” (malar) rash •restrictive lung disease
multisystem disease & a (+) ANA •panniculitis (lupus profundus), fingertrip •alveolar hemorrhage
lesions, periungual erythema, splinter
Risks: hemorrhages Lupus profudunfs *face is MC area Cardiac
•young females *before menopause •cutaneous LE lesions *sun exposed •firm, deep nodules resulting in •pericardium  pericarditis
•african ameriacns, Hispanic, native •alopecia dented scars (lipodystrophy) as fat •HF from myocarditis and HTN
americans cells are destroyed •arrhythmias
•sun exposure
•estrogen (OCPs) GI: mesenteric vasculitis

Neuro: CNS, psychosis, seizure


•severe depression and psychosis
with steroids

Renal: glomerulonephritis,
interstitial nephritis

Diagnostics Management
• (+) ANA Diagnostics Criteria: Need 4/11 •Refer rheumatology Complications/flare-ups:
• (+) antibodies to double-stranded “SOAP BRAIN MD” •Prednisone 40-60mg po daily
DNA and to Sm S-serositis Skin: topical corticosteroids
•depressed complement C3 and C4 O-oral ulcers Joint: NSAIDs and rest Resistance:
A-arthritis Rash/joint: anti-malarial •Cyclophasphamide
P-photosensitivity (hydroxycholorquine) •Mucophenolate mofetil
Drugs: “CHIMMP-Q” B-blood disorders *mainstay for treatment (CellCept)
•Hydralazine, Procainamide, R-renal involvement •Azathioprine (Imuran)
Isoniazid, Chloropromazine, A-antinuclear antibodies •Belimumab (Benlysta)
Methyldopa, Minoclycline, Quinidine I-immunologic phenomena
N-neurologic disorder
M-malar rash
D-discoid rash

Arthritis About Clinical Features Labs


Rheumatoid •Chronic, systemic Systemic: fever, fatigue, wt loss, anorexia Rheumatoid Nodules 20% •Anti-CCP antibodies & RF 70-80%
Arthritis inflammatory disease •occur over bony prominence •Anti-CCP MOST SPECIFIC
(RA) Joint Symptoms: •lungs, sclerae, other tissues •RF: can occur in other disease
•MC in women •Synovitis of multiple joints-BOGGY •correlate with RF in serum *best initial test
•peak 40-50yo F, 60-80 M •Symmetrical swelling of multiple joints •Elevated ESR & CRP
with tenderness and pain
Causes: genetics (HLA) *differentiates from OA Joint fluid analysis: CONFIRMATORY
-monoarticular disease of inflammatory nature of arthritis
Pathophysiology: -stiffness for >30min-1 hour in morning
•formation of hyperplastic Imaging: X-Ray
synovial tissue (pannus) •MC in PIP joints fingers, MCP joints, •radiographs may be normal in 1st 6mo
which erodes cartilage, wrists, knees, ankles, MTP joints Ocular •earliest changes in hands or feet
bone, ligaments, tendons •spared DIP •dryness of the eyes, mouth, and other •later changes of uniform (symettric)
mucous membranes joint space narrowing and erosions
Felty Syndrome: Complications:
RA + splenomegaly + - ulnar deviation Other Manifestations: Treatment: DMARDs + NSAIDS
neutropenia - swan neck deformity •ILD: cough & dyspnea •DMARDS: MTX, Leflunomide
- boutonniere deformity •pericarditis and pleural disease
Caplan Syndrome: •small vessel vasculitis Corticosteroids: Prednisone 5-10mg qd
RA + pneumoconiosis + *hemorrhage in nail fold or finger pulps •Low dose help with inflammation and
pulmonary nodules slow the rate of articular erosion

RA Pharm About Side Effects


DMARDs MOA: folate antagonist •teratogenic  pregnancy test, mandatory contraception
Methotrexate *GIVE FOLIC ACID SUPP. •pantocytopenia  ORDER CBC
Leflunomide •7.5mg PO once weekly •hepatotoxicity w/ fibrosis & cirrhosis (monitor CBC & LFTs q12wk)
Hydroxychlooquin •Max dose 20-25mg/week •lung: interstitial pneumonitis
e •MC: gastric irritation, stomatitis
1ST LINE
Sulfasalazaine •0.5g po BID, increase by 0.5g each week until symptoms improve, max 3g •neutropenia, thrombocytopenia, hemolysis with G6PD deficiency
2nd LINE
CI: ASA allergy
Monitor: CBC q2-4wk for 3 mo, then every 3 mo

TNF Inhibitors MOA: TNF (a pro-inflammatory cytokine) inhibitor •reactivation of tuberculosis, hepatitis B, hepatitis C
Entercept *added to Methotrexate *greater efficacy in combo
Infliximab CI: multiple sclerosis
Adalimumab Methotrexate + TNF Inhibitor
“-mab”

Arthritis About Clinical Manifestation Diagnostics Treatment


Gouty •Metabolic disease  abnormal •Sudden onset and usually nocturnal Labs: Acute Attack: *treat arthritis
Arthritis amounts of urates in the body •serial serum uric acid -NSAIDS (Naproxen 500 or
*monoarticular MC in MTP joint of big toe “podagra” •joint fluid analysis w/ aspiration: Indomethacin 25-50mg)
also common in feet, knee, ankle *SENSITIVE -Colchicine
Triggers: purine-rich foods (alcohol, •Pain intensifies  swollen joint, tender,  sodium urate crystals; -Corticosteroids
liver, seafood, yeast) red, warm, possible fever needle-like and (-) birefringent
Between Attacks:
Meds: thiazide and loop diuretics, •Characteristic lesion=tophus: nodular Imaging: *minimize urate deposition
ACEI, Pyrazinamide, Ethambutol, ASA deposit of monosodium urate crystals •Early: normal •diet: avoid alcohol, foods with
•Late: “rat bite”-punched-out erosions high purine; drink lots of water
Two types: *MC in men >30yo with overhanging rim of cortical bone •avoid hyperuremic meds
1. Primary: heritable, affects genes that • Allopurinol, Uloric 1st line
regulate urate in the kidney

2. Secondary: acquired by
hyperuricemia *obese, alcohol, chemo

Pseudogout •calcium pyrophosphate deposition •Acute, recurrent arthritis involving large X-Ray: linear calcifications of the •Acute: NSAIDS
(CPPD) in fibrocartilage and hyaline joints cartilage (chondrocalcinosis)
cartilage  crystal-induced arthritis •Prophylaxis: Colchicine
*MC in knees and wrists. MCP joints Joint fluid: (+) birefringent
Risks: rhomboid-shaped crystals
•hemochromotonosis
•hyperparathyroidism, hypomagnesium

Pharm Mechanism of Action/Indication Side Effects/CI/Dosing


Colchicine •Mechanism is unknown •GI effects: diarrhea, nausea, cramping, abdominal pain, vomiting
- possibly interferes with inflammasome complex present in
neutrophils and monocytes preventing activation of IL-1B •Dose: Load 1.2mg followed by dose of 0.6mg 1 hour later *only take three pills

Xanthine Oxidase •Lowers plasma uric acid level by blocking the final enzymatic steps •Allopurinol SE: hypersensitivity in first few months (rash  TEN)
Inhibitors in production of uric acid
Allopurinol, Uloric Goal uric acid: 6mg/dL •Allopurinol Dose: 100mg/d orally and titrated up every 2-5 weeks; max 800mg
Arthritis About Clinical Features/Diagnostic Treatment
Osteoarthritis •MC joint disease •Insidious onset, damage over years Labs: Lifestyle:
(OA) •MC in weight bearing joints •SX Pain  physical findings  loss of •does NOT cause elevated ESR •Regular exercise program
•Loss of articular cartilage and function • (+) gram stain and cx *walking, aerobics
hount degeneration •Weight loss
•Pain with motion of joint •synovial fluid non- •assistive devices
Risks: •worse with activity or weight bearing; inflammatory:
•obesity *knees and hips Relieved with rest -yellow, transparent Pharm:
•participation in contact sports •Reduced range of motion, crepitus -high viscosity •Acetaminophen 3-4g/day 1st LINE!
•frequent bending & carrying •NO systemic manifestation -WBC <2000, PMNs <25% (2 extra strength tab 3-4x/day)
•increasing age -no crystals *caution  liver dx, heavy alcohol use
Exam:
Pathogenesis: •hard, bony joint, decreased ROM, crepitus Imaging: x-rays •NSAIDS: Voltaren gel (topical), Meloxicam
degeneration of cartilage and •Bony enlargements of DIP (Heberden *STUDY OF CHOICE *caution with GI bleed and renal
hypertrophy of bone in nodes) and PIP (Bouchard nodes) -asymmetric joint narrow insufficiency
articular margins -marginal osteophytes
 altered mechanics -subchondral bone sclerosis Injections:
-bone cysts •Hyaluronic Acid: injections into the joint to
increase viscosity of synovial fluid
•Intraarticular corticosteroids

Surgery
•total hip and knee replacements
Osteoporosis About Types Clinical Findings Diagnostics
Osteoporosis •Disease of low bone strength Primary • “Dowager hump” suggests multiple DEXA-GOLD STANDARD
•Inadequate bone mass & quality •usually due to increasing age ad vertebral fracture & decreased volume •ALL females >65 years old
*mineral and matrix decreased gonadal function •hyperparathyroidism, bone loss
•menopause; inadequate Ca, sedentary, •upper or mid-thoracic pain associated disease, medication cause bone loss
•Leads to microarchitecture tobacco and alcohol with activity, aggravated by long sitting •postmenopausal with 1+ risks
deterioration or standing, relieved with rest
-increased fragility Type I: (Postmenopausal) •spine compression-loss of vertebral ht •osteopenia: T-score -1.0 to -2.5
-increased risk of fracture •MC in women •osteoporosis: T score <-2.5
-deformity, pain, loss of •estrogen & testosterone def. Bone Fractures:
independence, premature death •vertebral compression fracture or •pathologic fracture, back pain, deformity •T score: compares to young
fractures of the distal radius *VERTEBRAE MC •Z score: compared to peers
Pathogenesis:
•Accelerated bone loss during the Type II: (Senile) Physical Exam: Diet and Exercise:
perimenopausal period (mid-50s •2x more in women, >70yo •lid lag: hyperthyroidism •Maximize bone formation during
to 60s in women and 70s in men) •altered calcium metabolism and •moon face, buffalo hump: cushings youth with proper diet and exercise
intrinsic issues with bone forming •cachexia: cancer, eating disorder (peak reach 25-28yo)
Causes: •hip pelvic fracture •pelvic exams •adequate calcium and D intake
•Genetic & environmental factors •fx late findings: vertebrae, forearm, •regular weight bearing exercise
•Hormone deficiencies, nutrition, Secondary femoral neck, proximal humerus •avoid tobacco & alcohol
decreased physical activity, •results from CHRONIC condition
comorbidity, drug effects •MC in males & long-term steroids X-ray: radiolucency, cortical thinning, Vitamin D: 800-1200/d
•hyperthyroidism, hyperparathyroidism, occult fractures •enhances muscle function and
multiple myeloma, osteomalacia, absorption of calcium
osteogeneis imperfecta Labs: *rule out or find secondary •higher is deficient (2000-4000)
•Heparin, Phenytoin, Lithium, •CBC, CMP, testosterone, proloactin, •Vit D2 50,000 IU once weekly
Levothyroxine PTH, calcium, vitamin D, urine
electrophoresis, TSH, T4 Pharm: Bisphosphonates: 1st line

Medications MOA and Indications Names, Route, Dose CI, SE, Other
Estrogen •Adequate levels are the single most important Estradiol patch: 0.05mg/week topical •CI: hx breast cancer, estrogen-dependent neoplasia,
therapy Conjugated estrogens: 0.0625-1.25mg/d orally abnormal bleeding, thromboembolic

Calcitonin •Hormone directly inhibits osteoclastic bone Treatment until pain is controlled, then taper •Decreases further bone loss at vertebral and femoral
resorption *alternative for estrogen following resolution for 4-6 weeks sites in patients with osteoporosis
-provides analgesic effect Nasal: 200 IU/d instanasally
-good for ACUTE osteoporatic fracture Injectable: 50-100IU/d subcutaneous or IM •SE: nasal-congestion, rhinitis; inject-GI effects

Bisphosphanates •Anti-resportive agents Alendronate (Fosamax) *1st line •Foods and liquids can reduce absorption
1ST LINE •Prevent bone loss associated with estrogen -75mg po/wk; cheap & tolerated -Give with water 30min before the first meal
deficiency, steroids, and immobilization Risedronate (Actonel)-35mg weekly, 150mg/mo -don’t lie down for 30min-less esophagus irritate
•Bind permanently to mineralized bone Ibandronate (Boniva)-2.5mg daily, 150mg/mo
surfaces and inhibit osteoclast activity $$$ Soledronic Acid (Reclast)-IV-5mg IV q12mo •SE: osteonecorisis of jaw
SERM • Agonist-antagonist on estrogen receptor Ralocifen •SE: vaginitis and hot flashes
-agonist on bone
-antagonist on breast & endometrium •DC 72 h before surgery  thromboembolic
-inhibits trabecular and vertebral bone loss •Causes decreased LDL, no HDL effect
NEURO 6%
Disease Causes Clinical Findings
Vertigo False sense of motions Peripheral: BPPV: episodc vertigo, no hearing loss
•hortizontal nystagmus (beats AWAY from affected side)
2 types: •fatigable Menieres: episodic vertio, hearing loss
•peripheral: labyrinth or vestibular nerve •sudden onset of tinnitus and hearing loss can occur
-BPPV Vestibular neuritis: continueous vertigo, no hearing loss
-MEnieres Central
-Vestibular Neurotis •vertical nystagmus Labyrinthitis: continuous vertgo, hearing loss
-Labyrinthitis •nonfatigable (continuous)
-Cholesteatoma •gait issues
•positive CNS signs
•central : brainstem or cerebellar
-migraine, MS, tumor

NEURO
Parkinson’s •Uncommon under age 40, increase after •resting tremor, rigidity, bradykinesia Clinical features from hx •Aim: blocking the effect of Ach or
Disease age 60 & PE administration of Levodopa
Tremor
•MC in men •resting tremor of distal muscles; “pill Must have bradykinesia •1st line: Levodopa & Carbidopa
roll” with either rigidity
•Reduced Risk: caffeine, coffee, ibuprofen, •less with voluntary movement and/or tremor •Dopamine Agonist: <65 and mild sx
statins, cigarette smoking, moderate alcohol •spreads unilaterally up the body -Bromocriptine (Cycloset)
-Pramipexole (Mirapex)
•Increased Risk: age, fhx, Rigidity -Ropinirole (Requip)
herbicide/pesticide exposure •unilat  spread up then contralateral -Amantadine
• “cogwheel” rigiditiy & lead pipe
Pathophysiology: •Anticholinergic-Benztropine
Degeneration of the dopamine-producing Bradykinesia 80% of patients -young patient w/ tremor as primary sx
neurons in the pars compacta of the •generalized slow movement
substantia nigra and locus coeruleus in •starts distally: fingers  loss of arm •COMT Inhibitors
brainstem Dopamine/Ach imbalance swing  feet and legs (shuffling) -Talcapine, Entacapone
-extend life of Levodopa; adjunct tx
•Lewy bodies within neurons

NEURO About Cardiac/Other Neuro Diagnostics and Treatment


Syncope Cardiac: carotid sinus, Carotid sinus: slow SA and AV node due Basilary Artery: vertigo, visual Diagnostics:
aortic stenosis, HOCM to pressure on carotids (turtleneck) dysfunction, perioral numbness, ataxia •Head CT: focal neuro, hx SAH
•Position of patient at time of syncope
Neurologic: basilary artery, Aortic Stenosis: low CO due to stenosis Subclavian Steal: Stenosis of subclavian -lying: seizure, cardiac arrhythmias
subclavian steal syndrome artery proximal to the vertebral artery  -standing: orthostatic, vasovagal
HOCM: murmur increases with valsalva sx with left arm exertion  angina, finger
Other: orthostatic, ulceration Orthostatic vital signs: SBP drop >20
vasovagal Vasovagal: decrease in arterial pressure or SBP <90 or pulse increase >30bpm
and HR mediated by parsympathetic tone Orthostatic: rapid change from lying to
sitting or sitting to standing Treatment: varies on cause

NEURO About Clinical Presentation Diagnostics Treatment


Epidural •middle meningeal artery •LOC, HA, N/V, focal sx, CN III palsy •CT of brain: convex (lens) bleed •high ICP: Mannitol
Hematoma •MCC: temporal FX •LUCID INTERVAL  coma -does NOT cross suture lines

Subdural •cortical bridging veins •varies, may have focal neuro sx •CT of brain: concave (cresecent) •evacuate with massive 5mm+ midline
Hematoma •MC in elderly -DOES cross suture lines shift
•MCC: blunt trauma

Subarachnoi •berry (saccular) arterial •”thunderclap” sudden HA •CT of brain •supportive tx: bed rest, elevate head
d aneurysm of AVM •sudden, severe, worst HA of life •LP: CT (-)  xanthrochromia, high CSF •lower BP: IV Labetalol if SBP >160
Hemorrhage •MCC: rupture, AVM •meningeal symptoms, no focal neuro

Intracerebral •intraparenchymal •HA, N/V •CT: intraparenchymal bleed •supportive treatment: +/- Mannitol
Hemorrhage •MCC: HTN, AVM, trauma •DO NOT PERFORM LP (herniation) •gradual BP reduction

NEURO
Delerium •Acute alteration in LOC with change in •sleep-wake cycle disrupted-daytime •Goal: find underlying cause
cognition or perception somnolenece and agitation
“sundowining” at night Acute Agitation
•disturbance in attention (ability to direct, focus, •Haloperidol (Haldol)
sustain or shift attention) and awareness extrapyramidal & QT prolonging
(orientation), develops over hours to days •Lorazepam (Ativan)
*respiratory depression

Alzheimers •MCC form of dementia •short-term memory loss MRI: cortex atrophy TX: ACH inhibitors: Donepezil,
Dementia Risks: age, genetics, family history  long-term memory loss Rivastimine, Galantamine
•cognitive deficites Histology: •alternative: NMDA antagonist:
Pathophysiology: •amyloid-beta protein Memantine
•amyloid hypothesis: amyloid-beta protein deposition
deposition (senile plaques)
•tau hypothesis: meurofibrillary tangles
•cholinergic hypothesis: ACH deficiency 
memory, language, visuospatial loss

Vascular •disease due to ischemia & multiple infarction •SUDDEN decline in functions with a MRI: white matter •strict blood pressure control
Dementia stepwise progression lesions, infarcts
Risks: HTN, DM, CVA, atrial fib

FrontoTempora •localized brain degeneration of the frontotemporal •changes in social behavior, Histology: picks bodies:
l Dementia lobes personality, language round or oval aggreagate
(Picks Disease) •socialy inappropriate behavior of tau protein
•hyperorality (binge eat, change in food
prefernces)
Diffuse Lewy •progressive dementia characterized by the diffuse •visual hallucinations Cortical lewy bodies
Body Disease presence of lewy bodies •parkinsonism
*parkinsons is localized, not diffuse •autonomic dysfunction

NEURO
CVA A sudden onset of neurologic deficit •hemi/mono/quadriparesis •Oxygen Saturation •Keep NPO  IVF
resulting from a loss of blood flow •hemisensory deficit •Finger stick blood glucose •elevate head 30 degrees, supine
to a part of the brain •visual loss; one or both eyes, diplopia •Hypoglycemia: dextrose
•dysarthria, facial droop, ataxia, vertigo •CT brain w/o contrast •Blood pressure: Labetolol
*cell death and irreparable damage •aphasia (Brocas or Wernickes) -GOAL: w/n 25min of arrival •Cerebral edema: Mannitol
to brain tissue w/n 5min •decrease in LOC •Seizure: Lorazepam
Other Immediate Studies:
Types: Exam: •CBC, BMP, PT/PTT, troponin Ischemic: SBP < 185 + DBP < 110
Ischemia: thrombotic, emboli •ABCs and vitals •EKG/cardiac monitoring •TPA (Altepase): maintain BP
*MCC is atrial fibrillation <180/105 for 24 hours after
•Skin: petechial, janeway, osler, livedo Additional Work-Up: *within 3 HOURS
reticularis, purpura •CTA, MRA, MRI *do not treat BP if not elgible for TPA,
•carotid duplpex •Echo only if BP >220/120
•HEENT: trauma, funcoscope, mouth •CI: BP >185/110, recent bleed,
•Cardio: rhythm, M/R/G, bruit Additional Labs: *based on pt bleeding disorder, recent trauma
Hemorrhagic: intracerebral (HTN),
•Respiratory: breath sounds •toxicology, blood alcohol, LP, •>3-4.5 hours: ASA
subarachnoid (aneurysm, AVM)
ABG, hCG, CXR, EEG, UA/Cx
•Neuro: CNs, NIHSS scale Intracerebral : lower BP w/n 1hr to 140
0: no sx, 1-4: minor, 5-15: mod
16-20: mod-severe, 21+ severe

Lacunar •small vessel disease of the Pure Motor (MC): hemiparesis or Sensorimotor: weakness & numb •ASA
Infarction penetrating branches of the cerebral hemiplegia in absence of sensory/cortical of face, arm, leg on onse side
arteries in the pons & basal ganglia signs (aphasia, agnosia, apraxia) •Control risk factors
Dysarthria “clumsy hand”:
Risks: HTN, DM Ataxic Hemiparesis: ipsilateral weakness dysarthria, facial weakness,
and clumsiness legs >arm dyaphgia, clumsy hand

Sensory: numb face, arm, leg on onse side

Transient •transient episode of neurologic •Assessing severity of symptoms (look for 1. Order US of carotids (CAD) Pharm:
Ischemic dysfunction caused by cerebral persistent neurologic deficits) 2. EKG, Echo (emboli) •Plavix, ASA, Agrenox *antiplatelet
Attack (TIA) acute infarction (blood supply 3. MRA/CTA (vessel disease) •Control BP, Statin
temporarily blocked  lack of O2 •Neurologic deficits <24 hours
-embolic, thrombotic, or lacunar *most last a few minutes with complete ABCD2 Score: 3+ admitted •TPA if neuro deficit potentially
-most resolve within 1-2 hours resolution in 1 hour •Age 60+ disabling
*body immediately breaks it down •BP >140 or >90
and so symptoms are short •clinical SX
•duration: >10 min (+1), >60 (+2)
•diabetes
Carotid •Most severe within 2cm of the •Symptoms due to reduced blood flow Imaging: Asymptomatic: antiplatelet agents
Artery bifurcation of the common carotid and/or superimposed thrombus formation 1. Carotid Duplex US Symptomatic: endarectomy or stenting
Stenosis artery 2. Order MRA or CTA to see true
Physical Exam: stenosis of the vessels Stenosis 60-99%: vascular surgery for
•HX of stroke-like sx or benign •carotid bruit or palpable sclerosis 3. Cerebral angiography carotid endarcectomy
*GOLD* but not used b/c invasive • <60 and >99 then do nothing

NEURO About Types Symptoms and Diagnostics Treatment


Seizure Episode of abnormal neurologic Type I: Focal (Partial) Seizures-ONE AREA 1. PRE-ICTAL/Aura *simple Emergency Room:
Disorder function cause by inappropriate -simple: no LOC; discongitive: LOC Frontal: wave sensation, smell •turn on side, 2 IV large bore access
electrical discharge of neurons Parietal: numb or tingling, taste •Lorazepam x2 DOSES
Type II: Generalized-ALL AREAS, no aura Occipital: visual •Diazepam, Midazolam
Seizure: One-time event Temporal: déjà vu, hearing •2nd line: Fosphenytoin, Phenytoin
1. Absence (Petit Mal) Seizures (CHILD!) *phenytoin incompatible with benzos
Disorder (Epilepsy): Multiple •staring  normal when seizure ends 2. ICTAL *during seizure
•DX requires at least 2
“unprovoked” seizures 2. Myoclonic Seizures *no LOC 3. POST-ICTAL: body relaxes AED Drugs: * after 2+ unprovoked
•sudden brief jerks/twitches -Benzos, Barbituates, Clonazepam,
Non-Epileptic Seizure (+) sx =contractions, (-) sx=relaxations •Todd’s paralysis: transient focal Gaba, Phenobarbital, Valproic Acid,
•Triggered (“provoked”) by a deficit after a simple or complex focal ***Phenytoin, Carbamazepine
disorder, event, or other 3. Atonic (Astatic) Seizure “Drop” zeisure
condition within 7 days •slumping or nodding >1 second AED SE:
Diagnostics: HISTORY IMPORTANT •drowsiness, dizziness, diplopia,
Status Epilepticus: 4. Tonic: muscles (arms, legs, back) tighten Labs: imbalance, N/V, teratogenic
•seizure activity for 5+ minutes •HX: glucose, drug levels, Hcg •gingival hyperplasia (Phenytoin)
or 2+ seizures without 5. Clonic Seizures: convulsive movements  •NO HX: “ + BMP, Mg, toxicology
regaining consciousness rhythmic, jerking, muscle movements AED Monitoring:
•CT scan of head without contrast •CBC, CMP, albumin, depression
Refractory: persists despite IV 6. Tonic-Clonic (Grand-Mal) •MRI *progressive or new in pt >20yo
of 2 antiepileptics •LOC, shaking, body stiffening, bladder/bowel •LP Discontinuing AED:
•after at least 2-year seizure free
Risk Factors: EEG: *BEST TEST •slow rates of AED taper (6 months)
•age (MC child, adults >60) •electrodes  sense & record activity
•brain infections, tumor, injury •prep: wash hair, no caffeine, take med
•family history
•inadequate O2 supply to brain
•cerebral edema, dementia
•vascular disease
•prescription drugs

NEURO About Clinical Presentation Management


Bells Palsy •CN VII (Facial nerve) palsy •ipsilateral hyperacusis (ear pain) •no treatment needed
•lower motor neuron disorder •hemifacial weakness and paralysis •supportive: artificial tears
*unable to life affected eyebrow, loss of nasolabial
Causes: idiopathic, HSV infection fold, drooping of corner of mouth •Prednisone w/n 72 hours reduces time to recover
•taste disturbance (anterior 2/3)
Risks: DM, prego (3rd T), URI, dental nerve block ONLY AFFECTS THE FACE Severe: Acyclovir + Prednisone
NEURO
Tension HA Trigger: •Band-like/vise-like or tight in quality Clinical diagnosis •NO TRIPTANS
•STRESS •Pericranial tenderness, poor concentration •Analgesics (NSAIDs, Tylenol)
•depression •gradual onset •Ketoralac + Metoclopramide +
•anxiety •Bilateral, generalized, non-pulsatile Diphenhydramine IV
•NO FOCAL/NEURO COMPLAINTS
•relaxation

Cluster HA •activation of cells in ipsilateral •Minutes  hours, severe, unilateral, temporal •Typically occur at night & •100% O2 via non-rebreather
hypothalamus, triggering trigeminal HA in grouped attacks over a period of time awaken patient •Sumatriptan (Imitrex)
autonomic vascular system •recurring attacks daily and remitting x4 weeks •Dihydroergotamine (DHE)
*MC middle aged men (30-40yo) •average time between attacks is 6mo-2 years •Clinical Diagnosis -CI: HTN, IHD, PAD, prego

•Trigger: alcohol, vasodilators IPSITALERAL SYMPTOMS: Preventative:


•ptosis, miosis, lacrimation, conjunctival •lithium, verapamil, Topamax,
injection, rhinorrhea, congestion, horners Valprate, prednisone. Ergotamine

Migraine HA Neuronal dysfunction in the Classic migraine attack typically has 4 phases: Criteria without aura: •NSAIDS (FIRST!)
trigeminal system vasoactive 1. Prodrome (24-48h prior to HA) A. 5 attacks + B-D •Triptans (FIRST LINE)
neuropeptides such as calcitonin gene- •euphoria, depression, irritability, cravings, B. HA last 4-72h -CI: prego, CAD, no control HTN
related peptide leading to neurogenic constipation, stiffness, and yawning C. has 2/4 following: •Ergotamines
inflammation, sensitization, and HA -unilateral •Antiemetics (Zofran!)
2. Aura (75% don’t have) -pulsating
•3x more common in women •attributed to cortical spreading depression -moderate or severe ER:
•90% have a family history •transient neurologic sx -aggravated by activity •ketoralac 30mg + Metoclopramide
*MC visual; sensory, verbal, motor D. 1+ of following: 10mg + Diphenhydramine 25mg IV
•MC type is WITHOUT an aura -nausea, vomiting, or both
3. Headache -photophobia or phonophobia *sx treatment: rest in quiet, dark
Triggers: •unilateral, throbbing, or pulsatile room until sx subside, cold cloth
stress, menstruation, visual stimuli, •anorexia, N/V, photophobia, phophobia, Imaging:
weather, nitrates, fasting, wine, sleep cognitive treatment, cutaneous allodynia (touch •CT of the head if abnormal Prophylaxis indication:
disturbances, aspartame hurts), blurry vision, hyperalgesia, blurry vision neuro exam, severe HA of •Topiramate, valproic acid
sudden onset, •Propranolol (BB), CCB, TCA
4. Postdrome-drained, exhausted, euphoria immunocompromised
Intracranial Syndrome of increased ICP w/o a •Fever, Night sweats DX: MRI •Repetitive LPs/CSF volume
Tumor space occupying lesion •throbbing headache *worse with straining removal until target pressure of 10-
“Pseudotumo •excessive CSF, defective absorption, •transient visual obstructions Lumbar puncture 20cm H2)  1ml CSF lowers
r Cerebri” venous sinus pressure •photopsia  flashing lights •high opening pressure >250 pressure by 1cm H2O
•Immunocompromised/ malignancy hx •CSF is normal
•HA that are typically worse w/ Combo: Oral acetazolamide,
awakening or lying down Exam: papilledema, visual field, CN VI palsy, Furosemide, Steroid, Topiramate
•May awaken person at night high ICP
•New onset HA age >40 New dx: admission
Previous dx: discuss with neuro
Risks:
•obese women of childbearing age
•vitamin A toxicity

Disease Mechanism of Action Drugs CI/BBW Caution Education


Triptans •Agonistics effect on serotonin of •Sumatriptan (Imirex) *ER CI: •medications for HTN •pain at site & tingling 30min
meningeal arteries (5-HT1B) and -SC, onset 10-15minutes •CAD, PVD, IHD •SSRIs or SNRIs •don’t use if MOA w/n 14d
trigeminal nerve (5-HT1D) •Zolmitriptan (Zomig) •don’t use within 72h of •wait 2 hours after taking one
-nasal spray *pt for home •stroke or risks (HTN, DM, Ketoconazole before taking another
•inhibit proinflammatory •Treximet (Imitrex+Naproxen) HLD, TIA, obese) •will only tx HA once it has
neuropeptide release *tablet Pregnancy Cat C begun (not prophylaxis)
•prinzmetal angina •may breastfeed 12h after •do not use <18yo or >65yo
Rest are oral; end in “-triptan” dose, but discard milk •may impair think/reactions
•ergot compound meds within the 12h span *driving caution

Ergotamine •Agonist, binding to several •Cafergot (Ergotamine w/ BBW: peripheral ischemia •Elderly SE:
different receptors, producing caffeine) with CYP3A4 inhibitors and •Cardiac disease risk HTN, coronary vasospasms,
*cant give peripheral vasoconstriction and •Migergot with caffeine marcolides •Valvular heart disease peripheral ischemia,
right away decreased blood flow •Dihydroergotamine (DHE) dependency, HA exacerbatib,
CI: PVD, CAD, HTN, renal valvulopathy, N/V, abd pain, leg
•However, may be a vasodilator impairment, hepatic, sepsis, weaknes, myalgia, numbness, int
in large amounts pregnancy (cat X), claudication, photosensitivity
•Similar to Epi & serotonin breastfeeding
Preventative •Alteration of central •botox •Topiramate (Toapamx) •CCB •TCA (Amitriptyline)
neurotransmission •acupuncture •Valproic Acid (Depakote) •Candesartion •Venlafaxine *EVIL!
*continue for months if helps •Propranaol •Guanfacine (Intuniv) •Riboflavin

NEURO
Essential •MC adult onset movement disorder •Rapid action/kinetic tremor bilaterally Clinical Diagnosis •1st line-Propranolol (
Tremor •autosomal dominant -trouble writing, eating, fine motor skills
•Primidone + Propranolol if no response
Defined by: •tremor worse w/ intentional movement *refer if 1st line options don’t work
•enhanced by emotional stress •NO tremor at rest
•decreased with ETOH •2nd-Alprazolam, Topiramate, Gabapentin
•fhx common (dominant) •3rd line-Deep brain stimulation, botox
•no other abnormal findings

NEURO About Clinical Presentation Diagnostics Management


Concussion •mild TBI  AMS •headache, dizziness, brief amnesia CT of Head W/O Contrast: •cognitive and physical rest
•may result after blunt force or •confusion: blank expression, blunted affect •study of choice •resume strenuous activity when
acceleration/deceleration injury •delayed responses & emotional changes resolution of symptoms & memory
MRI: sx lasting >7-14d recovery
Increased ICP: vomit, HA, increasing
disorientation, changing LOC

Anterior Cebebral Artery: •Contralateral motor and sensory, ipsilater CN deficits


•Contralatl arm/leg weak (LE >UE) & sensory, contralat homonymous heminaopsia Middle Cerebral (MC!): MCA is Most Common Artery
•Urinary incontinence, personality changes •Contralateral sensory and motor (face and arm > LE)
•Gaze preference toward side of leasion
Posterior Cebebral Artery: (Vs) •Left hemisphere: Aphasia (brocas-expressive, wernickers-sensory), math, agraphia
•Vomiting, visual changes, vertigo •Right hemisphere: neglect of other side, flat affect, impaire judegement and impulsivity
DERM 5%-dyshydrosis, karposi sarcoma, paronychia, pilonidal disaese

DERM About Clinical Presentation Diagnosis Treatment


Hidradenitis •suppurative disease of the •deep, inflammatory nodules •clinical diagnosis •dietary changes, smoking cessation, local skin
Suppurativa apocrine gland bearing skin areas •draining tracts, open comedones care, eliminate irritatints, reduce skin friction
•fibrotic hypertrophic scars Complications
sites: axillae, inguinocrural and •sinus tracts lead to dermal contractures •secondary infections Mild: topical Clindamycin +/- intralesional
anogenital regions; scalp (rarely) and ropelike elevation of the skin •fistulas Triamcinolone
•inflammatory reactions
Risks: obese, females, smoking Systemic: Tetracycline, Clindamycin +
•history of acne Rifampin
•family history
•mechanical frition Deep, recurrent: punch debridement
Painful abscess: I&D

Epidermal •encapsulared subepidermal •skin colored freely mobile, compressible •clinical •complete surgical excision of cyst wall intact
(Pilar, nodules filled with fibrous and cyst or nodule-visible central punctum
Sebaceous) keratinous tissue/material •possibel foul, smelling yellowish cheese- histology: stratified squamous
Cyst like discharge epithelium

Lipoma •benign collection of adipocytes •soft, painless, slow growth, easily mobile •Not painful: leave alone
inside thin fibrous capsule •subcutaneous nodules
•MC soft tissue tumor •1-10 cm in size •Painful/growing: remove

MC location: trunk, UE

Acanthosis •Common benign disorder of the Types •Symmetric, dark brown Lifestyle:
Nigricans skin characterized by velvety •Obesity hyperpigmented plaques with •dietary changes, control blood sugar
hyperpigmented plaques *MC with DM, insulin resistance, BMI a velvety, verrucous
metabolic syndrome, PCOS •topical retinoids and/or vitamin D analods
•typically located in flexural and •Typical in neck folds and •refer to GI for endoscopy if cancer
intertriginous regions •Malignancy: gastric carcinoma axilla

•Stimulation of insulin-like •Drug Induce: Niacin Diagnostics:


growth factor receptors and •Skin biopsy diagnostic
tyrosine kinase Syndromic: Type A and Type B *rarely needed
•A: hyperandrogenism, insulin resistant, •A1C or fasting insulin and
•MC in African Americans acanthosis, acromegaly glucose
•B: autoimmune (lupus) & DM •plasms T and
dehydroepiandrosterone sulfate
test in women with signs of
hyperandrogensim  r/o
PCOS

DERM About Clinical Presentation Diagnostics Treatment


Pemphigus •Autoimmune disorder of mucous 1. painful erosion/ulceration of Skin Biopsy: •Prednisone 2-3mg/kg + local wound care
Vulgaris membrances and skin mucous membrane (intraoral) •intraepithelial splitting with *continue steroids until cessation of new blister
•type II hypersensitivity reaction  IgG acantholysis formation and disappearance of (+) Nikolsky
against desmoglein (component of 2. painful, flaccid skin bullae
desmosome)  acantholysis (separation) *easily rupture & bleed Direct Immunofluorescence: Wound Care: wet compression, routine bathing,
•IgG throughout epidermis anticipate infection
Risks: •spares palms and soles •basal keratinocytes
•patients 30-40yo, middle eastern •Concomitant Immunosuppresive
•meds: Penicillamine, Captopril, Exam: ELISA: anti-desmoglein or -1st line: Azathioprine or MEthotrexate
Cephalopsorins, Phenobarbital •(+) Nikolsky: superficial anti-epithelial autoantibodies
detachment of skin under Complications: fluid, electrolytes, secondary
MC areas: pressure/trauma bacterial, osteoporosis
•scalp, face, chest, axillae, groin, umbilicus

Bullous •MC bullous autoimmune disease Prodrome: pruritis, urticaria BIOPSY (Gold Standard) Mild (<20% BSA): Topical Steroids (high)
Pemphigoi •Interaction of autoantibody with BP •linear IgG and C3 along
d antigen  BPAG1 and BPAG2 Classic cutaneous lesion dermal-epidermal junction Severe: systemic steroids (Prednisone)
•large, tense, firm-top bullae •subepidermal blister and
•Type II reaction: IgG autoantibodies •serous or hemorrhagic fluid eosinophilia •may couple with Azathipoprine 150mg/day
against hemidesmosome and basement •painful erosions after rupture *same complications as pemphigus vulagris
membrane  subepidermal blistering ELISA: BPAg 180 & 230
Exam: (-) nikolsky *highly sensitive & specific
Risks: elderly

•MC sites: axilla, medial thigh, groin,


abdomen, ventral forearm, lower legs

About/Causes Clinical Presentation Types/DDX/DX Treatment


Lichen •Acute or chronic •6 PS: purple, polygonal, planar, pruritic, Primarily clinical diagnosis •topical steroids under occlusion
Planus inflammatory dermatosis papules, plaques with fine scales -Triamcinolone
involving skin and or mucous DX: punch biopsy & immunofluorescence (cotton gloves, cotton socks, wraps)
membranes •Papules, flat top-polygonal, oval, annular •saw-tooth lymphocyte infiltrate at the
•white lines (Whickham striae) dermal epidermal junction 2nd line: intralesions steroids, topical
Causes •dark phototype: hyperpigmented tretinoin, UV lihgght
•idiopathic MC
*cell mediated Location: volar wrists, lumbar region, shins, Mouth: Cyclosporine & Tacrolimus
•drugs scalp, glans penis, mouth solution mouthwash
•metals (gold and mercury)
•infection (hepatitis C) Koebners phenomenon:
•new lesions at site of trauma

Lichen Skin thickening in patients •repetitive rub & scratching “itch-scratch” clinical •Avoid scratching
Simplex with atopic dermatitis •scaly, well-demarcated, rough •topical corticosteroids
Chronicus hyperkeratitic plaques with exaggerated (Triamcinolone)
skin lines

About Clinical Presentation Diagnostics Treatment


Seborrheic •MC benign tumor •“stuck-on” appearing papules and Usually a clinical diagnosis •No treatment needed
Keratosis •MC on chest abd back plaques with well-defined borders •best appreciated with side-lighting •Can treat for cosmetic or
•well-demarcated symptoms
Risks: genetics, age, fair skin, •velvety, warty lesions “postage stamp” Dermatoscope:
sun exposed •skin-colored, pink, light brown, yellow- •ridges, fissures with comedone like •Cryosurgery 1st line
brown, and brownish-black to black openings  cerefriform pattern *may cause dyspigmentation
Pathophysioloy: benign •looped, hairpin vessels •Curettage & cautery
proliferation of keratinocytes lichenoid keratosis: inflamed SK  pink •Chemical peels
shiny papule or plaque, appears like BCC Biopsy: well-demarcated proliferation of •Laser therapy
•MC associated malignancy is keratinocytes with small keratin-filled cysts
adenocarcinoma of GI tract Dermatosis papulosa nigra: papular SK •Multiple eruptive SKs with a
(dark brown 1-3 mm papules) on the face of visceral cancer is sign of Leser-
individuals with darker skin phototypes Trélat

Actinic •Neoplastic condition of •dry, rough macules or papules DDX: SCC (lips and ears MC) •Avoid sun exposure
Keratosis precancerous epithelial lesions •“sandpaper” rough feel (feel before see) •SPF 30+ when in sun
on sun-exposed areas •transparent, yellow scaling CLINICAL DIAGNOSIS
“Solar •Easily palpated “gritty” scales Treatment:
Keratosis” •MC pre-malignant skin Punch/shave biopsy: atypical epidermal Lesion-Targeted
condition *can lead to SCC Pigmented: keratinocytes, hyperchromatic pleomorphic •cryosurgery liquid nitrogen
similar to nonpigmented AK with the nuclei from basal layer •curettage and electrosurgery
MC Areas: addition of moth-eaten or sharp borders •shave excision/biopsy
•ears (face), scalp, eyelid, neck, and gray dots/granules •biopsy if: >1cm, indurated, ulcerated, rapid
upper chest, forearm, hand, shins growth, fail to respond to therapy Field Therapies *multiple
Actinic cheilitis: lower lip •5-FU 0.5% cream BID x3-4wk
*white scaly flat papules or confluent plaque Dermoscopy: White-yellow surface scale, •Imiquimob cream 5%
erythema revealing a pseudo-network around
hair follicles, linear-wavy vessels

About Clinical Presentation Diagnostic Treatment


Common •benign overgrowth of skin cells •asymptomatic without •dermatoscopy •none if diagnosis is confirmed
Melanocytic change -will not appreciate
Nevi Types: •symmetric neoplastic changes Indications for excision:
•congenital (CMN): develop defect in melanoblasts •sharp borders •locations: scalp, anogenital, mucosa
 increased risk of melanoma development if large •uniform color •rapid change
•irregular borders
•Acquired (MN): develops early in childhood •erosions
 often regresses by age 6 •persistent itching, pain, bleeding
Dysplastic •pigmented lesion from proliferation of atypical •asympatomatic •clinical diagnosis & •observe with dermoscopy +/- digital
Melanocytic melanocytes •irregular shape confirmed w/ histopathology
Nevi (DN) •MC onset in late childhood-early adulthood •sharp and ill-defined •surgical excision w/ biopsy-r/o melanoma
•precursor to superficial spreading melanoma (SSM) bordered Education: -indications: changing, cant observe
-one lesions increase risk by 2x •varigated color •monthly self-exams
-10+ lesions increase risk by 12x •maculopapular •sun protection •routine exams q3-12mo
•family member skin checks -3 if fhx of DN or melanoma, 6-12 if
sporadic DN

About Clinical Presentation Diagnostic Treatment


SCC •Malignancy of cutaneous Differentiated: Tumor Subtypes: •excision w/ margins (3-5mm) & Mohs
epithelial cells •Hard, firm papule, plaque, nodule with a •bowen diseae (SCCIS) *TREATMENT OF CHOICE
(keratinocytes) on sun- thick adherent keratotic scale •acantholytic, adenoid, psuedoglandular -6mm margin in high risk
exposed areas •Erythematous, yellow, skin colored •well v poorly differentiated
•Greater risk of mets on the lip •LAN with mets Non-surgical Candidates
and oral mucosa Biopsy BEST TEST Superficial: electrodessication, curettage
Undifferentiated: *large, irregular border •atypical keratinocytes & malignant cells, (x3) with margins of 3-4mm, radiation
MC Areas & people: •soft, fleshy, erosive papule/nodule pleomorphic and hyperchromatic SCC
•face and dorsal hands •papillomatous, cauliflower, bleeds easy with variable nuclear size Others: Imuquimob, 5-FU, IF-alpha,
•MC skin cancer in AA •found on less sun exposed areas •loss of full-thick epidermal maturation electrochemotherapy, Intralesional
-CAN NOT USE PICATO
Risks: Squamous in situ (SCCIS): BOWENS
•sun exposure, fair skin •confined to epidermis
•fhx skin cancer, increased age •more frequent & aggressive in
•scarring processes immunosuppres
•radiation, immunosuppression •organ transplant and HIV/AIDS
•HPV, tattoos, piercing, burn

Kaposi •vascular cancer •macular, popular, nodular plaque-like Biopsy: angiogenesis, inflammation and •HAART therapy
Sarcoma •associated with HHV-8 brown, pink, red, or violaceous lesions proliferation (whorls of spindle-shaped
cells with leukocytic infiltration) •chemotherapy, radiation
Risks: immunosuppressed

About Types Clinical Presentation Diagnostics and Treatment


Melanoma •aggressive malignancy of Superficial spreading melanoma (MC) Classifications: 3 ways:
melanocytes •asymmetric macule with variegated •de novo melanoma (70%): new •Clark
•MC cancer in young women pigmentation & notched/ragged borders pigmented papule, plaque or nodule •TNM (tumor, node mets)
between 25-29yo •MC trunk in men and LE in women •precursor melanoma (30%): develop •Breslow (>0.76mm)
from precursor (DN or CMN)
Sites:skin, mucous, nail, eye Nodular melanoma (2nd MC) Clark level
RAPID GROWTH 2 phases of growth: •Level I: in situ (within the epidermis)
Associations: •dark brown to bluish-black nodule •radial (thin): remains in epidermis •Level II: invades the papillary dermis
•genetics •ulcerate or bleed •vertical: extends to dermis/vessels •Level III: reaches reticular dermis
•exposure to UVA/UVB in •MC on trunk, head, and neck •Level IV: invades the reticular dermis
light skin types ABCDS: •Level V: invades the subcutaneous fat
Lentigo maligna melanoma •asymmetric
Risk Factors: SLOW GROWTH •borders irregular Treatment:
•age, light skin, fhx, UV •asymmetric brown/black macule or patch •change in color Surgical excision (treatment of choice)
• > 25 nevi, atypical nevi with color variegation & irregular borders •diameter large (>6mm) •In situ: margins at least 0.5cm
•immunosuppression •dermal induration or nodularity •evolution •tumor <1mm: margins of 1cm
•tumor 1-2mm: margins 1-2cm
MC Sites of Mets Acral lentiginous melanoma Best test: •tumor 2+: margins 2cm
•skin/subcutaneous SLOW GROWTH •full-thickness wide excisional biopsy
•lymph, lung, liver, brain •Asymmetric brown/black macule with •shave biopsy discouraged Skin Exams:
variegated pigment and irregular borders •age 18+ should have yearly skin exams
•MC on palms, soles, or nail apparatus •BCC and SCC: skin exam every 6 mo
•melanoma: skin exam every 3 months

About Clinical Presentation Diagnostics Treatment


Basal Cell •MC SKIN CANCER Nodular (MC!) Superficial Multicentric •Good prognosis w/ tx
Carcinoma •neoplasm of basal keratinocytes •translucent “pearly” papule/nodule *can rub alcohol pad over it •likely recurrence in 5 years
•well defined borders (rolled-edge) •thin plaque/patch, pink/red
(BCC) MC locations: •smooth, firm, telangiectasias •+/- scaling Education:
•head and neck •Avoid sun exposure
•upper chest and back •Followed by dermatology
•upper extremities •watch for suspicious lesion
•SPF30+
Risks: •protective clothing
•light skin phototype Ulcerating
•sun exposure •translucent-pearly, smooth, firm, Pigmented Treatment:
•radiation telangiectasia with a central ulcer •firm papule/nodule +/- umbilications •MOHS SURGERY
•advanced age •+/- elevated border (rodent ulcer) •smooth pearly surface •electrodisection
•immunosuppression •generally pigmented or stippled globules •surgical excision
•personal hx •Vismodegib (Erivedge) or
Sonidegib (Odomzo)
Hereditary conditions associated: MOA: hedgehog pathway inhibitors
•albinism Dose: once daily
•xeroderma pigmentosum Sclerosing
•nevoid basal cell carcinoma •plaque, scar like lesion
•rasmussen syndrome •pink/white in color, telangiectasia
•rombo syndrome •ill-defined borders Diagnosis:
•bazex-dupre-christol syndrome
•punch or shave biopsy
•darier disease
•nests and cords of basaloid keratinocytes
with peripheral palisading and a central
arrangement

DERM About Clinical Presentation Risks/Diagnostics Diagnostics/Management


Impetigo •highly contagious, superficial Bullous: *exfoliative toxin A (-) nikolsky •warm water soaks 15-20min then •topical
vesiculopustular skin infection •vesicles  large bullae with rupture Bactroban TID x10d
•MC in children •”varnish-like crusts” DX: gram stain & cx
•fever, diarrhea Widespread infection:
Risks: poor hygiene, poverty, Complications: •Cephalexin, Dicloxacillin, Erythromycin
crowding, warm/humid weather, Non-bullous: MOST COMMON FORM •cellulitis
skin trauma •papules, pustules, vesicles with weeping •acute Prevent: BPO wash, check family members,
 “honey colored, gold crust” glomerulonephritis ethanol or Isopropyl gel for hands
Bugs: S. aureus, GABHS •usually at site of superficial skin trauma
•scattered, discrete lesions  satellite

Molluscum Molluscipoxviru (Poxviridae) •Small, smooth, dome shaped papule with Histology: •Self-limiting  NO TREATMENT
Contagiosum •Benign viral infections umbilicated center •Henderson-Paterson *6mo-2 years! DON’T PICK
•Pink/pearly/flesh color bodies (keratinocytes
•Skin-skin: bathing, sexual with eosinophilic •cryotherapy/curettage (cause scarring),
•HIGHLY contagious •Grouped or linear inclusion bodies) •Podofilox 0.5%
•MC in young children •Palms and soles are spared •salicylic acid (compound W)

DERM About Clinical Manifestation Diagnostics Diagnostics and Management


Atopic •Defetive skin barrier susceptible to •HALLMARK: pruritis •not necessary, clinical dx •avoid triggers: hot water, stress, infection,
Dermatitis drying  rash, pruritis, inflammation •scaly erythematous, ill- irritants, sweating, env alelrgies
(Eczema) •Impaired barrier function leads to defined blisters, papules, •can do IgE
increased water loss and infection pustules •Skin biopsy can help & culture 1st line: topical corticosteroids
•IgE mediated •”itch that rashes” •Triamcinolone (medium potency)
•MC in persons with darker skin •lichenification in chronic •Desonide (low potency-FACE)
•SE: atrophy, hypopigmentation, striae
ATOPIC TRIAD: atopic dermatitis, Nummular: •alternative: Tacrolimus, Pimecrolimus
allergic rhinitis, asthma •Round or coin-shaped *no atrophy
erythematous scaly plaques
Locations: face, scalp, torso, popliteal •sharply defined Itching: antihistamines
folds, antecubital •Minute fissures, round •Benadryl, Hydroxazine, Allegra, Zyrtec
•MC in FLEXOR creases erosions, crusts located within
•Papules/vesicles  plaques Cleansers/Emollients: twice daily
Environmental triggers: heat, humidity, •CeraVe, Cetaphil, Vanicream
detergent, soaps, abrasive clothing, Location: dorsum of hands,
chemicals, smoke, stress feet, extensors (elbow, knees)

Acute •Recurrent pruritis vesicular rash •puriritic “tapioca-like” small •clinical diagnosis •Topical corticosteroid ointments:
Palmoplantar affecting the palms and/or soles tense vesicles Triamcinolone (medium potency)
(Dyshidrotic) •palms, soles, fingers
Eczema Triggers: sweat, stress, warmth, metals •desquamation later •cold compress, burrows solution

Contact •Inflammation of the dermis and Acute: erythema, vesicles, •clinical diagnosis •identification and avoidance of irritants
Dermatitis epidermis from direct contact w/ irritant bullae •patch testing *allergy referral
or allergy •prutitis, stinging, burning 1st line: topical corticocteroids
*oral if severe or extensive reactions
Irritant: MC form  ONE exposure Chronic: scaling,
•chemical, alcohols, creams lichenification, fissures, cracks Review medications: OTX, RX, homeopathic,
•diapers hot water, humidifier, Antihistamine
Toxicodendron: (Hydroxyzine or Benadryl), animals
Allergic: delayed type IV; 24-48h •caused by poison ivy /oak
•nickle MCC
•poison ivy, oak, sumac
•detergents, cleaners, acids, water

Perioral •MC in young women •erythematous grouped •eliminate topical corticosteroids and irritations
Dermatitis papulopustules
Risks: •spares vermillion border Topical: Pimecrolimus, Metronidazole,
•history topical corticosteoirds Erythromycin
•fluoride toothpaste

About Clinical Presentation Treatment


Psoriasis •immune-mediated Progression: Pustular Psoraisis Mild-Moderate:
multisystemic disease 1. Normal •increase in PMN leukocytes •topical corticosteroid (Clobetasol)
•inflammatory disorder causing 2. psoriatic skin without active lesion •caused by corticosteroid withdrawal •Vitamin D analogs (Calcipotriene)
hyperproliferation of •slight dilation, increase mast cell; thick •erythematous patches or thin plaques -apply ointment to wet skin, cover with
keratinocytes in the epidermis 3. developing lesion  studded with numerous pinhead- plastic wrap and leave overnight
•capillary dilation, mast cells, T-cells, sized sterile pustules -re-apply steroid to lesion in AM and
Pathophysiology: macrophage; increased thickness leave uncovered
•keratin hyperplasia & 4. fully developed lesion Generalized “von zumbusch”
proliferating cells in the stratum •pusutles coalesce into “lakes” of pus Moderate/Severe
basale & stratum spinosum Chronic/Plaque (MC): raised, well- • (+) Nikolsky sign in generalized •phototherapy, UVB, PUVA
•T-cell proliferation of the demarcated, pink-red plaques or papules •life threatening
epidermis  over production of with thick silvery white scales •relapses & remission occur over yrs Severe:
epidermal cells & turnover *MC on extensor surfaces •systemic: Cyclosporine, Retinoids,
Eruptive/Guttate biologics, MTX
Risks: Auspitz sign: punctate bleeding with •lots of salmon-pink small teardrop
•MC 20-30yo, 50-60yo removal of plaque or scale papules with fine scales Scalp: Tar shampoo  steroid lotion
•Usually family history & •spontaneous remission
stressor Koebner’s phenomenon: new isomorphic •follows strep pharyngitis Palmoplantar Pustolosis: PUVA soak
lesions at the sites of trauma •most often evolves into chronic
Inverse/Genital: short topical steroid
Nail involvement: pitting nails, yellow-brown Inverse:
under nail (oil spot-pathognomic) •erytheamtosu (lacks scale) Nails: PUVA, oral retinoid, MTX or
cyclosporine

Vitamin D MOA Indication and Application SE/CI Vehicle


Calcipotreine •binds to vitamin D receptors and Indication: psoriasis SE: burn, itching, irritation, •solution, ointment
(Dovonex, regulated cell growth photosensitive, high Ca+ cream, foam
Calcitrene) -inhibits proliferation of keratinocytes Directions: thin layer to rea BID  avoid apply to •Solution best for
-inhibits proinflammatory cytokines <40% body surface area-max 100g/wk CI: hypersensitivity scalp
0.005%
Calcitriol •binds to vitamin D receptors and Indication: psoriasis SE: hypercalcemia, photosensitivity
regulates cell growth
0.0003% -inhibits proliferation of keratinocytes Directions: apply thin layer to affected area BID CI: none
ointment -inhibits proinflammatory cytokines 20g/wk max

DERM Risks Risks Immediate V Delayed Types


Adverse Drug •medication induced changes in •foods Immediate: <1hr of last dose •Type I: IgE mediated-urticarial,
Reactions the skin and mucous membrances •animal/insect bites •urticaria, angioedema, anaphylaxis angioedema *immediate
•most are self-limited •drugs •Type II: cytotoxic, antibody-
•environmental Delayed: occurring after 1 hour, usually mediated (drugs)
•exercise induced before 6 hours, occasionally weeks-months •Type III: immune antibody-antigen
•infections •exanthematous eruptions v. fixed drug comples (serum sickness)
eruptions v. systemic reactions •Type IV: delayed (cell mediated)
Ex. erythema multiforme

About Presentation Information Treatment


Exanthematou MC drug reaction; Type IV •morbilloform or maculopapular •High: PCN, carbamazepine, allopurinol, gold •withdrawal of offending agent
s Drug •macules/papules after drug
Eruption •5-14 days after drug initiation inititation •Medium: sulfonamides, NSAIDS, isoniazid, Symptomatic treatment:
•1-2 days if previously sensitized •bright red macules and chloramphenicol, erythromycin, streptomycin •antihistamine (H1): Benadryl
papules  plaques
Causes: Penicillin, Bactrim, •Low: barbituates, BZDs, phenothiazines, Severe: oral steroids
NSAIDs, Allopurinol tetracyclines

Angioedema •self-limited, locazlied Mast cell: other allergic sx •affect mucosal tissues of face, lips, tongnue, •airway protection
subcutaneous swelling resulting •urticaria, flushing, pruritis larynx, hands, feet, genitalia
from extravasation of fluid •bronchospams, stridor Mast-Cell:
•epinephrine, steroids, antihistamine
Types: Bradykinin: NO allergic sx
•mast-cell: allergic Bradykinin:
•bradykinin: ACEI, hereditary •Ecallantide, Icatibant

Fixed Drug Eruption Pustular Drug Eruption


Other Types •Solitary erythematous Management *remove agent •An acute febrile eruption Labs: leukocytosis
patch/plaque that will recur at •non-eroded: steroid ointment
same site if re-exposure •eroded: topical ABX Clinical Findings: Course/Prognosis:
(Bactroban or Mupiciron) •sterile pustules on an erythematous base, •pustules resolve over 2 weeks then
Lesion: •SX: antihistamines for pruritis often start in the intertriginous fold and/or face desquamation for 2 weeks
•sharply marginated macule liquid compound for oral lesions •fever
•erythema; dusky-red color
•edematous & bullous  erode
DERM About Clinical Presentation Diagnostics Treatment
Urticaria •edema of superficial layers of the skin due •circumscribed hives or wheals Clinical diagnosis Acute Urticaria
to histamine-related increase in vascular •blanchable, raised, erythematous areas on •ED evaluation, can be life-threat
permeability skin or mucous membrances •Triple Regimen: H1(initial)+ H2+steroid
•Type I IgE mediated •intense pruritis
•transient hives (disappear in 24h) H1: Diphenhydramine, Hydroxyzine,
Pathogenesis •+/- dermatographism Chlorpheniramine
•mast cells and basophils release vasoactive H2: Ranitidine
substances  extravasation of fluid Acute: <6 weeks (infection, allergy)
Chronic Urticaria
Triggers: food, meds, head, cold, stress, Chronic: recurrent and >6 weeks (physical, •antihistamines (Allegra, Claritin)
insect bites, environment, infection autoimmune, idiopathic) •refer to dermatology

About Clinical Presentation Diagnostics Treatment


Melasma •Acquired light or dark brown •mask-like hypermelanotic Tests •Tri-Luma (1st line)
pigmentation due to increased symmetrical macules •Woods lamp à unchanged -Fluocinolone 0.01%
melanin •MC on malar and frontal areas of the •Histology: increased melanin -Hydroquinone 4%
•MC on the face face -Tretinoin 0.05%
•MC in females & hot climates
Counseling
Risks: •Avoidance of sun
•Increased estrogen (prego, OCPs) •Sunscreen >30spf q80m
•Genetics *Titanium dioxide and zinc oxide
•Amount of sun exposure •Remove estrogen exposure
•Medications (diphenylhydantoin)

Vitiligo •depigmenting disorder •individual “chalk” white macules Wood’s lamp: fluorescence Course and Management: No cure
characterized by a patchy absence with sharp margins •possible spontaneous repigmentation
of melanocytes •loss of color to mucosal membranes, Skin biopsy: normal skin with •SPF >30 recommended
•destruction or discontinued function retina or hair overlying areas of lack of melanocytes
of the melanocyte depigmented skin Repigmentation
•Painless and without pruritus •TSH, T4, fasting glucose, •Topical Glucocorticoids
•½ of all cases begins b/w 10-30yo •Often seen first in sun-exposed ANA, CBC ACTH stimulation *Intermittent high potency steroid, D/C if no
•new macules in areas of recent trauma if autoimmune response in 2 months
3 Theories (Koebner phenomenon)
•Autoimmune : Selected •Topical Photochemotherapy: 8-MOP + UVA
melanocytes are destroyed by certain Generalized (MC): symmetrical
lymphocytes •”lip-tip” pattern (outh, fingers, toes, •Systemic Photochemotherapy: 8-MOP + UVA
nipples, genitalia) *tx x1yr with poor results for “lip-tip”
•Neurogenic : Interaction of
melanocytes & nerve cells Segmental: one side/part of body •Narrow-band UVB: TOC child >6 years old

•Self-destruction : Melanocytes Localized: 1-3 macules in single sight Depigmentation


destroyed by toxic substances •bleaching of normal pigmented skin
Vitiligo Universalis: confluence •hydroquinone 20% (MEH) cream

DERM About Stages Treatment


Pressure •Breakdown of the skin and underlying I: III: •reduce/eliminate underlying risk factors
Injury tissue resulting from unrelieved soft tissue •superficial, nonblanchable •full thickness of the skin •optimize nutrition
pressure between bony prominence and •redness, intact skin •possible subcutaneous layer •redistribute pressure every 2hr
external surface •pain control
II: IV: deep, beyond fascia,
Risks: elderly, immobile, incontinent, •epidermal damage muscle, tendon, bone Local wound care
aging skin •extending into dermis (blister or Stage I: cover with film for protection
abrasion)
Location: sacrum/hip, bony prominences Granulation: scar tissue Stage II: dressing with moist environement
Unstageable: Eschar: dry, dark scab of skin •transparant film or hydrocolloid dressing
Pathophysiology: •tissue loss obscured by slough or eschar Slough: moist, thick, stringy,
•pressure impairs oxygen and nutrient •do not remove a stable eschar Epibole: rolled wound edge Stage III/IV: debridement
delivery & waste removal

DERM About Clinical Presentation Diagnostics Treatment


Verrucae •cutaneous warts Common & Plantar: Clinical Most resolve within 2 years
•HPV: 1, 2, 27, 57 •firm, hyperkeratotic papules 5-10mm •Biopsy can to r/o SCC •Cryotherapy q4wk  debride surface
*MC IN KIDS •red-brown punctations •Electrosurgery
3 types: (thrombosed capillaries) Histology:
•verruca vulgaris (common) •koilocytotic squamous cells Topical: salicyclic acid
•verruca plantaris (plantar) Flat Warts: sharply defined with hyperplastic •desquamation of hyperkeratotic skin
•verruca plana (flat warts) •numerous, flesh-color papules 1-5mm hyperkeratotis •soak  file  apply

Condyloma •HPV 6, 11; Others: 16, 18, 31, 22 •usually asymptomatic CLINICAL •Imiquimod, podofilox, tricloracetic acid
Acuminatum •MC in sexually active YA •painless papules  soft, fleshy •dermoscopy (HALLMARK) •cryotherapy q2-4wk x3mo
cauliflower-like lesions •shave biopsy if refractory •electrosurgery
“Genital warts” Transmission: invasion of the basal •skin colored/pink/red/tan/brown •surgical removal (best if >1cm)
cells of epidermis by microabrasion Acetic Acid: •laser: recurrent or resistant
•active lesions NOT required for Types:small popular •whitening of lesion •curetage (pairing)
transmission •cauli-flower-floret
•keratotic warts Histology: koilocytotic Prevent: pap smear, Garadasil 9, condoms
Risks: sex, partner with HPV, STIs •flat topped papules/plaques squamous cells with < 15 years old: 2 doses 6 months apart
hyperplastic hyperketatosis > 15 years old: 3 doses (0, 2, 6 month)
Pathogenesis: Laryngeal: HPV 6 and 11
•Low risk: one partner, •MC on vocal cords, age <5 or >20
immunocompetent, regular protection •risk of SCC, squamous cell in situ
•High risk: IVDU, multiple partners,
immunocompromised

PHARM MOA Application SE/CI Education


Imiquimob 5% •Induces immune systems response Apply small amount at night 3x/week and rub until gone •SE: localized reaction •Avoid sexual contact during
Not in SCC to recognize and destroy lesions -wash off in morning with mild soap (6-10h) -If too severe, can take a application times
holiday then resume

Podofilox •Prevents cell division and causes •apply q12h for 3 days then off 4 days, then repeat MC: mild-severe irritation •Apply initial tx in office to
tissue necrosis •apply to normal skin between lesions; avoid open wounds MC systemic SE: HA educate on proper application
•wash medication off after 1-4 hours •Avoid sexual activity
•tx area <10cm AND total volume <0.5ml/d CI: pregnancy

Tricholoracetic •Burns, cauterizes, erodes •Apply Vaseline around lesion to create barrier then apply VERY EFFECTIVE
Acid medication to area with cotton top applicator x6-10weeks

5-FU •Blocks DNA synthesis: apoptosis •BID to affected region x2-4wk •localized skin reaction Education: success is parallel
and selective cell death -Alt: daily for face & anterior scalp only burn, itch, erythema, to compliance
•D/C once erythema, erosion, crusting, and necrosis have erosion, crusting, ulceration
occurred •F/U in 2 weeks

Ingenol 1. disrupt cell membrane & •0.015% gel: face/scalp once daily x3days •Localized skin irritation; AK ONLY
Mebutate DNAcell necrosis •0.05% gel: trunk/extremities, apply nightly x2d erosions
(Picato) 2. Neutrophil-mediated cytotoxic •Plant derivative
•Caution: risk of SCC?
Diclofenac •COX-2/prostglandin inhibit •3% Gel-Apply to treatment area BID x60-90d •Localized skin reaction

DERM About Clinical Presentation Diagnostics Treatment


Pediculosis Bug: Pediculous humanus capitis •Maculae cerulae or pruritis stains on •demonstration of lice or nits •Primetherin 5% (off-label)
Capitis •Live 30 days on the host and <1 day skin of occipital scalp and nape of neck on hair visually or under •Spinosad (>4yo)
“Head lice” off the host microscope •Malathion 0.5% lotion
•Occipital and cervical LAN •Ivermectin-formulated lotion
Transmission: person •Pyodermas in the scalp Nits: white, oval-shaped egg •Oral Invermectin (off label)
•Louse, Nits, hair casts capsules *seen on woods lamp
Risks: girls > boys •Manual nit removal with comb
•less risk: African american Hispanics lice: oblong structure attached
•MC in warmer months to hair Eyelid: Petrolatum BID to margin for at
least 8 days

Pediculosis Bug: Pediculus humanus corporis •Extremely pruritic, rash concentrated •clinical 1st line: hygiene improvement
Corporis where clothing is
“body lice” •live in/lay eggs on clothing Tests: examine clothing *MC •wash all clothes and linins
•2-4mm, wingless, blood-sucking Maculae cerulean: blue grey macules at seams •iron seams to destroy lice
arthropods are pathognomonic for lice •shake clothes over paper & •Permethrin 5% cream if heavy infestation
examine
Risks: poor hygiene, poverty, “vagabond disease” •woods lamp: fluoresce
homelessness •chronic (parasitic melanoderma) yellow-green
•thickened and darkened skin after a •confirms w/ finding live
Transmission: usually sex long period of bites and rubbing louse or nit
•examine pubic and head lice

Pediculosis •Highly contagious •Live on hair between pubic and perianal •finding louse or nit •CDC-sexual partners w/n 1mo notified
Pubis Bug: Phtirus pubis regions, eyelashes, eyebrows, and face •nits: brown & full(nymph) •wash, clean all clothes and linens
“pubic lice translucent and flat (cast) •Permethrin 1% or Pyrethrins
or crabs” Transmit: person-person •Extreme pruritus (MAIN) due to
•usually sexual saliva or anticoagulant during feed

Scabies Bugs: Sarcoptes scabiei •Itching is worse at night, especially •clinical diagnosis •Treat infected & close contacts, repeat
after just getting into bed treatment in one week
Pathophysiology: •Typically, 10-20 mites Skin scraping: mites, eggs,
•female mites burrow into the stratum feces •1st line-Permethrin 5% or Rid: use about
corneium of the epidermis to lay eggs, Exam: 8-14hr prior to shower
feed, and defecate  reaction •multiple, small erythematous papules Dermoscopy: darker, V- •Oral Ivermectin 200ug/kg PO single dose
•linear burrows shaped structure  “jetliner •Last Resort-Lindane
Risks: children, sex, poor hygiene, *PATHOGNOMONIC with its trail” -CI: teratogenic, breastfeeding, <2yo
long-term care facilities, overcrowding  MC in intertriginous zones, web -SE: seizures *do not use after shower
spaces *avoid in crusted due to CNS toxicity
Transmission: person to person •usually spares face and neck
•red, itchy papules or nodules on Patient Education:
scrotum, glans, body folds •symptoms should improve within 3d,
pruritus can persist for 4 weeks after tx
•put all clothing & linens in bag x72hr

DERM
Tinea Capitis •MC in children & African Americans Endothrix: occurs within hair shaft Clinical diagnosis PO Antifungal:
•MCC Trichophyton Black Dot: broken hair near scalp  swelling •Oral Griseofulvin
•diffuse & poorly circumcised KOH: MC initial test *monitor CBC, Cr,, LFTs
Non-inflammatory: -T, tonsurans, T. violaceum •fungal element insure or •Terbenafine (Lamasil)
•scaling, prutitis, alopecia, adenopathy surrounding base of hair •Itraconazole, Fluconazole
Kerion: inflammatory mass & loose hair
Inflammatory: pain, tenderness, alopecia •boggy, purulent, inflamed nodules & plaque Woods lamp: microsporum Antifungal shampoo:
•painful, drain pus; hair falls out when pull *T. tonsurans does not •Ketoconazole
-T. verrucosum, T. mentagrophytes fluoresce
Prevention:
Favus: “honeycomb” thick/yellow crust Culture: DEFINITIVE -wash furniture, clothes, linens
•avoid pillow cases, head to head
Ecothrix: occurs outside hair shaft •disinfect combs
“grey scaly patch”  scaly

Tinea Cruris •MC in inguinal folds  thighs •Large scaling, well demarcated dull Clinical diagnosis •Topical Ketoconazole,
“Jock Itch” T. rubrum red/tan/brown plaques, sharply demarcated Clotrimazole, Butenafine
with a raised border KOH: MC initial test •oral Griseofulvin or Terbenafine
Risks: males, copious sweating, •segmented hyphae •put on socks before pants
immunocompromised •PRURITIS •avoid tight fitted clothing
Culture: DEFINITIVE •use cotton underwear

Tinea •Fungal/dermatophyte infection involving •Asymptomatic KOH: segmented hyphae •Topical Clotrimazole,
Corporis anywhere on the body •Pruritis, well-defined plaques or patches Ketoconazole (use first)
(Ring Worm) •T. rubrum with central clearing Culture: DEFINITIVE •Oral Terbinafine, Itraconazole

Tine Pedis •MC dermatophyte infection Interdigital (MC): pruritis, erythematous Clinical diagnosis Topical Antifungal x2-4 weeks
(Athletes foot) T. rubrum, T. interdigitale erosions or scales between the toes •Ketoconazole and Econazole
•MC 20-50yo *MC between 4th and 5th KOH: skin scrapping
•segmented hyphae Oral Antifungal: *if topical fail
Risks: hot, humid climate; occlusive Hyperkeratotic: soles, lateral & medial •Terbinafine (Lamisil)
footwear, hyperhidrosis surfaces, with “moccasin” distribution Culture: DEFINTIVE •Fluconazole, Itraconazole

Vesiculobullous/Inflammatory: pruritic Prevention: BPO, antifungal


vesicur or bullous eruption, medial foot powder, shower shoes
Tinea *NOT caused by dermatophytes •hyper or hypopigmented, well-demarcated KOH: hyphae & budding •Selenium sulfide
Versicolor •Overgrowth of Malassezia furfur round or oval macules with fine scaling •Zinc Pyrithione
Woods lamp: yellow-green •Sodium Sulfacetamide
Risks: climate, sweating, immunodef. •MC on trunk and proximal extremities fluorescence •Itraconazole or Fluconazole if
products, steroid use, oily skin widespread or failed topical

Cutaneous •Superficial fungal of intertriginous skin Areas: genitocrural, gluteal, interdigital, Clinical diagnosis Mild-Mod: topical antifungal
Candidiasis •MC neonates and age > 65 inframammary, axilla, pannus •Ketoconazole, Clotrimazole
•Candida Albicans *NOT TINEA KOH: skin scrapping
•Pruritic, tender, painful •budding yeast +/- Severe: oral-Fluconazole
Risks: warm, moist environment, •Erythematous, “beefy red” plaques and pseudohyphae Prevent: keep dry (powder-
immunocompromised (DM), obese erythematous satellite lesions Zasorb AF)

DERM Cause/Incubation/Tran Contagious Presentation Diagnostics/Treatment


Hand Foot and •Incubation: 3-7d •very contagious in •oral painful, red, ulcers surrounded by red halo lesions on •supportive
Mouth •Tranmission: person, secrete first week tongue, gums, palate, buccal mucosa, palms, feet, and butt •MC complication: dehydration
Coxsackie Virus •MC in summer/autumn •contagious until
blisters disappear

Herpangina Coxsackie Virus A •sudden onset high fever •Supportive treatment


•MC in child 3-10 years old •stomatitis: small, yellow-white papulovesicular lesions on •Complication: aspect meningitis, GB
•MC in summer, early fall posterior pharynx that ulcerate

Herpes Simplex •HSV-1 •active lesions very •grouped vesicles on erythematous base •supportive, lasts about 7-14 days
(HSV-1) contagious •ulcers, perioral area, lips; LAN; stinging/burning

Varicella Incubation: 10-21d •can spread 1-2 days •itchy, fluid filled; rash  papule  vesicle  crust •varicella vaccine (12-15mo, 4-6yr)
Varicella Zoster Transmission: respiratory before rash and until •dew drop on a rose petal •Acyclovir
all have scabbed

Rubeola Children <5; Paromyxovirus 8 days total: 4d with •Koplik spots-white/blue papules with red base on mucosa •MMR vaccine (12-15mo, 4-6yr)
(Measles) (1st) incubation: 7-14d; airborne rash, 4d w/o rash (no •Red blotchy: face  arm  trunk  leg •MC complication: OM, diarrhea
Rubeolla Virus school x4d after rash) •high fever, cough, coryza, conjunctivitis •PNA MCC measles-related death

Mumps Incubation: 12-15d •5 days isolation after •swollen parotid glands •MMR vaccine, supportive care
transmission: airborne, saliva swelling •fever, HA, myalgia, anorexia •Complication: orchitis, encephalitis

Rubella (3rd) Transmission: airborne, •10d before rash  •postauricular and occipital LAN •Complication: rubella syndrome
contact, pregnancy 1-2 weeks after gone •fine pink rash face  trunk  arms  legs (deaf, cataracts, heart defects), arthritis

5ths: Erythema •Parovirus B19; child 5-7yo •most contagious •prodrome: fever, HA, rhinitis •can suppress RBC aplastic crisis
Infectiosum Incubation: 1-2 weeks BEFORE rash •fiery-red facial erythema “slapped cheek”  lacy, •supportive, IVIG if
reticular on extremities immunosuppressed

Roseola 6ths •Children 6mo-3yr •high fever 3-5d  rash (rosey pink nonpruritic •supportive
(HHV 6 and 7) Incubation: 5-15 days; maculopapular rash) •MC complication: seizures
Transmission: airborne
About Symptoms Causes, Risks, Complic. Treatment
Onycho- •Fungus of the nail, MC on toes •thick, opaque, discolored •Fungal culture + rapid •topical antifungal: Ciclopirox or Efinaconazole
mycosis •lifting of the nail from bed test (KOH or PAS)
Cause: *culture by clip/scrap •oral antifgunals: Terbinafine (Lamisil)
•Trichophyton (T. rubrum MC) •must rule out melanoma -6 weeks for fingers, 12 weeks for toes
•Epidermophyton *longitudinal melanichia  Periodic Acid Schiff (PAS): -monitor CBC and LFT at baseline and monthly
biopsy •MOST SENSITIVE -risks: hepatotoxic, pantocytopenia, agranulocytosis
Risks:
•tinea pedis, psoriasis home remedy: 50/50 apple cider vinegar and water
•fhx, old age, climat 10min soaks x6months
•poor health, trauma, immunosupress
•fitness, communal bathing, footwear

DERM About Clinical Presentation Risks/Diagnostics Diagnostics/Management


Erysipelas •Acute superficial infection Prodrome: fever, chills, anorexia, malaise •clinical diagnosis •MRSA: IV Vancomycin, Daptomycin
(dermis & dermal lymph vessel) General: +/- sepsis -Oral: Clindamycin, Bactrim, Doxycycline
•MC in young children & adults Labs: CBC, CMP, ESR,
Lesion: painful, tender, hot blood cx, US or MRI •MSSA: IV Cefazolin, Clindamycin, Nafcillin
Bugs: GABHS (MC!), S. aureus -bright red, raised, swollen, indurated plaque -Oral: Cephalexin, Nafcillin, Clindamycin
with SHARP borders
Cellulitis •dermis and subcutanoues tissue Prodrome: fever, chills, anorexia, malaise Complications: •Dog/cat: Augmentin *Altnerative:
•MC in middle age adults General: +/- sepsis •abscess formation Doxycycline
•bacteremia, endocarditis •Human bite: Augmentin
Causes: Lesion: painful, tender, hot •osteomyelitis •Fresh water: Ciprofloxacin
•S. aureus), GABHS -bright red, raised, swollen, indurated plaque •metastatic infection •Salt water: Doxycycline
•Cat/dog: Pasturella multocida with INDISTINCT borders •sepsis, toxic shock
•Freshwater: Aeromonas syndrome
•Salt water: V. vulnificus

About Clinical Presentation Diagnostics Presentation


Alopecia •T-cell mediated autoimmune disorder •Smooth, discrete, circular patches of Subtypes: •Spontaneous remission, can reoccur
Areata •associated with other autoimmune complete hair loss •Alopecia areata: solitary or multiple
•MC in children <25yo •gradual over weeks to months •AA totalis: total loss of terminal scalp Pharm
•Usually family history •oval, round defined border •AA universalis: total loss of all Local: intralesional steroids
terminal body and scalp hair Extensive: Topical corticosteroids
Causes: Nails: pitting, fissure, trachyonychia •Ophiasis: bandlike pattern of hair loss (Triamcinolone) with Minoxidil
•definicieny in vitamins over peripheral of scalp
•high and low temperature Areas: scalp, beard, eyebrow, extremities •Nails: fine pitting (hammers brass) of Others:
•disease of digestive system, endocrine dorsal nail plate •ILK (intra-lesional kenalog)
•hormonal changes, genetic Exam: •Minoxidil 5% (best if combo)
•hairstyles •Black dots: no scaring Diagnosis: clinical •Anthralin: safe in kids
•Dermoscope: hair breaks before surface •punch biopsy is DEFINITIVE-
Pathophysiology: •Exclamation hairs: blunt distal end and peribulbar lymphocytic inflammatory
•Damage to hair follicle in anagen stage taper proximally near scalp infiltrates
•tranform to catagen & telogen •appear when browkne hair (black dots)
•NO SCARRING are pushed out of follicle
Androgeneti •genetically predetermine progressive Men: Clinical diagnosis Topical:
c Alopecia loss of terminal hairs •bitemporal thinning of frontal scalp then •Minoxidil/Rogain 2%, %5 BID
•Male and Female pattern baldness involves the vertex Labs: DHT, T, DHEA, Prolactin MOA: widens blood vessels to allow
oxygen and nutrients
Pathophysiology: Women:hair between frontal and vertex Dermatoscope: miniaturized hair and *hair loss first before regrowth
•dihydrotestosterone (DHT) key brown perihilar casts
angrogen Oral:
•activation of androgen receptor Biopsy: telogen & atrophic follicles •Finasteride 1mg po daily (2nd line)
shortens the anagen  tellagen phase *MEN ONLY , inhibits T & DHT
Trichogram: increased telogen hairs •Spironolactone 50-100mg QD
•Men: after puberty, 20s-40s MOA: blocks action of DHT
•Women: MC after 50 Hormones: T, DHEA, Prolactin CI: breast cancer; SE: K+ levels

•Hair transplant, hair piece/wig


Recurrent: antiviral qd
BILAT & SYMMETRICAL Exam: (-) Nikolsky Complications:
•infection, fluid/electrolyte

About Clinical Presentation Diagnostics Treatment


Rosacea •Chronic, inflammatory •acne-like rash (papulopustules), centrofacial erythema, •Clinical diagnosis Lifestyle:
•hyperreactivity of capillaries flushing, telangiectasias, skin coaorsening •Definitive is biopsy •avoid triggers
•burning and stinging, red eyes •camouflage w/ green/yellow-tint makeup
•MC in lighter skin & females •cleaners, moisturizer , daily sunscreen
Exam: NO comedomes
Triggers: •rhinophyma: red, enlarged nose Telangiectasia/Erythema: Brimonidine
•hot or cold weather, sun •gnathophyma: chin Rosacea Fulminans: Prenisolone
•hot drink, spicy, caffeine, alcohol •metophyma: forehead
•hot baths •blepharophyma: eyelids Mild/Moderate:
•medication: hrmones, Niacin •otophyma: ear •Topical Metronidazole (1st line)
•emotions •15% azelaic acid gel
•exercise Types: •Ivermectin cream
•erythematotelangiectatic (MC): persistent erythema of •Sodium sulfacetamide with 5% sulfur
central face, telangiectasia, sting •Brimonidine gel (Mirvasa)
•papulopustular: papules, pustules, edema
•phymatous: enlarged cobblestones, nose Moderate-Severe:
•ocular rosacea: blepharitis, conjunctivitis, hyrpemia •Oral Tetracycline antibiotics *1st line
•Metronidazole, Azithro, Isotretinoin

DERM About Clinical Presentation Diagnostics/Management Management


Folliculitis •Infection of hair follicle •singular or clusters of perifollicular papules •clinical Mild (few): warm compress, soap
•surrounding erythema on hair bearing skin •Topical Bactroban
Bugs: S. aureus, step, gram (-), •Non-tender or slightly tender, pruritis •gram stain, C/S •Clindamycin, Erythromycin, BPO
fungi, pseudomonas •KOH if fungal
•Hot tub: Pseudomas (trunk) *KOH dissolves the skin so you see Severe/Refractory:
Predisposing factors: •Viral: herpetic and molluscum the actual fungus •oral Cephalexin or Dicloxacillin
•shave hairy areas, hair occlusion •Fundal: candida, malassezia
•hot tub *Pseudomonas •Other: syphilis MRSA: Doxycycline, Bactrim
•topical CS, systemic ABX
•diabetes, immunocompromised

DERM About Clinical Presentation Diagnosis Treatment


Brown •MC in southwest & Midwest •lack severe symptoms: possible fever, chills, N/V •clinical •local wound care & pain control (NSAIDs)
Recluse •Loxosceles reclusa: violin pattern • “red, white and blue”
Bite on anterior cephalothorax GOLD: wound care
1. Local: burning, erythema at bite  blanching of area spider •clean with soap and water
Pathophysiology: (vasoconstriction) collection & •cold packs
•venom is cytotoxic and hemolytic 2. erythematous “red halo” for 24-72 hr indentificatio •elevated or neutral position
 local sx, necrosis 3. hemorrhagic bulla  eschar formation  DIC n

Black •Latrodectus Hesperus Latrodectism: pain at bite  systemic and neurologic “ Mild: wond care and pain control
Widow •red hourglass shape on underside symptoms in 30min-2 hours
Bite of belly Moderate/Severe:
•muscle pain, spasms, rigidity •muscle relaxants (Benzos, Methocarbamol)
Pathophysiology: neurotoxin •blanched, circular patch with surrounding red perimeter
and central punctum (target) •antivenom reserved to pts not responsive to above
About Clinical Presentation Causes/Location Clinical Presentation
Acne •Inflammatory skin condition •open comedones (blackheads): incomplete •COMEDONES MUST BE •Acne often resolves after teen years
Vulgaris involving pilosebaceous units •closed comedones (whiteheads): complete PRESENT! •nodulocystic acne will require more
aggressive treatment
•Women > men b/c hormones Inflammatory: papules and pustules •Definitive-skin biopsy •acne requires consistent, regular care over
months
•Begins around puberty due to nodules and cysts: >5mm can cause scarring, •Itchy back, shoulder, scalp  scrap
androgens stimulation including pitted or hypertrophic scar for potassium hydroxide (KOH) for Mild: topical retinoids, BPO, topical
pilosebaceous unit changing Pityrosporum folliculitis  fungal antibiotics (Clindamycin, Erythromycin)
keratinzation at follicular orifice shampoo
Moderate: + oral ABX (Doxycyline)
•MC on face, back, upperchest
Severe: oral Isotretinoin
Pathogenesis:
1. increased sebum (oil)
2. Follicular hyperkeratinzation
3. proliferation of the bacterium
Propionibacterium acnes
4. inflammation

Class Drugs Indication/About MOA SE/CI


Topical Tretinoin, Tazarotene •1st line for comedonal & inflam decreases cohesion and increases turnover Side effect: dryness, photosensitivity
Retinoids Adapalene Gel •every 3rd night  nightly of epidermal cells *speed up exofiliation CI: pregnancy

Benzoyl •Start with lowest concentration •no baceterial resistance  Release free- SE: skin irritation, bleaching of hair/clothing
Peroxide (2.5%) than increase as tolerable radical oxygen oxidized protein
(BPO)
Topical Clindamycin (BID, foam qd) •Mild-moderate inflammatory acne •reduces number of P. acne in SE: skin irritation
Antibiotics Erythromycin (BID) (papulopustular) pilosebaceous unit
* use with BPO
Oral ABX Doxycyline or minocycline Moderate acne with inflammatory •Inhibits P, acnea SE: upset stomach, photosensitivitiy, lupus
*add 100mg BID papules or deeper-seated lesions •Quickers results than topicals

Tetracyclines *1st line •Anti-inflammatory and ABX CI: pregnancy and young children
SE: photosensitivity, teeth staining in children

Macrolides Erythro &Azithromycin *1st Increased resistance Pregnancy category B

Bactrim *2nd line •Severe acne unresponsive to SE: SJS, TEN; CI: Avoid in pregnancy
others
Oral Isotretinoin •Severe resistance nodular/cystic •Decrease in P. acnes SE: dryness of skin and mucous membranes,
Retinoids (Claracis or Acutane) •Inhibition of sebaceous gland headaches, suicide & depression, teratogenic
*monotherapy •2 contraceptives *can decrease athletic performance •increased TG and cholesterol
*after FAILED oral •2(-) pregnancy tests & monthly
test •no blood donation Labs: CBC, LFT, lipids monthly CI: tetracycline; pregnancy  birth defects

ENDO 5%
ADRENAL GLAND: GFR-ACE (out to in): Glomerulosa-Aldosteron, Fasiculata-Cortisol, Reticularis-Androgens (E/T)
Hypothalamus (CRH)  Anterior Pituitary Gland (ACTH)  Adrenal Gland

Disease Causes/Types Presentation/Labs & Testing Work-Up Treatment/Management


Primary Bilateral idiopathic TRIAD: HTN + HYPOKALEMIA + BMP: High NA+ and bicarb,low K+ Unilateral Adrenal Adenoma “ Conn”
Hyper- adrenal hyperplasia METABOLIC ALKALOSIS •surgical excision + Spironolocation
aldosteronism *increased urinary hydrogen excretion Plasma Renin & Aldosterone
Primary: *in the morning in seated position Bilateral Adrenal Hyperplasia (MC)
•Renin-independent •Refractory hypertension (young age •aldosterone:renin ratio (ARR):>20:1 •Medical management
•idiopathic(MC) w/o risk factors) *high aldosterone, low plasma renin •Low Na+ diet
•conn syndrome •K+ spare diuretics:
“unilateral adenoma” Hypokalemia: CT Scan-Adrenal Spironolactone or Eplerenone
•proximal muscle weakness <4-Conn, >4- carcinoma •HTN: ACEI, CCB, ARB
Secondary: •polyuria, constipation •thickening-hyperplasia
•increased renin  •decreased DTR •negative CT  adrenal vein sampling Adrenal Carcinoma: Oncology referral
increased aldosterone •hypomag
in RAAS Adrenal Vein Sampling Referrals: Endo, Cardio
*perform if negative CT scan
•assess aldosterone in blood Monitor: HTN and electrolytes

Confirmatory:
•sodium loading test (oral preferred)
-high urine aldosterone >12  primary
*If no oral, then IV over 4 hours while
seated, >10 is confirmatory
Pheo- Catecholamine Triad: Plasma Free Metanephrines Management: resection of tumor after 1-2
chromocytom secreting adrenal episodic palpitations, HA, diaphoresis •sit 15min; elevated  assess urine weeks of hypertensive therapy
a tumor  NE and E •Anxiety, pallor, syncope, tachycardia
*Adrenal •Paroxysms <1hr 24h Urine Metanephrine and Creatinine Awaiting Surgery: Maintain BP < 160/90
Gland Rule of 10s: • (+) if >2.2mcg/mg of creatinine •Alpha-Adrenergic Blockers:
•10% bilateral *triad + HTN is HIGHLY suggestive Cardura, Minipress, Hytrin
•10% extra-adrenal Imaging: CT/MRI with contrast
•10% malignant Diet-high salt and water intake

Adrenal Nonfunctional (MC) Nonfunctional: Asymptomatic •Detailed HP Surgery referral


Adenoma/ Benign, does NOT •Labs based upon suspected adrenal
Carcinoma secrete steroids Functional: *related to cellular zone affected
involvement
LOW ACTH Functional: •Glomerulosa-hyperaldosteronism •Fine Needle Aspiration
•Benign >1cm •Fasiculata-cushings *only to r/o metastasis in patient with
•Secrete steroids ind. •Reticularis-hyperandrogenism known malignancy
ACTH/ RAAS •Medulla-pheochromocytoma *must r/o Pheochromocytoma

ENDO
Addison’s Autoimmune disease *glucocorticoid & mineralcorticoid deficiency •CBC: Eosinophilia, lymphocytosis Chronic: Oral glucocorticoids
Disease of adrenal gland •CMP: -Hydrocortisone 15-30mg daily
(Primary •skin and mucosal hyperpigmentation low Na, & glucose; high K+, BUN/Cr 2/3 in morning, 1/3 later
Adrenocortical ↓ cortisol •Anorexia, weight loss, fatigue, low stamina •Culture: Blood, sputum, urine
Insuffciency) ↓ aldosterone •dizzy, hypoglycemia, weakness, fever, lymphoid •Plasma Cortisol and ACTH @ 8am •Glucocorticoid Stress therapy
hyperplasia, N/V/D *ACTH elevated in primary, decreased -Increase dose by 50% with stressors,
HIGH ACTH •orthostatic hypotension (dehydration) in secondary surgery, or illness
*Adrenal •Serum DHEA (<1000)
Gland Adrenal Crisis: Chronic: •Plasma Renin @ 8am •Mineralocorticoid: Fludrocortisone 0.05
MCC: abrupt •vitiligo (10%)  ACTH binds to melanocyte •ACTH Stimulation (Cosyntropin) *can be added only in primary
withdrawal of •Changes in hair *loss CYP11+17, gain CYP21 *(+) if low or absent rise in serum
steroids •Generalized pain cortisol (<18) after ACTH Acute Adrenal Crisis
•Psychiatric: anxiety, irritable •Hydrocortisone: Loading IV  IV q6h
Causes: •Neurologic: gait, coma Adrenal Crisis: for 24h  oral when tolerate oral food
•autoimmune (MC) Low Na and glucose, high K+ •Fludricortisone, IVF (normal saline)
•infection (TB, HIV) Acute: Sudden “Adrenal Crisis” •Broad ABX, tx electrolytes/glucose
-MC symptom: Shock  low BP and volume
Education: medical alert bracelet

Acute Adrenal •Sudden worsening •Hypovolemic Shock (MC!) BMP •IVF: normal saline (D5NS if
Insufficiency of adrenal •Hypotension, Hypovolemia •hypoglycemia hypoglycemia)
“Addisonian insufficiency due to •Abdominal pain, N/V •hyponatermia
Crisis” “stressful” event •Fever, Weakness, lethargy, confusion •hyperkalemia •IV HIGH dose Hydrocortisone

*normal response is Triggers: abrupt steroid withdrawal, surgery, Confirm: cortisol and aldosterone •Reversal of electrolytes
a 3x increase trauma, volume loss, hypothermia, MI, fever, •Fludricortisone
sepsis, hypoglycemia

Cushing’s ↑ cortisol •Fatigue, reduced physical endurance CBC: leukocytosis, low eosinophils Cushing SYNDROME Management
Disease HIGH ACTH •Central obesity with protuberant abdomen, CMP: elevated glucose, hypokalemia •hydrocortisone=DOC
plethoric “moon face”, supraclavicular fat
pads, “buffalo hump” Dexamethasone suppression test Cushing DISEASE Management
*Adrenal Cushing Syndrome: •Large purple striae, thin skin •Syndrome: 1mg @ 11pm-check cortisol •surgical removal of pituitary tumor
Gland •Exogenous long- •Hirsutism, male hair loss at 8am: (+) if cortisol elevates >5 •Hyperandrogenism: Flutamide
term ortisol excess •Muscle weakness proximally •Hypercortisolism: Metrypone
•Elevated BP, immune suppression •Disease: high ACTH and suppression •HTN: K+ sparing diuretics, CCB
Cushing Disease: of cortisol on high dose
•Increased ACTH Monitor/treat cortisol related conditions:
secretion from •24 hour urine free cortisol osteoporosis, psych, DM, hypokalemia,
anterior pituitary *most specific (+) if 3x normal muscle weakness, infections

Causes: •Late night 11pm salivary cortisol


•Iatrogenic (steroid)
•Pituitary adenoma •Serum ACTH
•Ectopic ACTH <20  adrenal CT
•Tumor (SCLC, >20  MRI pituitary
medullary thyroid)

ENDO
Type I DM no insulin due to destruction of beta-cells (islet 3 Ps: Monitor: •insulin (multiple-dose basal bolus)
cells of the pancreas) *antibodies •polyuria (pee) •check glucose 3x daily -short acting: Humalog, Novalog
•polydipsia (thirst at home, increase fiber -basal: Glargine, Lantus, Levemir
•Islets of Langerhans infiltrated by lymph •polyphagia (hunger) •GFR + urine albumin/yr •Pramlitide (Symlin) *adjunct only
islet cell autoantibodies, T-lymph proliferate,
release of cytokines within infiltrated islets Labs: Dawn Phenomenon:
•Weight loss (low
•nml glucose  HIGH at 2am-8am
water) • glucose >200
preproinsulin (A, B, and C)  proinsulin (A & •decreased insulin sensitivity & nightly hormones
B)  insulin (C chain cleaves off) •postural hypotension •fast>126 (GOLD) •TX: bedtime injection of NPH, increase dose
•weakness, blurred •A1C >6.5%, GGT
Type IA (MC): autoimmune vision, neuropathy, •C-pep/Insulin ratio (low) Somogyi effect:
Type IB: non-autoimmune beta cell destruction fatigue *longer t ½ then insulin •hypOglycemia  rebound HYPER due to GH
(+)GAD-65 with type IA •TX: decreased night NPH, move earlier, or snack

Type II DM Risks: •polyuria, polydipsia •glucose >200 1st line-Metformin + diet + exercise
•(+) fhx; N. American, AA, Hispanics, Asians •blurred vision •fast>126 (GOLD) GLP1:“-tide”, DDP4: “-gliptin” SGLT2: “-glifozin”
•overweight (obese), birth wt >9lbs •poor wound heal •A1C >6.5%, GGT
•hyperglycemia •neuropathy Hypoglycemia: Sulfon, Meglit, Insulin
•dry and itchy skin •check glucose 1-2x/d Affordable: Metformin, Sulfonylureas
Insulin functions: glucose transport; stops •weight loss (5-10%) Weight Loss: GLP-1, SGLT2 inhibitors;
protein & TG breakdown, and gluconeogenesis •caloric restriction Weight Gain: Sulfonylureas, TZDs, insulin
•eye exam, neuropathy
•A1C 2x/year Screening:
Incretin: hormone that stimulates the release of
ADA: All adults 45+ q3 years or any adults with
insulin and decrease blood glucose
BMI 25+ and 1 additional risk fractor
Yearly eye exams USPSTF: 40-70yo q3yr if overweight or obese

Diabetic •metabolic acidosis 1+ days of ↑ thirst with •Glucose: 350-900 1-Fluid: 1L/hr 0.9% NS or LR 1-2h
Ketoacidosis •insulin insufficiency and counterregulatory polyuria and polydipsia •pH: acidic (6.8-7.3) *if Na rises, change to 0.45%
hormone imbalance
GI: N/V, fruity breath •Potassium nml or ↑ 2-IV regular insulin 0.1U/kg/hr
•Relative/absolute insulin def. •Sodium ↓Bicarbonate ↓ *change to dextrose when glucose 200mg/dL
•Excessive counter hormones Cardio: ↓ BP, ↑ HR Goal glucose: 150-200mg/dL
•↓ Bicarbonate b/c depleted by ketoacids Renal: ↑BUN/Cr, ↓GFR
•↓ Insulin, ↑Glucagon Neuro: alert, drowsy, 3-Potassium 10meq if <5.2
-↑ hepatic gluconeogenesis and glycogenolysis confusion, coma Ketones: acetone, 4-Bicarb if pH <7.0 (50meq/L/2hrs)
-↑ free FA release -> ↑ ketones acetoacetate, B-hydroxy *important in determining sevirty of DKA
*triggered during increased physiologic need Respiratory: kussmaul *serum checks B-hydroxy,
SIPS: saline, insulin, potassium, search for cuase

Hyperglycemi Insulin deficiency and inadequate fluid intake polyuria, polydipsia, Glucose: 600-1200 1-Fluid management
a weakness, decreased oral pH: nml or minimal acid Infusion 1-3L/hr 0.9% NS 1-3h *Na >150  0.45%
Hyperosmolar Triggered by stress: intake, wt loss
State •infection (MC for both), infarction, ↓ water •K+ and bicarb nml 2-IV regular insulin 0.1U/kg/hr *NO if K <3.3
•Neuro: altered mental *change to dextrose when glucose 250mg/dL
Renal: ↑BUN/Cr, ↓GFR Goal glucose: 250-300mg/dL
•Cardio: ↓ BP, ↑ HR
Osmolality: 330-380 3-Potassium-10meq/hr, ↑ if K+ <3.5

Drug MOA Advantages Disadvantages Contraindications


Binguanides Inhibits hepatic gluconeogenesis Improves glucose N/V/diarrhea, anorexia, metallic taste Allergy
Metformin Decreases intestinal absorption *no hypoglycemia B12 deficiency *check levels! Acidosis
*FIRST LINE Inexpensive BBW: lactic acidosis CHF
Weight loss *avoid CRF, liver failure, ETOH CKD (GFR <30)
Improves lipids (especially TG) Nephrotoxicity: STOP if hospitalized, Hospitalization
dehydrated, or contrast Radiocontrast

Thiazolidinediones “unlock” muscle and fat cells and help Improves glucose Weight gain Allergy, CHF
“Locksmiths” them ultize glucose *no hypoglycemia Peripheral/macular edema, CHF
Rosiglitazone  Improve insulin sensitivity Pioglitazone improved HDL Rosiglitazone worsens TC, LDL, increases Caution:
Pioglitazone  Increase glucose uptake HDL Osteoporosis
BBW: both-CHF, Ros-MI Liver disease

Sulfonylureas “Electricians”: Improves glucose Associated with hypoglycemia Allergy to sulfa


Glimepiride “zap” pancreas to stimulate increased Inexpensive *Glipizide and glumpiperide lower risk Diabetic ketoacidosis
Glipizide, Glyburide production of insulin Short onset Weight gain
Meglitinides Caution:
Repaglinide Bind to K+ channels of B-cells -> Liver/renal disease
Nateglinide depolarization -> insulin release Cardiovascular disease
a-Glucosidase “Block” the breakdown of starches in Improves PPBG Flatulence, elevated LFTs Allergy
Inhibitors intestine *no hypoglycemia hypoglycemia w/ sulfons or insulin Diabetic ketoacidosis
“Offensive Linemen”  Delayed carb absorption Cirrhosis
Acarbose, Miglitol  Lower postprandial glucose Caution: CKD, liver disease Major Chronic GI!

SGLT2 inhibitors “Halt” renal glucose absorption Lower glucose ind. Of insulin Dehydration GFR <45-60, Allergy
“The Traffic Cops” increase renal excretion by inhibiting the Weight loss (2-5kg) Genital yeast infections & UTIs
*end in –glifozin” protein Can lower blood pressure Increased LDL Caution:
Act of proximal tubule Masks ketoacidosis Osteoporosis, CKD

GLP-1 Agonists “mimic” incretin GLP-1 Improves both FBG and PPBG *INJECTION! Allergy
“Imitators” increase insulin release, decrease *no hypoglycemia Risk of hypoglycemia with sulfons Phx, fhx thyroid CA or
all in end “-tide” glucagon release, decrease gastric Weight loss 3kg N/V/D, slows motility MEN2
empyting, increase satiety Decreased appetite BBW Thyroid Cancer

DDP-4 Inhibitors “stretch” out the effects of endogenous Improves FBG and PPBG GI SE *less than GLP-1 allergy
“Stretchers” incretin GLP-1 Weight neutral May be expensive
*similar to GLP-1s but oral dosing Pancreatitis

Amylin Analogs “impersonate” the effects of amylin Improves FFBG and PPBG SC injection! Allergy
“Impersonators” decrease glucagon release, decrease weight loss BBW-hypoglycemia with insulin Gastroparesis
Pramlintide (Symlin) gastric emptying, increase satiety may be used with T1DM or T2 Nausea, vomit, anorexia Unknown hypoglycemic

SECOND LINE: SGLT2 inhibitors, GLP-1 agonists, DDP-4 inhibitors

Insulin Drugs Onset Insulin Coverage


Rapid-Acting Lispro (Humalog) Onset: 5-15 min Peak: 45-57 minutes Duration: 2-4 hours •Given at the same time of the meal
Aspart (Novolog) *usually with intermediate or long acting

Short-Acting Regular Insulin Onset: 30 min Peak: 2-4 hours Duration: 5-8 hours •Given 30-60 minutes prior to meal
*usually with intermediate or long acting

Intermediate NPH Onset: 2 hours Peak: 4-12 hours Duration: 8-18 hours •Covers insulin for ½ day
Lente *combined with rapid or short actin
*NPH usually given at bedtime

Long Acting Detemir Onset: 2 hours Peak: 3-9 hours (Detemir), no peak (Glargine) •Covers insulin for 1 full day
Glargine Duration: 6-24 hours (Detemir), 20-24 hours (Glargine) •Glargine causes less hypoglycemia than NPH
Lantus, Levemir *Should no be mixed with other insulin in same syringe

Complication DM About Characteristics Treatment


Hypoglycemia Blood glucose <70 Autonomic: sweating, tremors, palpitations, nervous, tachy, cool •15-20 G fast-acting carbs
CNS: HA, lightheaded, confusion, slurred speech, dizzy, irritable •IV D50W or glucagon
Diabetic Retinopathy Nonproliferative: initial stage Non-proliferative: Microaneurysms, Dot hemorrhage, Retinal exudates
*leading cause of blindness •Retinal capillaries leak proteins, lipids, or RBC
Proliferative: same plus growth of new vessels, cotton wool spots
•Proliferative: new capillaries release VEGF

Diabetic Nephropathy *more likely in T1DM Initial: proteinuria  24hr urine for protein=best
*can lead to nephrotic syndrome with massive
proteinuria, edema, HLD, hypoalbuminemia Biopsy: Kimmel-Wilson leasion (pink hyaline material)

Peripheral Neuropathy Distal symmetric: most common! Distal symmetric:


•Denervation of foot muscles -> toe clawing and •Symmetric, bilateral loss of sensation, vibratory sense, and temp
displaced fat pabs  ↑pressure  ulcer • “stocking-glove” distribution, Charcot arthropathy

Autonomic Neuropathy •Gastoparesis: delayed gastric emptying •Urinary: incomplete bladder emptying *control hyperglycemia
DX: upper endoscopy  nuclear scintigraphy •Orthostatic hypotension
TX-botox, Metoclopromide, Erythromycin •Erectile dysfunction

Cardiovascular •5x risk of complications •Hyperglycemia, hyperlipidemia, inflammation, HTN, oxidative stress,
•3x risk of cerebrovascular complications abnormal platelet function and coagulation
•30x risk of gangrene of the feet •Treat: aspirin if >10% 10-yr risk

Immune More frequent, more severe infections •Impaired phagocytosis; impaired flow  immune cells cant travel
•High glucose helps organism growth

Skin •Candida
•Eruptive cutaneous xanthomas
•Pigmented pretibial papules/diabetic
dermopathy/shin spots
•Acanthosis nigricans
•Necrobiosis lipidica diabeticorum

TSH: best test for screening, Free T4: metabolically active hormone, ordered when TSH is abnormal , Free T3: use if TSH low and T4 normal
Thyroid Antibodies: Graves (Anti-TSI), Hashimoto (Anti-TG and anti-TPO)
Primary: HIGHs TSH and low T4, Secondary: low TSH, low T4
Radioctive Iodine: diffuse uptake: Graves, TSH adenoma; decreased uptake: thyroiditis; hot nodule: toxic adenoma; multiple: toxic multilobar; cold: r/o malignancy

Subblicnical Hypothyroidism: elevated TSH alone, normal free T3/T4; treatment is observation

ENDO Etiology Clinical Presentation Complications/Management


Cretinism Untreated congenital •mental development delays •high TSH •Levothyroxine
hypothyroidism •short stature •low T3, T4
•microglossia, coarse facial features, umbilical
Causes: hernia
•lack of maternal iodine •hypotonia (decreased DTRs)
•dysgenesis of thyroid gland •jaundice, feeding problems

Subacute •Likely due to viral infection (URI) •Acute enlargement of thyroid •↑ ESR level High-Dose aspirin, NSAIDS
(Granulomatous •hyper  euthyroid  hypothyroid •Pain: worse with head movements & swallow •(-) antithyroid antibody titer +/- corticosteroids if severe
, DeQuervans) •starts in lower neck and radiates to jaw & ear BB for acute symptoms
Thyroiditis *MC-young & middle-aged women •Dysphagia Biopsy: granulomatous
*summer •Malaise, low-grade fever, Thyrotoxicosis inflammation with Severe: iodinated contrast agents
multinucleated giant cells

Suppurative Bacterial infection of thyroid gland by •Severe pain, tenderness, redness, and •↑ ESR level; ↑ leukocytes •Antibiotics
Thyroiditis Gram (+) Staph. aureus •fluctuance •Normal thyroid & antibodies •Surgical drainage of abscess
•Fever, chills, pharyngitis
*only two causes of painful thyroids Thyroid US-abscess presence Complications:
are subacute and suppurative •Abscess
FNA biopsy + gram stain + Cx •Chronic sinus tract formation

Riedel •systemic fibrosis of thyroid and •Asymmetric, stony, adherent gland •TSH varies •Tamoxifen and/or steroids
Thyroiditis adjacent neck structures •non-tender, rapid growth •IgG4
•Dysphagia, dyspnea, pain, hoarse •surgery for decompression
•middle aged or elderly women Biopsy: dense fibrosis
*clinically similar to anaplastic thyroid CA *distinguish from anaplastic CA

Sick Euthyroid abnormal thyroid function studies in MANY! •Cardo, renal, pulm, GI, liver •decrease T3
Syndrome the setting of severe non-thyroidal •Sepsis disease •increased reverse T3
illness •Starvation/anorexia •Inflammatory •normal T4
•Burns •Trauma •TSH varies
Pathophysiology •Cancer •Surgery
•severe illnesses decrease peripheral *rT3 ↑ with stressor
conversion/deiodination of T3/T4

Toxic Adenoma •multiple hyperfunctioning •symptoms of thyrotoxicosis •decreased TSH and high T4 •RAI ablation
autonomous nodules •obstructive symptoms: dyspnea, dysphagia, •surgery for compressive sx
hoarsness, stridor RAI: multiple areas of iodine
•palpable thyroid gland uptake “hot nodules”

Thyroid Ultrasound: nml or decreased vascularity; RAI Uptake: typically low RAI uptake; Thyroiditis Complications: thyrotoxicosis, thyroid storm, hypothyroidism, depression, cancer
ENDO
Hyper- Causes: HIGH BMR, increased DTRs Thyroid labs: Treatment: PTU
thyroidism •Graves Disease (MC) •Heat intolerance •Primary: ↓TSH, ↑ FT4
•Toxic multinodular goiter •Weight loss, hyperglycemia Hypercalcemia, ↑ alk phos, Anemia Graves: radioactive iodine
•TSH secreting pituitary adenoma •Warm, moist skin & hair Decreased granulocytes
•Excess intake •Hyperactive: anxiety, tremor, nervous,
•Iatrogenic thyrotoxicosis fatigue, weakness •Graves-(+) TSI,(+) anti-TPO, anti-Tg Thyroid Storm Definitive tx:
•Amiodarone •Tachycardia, high output HF, A-Fib increased RAI and vascularity radioactive iodine surgery
•Scanty period, diarrhea
Graves: •Thyroiditis: ↑ ESR
•autoimmune disease of TSH receptor Eyes: lid lag with downward gaze
antibodies target and stimulate TSH
receptor Graves dermopathy (pretibial
myxedema): red, rough plaques
Thyroid •Potentially fatal complication of HYPERMETABOLIC •Insulin resistance 1. Propranolol *reduce cardiac
Storm untreated thyrotoxicosis •N/V/D, high fever •Increased free T3 & T4 sx
•Tremors, psychosis (AMS) •Decreased TSH 2. PTU *block new T3/T4
Triggers: surgery, trauma, infection, •Lid lag 3. Iodine *blocks hormone relase
illness, pregnancy •Palpitations, tachycardia 4. Glucocorticoids *reduce
•Liver failure peripheral conversion

Definitive: radioactive iodine

Hypo- Causes: LOW BMR, decreaed DTRs •serum TSH +/- FT: ↑ TSH, ↓ FT4 Treatment: Levothyroxine
thyroidism •Iodine deficiency •Cold intolerance
•Hashimotos thyroiditis (autoimmune) •Weight gain, hypoglycemia Hashimoto: •Treament Myxedema:
•Potpartum thyroiditis •Constipation, menorrhagia • (+) (anti-TPO) *MC and (=) anti-Tg IV levothyroxine (LT4)
•Pituitary hypothyroidism •Dry, thick skin; brittle nails
•Hypothalamic hypothyroidism •Loss of outer 1/3 eyebrows Imaging:*not indicated in simple cases •Supportive Myxedema: blankets,
•Non-pitting edema (Myxedema) •thyroid US intubation, tx underlying
Primary: failure of thyroid T3, T4 •Hypoactivity: fatigue, sluggish, •CT MRI views gland
Secondary: failure of pituitary TSH memory loss, depression
Tertiary: Failure of hypothalamus TRH •Bradycardia, low CO

Hashimoto Autoimmune thyroid disease due to anti- •Diffusely, enlarged, firm, finely (+) anti-TPO and (+) anti-Tg Observe if asymptomatic
s TPO and anti-TG nodular thyroid WITHOUT pain
*most common thyroid disorder •Hypothyroid sx, depression, fatigue RAI: diffuse decrease uptake: Hypothyroidism-•Levothyroxine
•loss of outher 1/3 of eyebrow
painless postpartum: autoimmune •+/- galactorrhea Thyroid US: diffuse heterogenous density Large gland/goiter
•Goiter symptoms: hoarseness, dyspnea •Levothyroxine suppression
Painless sporadic: FNA biopsy: lymphocytes, hurthle cells
subacute form of Hashimoto, not Postpartum-hyper  hypo *higher risk of spontaneous
associated w/ pregnancy Sporadic: hyper x1-2mo hypo Postpartum: (+) anti-TPO miscarriage in 1st trimester

Myxedema •extreme form of hypothyroidism •bradycardia & obtundation •LOW Na+ and glucose •IV levothyroxine
Coma •hypthermia •increased TSH, LOW T4 •warming, supportive
MC in elderly women with •hypoventilation, hypotension
hypothyroidism in the winter •hypoglycemia, hyponatreamia

Disease Signs/Symptoms Labs Imaging Results Management


Thyroid >90% of nodules are benign Order TSH *FIRST TEST Multinodular Goiter •Follow-up with regular palpation
Nodules and Follicular adenoma MC benign •if normal or high  FNA biopsy •Multiple hyperechoic nodules and •US every 6mo
Goiters •Enlargement of isthmus and
Risks: Thyroid US: *usually after labs thyroid lobes •LT4 suppression for nodule >2cm and
•exteme ages •determines if FNA is needed normal or high TSH
•history head/neck radiation •concerning-irregular margins, hypoechoic, Thyroid nodule *reduces emergence of new nodules
central vascularity, growth, >1cm •Single nodule, color flow Doppler
Symptoms: •Ethanol Injection *shrink benign
•most are asymptomatic FNA Biopsy *BEST TEST Solitary Thyroid nodule nodules
•compressive symptoms (large •nodules >1.5cm with normal TSH or •Discrete hypoechoic nodule
multinodular goiters) highly suspicious nodules Toxic Multinodular Goiter:
Multinodular Goiter •Methimazole +/- BB
Exam: RAI Uptake *hyperfunctioning tissue •SVC compression •Surgery
•benign: varies •hypofunction=cold, high cancer risk •Tracheal deviation
•malignant: rapid, fixed, no •hyperfunction=hot, low cancer risk Cancer, Hyperfunction, Toxic: Surgery
movement with swallowing
Toxic adenoma, Tosic MNG, Graves
•RAI therapy *shrinks nodules by
~60%

Thyroid papillary thyroid carcinoma *MC •Palpable, firm, nontender nodule/mass Serum Tg Surgery-First Line
Cancer •single thyroid nodule, painless •↑-metastatic papillary and >1cm cancer: total
•MC after radiation exposure Symptoms: follicular <1cm cancer: lobectomy
•least aggressive, slow, confined •Anaplastic-s/s of metastasis <4cm inderterminate: lobectomy
•Medullary-flushing, diarrhea Serum Calcitonin >4cm indeterminate: total
Follicular Thyroid Carcinoma *Medullary can be associated with Men IIa •↑-medullary thyroid carcinoma
•metastasize (distant MC) and b RAI therapy
• ↑ iodine uptake Serum CEA CI-prego, nursing
MC Met Sites: lung, lymph, bone •↑-medullary thyroid carcinoma *low iodine diet for 2wkbefore
Medullary Thyroid Carcinoma
•Secrete calcitonin, prostaglandins, Thyroid US *size, location of mass Chemo: aggressive CA
5HT, ACTH, CRH, other chemicals
•Local mets, NO good iodine uptake RAI Scan *use after thyroidectomy
•reveal metastatic tissue
Anaplastic Thyroid Carcinoma High uptake: follicular
•Most aggressive, worst survival
•Rapidly enlarging mass in MNG CT or MRI: Distant metastases
•Does not have good iodine uptake
•hard rock thyroid mass CA: COLD & LOW uptake

TSH •benign pituitary adenoma that •diffuse goiter HIGH TSH and T4 •transphenoidal surgery
Secreting secretes TSH •hyperthyroidism SX •somatostatin analogs
Pituitary •compression os structures  bitemporal Radioactive iodine: DIFFUSE
Adenoma hemianopsia (optic chiasm), headache
Order MRI to detect adenoma

PHARMACOLOGY ***all BBW against the use for obesity *don’t take with food Monitoring: TSH every 4-8wks after change/start, then q6-12h
Drug Indication/MOA Monitor SE CI
Levothyroxine (T4) Hypothyroidism Monitor: Cardio: angina, palpitations, tachycardia, arrhythmia, CHF, Hypersensitivity
(Synthroid, Levoxyl, TSH suppression •check TSH levels every 6 weeks flushing Acute MI
Levothroid) Thyrotoxicosis
MOA: synthetic •best taken in the morning on an EMPTY CNS: anxiety, fatigue, insomnia, irritability, Adrenal insufficency
FIRST LINE thyroxine (T4) STOMACH pseudotumor cerebri (peds)

Endocrine: menstrual irregularities, wt loss


GI: abd cramps, diarrhea, vomiting, ↑appetite

Liothyronine (T3) Hypothyroidism 25mcg qd; titrate 12.5-25mcg/1-2wks Cardio: hypotension, tachycardia, arrhythmia, MI, CHF
(Cytomel, Triostat) Myxedema *take with water 1hr before food
Goiter suppression CNS: twitching, nervousness, irritability,
elderly/CAD-
5mcg qd, titrate 2mcg per 2wks Endocrine: menstrual irregularities, wt loss
GI: abd cramps, diarrhea, vomiting

Liotrix Hypothyroidism 25/6.25mcg daily; titrate every 2-3wks Cardio: tachycardia, arrhythmia, HTN, palpitation Hypersensitivity
(Levothyroxine/ *take with water 1hr before food Thyrotoxicosis
Liothyronin) CNS: anxiety, insomnia, nervousness, irritability Adrenal insufficiency
(Thyrolar) elderly/CAD-
12.5/3.1, titrate 2-3wk Endocrine: menstrual irregularities, wt loss
GI: abd cramps, diarrhea, vomiting
Desiccated Thyroid Hypothyroidism 15-30mg daily; titrate every 6wks Cardio: angina, palpitation, tachycardia, arrhythmia, CHF Hypersensitivity
(armour thyroid, *take with water 1hr before food Thyrotoxicosis
nature-throid, CNS: anxiety, fatigue, insomnia, irritability, Adrenal insufficiency
westhroid) *don’t use in elderly Beef or pork allergy
Endocrine: menstrual irregularities, wt loss
65mg=100mcg levo GI: abd cramps, diarrhea, vomiting, ↑ appetite

Thiourea Drugs MOA SE CI &Monitoring


Methimazole Inhibits organification of iodine, blocking formation of thyroid hormone Methimazole carries greater risk of teratogenicity and Hypersensitivity
(tapazole) goes more into break milk than PTU Breast Feeding (Canada)
•prevents thyroid hormone synthesis •Pruritis, rash, urticarial, Joint pain *PTU only
•Abnormal taste, N/V, fulminant hepatitis
USE IN 1st TRIMESTER & STORM •Agranulocytosis, aplastic anemia Monitor:
•Thyroid
Propylthiouracil Inhibits organification of iodine, blocking formation of thyroid hormone PTU greater risk hepatotoxicity than methimazole •LFT
(PTU) AND decreases conversion of T4 -> T3 BBW hepatotoxicity •CBC
*first line for prego or breast feeding

PSYCH 5%-insomnia

Disorder About Presentation Findings Treatment


Anorexia •Refusal to maintain a normal body weight LOW BMI (>17.5kg/m2) or body weight <85% ideal •metabolic alkalosis 1st line-multimodal
Nervosa •Morbid fear of gaining weight •Leukocytosis & leukopenia (therapy, SSRI, nutrition)
•distorted body image •emancipation (thin), depressed, fatigue •hypogonadotropic
•MC in women and teenage girls •bone pain, osteoporosis, amenorrhea hypogonadism Hospital:
•highest death rate of all psych conditions! •abd pain, constipation •Anemia •<75% ideal body wt
•hair loss, brittle nails, lanugo •Hypokalemia •electrolyte imbalances
Types: •Russel Sign •Hypothyroidism •cardiac abnormalities
•Restrictive: reduced calorie intake, dieting, •hypothermia, decrease HR and BP •Increased BUN
excessive exercise, diet pills (dehydration)
•Purging: primarily engages in self-induced
vomiting, diuretic, laxative, enema
Bulimia •MC in females and late-teens •NORMAL/HIGH BMI •Metabolic alkalosis 1st line-multimodal
Nervosa •Recurrent binging & compensation •abdominal pain, constipation, hair loss •Hypokalemia •therapy
ONCE WEEKLY x3 MONTHS •teeth pitting or enamel erosion •Hypomagnesemia •SSRI-Fluoxetine
•increased heart rate •increased amylase •nutrition
Types: •puffy cheeks (parotid gland)
*self-worth influenced by
•purging: laxatives, vomiting, diuretics
shape and body weight
•non-purging: low kcal, diet, fast, exercise Russels Sign: calluses on dorsum of hand from vomit

Disorder About Criteria Treatment


Intimate •1/4 women, 1/7 men one in lifetime Clues: •screening, assessing, referring
Partner •women who leaves abusive partner has 70% > risk of being killed •contusions to chest ,breast, abdomen, face, neck •adress pateints directly with non-
Abuse •pregnancy abuse can make up 10% pregnancy-related hospital •MSK and “accidental injuries” threatening questions
admissions •multiple injuries in various stages of healing

Sexual •common ages 9-12 years old Clues:


Abuse •MC perpetrators: males, relatives, known by child •children with sexual knowledge
•initiate sex acts with peers or show knowledge
•bruises, pain, pruritis in genital or anal area

Physical •abuser often female and primary caregiver •cigarette burns, burns in stocking glow pattern
Abuse •laceration, healed fracture, subdural hematoma
•multiple bruises, retinal hemorrhages

Child •failure to provide basic needs of a child •malnutrition, failure to thrive, withdrawal, poor
Neglect hygiene

Disorder About & Criteria Symptoms Treatment


General •MC in females •restless/on edge •irritable •SSRIs (Paroxetine, Escitalopram,
Anxiety •fatigue •sleep disturbance Fluoxetine), SNRI (Venlafaxine)
Disorder •Excessive worry or worry a majority of days for •difficulty concentrating •headaches •Buspirone as adjunct *NO sedation
(GAD) 6+ MONTHS with 3+ symptoms •muscle tension •shakiness •Benzos *short term
•CBT x6-12 months
Panic Panic Attacks: Panic Attacks: 4+ hallmark sx of sympathetic overdrive Panic Attacks:
Attacks/ •Episode of intense fear or discomfort that develops •rule out life-threatening conditions (MI, thyrotoxicosis) •Benzos (Lorazepam, Alprazolam)
Disorder abruptly; usually peaks within 10min, last <60min
Physical Symptoms: Panic Disorder
•dizzy, trembling, choking, paresthesias, sweating, SOB, CP, •SSRI (Paroxetine, Sertraline,
Panic Disorder: chills, fear of losing control, fear of dying, palpitations, Fluoxetine, Citalopram)
•average age 20s, MC in females tachycardia, nausea, depersonalization or derealization •CBT
•Recurrent, unexpected panic attacks •combo is best
•2+ attacks x1 month and 1+ criteria sx (concern
about future, persistent worry, maladaptive behavior) Panic Disorder Criteria Symptoms Acute Attacks: Benzos
•4/13 physical symptoms •persistent concern/worry of another attack
•worry of losing control during attack
•maladaptive behavior

Phobic •Symptoms 10-15min prior to specific stress event •out of proportion 1st line: CBT/exposure therapy
Disorder •causes fear 2nd line: SSRI, Benzos
•6+ MONTHS of fear/anxiety about an situation •actively avoided, distress Specific, predictable trigger: Benzos

Social •6+ MONTHS of intense fear of social situation or •MC type of phobia-public speaking •CBT (1st line), SSRI/SNRI or both
Phobia/ performance where you can be scrutinized Performance: Beta Blockers
Anxiety
Agoraphobia •Fear of going into places where you cant escape Scenarios: •SSRI, CBT, or both
•public transportation •open or closed spaces
•6+ MONTHS of fear/anxiety about 2+ scenarios •crowd/lines •outside home

Disorder About Criteria Treatment


Bipolar I/II Risks: FHX, men=women, 20-30 year old Bipolar I: 1+ FULL MANIC or mixed episode and •1st line Mood Stabiliziers: Lithium
occasional depressive *reduces SI risk
Mania: abnormal or persistently elevated, expansive or •marked impairment in social/occupational function
irritable mood for at least 1 WEEK •mood (euphoria), thinking (racing thoughts, •Others: atypical antipsychotics
disorganized, distracted, grandiose), behavior (no (Ripseridone, Quetiapine, Olanzapine,
Hypomania: mania sx for < 1 week; no impairment sleep, impulsive) Ziprasidone)

Mixed: criteria for one and 3+ sx of other Bipolar II: 1+ MDD and 1+ hypomania episode
*Antidepressants may precipitate mania
Rapid: 4+ ep/yr

Cyclothymic •Similar to bipolar II but less severe 2+ YEARS of hypomania and depressive symptoms •Mood stabilizers (Lithium) and
•Gender: men=women (no more than 2 free consecutive months neuroepileptics

Drug MOA Indication Side effects DDI CI


Benzodiazepines Enhance GABA •Anxiety, panic •Drowsiness •ETOH •Pregnancy
Short: at receptor •insomnia, •dizziness •Opioids •Allergy
-Midazolam •ETOH withdrawal •decreased motor coordination •CNS depressants •Myasthenia gravis
-Triazolam •agitation •decreased libido •Anticonvulsants •Glaucoma
Long: •seizure •disinhibition •antidepressants
-Diazepam •procedural sedation •rebound anxiety, SI •antifungal
-Flurazepame
-Chlordiazepoxide Rare: respiratory depression

Buspirone •5HT-1a Anxiety •Dizziness, headache •Other Psych meds Allergy


receptor agonist •Drowsiness, nausea, HA •CNS depressants
•dopamine *NO SEDATION •Serotonin Syndrome
receptors
Hydroxyzine Histamine •Anxiety •Drowsiness •Potassium Allergy
(Vistaril, Atarax) receptor •muscle relax •dizziness •MAOIs 1st trimester prego
antagonist •antihistamine •dry mouth •CNS depressants *only use po route
•antiemetic •rash
•insomnia •respiratory depression

•Antidepressants take about 4-6 weeks to reach efficacy


•Duloxetine first line if depression + neuropathic pain

Serotonin Syndrome: increased serotonergic activity in the CNS due with initiation or change in srotonergic drugs (SSRI, SNRI, TCA, MAOI, Buspirone, triptans or combo)
- AMS, agitation, confusion, hyperthermia, tachycardia, diarrhea, N/V, spontaneous/inducible conus, hypertonia (increased DTR), tremor, mydriasis (dilated pupil)
- TX: discontinue medication, supportive care; NO antipyretics for hyperthermia

Disorder About Criteria Treatment


Major Risks: FHX, female, 20-40yo 2+ WEEKS of 5+ SX •Psychotherapy
Depressive •must include anhedonia or depressed mood
Disorder Screen: •SSRI x4 weeks
(MDD) •PhQ2  PhQ9 if (+) •other symptoms: depressive mood, anhedonia, fatigue, *if not affects after 4 weeks then adjust
•Zung Self-Rated insomnia or hypersomnia, guilt or worthlessness,
recurring thoughts of death, psychomotor agitation,
Pathophysiology: weight loss, appetite change
•Alteration in neurotransmitters serotonin, epi, norepi, •Somatic SX: constipation, skin changes, pain
dopamine, ach, histamine
•Neuroendocrine dysfunction (adrenal, thyroid, GH) Catatonic: motor immobility, stupor, withdrawal
Melancholia: anhedonia, lack of reactivity, depression,
weight loss, excessive guilt, agitation
Seasonal •depressive symptoms at the same time each year •SSRI, light therapy, Buproprion
Affective (MC is winter “winter blues”)
Disorder
Atypical •mood reactivity (improved mood) in reposonse to •weight gain, appetite increase •MAO inhibitors (Phenelzine,
Depression positive events •hypersomnia Tranylcyprimine, Isocarboxazid)

Persistent •Patients are usually able to function Persistently depressed for 2+ YEARS with 2+ •SSRI and therapy
Depressive •No SX of hypomania, mania, or psychotic features symptoms (no more than 2 consecutive free months)
Disorder
(Dysthymia) •MC in women and teens/early adulthood

Grief •altered emotional state in response to loss •usually resolves in 6 months to 1 year •Psychotherapy
Reaction •peak in the first couple months •short course Benzos for insomnia
5 Stages: denial, anger, bargain, depression, acceptance Abnormal Grief: >1 year or positive SI

Premenstrua PMS: cluster of physical, behavior, and mood changes with •Physical: bloating, fatigue, breast swelling or pain •lifestyle modifications
l Dysphoric cyclical occurrence during luteal phase of menstrual cycle •Emotional: irritability, depression, anxiety, hostility, •SSRI (1st line for emotional)
Disorder libido hanges, aggressiveness •OCPs (Drospirenone contain)
PMDD: severe PMS with functional impairment •Behavioral: food cravings, poor concentration

Diagnosis: onset 1-2 weeks before menses, relevied


day 2-3 menses, 7 days free during follicular

Suicidal •recognizing the problem, assessment of risk, and SADPERONS: 5+ points is high risk Low risk: indicates remorse, shame or
Patient development of a treatment plan S: sex: male *female more often, male more affective embarrassment at suicide attempt
A: age: middle (45-64)
Recognition of SI D: depression High risk: sits quietly, engages poorly with
variable presentation: P: previous attempt *strongest predicitive factor physician, voices regret for surviving
•obvious attempt to cause self-harm E: ethanol use •require hospital admission to psych
•less obvious presentation: MVA or exchange of gunfire R: rational thinking loss service or medical with psych consult
•vague complaints, unwillingness to expand on thoughts S: social support lacking
O: organized plan Assure patient safety
• general questions about the patients emotional state N: no spouse (alone)
“what are your feelings about dying? S: sickness
“what has stopped you from killing yourself so far?”

Depression MOA CI Side effects Differences


SSRIs Selectively decreased •Allergy •N/D, anorexia •Sertraline: diarrhea, less QT, drowsy
FIRST LINE action of 5-HT reuptake •MOAI w/n 2 weeks •Sleep changes, HA, anxiety, dizziness •Citalopram/Escitalopram: more QT, least liver
*Fluoxetine 5wks •Decreased libido, anorgasmia, ED •Fluvoxamine: shorted t ½ and CYP
•Prolonged QT, WT gain, bleeding •Fluoxetine: long t ½ and don’t use with Tamoxifen
•Serotonin syndrome, increased SI •Paroxetine: anticholingeric SE, CYP, don’t use with
Tamoxifen *panic disorders 1st line
SNRIs Block reuptake of 5-HT •Allergy •N/D/V, constipation, dry mouth •Venlafaxine: high SE, elevated BP
2nd line if cant and NE (Milnacipran and •MOAI w/n 2 weeks •Sleep changes, HA, anxiety, dizziness •Desvenlafaxine: less HTN
tolerate SSRIs Levomilnacipran greater) •Angle closure •Decreased libido, anorgasmia, ED •Cymbalata: least associated with BP
glaucoma •Diaphoresis, HTN, SS syndrome •Milnacipran/Levomilnacipran: anticholinergic SE
•LESS SEX and NO WEIGHT GAIN!

Atypicals Buproprion: •Buproprion:, seizure, Buproprion: *NO WT GAIN OR SEX


Buproprion dopamine and NE reuptake anoremia, MAOI 2 weeks •dry mouth, insomnia, nausea
Remeron inhibitor •seizures, tobacco cessation
•Remeron: MAOI 2 wks
Remeron: antagonizes Remeron:
alpha-2 and 5-HT2/3 •dry mouth, drowsiness, sex dysfunction
•wt gain, increased appetite

Serotonin Nefazadone/Trazadone: •Allergy HA, N/D, SI risk, serotonin syndrome •Nefazadone: BBW-hepatotoxicity
Modulators Antagonize 5-HT •MOAI w/n 2 weeks  Drowsiness, xerostomia, hypotension
*with initiation and increase in dose •Trazadone: SEDATION, dry mouth, WT NEUTRAL
Vilazadone/ Vortioxetine:  Rare: priapism, cardiac arrhythmia
Partial agonist 5-HT •Vialazdone/Vortioxetine: N/V/C/D, sex dysfunction
 Faster onset and less sexual dysfunction

MAOIs MAOa: Break down •Allergy *MANY DDI INTERACTIONS Selegiline (Eldepryl): low doses for Parkinsons
Parnate serotonin and NE •Serotonin w/n 2 weeks •hypotension  Less CI than other MAOIs
Nardil •Cardiovascular •GI, urinary hesitancy  Less hypertensive crisis with transdermal
Marplan MAOb: •Pehochromocytoma •HA, myoclonic jerks
Eldepryl Break down dopamine •Hepatic/renal •edema
•Hypertensive crisis-foods with
tyramine

TCAs Inhibits reuatake of •Allergy •Anticholinergic, drowsiness, sweating Nortiptyline and Desipramine: highest tolerability
5-HT and NE •MOAI w/n 2 weeks •sexual dysfunction, wt gain & appetite
•Acute recovery of MI •tremor, OD fatality Tertiary(5-HT): Amitriptyline, Doxepin, Imipramine
•Cardiotoxicity (QT) Secondary (NE): Nortrip, Despiramine, Protriptyline

TeCAs •Ludiomil: •Less anticholinergic and more *have extra cyclic ring
block NE & 5-HT antihistaminic than TCAs *last resort, don’t ever really prescribe
Ludiomil •SI risk
Asendin •Asendin: blocks NE,
dopamine

Bipolar Labs/Indication Side effects DDI CI


Lithium Indication: Acute: GI, tremor, thirst, polyuria, weight gain, •Diuretics •CKD, dehydration, sodium
•Acute mania/hypomania or loose stools •NSAIDS depletion
maintenance •ACEIs •cardiovascular disease
Long term: LITH-PA •Tetracyclines •pregnancy
•Antidepressant *several wk onset •Leukocytosis, insipidus (renal), tremor, •Metronidazole •Ebstein’s anaomly
•Reduced SI risk & relapse risk hypothyroidism, parathyroid, arrhythmia •theophylline •increased lithium toxicity
Valproate •Bipolar I/II N/V, HA, hair loss, bruising, weight gain, •TCAs •Allergy
*Depakote tremor, dizziness •anticonvulsants •Liver
enteric coated Labs: •Mitochondrial
*increase GABA •Serum drug. LFT Rare: hepatotoxic, pancreatitis, low platelets •Pregnancy

Lamotrigine •Bipolar I/II Nausea, rash, pruritis, drowsiness, MANY Allergy


*inhibit *safest for pregnancy
glutamate Labs: Serum drug, LFT, renal Rare: multiorgan hypersensitivity. SJS, TEN

Carbamazepine •Bipolar I/II Nausea, rash, pruritis, hyponatremia, fluid MANY •Allergy
retention, leukopenia •TCAs
Labs: •MAOI w/n 2 weeks
•Serum drug levels Rare: bone marrow suppression, aplastic •Bone marrow suppression
•LFTs, CBC, sodium anemia, SJS, TEN •Pregnancy

Alcohol Dependence: When withdrawal symptoms develop or tolerance


Substance About Intoxication Withdrawal Treatment
Alcohol Drink: Alcohol Intoxication: •uncomplicated (6-24h): increased CNS Screening: CAGE 2+ is (+)
•12oz beer •8oz malt liquor •Slurred speech activity (tremors, anxiety, diaphoresis, C: cut down
•5oz wine •1.3oz spirits •incoordination, unsteady gait palpitations, insomnia) A: annoyed
•nystagmus G: guilt
At-Risk Drinking: •impairment in memory or attention •withdrawal seizure (12-24h): E: eye opener
Continuous use of alcohol •coma or stupor MC tonic-clonic
-men: 4 drinks/day or 14 drinks/week Withdrawal:
-women: 3 drinks/day or 7 drinks/week Wernickes TRIAD: •alcoholic hallucinosis (48h): hallucinations •Benzos, Clonidine,
confusion, ataxia, ophthalmalgia with normal vital signs Barbiturates, Anticonvulsants
Moderate-Severe Drinking: •IV fluids, Thiamine & Mg
•Recurrent use of alcohol despite risks Karsakoff: •delirium tremens (2-5d): delirium, •Glucose
amnesia, aphasia, apraxia, agnosia hallucinations, agitations, abnormal vitals
Tolerance: need for increased amounts Chronic use TX: Naltrexon,
of alcohol; diminished effect with S/S: sweating, tachycardia, increased hand Campral, Antabuse
continued use of same amount tremor, insomnia, N/V, hallucinations, •Thiamine
agitation, anxiety, seizure

Substance Intoxication Withdrawal Treatment


Marijuana •Takes effects in about 10-20 minutes Withdrawal *usually only with heavy use Urine drug test can detect for 4-6
•Cannabis binds to CB1/2 cannabinoid receptors •irritable days and up to 50d in chronic users
•insomnia & restless
Intoxication •diaphoresis Treatment:
Moderate dose: •diarrhea Symptomatic
•euphoria, giddiness •twitching
•dry mouth (cotton mouth)
•conjunctival erythema Complications of long-term use:
•tachycardia, hypotension •laryngitis & rhinitis
•low testosterone and low sperm count
High Dose: hallucinations, paranoia, delusions

Tobacco Withdrawal Symptoms: (+) effects: decrease anxiety and appetite, increase mood, •Nicotine Replacement
•restless alertness, soling •Buproprion (Wellbutrin)
•anxiety •Verenicline (Chantix)
•irritable (-) effects: CA, DM, COPD, asthma, dental, infection
•sleep abnormalities
•depression Tolerance due to UP REGULATION of receptors
•nicotine craving
•weight gain

Alcohol Chronic Use


Drug MOA Side Effects CI/DDI
Thiamine •Low BP
(B1) •Effect glucose metabolism

Naltrexone Blocks dopamine release, antagonizes mu BBW: hepatocellular CI: opioid dependency
1st LINE receptor  decreases craving and reward •N/V/D/C, abd pain DDI: opioids
•dizzy, HA, anxiety, fatigue
Acamprosate Restores glutamate •Diarrhea, nausea, abd pain CI: renal (Cr <30)
(Campral)  Stops withdrawal S/S •fatigue, HA, amnesia, mood
1st LINE
Disulfiram Inhibits enzyme aldehyde dehydrogenase  •Metallic taste CI: heart or CAD, ethanol
(Antabuse) increases acetaldehyde  Fs you up •Effects of drug  sweating, HA, dyspnea, low BP, DI: “WAM”-warfarin, amitriptyline, metronidazole
2nd Line flushing, palp

Substance Intoxication Withdrawal Treatment


Hallucinogen PCP (NMDA receptor antagonist) *15-30 minutes PCP Labs: CK-MB, AKI, CBC, CMP.
PCP, LSD •S/S: euphoria, numbness, disorientation, physical S/S: depression, anxiety, irritable, restless, sleep disturbance urine drug test
detachment, body distortion, unusual strength, dilated
pupils, multidirectional nystagmus LSD Treatment:
•no withdrawal because it does not affect dopamine •Haloperidol
LSD (5-HT receptor) •Benzos
•S/S: visual hallucination, see sound as color, dilated •Low stimulus environment
pupils, delusions

Inhalent Intoxication Common Chemicals Antipsychotics if severe aggression


Paint, Mild/moderate: euphoria, slurring speech, confused, •Toluene
petroleum, hallucinations, watery eyes, impaired vision, rash •Butane, Propane
toluene, glue, •Fluorocarbons
nail polish High dose: cardiopulm failure, liver, kidney, bone •Chlorinated hydrocarbons
marrow suppression *all organ failure •Acetone

Opioid MOA: mu receptor agonist Withdrawal Intoxiation:


heroin, •lacrimation Naltrexone (1st line)
oxycodone, Intoxication: •HTN, tachycardia Methadone,
morphine, •euphora •pruritic Buprenorphine
meperidine, •sedation & impaired memory •piloerections (goose bumps)
codeine •slurred speech •pupil dilation OD: Nalaxone
•impaired social function •flu-like symptoms
•pupil constriction •yawning Withdrawal:
•respiratory depression Methadone
•bradycardia & hypotension Clonidine
•nausea, vomit, flushing Lofexidine
Suboxone
Biots breathing: quick, shallow inspirations  apnea

Sedative/ GABAa channel-increased frequency of opening Withdrawal: Intoxication:


Hypnotic/ •rebound anxiety •Flumazenil (GABA antagonist)
Anxiolytic Intoxication: •seizures and tremors
Benzos •respiratory depression Withdrawal:
•hypotension •long acting benzo (Clonazepam)
•ataxia, amnesia (forget things), coma, death

Stimulants Intoxication: Withdrawal: Intoxication: *do not restrain


Cocaine •mydriasis (pupillary dilation) •depression and SI •antipsychotics
Amphetamine •hyperactivity, euphoria •hyperphagia •benzos
Meth •perspiration •hypersomnolence & fatigue •antihypertensives (Labetolol)
•excessive talking  depression •craving •vitamin C
•weight loss, anorexia •vivid unpleasant dreams
•dry mouth Withdrawal:
•nose bleeds and sepal perforation (COCAINE!) Kindling: hallucination and paranoia •Bupropion •Bromocriptine

Opioid Overdose Treatment


Drug MOA Dose Side Effects
Naloxone Short-acting opioid antagonist Cardiorespiratory: 2mg
ventilations: 0.05mg IV
Opioid Use Treatment
Drug MOA Dose Side Effects
Naltrexone Blocks dopamine release, antagonizes mu receptor 25-50mg daily BBW: hepatocellular
1st LINE  decreases craving and reward Vivitrol (IV): 380mg IM/4wks N/V/D/C, abd pain, dizzy, HA, anxiety, fatigue
*opioid antagonist, completely blocks effects

Methadone Long-acting opioid agonist 20-30mg po, titrate up (80-120mg) •Constipation, drowsiness, sweating
•peripheral edema, hyperalgesia
•reduced libido, ED
•QT prolongation, OVERDOSE

Buprenorphin Partial agonist *often in combo with Naloxone •HA, nausea, pain
e *take home therapy 4mg B/1mg N daily •insomnia
most stabile on 16-20mg/d B •withdrawal syndrome

Taper by reducing 2mg/1-2wk Rare: liver, necrosis, anaphylaxis

Tobacco Use Treatment


Drug Use Dose Side Effects
Transdermal Apply to skin once daily, avoid hair, change places each •>10 cig: 21 x6wk, 14 x2wk, 7 x2wk •Skin irritation
Patch day •10 or less cig: 14 x6wk, 7 x2wk •Insomnia, vivid dreams

Oral Nicotine DON’T CHEW •Smoke w/n 30 min: 4mg •Mouth irritation
Lozenge •All others: 2mg •N/V/D, HA, insomnia
•Max: 5 every 6hrs or 20/day •palpitations

Oral Nicotine Diminishes withdrawal •25+ cigs: 4mg •N/V/D, HA


Gum “chew and park” method •all others: 2mg •excess salivation, mouth irritation
*avoid TMJ, poor dentition, dental appliances

Nicotine Absorbs through mucosa 6-16 cartridges/d for 6-12 wk Oropharynx irritation, bronchospasm
Inhaler Satisfies behavioral & sensory cravings *avoid RAD (asthma)

Nicotine Absorbed through nasal mucosa 1-2 sprays/3mo •Nasal and throat irritation
Nasal Spray Max: 10 sprays/hr or 80/day •sneezing, tearing

Buproprion Blocks dopamine and NE reuptake 150mg/d x3d, 150mg BID x12wk •Insomnia, agitation
(Wellbutrin) Antagonizes nicotinic rec. •dry mouth, HA
*start 1wk before quite date
•CI: Epilepsy, seizure, h/o anorexia/bulimia

Varenicline Partial agonist of nicotinic rec. 0.5mg x3d, 0.5mg BID x4d, 1mg BID •Vivid dreams
(Chantix) -Decreases withdrawal x12wk •nausea, insomnia, syncope
-Interferes with reward *start 1wk before quite date
Serious: neuropsych (SI, mood, behavior)
Disorder About & Criteria Symptoms Treatment
Post- •trauma at any time with symptoms > 1 MONTH AVOIDANCE (1+): effort to avoid measure, effort to avoid 1st line-CBT with exposure
Traumatic •Men: combat experience, urban violence reminders
Stress •Women: rape or assault •SSRI/SNRI
Disorder EXPOSURE (1+): direct experience, witness, learn it happened (Paroxetine, Sertraline, Fluoxetine)
(PTSD) PTSD Criteria: to someone close, extreme/repeated exposure
1. Exposure to actual or threatened death, serious •Trazadone is good to use insomnia
injury, sexual violence INTRUSTION (1+): distressing memories, distressing dreams,
- Direct experience of the event feeling or reoccurrence, psych reaction or stress
- Witness the event
- Learn the event happened to someone close NEGATIVE CHANGE IN MOOD/COGNITIVE (2+):
- Experience extreme or repeat exposure memory loss, exaggerated, distorted thoughts, negative state
2. Presence of symptoms (avoidance, exposure, constantly, inability to be positive, decreased interest,
intrusion, negative change, arousal/reactivity) detachment
3. ALL have to occur for ONE MONTH
AROUSAL/REACTIVITY (2+): irritable, reckless, startled
easily, sleep disturbance, difficulty concentrating

Acute Stress •traumatic event occurs <1 MONTH ago and •similar symptoms to PTSD •counseling & psychotherapy
Disorder symptoms last < 1 MONTH *different between the two is the timing

Adjustment •maladpative emotional or behavioral reaction to an •marked distress out or proportion to the severity of stressor •psychotherapy
Disorder identifiable stressor •may manifest as depressed, anxiety, conduct disturbance
•disproportionate response within 3 MONTHS,
usually resolves by 6 MONTHS

GU 5%-balanitis
GI
Hernia (Inguinal) Indirect (MC!): Direct: Incarcerated: •Surgical repair
•internal inguinal ring •Hesselbachs triangle: “RIP: rectus abdominis, •painful, enlarged and
•lateral to inferior epigastric inferior epigastric, pouparts (inguinal) ligament irreducible •Strangulated are surgical
•persistent patent process •medial to inferior epigastric emergencies
vaginalis •Does NOT reach scrotum Strangulated: systemic toxicity

Femoral Hernia Protrusion of the contents •below the inguinal ligament •often become incarcerate or
through the femoral canal •MC in women strangualted b/c small ring

Umbilical Hernia Occurs after closure of the •Increases steadily in size High rate complications: Observation: usually resolve by 2yo
umbilical ring •Umbilical hernias usually contain omentum •large size of the hernia
MC in women •Incarceration and strangulation are common •old age or debility of pt Surgical repair
•obesity •persistent after age 5
Risks: mult pregnancies, •Umbilical hernias with tight rings are often •intra-abd disease
ascities, obesity, intra- associated w/ sharp pain on coughing or straining
abdominal tumors •Aching sensation

Ventral Hernia Herniation through weakness Risks: •Small incisional hernias should be
(Incisional in the abdominal wall •Poor surgical technique, post-op complications treated early
hernia) •Postop wound infection •Elastic bind if CI to surgery
MC in ventral incisions in •advanced age, general debility, obesity
obese patients •placement of drains or stomas through primary *typically, surgeons do not fix this
operative fascial defect due to rectus abdominus fascia
•defects in collagen

GU
Glomerulo- Inflam glomeruliprotein, RBC in urine Classic presentation: UA: •High dose corticosteroids
nephritis •HTN •hematuria, proteinuria (<3g) *slows immune process
*intrinsic Types: •Edema •RBC casts
renal injury IgA nephropathy (Buergers)-MCC (scrotal, periorbital) •high specific gravity •Cytotoxic agents if cell mediated
•young males after URI or GI •hematuria (cola or tea-
Post-Infectious(high ASO, low comp.) colored urine) Labs: high BUN/Cr •Plasma exchange: Goodpasture
•MC with Group A Strep •Proteinuria •Complement low in immune complex disease and pauci-immune
•2-14yo boys, facial edema, cola/dark urine •RBC, WBC, RBC casts •ASO titers (post-infectious)
Membranoproliferative: lupus, hep(HCV) •Anti-GBM antibodies
Rapid Progressive (RPGN) •ANCA levels (Pauci-Immune)
•crescent formation on biopsy
GoodPastures Disease (anti-GBM) Renal Biopsy: GOLD
•glomerulonephritis + hemoptysis •IgA: IgA mesangial deposits
Vasculitis: polyangitis (P-ANCA) and •Poststreptococcal: immune humps, hypercell
wegeners (C-ANCA) •Goodpastures: IgG in basement membrane
Pauci-Immune: churg-strauss

Masses About/ Causes Signs/ Symptoms Diagnosis Treatment


Testicular •MCC testicular tumor in 18-40 •Painless enlargement of testis Labs: •Inguinal exploration with vascular control
Tumor/ -testicular or scrotal heaviness •Alpha-fetoprotein (AFP) of spermatic cord
Cancer Types: -painless nodule on testicle *nonseminomas •Radical inguinal orchiectomy
•germ cell: seminoma & non •Hcg *nonseminomas •Radiation/chemo with seminoma
•nonseminomas: high AFP & b- •acute testicular pain -10% •LDH
hcg, resistant to radiation •metastatic symptoms-10% •Advanced: anemia, LFTs, renal Follow-Up:
•seminomas: 4s: simple (lack •Monthly for 1st 2 years, bimonthly 3rd yr
AFP), sensitive (to radiation), MC site of metastasis-retroperitoneal Imaging/diagnosis: •Tumor markers at each visit
slow, stepwise spread abdominal lymph nodes •Scrotal US-initial evaluation •CXR and CT every 3 months
-Back pain, cough/dyspnea, anorexia, *transscrotal biopsy contraindicated •80% relapse in 1st 2 years after tx
Risks: N/V, bone pain, LE swelling
•Cryptorchidism •Definitive: -radical inguinal Prognosis: 90% + 5 years survival rates
•Exogenous estrogen in prego orchiectomy
•Infertility, Fhx HIV, ethnicity

Disease Causes/Risks Signs/Symptoms Labs/Imaging Treatment


Benign •More common with increasing age Usually begin slowly, progress gradually Labs: Pharm:
Prostatic *more sensitive to androgens & GF, •UA: normal, possible hematuria •Alpha-1 Blockers: Tamsulosin
Hyperplasi stops nml cell death Obstructive voiding-hesitancy, •5-alpha Reductase: Finasteride
a (BPH) •Increase in both glandular and stromal decreased force, incomplete bladder •PSA: can be elevated in BPH *reduce size of prostate
component emptying, double voiding (urinating
within 2 hr), straining, post-void dribble Surgical Procedures:
Risks: •Transurethral Resection of the
•Genetics (African americans more Irritative: urgency, frequency, nocturia Prostate (TURP) *MC!
likely, Asians less likely) *detrusor muscle hypertrophy and 2 mechanisms of obstruction: *can cause retrograde ejaculation
hyperplasic, collagen deposit •Mechanical: narrowed urethral •Tranasurethral Incision (TUIP)
•Increase Risk: Higher PSA levels, lumen or bladder neck •Open Simple Prostatectomy
prostatitis, heart disease, BB use, lack of DRE: smooth, firm, symmetric, elastic
exercise, obesity enlargement of prostate •Dynamic: A-receptor stimulation Minimally Invasive:
causing increased tone •Laser Therapy (TULIP)
•Decreased risk: NSAIDS, excessive Neuro: r/o neurogenic bladder (constriction) to prostatic urethra •Transurethral Needle Ablation of
ETOH, smoking, exercise Lower abd exam: evaluate distention Prostate (TUNA)
*alcohol & smoke suppress T •Transurethral Electrovaporization
•Hyperthermia
•Implant to Open Prostatic Urethra
Drug Class Drugs in class MOA SE Notes
SELECTIVE Prazosin (Minipress) Help with •Orthostatic hypotension, dizziness, combo of alpha blocker and 5-alpha reductase is
Alpha-1-blockade Doxazosin (Cardura) symptoms tiredness, retrograde ejaculation, rhinitis, HA first line
“-osin” Terazosin (Hytrin)
*doxazosin and Terazon more •Intraoperative floppy iris syndrome
effective but more SE

NON-SELECTIVE Sildosin (Rapaflo) DDI: Antihypertensives & PDE-5 inhibitors


Alpha-1a-blockade Tamsulosin (Flomax) *drop BP when taken with alpha blockers
“-osin” Alfuzosin (Uroxatral)

5-alpha-reductaste Finasteride reduces size of •Decreased libido •Don’t work overnight and may take up to 6 months
inhibitors Dutasteride *more efficacious prostate gland •Erectile or ejaculatory dysfunction to see treatment
Jalyn (combo of dutasteride and *more effective for very large prostates because
tamsulosin) reduce PSA by 50% and size by ~20%

PDE 5 Inhibitors Tadalafil (Cialis) BPH + ED •Not covered well by insurance

Phytotherapy Saw Palmetto •If doesn’t want to take meds- try this
*approved in Europe

GU About Clinical Presentation Diagnostics Treatment


Renal Types: •Acute, severe pain (flank) Labs: Supportive: pain (NSAIDS, opioids), fluids, antiemetic
Calculi •Calcium oxalate- MC! •radiate to abdomen or groin •hematuria (micro or gross)
(Nephro- •Uric Acid: protein, gout, chemi •Urgency and frequency •<5.5: uric acid or cysteine Pharm:
lithiasis) •Struvite: Mg Ammonium Ph •Nausea/Vomiting • 5.5-6.8: calcium oxalate •Alpha-1 blocker-Tamsulosin
•Cysteine: rare •Hematuria • >7.2: calcium phosphate/struvite •Steroids-Prednisone 10mg daily for 3-5d

Risks: Exam: CVA tenderness Frequent or + family history: Surgery:


•high protein and salt •Low Na & protein, high fluid •extracororeal shock wave lithotripsy
•inadequate hydration •<5mm pass spontaneously •uretroscopy w/ w/o stent  immediate relief if obstruct
•humidity & elevated temp. •10mm+ do NOT pass Diagnostics: •percutaneous nephrolithotomy  large >10mm, struvite
•sedentary lifestyle *stone size does NOT •KUB abdominal xray (cheapest)
•GI malabsorption syndromes correlate with severity •Renal US (good if noradiation) Prevention:
•HTN, obesity, gout •Non-Contast Spiral CT-BEST! •Increase fluids-best thing to do! (1.5-2L of urine/day)
•Diet (Eat bran to decrease calciuria)
*do not need to decrease calcium!

GU About Presentation Diagnostics Treatment


Acute •non-gonococcal v gonococcal •Irritative voiding (dysuria) Labs: Empiric treatment
Urethritis •Pain/pruritus at urethral meatus •>2 WBC/hpf *usually treat both G & C together
Bugs: •Gram (-) intracellular
•Neisseria gonorrhoerae (MC!) •Urethral discharge: diplococci Gonorrhea:
•Chlamydia trachomatis (2nd MCC) -nongonococcal: scanty/thin/watery •Ceftriaxone250mg IM x 1 dose
•Mycoplasma genitalium -gonococal: thick/purulent/copious Nucleic Acid: BEST TEST
•Trichomonas vaginalis •first catch or first void Chlamydia:
Exam: •Azithromycin 1g po x 1 dose
Risks: men, young & sexually active •+/- inflammation at urethral meatus UA: •Doxycyline 100mg po BID x7days
•Thick, purulent, copious discharge •+ WBC esterase & pyuria >10
MC route: ascent up urethra *gonorrhea •+/- hematuria

Epididymitis •often occur simultaneously due to an •gradual onset of milt to severe Ultrasounds of Scrotum Outpatient
and Orchitis underlying bacterial etiology unilateral testicular pain •r/o torsion <35: Ceftriaxone+Azithromycin or Doxy
-lower abd, inguinal canal, scrotum •enlarged epididymis >35: Fluoroquinalone, Bactrim
Orchitis: •+/- fever, dysuria, discharge (UTI) •increased blood flow
•isolated orchitis is often viral of •affected testis will hang low •elevation, ice, NSAIDs, stool softener
syphilitic (rare to find alone) *inflammation and edema pull down UA with C/S: pyuria •avoid lifting heavy objects and straining
•viral orchits MC due to mumps •follow up with urology in 5-7d f
(5d post parotitis; unvaccinated) Exam: Nucleic Acid Amplification
•swollen, tender, warm •test for G/C Admission  toxic or septic
Epididymitis •cremasteric reflex normal •fever, hypotension, tachycardia (shock)
•bacterial infection is most common •Phren (+) -workup: CBC, CMP, blood culture
•men <35: G/C *h/o sexual risk <35: Ceftriaxone + Azithromycin
•men >35: E. coli and Klebsiella >35: Bactrim OR Levofloxacin

Acute Bugs: Gram – rods most common •Fever, chills, malaise Labs: IV: fluroquinalone +/- aminoglycoside, or
Bacterial •>35 years old: E. coli, Pseudomonas, •Pain-perianal, sacral, suprapubic •CBC: leukocytosis, left shift amp/gent
Prostatitis Proteus, Klebsiella, Enterobacter •Irritative & obstructive voiding •UA: pyuria, bacteriuria,
•<35 years old: G/C hematuria Oral (MC): Bactrim 800-160mg BID or
Digital rectal exam (DRE): *KEY! •Urine culture: + for bacteria Fluoroquinalone (Cipro)
MC route: ascent up urethra hot, exquisitely tender prostate
Risks: catheter, biopsy, stricture, UTIs *prostatic massage CI due to sepsis Imaging: rarely indicated

Chronic Bugs: •Dull, poorly located, in suprapubic, Labs: Fluoroquinolones or Bactrim x6wk
Bacterial •Gram – rods most common perineal or low back pain •Prostatic secretions: increased
Prostatitis •E. coli •Irritative & obstructive voiding WBCs (>10) with a (+) culture SE of prolonged use:
•Pseudomonas, Proteus, Klebsiella, •HX UTI or (+) cultures C. diff, CNS, tendinopathy
•Enterococcus UA: nml unless cystitis a
DRE: nml (MC), may see boggy Supportive: anti-inflam, sitz baths
MC route: ascent up urethra (spongy), tender, enlarged, and/or Urine culture: negative; + for
indurated prostate organism AFTER massage!

Type of Stones Causes About/Diagnosis Treatment


Hypercalciuric •Absorptive hypercalciuria Absorptive hypercalciuria: Increased urinary Calcium even without high •Absorptive hyperclacuria-thiazide diuretics or
Stones *calcium •Resorptive hypercalciuria dietary calcium intake, high absorption in S.I. cellulose phosphate (bind to Ca)
•Renal hypercalciuria
Resorptive hypercalciuria: secondary to hyperparathyroid Resorptive hypercalciuria-treat elevated PTH
Renal hypercalciuria: normal or low serum Ca Renal hypercalciuria-thiazide diuretics

Hyperuricosuric Dietary purine excess or uric •Allopurinol (gout medication)


Stones *calcium acid metabolic defects

Hyperoxaluric Primary intestinal disorders H/o chronic diarrhea, usually have IBD •Stop diarrhea, avoid excess ascorbic acid
Stones Absorb oxalate more than calcium *cut down vit C tablets and/or orange juice
*calcium •Calcium carbonate

Hypocitraturic Chronic diarrhea, chronic •Reduce potassium citrate supplements


Stones *calcium HCTZ treatment, acidosis •Oral lemonade
Uric Acid Calculi Hyperuricemia, Urinary pH consistently <5.5 •Increase pH > 6.2 *helps treat patients
myeloproliferative disease, •Potassium citrate (alkalizes urine)
cancer, abrupt weight loss Xray: Pure uric acid is radiolucent *cant see on xray •Allopurinol 200mg/day if hyperuricemia

Struvite Calculi MC-women w/ recurrent UTIs Urease-producing: Proteus, pseudomonas, providencia “PPP” *usually cause staghorn calculi
Cystine Calculi Abnormal cystine excretion Xray: radiolucent; “smooth-edged ground glass” Urinary volume foal 3-4L, goal pH >7.0

About Presentation Diganosis Treatment


Acute Bugs: •Irritative voiding (dysuria) Labs: Short term antimicrobial Therapy
Cystitis •E. coli •Frequency urgency & frequency •Pyuria •Nitrofurantoin 100mg po BID x5-7d
“Bladder •Proteus •Suprapubic pain •Hematuria *safe in pregnancy
Infection” •Klebsiella •+/- gross hematuria •Bacteriuria •Bactrim 800/160mg po BID x3d
“UTI” •Pseudomonas •Leukocyte esterase •Fosfomycin (Monurol) 3g po x1
•Staph saprophyticus Exam: •Urinary nitrate •FQs, Augmentin, Cephalosporins
•Suprapubic tenderness •Urine culture + bacteria
MC route: ascent up urethra •NO CVA! Analgesics: Phenazopyridine (Azo)
•NO TOXICITY! COLONY COUNT >100,000 Other: baths, fluids +/- juice, probiotics
Risks: women, sex, spermicides, pregnancy *epithelial cells indicate
(P and E), elderly, infants contamination *DO NOT TREAT IF NO SX

Acute Bugs: •Irritative void (dysuria) Labs: Outpatient:


Pyelo- •E. coli •Suprapubic pain •Pyuria, hematuria, bacteriuria •1st line: FQ (Cipro or Levo),
nephritis •Proteus •+/- gross hematuria •+/- WBC Casts Bactrim, Augmentin
“kidney •Klebsiella •Fever, chills, N/V/D •Leukocyte esterase & Urinary
infection” •Pseudomonas •Flank pain nitrate •Adjunct/Pregnancy Tx:
•CX: + heavy growth of bacteria 1g Ceftriaxone (Rocephin)
MC route: Exam: •CBC: leukocytes w/ left shift
ascent up urethra •Fever Inpatient:
hematologic route •Tachycardia Imaging: •IV ampicillin/gentamicin, FQ
•CVA tenderness •CT: renal inflam, abscess •IV carbapenem (Imipenem)
•US: hydronephrosis, abscess •IV cephalosporin (Cefepime)
HEME 4%

HEME
Iron •MC anemia worldwide •Fatigue, weakness, dyspnea, exercise •High TIBC, RDW Ferrous Sulfate
Deficiency intolerance •Low iron, ferritin, transferrin *take with Vitamin C
Microcytic Causes: •hair thinning •Low MCV, low MCH, nml MCHC
•MCC blood loss (menstruation, GI) •pallor, pica, cheilitis *microcytic, hypochromic
•decreased absorption (diet, celiac) •koilycia, plummer vinson syndrome
Bone marrow: absent iron stores
Pathophysiology: *DEFINTIIVE
•decreased RBC production due to lack of
iron and decreased ferritin (iron stores)

Lead Pathophysiology: Neuro: ataxia, fatigue, learning Serum lead: >10 •removal of the source of lead
Poisoning •enzymes cause cell death  shortening disabilities, difficulty concentrating Capillary initial  venous best
Anemia life span of RBCs •wrist or foot drop <44mcg: outpatient follow-up and
•causes acquired sideroblastic anemia Smear: microcytic hypochromic lifestyle modfications
GI: abdominal pain, vomit, anemia with basophilic stippling
constipation, loss of appetite •ringed sideroblasts inmarrow 45-69mcg: succimer

Butrons lines: thin, blue-black line at X-rays: lead lines


base of the gums near the teeth

Alpha •Decreased alpha-globin •¼-silent •microcytic hypochromic •Moderate: folate, avoid oxidative
Thalassemia •MC in S.E. Asians •2/4- minor (trait) •normal/increased iron, ferritin stress, avoid iron supplementation
•¾- HbH (Heinz bodies)
Microcytic -Symptomatic at birth Smear: target cells •Severe: transfusions, splenectomy
-HSM, pigmented gallstones iron chelation (IV Deferoxamine),
-Frontal bossing Electrophoresis: all equal
•4/4- hydrops fetalis •HbH with ¾ deletion

Beta •Decreased beta-globin •½: Minor (trait)-asymptomatic electrophoresis: Minor: no treatment


Thalassemia •MC in Mediterranean •2/2: major (cooleys) •increased HbA2 and HbF,
-symptomatic around 6 months Major:
Microcytic HbF=2a2y -HSM, Frontal bossing & maxillary smear: target cells, nucleated cells •blood transfusions
HbA=2a2b, HbA2=2a2d growth, osteoporosis, cardiac (high •folate & vitamin C supplements
output HF) X-ray: crew-cut skull appearance •iron chelating (Deferoxamine)
“hair on end” appearance from
extramedullary hemtopoieses Definitive=bone marrow transplant
HEME/ONC
Folate Pathophysiology: •fatigue, exercise intolerance, pallor Labs: Folate supplement
Deficiency •folate required for DNA synthesis •Glossitis, cheilosis •high MCV and homocysteine
Macrocytic •deficiency causes abnormal synthesis of •NO NEUROPATHY •normal MMA
DNA, nucleis acis, metabolism
Smear: hypersegmented neutrophils,
Causes: macro-ovalocytes
•MCC=diet  leafy greens, alcoholic
•increased requirements (prego, infant,
malignancy)
•impaired absorption
•impaired metabolism: MTX,
Trimethoprim
•loss from dialysis
B12 Source: animal meat, eggs, dairy •fatigue, exercise intolerance, pallor Labs: B12 (Cobalamin) Injections
Deficiency •glossitis, diarrhea, malabsorption •high MCV weekly until corrected  monthly
Macrocytic Pathophysioogy: •dereased B12
•intrinsic factor is released by the •NEUROPATHY (symmetric), loss of •high homocysteine & MMA •can switch to oral therapy after
stomach parietal cells which allows B12 vibratory, sensory, and proprioception symptoms resolve
absorption in ileum •decreased DTRS Smear: hypersegmented, macro- •if pernicious, then need lifelong IM
•B12 deficiency causes abnormal ovalocytes, basophilic stippling therapy
synthesis of DNA, nucleic acids, and
metabolism. B12 is needed to convert
homocysteine to MMA

Causes:
•decreased absorption: pernicious
anemia  damaged parietal cells  lack
of intrinsic factor
•diet (vegans)

HEME/ONC
Anemia of MCC of normocytic anemia •fatigue, exercise intolerance, pallor Labs: EPO if renal disease or low EPO
Chronic •normocytic normochromic anemia
Disease Pathophysiology: •increased hepcidin
Normocytic •increased hepcidin and decreased EPO •Increased ferritin
 not able to produce heme •Low iron and TIBC
•decreased RBC production

Caauses
•MCC: Chronic Kidney Disease (CKD)

Hemolytic Anemia: increased RBC destruction > bone marrow replacement


- Intrinsic (sickle cell, thalassemia, G6PD, spherocytosis); Extrinsic (autoimmune, DIC, TTP, HUS, PNH, hypersplenism)
- Labs: increased reticulocytes, increased INDIRECT bilirubin, low haptoglobin
HEME/ONC
Hereditary •autosomal dominant •varying degrees, may go undetected, Labs: All: folic acid (1mg daily)
Spherocytosis may present acute after infection •high reticulocytes
autoimmune Pathophysiology: •normal MCV, high MCHC Definitive: splenectomy
abnormally shaped RBC due to proteins in •anemia, jaundice, splenomegaly •(+) osmotic fragility, (-) coombs *could be partial
cytoskeleton  sphere instead of flexible •pigmented gallstones *delay until 5+yo or puberty
and biconcave  destroyed by spleen Smear: hyperchromic
microspherocytes -pneumococcal vaccine
-transfusions of EPO if severe
EMA binding: PREFERRED
•flow cytometry analysis
Sick Cell •Autosomal resessive •trait usually asymptomatic Electrophresis: •PCN at 2mo, folic acid at 1yr
Anemia •MC in African Americans •trait: HbA and HbS •supportive: folic acid, vaccines
Normocytic Disease: *begin as early as 6mo •disease: HbS
Types: •dactylitis-inflammation of •Hydroxyurea
•trait (heterozygous-AS) finger/toe joints Smear: Howell-Jolly, target cells, MOA: increases production of HbF,
•disease (homozygous-SS) •osteomyelitis (Salmonella) sickled RBC reduces RBC sickling
•aplastic crisis with parovirus B19 SE: myelosuppression, GI
Pathophysiology: •Jaundice, HSM, cardiomegaly Labs: increased reticulocytes
•Valine substitutes for glutamic acid on •Ill appearing, poor healing, Acute Crisis :
B chain  HbS has decreased solubility retinopathy, stroke, tibia skin ulcers Splenic Sequestration: HOP: hydration, oxygen, pain
under hypoxia  abnormal shape  •reticulocytosis
vaso-occlusive episodes  spleen Splenic Sequestration Crisus: *differentiates from aplastic crisis Definitive: allogenic stem cell
•acute splenomegaly •drop in hemoglobin
Triggers: SHITAE: stress, hypoxemia, •rapid decrease in hemoglobin •thrombocytopenia
infection, temperature changes, acidosis,
exercise Vaso-Occlusive Crisis:
•acute chest syndrome
•back, abdominal, bone pain
•priapism

G6PD •X-linked recessive; MC in AA males •EPISODIC hemolytic anemia Smear: Prevent: avoid oxidative drugs,
Normocytic •Jaundice, dark urine •Bite & Blister cells; Heinz Bodies triggers
•Deficit in G6PD enzyme •Malaise, weakness, pain
Labs: •Folic acid supplementation
•RBC is vulnerable to oxidative state  Triggers: •Low hgb & hct
hgb denatures  forms Heinz bodies  infection, food (fava beans), ABX •High reticulocytes & High MCH
damages membrane  spleen destroys •G6PD assay
Autoimmune •antibody forms against RBC (IgG) •abrupt, life-threatening Smear: Prednisone 1-2mg/kg/day orally
Hemolytic •fatigue, exercise intolerance, pallor •microspherocytosis *esp warm
Anemia Pathophysiology: •polychromataphils Severe: splenectomy
*autoimmune Warm:IgG, Cold:IgM for complement •hemolysis: hemoglobinuria,
• macrophages remove membrane  jaundice Labs: Comorbitities: treat underlying
spherocytes  destroyed by spleen •splenomegaly •low hgb, hct, RBC
•complement can tag Kupffer cells •high reticulocytes Transfusion: required
•high indirect bilirubin *give without type/cross matching
Causes:
•warm: idiopathic, meds, autoimmune DIAGNOSTIC: COOMBS (+)
•cold: infection (mycoplasma, EBV)
HEME/ONC
Polycythemia Acquire disorder causing •Fatigue, HA, dizziness, vertigo CBC: Management: Refer to Hematology,
Vera OVERPRODUCTION of ALL 3 •Generalizes pruritus due to •Elevated RBC, WBC, plt educate
hematopoietic cell lines histamine release •HCT> 54%males, 51% females
*worse in a warm shower/bath Treatment:
Pathophysiology: JAK2 gene mutation CMP: liver and renal Phlebotomy
•Bleeding-engorged vessels from plts •1unit decreased Hct by 3%
Causes: radiation, toxins, men, 50-60yo •Hypercoaguability Peripheral Smear: normal •Remove 1 units weekly until <45%
*increased blood viscosity and
MCC of death=thrombosis platelet dysfunction •Low EPO •ASA 81mg daily
•abdominal pain, discomfort  •Genetic mutation: (+) JAK2 •alternative=hydroxyurea
ulcers due to increased histamine, •Iron studies due to blood loss
hepatosplenomegaly Myelosuppressive chemo if:
Criteria: •Frequent phlebotomy
Primary v. Secondary Erythrocytosis: •Engorged conjunctival & retinal 3 major or 2 major + 1 minor •High risk or mod/severe thrombosis
Primary: low EPO, high WBC & plt vessels  vision changes •Major: increased RBC mass •Pruritis
Secondary: high EPO, normal other (hgb/hct), + BM biopsy, JAK2
•Plethora-reddish uneven •Minor: decreased EPO, increased
complexion to the face, palms, nail ALP, iron deficiency
beds, mucosa

Aplastic Failure of hematopoietic bone marrow •Decreased WBC-infection Labs: Supportive: RBC & PLT transfusion
Anemia due to suppression or injury of stem cells •Decreased RBC-anemia, fatigue, •low WBC, RBC, platelets
from T cell attack pallor •normal MCV, MCH Mild: bone marrow growth factors
•Decreased platelets-bruising, bleed •erythropoietic (EPO), myeloid
MC: idiopathic autoimmune Diseases, Bone marrow biopsy: BEST MOA: recombinant DNA
toxins, medications, infections (hepatitis Exam: •hypocellular fatty aspirate SE: HTN, thrombosis
•pallor, purpura, petechial “3 Ps” •low/no hematopeitic precurosors BBW: vascular, heart issues, tumor
Pathophysiology: *no hepatomegaly, splenomegaly, CI: uncontrolled HTN, aplasia
Hypoplasia of hematopoietic bone bone tenderness, lymphadenopathy
marrow  decrease WBC, RBC plts Severe: stem cell transplant

Primary Hemostatis:
Platelets form a plug at the site of vascular injury  platelet adhesion, activation, and aggregation
Platelets send out ADP and Thromboxane A  more platelets  platelet plug
- Diseases: ITP, TTP, HUS, DIC, vWF deficiency
- Labs: affects bleeding time, but NOT PT or PTT
- Classic Presentation: petechiae, mucocutaneous bleeding

Secondary Hemostasis:
Clotting factors respond in a cascade to form fibrin strains to strengthen platelet plug
Extrinsic pathways (1, 2, 5, 7, 10): prolongs PT *DIC, warfarin therapy, vitamin K deficiency, Intrinsic Pathway (1, 2, 5, 8, 9, 11, 12): prolongs PTT *heparin overdose
- Diseases: hemophilia, DIC, vWF
- Presentation: deep delayed bleeding (hemarthrosis), delayed bleeding after surgery
HEME About Clinical Presentation Diagnostics Treatment
Immune •antibodies against antigens on platelets •Bleeding, petechiae, bruising ISOLATED thrombocytopenia •Steroids
Thrombo- surface  destruction by spleen •mucosal bleeding <100,000
cytopenic Purpura •2nd line: IVIG, splenectomy
Causes: •NO splenomegaly •normal PT. PTT, INR
•primary: MC after URTI
•secondary: HIV, HCV, SLE, Marrow: megakaryocytes
antiphospholipid syndrome

Thrombotic platelets into thrombi in vessel  FATRN: Fever, anemia, Tests: Plasmapheresis with FFP
Thrombocytopenic shearing of erythrocytes thrombocytopenia, renal, neuro •vWF LOTS, ADAMTS13 low •removes antibodies, adds
Purpura (TTP) •LOW Hgb & platlets ADAMTS13 to serum
Pathophysiology: •Acute/subacute onset of neurologic •HIGH fibrinogen, bilirubin
Adults •low ADAMTS13 symptoms, anemia, or 2nd line: steroids,
Brain, Fever, vWF •high vWF thrombocytopenia Direct Coombs: negative (-) immunosuppression
*antibodies against ADAMTS13 •Fever 50% pts Smear: mod/severe schistocytosis
•Hemoglobinuria, splenomegaly 3rd line: splenectomy

Hemolytic-Uremic •Renal failure with microangiopathic •Prodromal gastritis Labs: Typical:


Syndrome (HUS) hemolytic anemia and (Fever, blood diarrhea for 2-7d •BMP: elevated BUN/CR Supportive care
thrombocytopenia  damages before onset of renal failure) •CBC: severe anemia, varying
Kids endothelial cells •Irritability, lethargy, Seizure thrombocytopenia Atypical:
Kidney •Acute renal failure, Anuria Plasma exchange ASAP
NO fever Typical: Shigella toxin, E. coli O157:H7 •HTN Smear: schistocytes (helmet cells)
Diarrhea •diarrhea in 2-3yo, acute renal failure •Edema *Antibiotics and anti-motility
•Pallor (due to anemia) •stool for E. coli and Shigella agents are avoided
Atypical: non-STx, sporadic or familial

Heparin Induced Pathophysiology: •Venous thrombosis: legs, cardiac, •new thrombocytopenia •anticoagulation: Argatroban
Thrombo- •Antibody to hapten of heparin and skin  necrosis, gangrene •platelet drop >50% *direct thrombin inhibitor
cytopenia (HIT) +PF4 forms neoantigen on platelet  •can be hypercoaguable because •venous or arterial thrombosis
antibodies  clearance  low destroyed platlets release granules •GOLD: 14-C-serotonin release
•onset 5-7d post therapy assay
HEME About Clinical Presentation Diagnostics Treatment
DIC Pathologic activation of coagulation system  •Widespread hemorrhage & thrombosis Labs: •Treat underlying cause
widespread microthrombi  consumes clotting •bleeding from venipuncture sites, •Increased PTT, PT/INR •FFP if severe bleeding
factors  thrombocytopenia and diffuse bleeding catheters, drains •Decreased fibrinogen •platelets if <20,000
*MC at venipuncture sites •Increased D-dimer
Causes: •gangrene or multi-organ failure •Thrombocytopenia
•infections (gram – sepsis)
•malignancies, obstetric (pre-eclampsia) Smear: fragmented RBC, shistocytes

vWF •Autosomal Dominant •recurrent nose bleeds, heavy •Decreased vWF, decreased VIII •DDAVP
Disease •MC inheritable bleeding disorder periods, prolonged wound bleeding •prolonged PTT and bleeding time •recombinant vWF or
•ineffective plt adhesion due to low vWF •family history •normal PT/INR Factor VIII
•use Celebrex for anti-
vWF function: promotes plt adhesion by GOLD: Ristocetin-induced platelet inflammatory
crosslinking GP1b receptor on plts with exposed aggregation *helps determine type
collagen on damaged epithelium
•factor VIII degradation

Hemophili •X-linked recessive; MC in children & males •hemarthrosis (joints, MC ankle) •prolong PTT & bleeding time • A: Factor VIII or DDAVP
a •A: factor VII deficiency (MC) •hemorrhage due to trauma •normal platelets • B: Factor IX
•B: Factor IX deficiency *christmas tree •possible purpura/petechiae • C: Factor XI
•C: factor XI deficiency *ashkenazi jews

HEME
Factor V Leiden •MCC inherited thrombophilia (hypercoag) •DVT, PE, Clotting •normal PT, PTT Anticoagulant indefinitely

Pathophysiology: Factor V: activated by thrombin and helps Activated protein C


•Mutation of Factor V  does not respond to convert prothrombin to thrombin resistance assay
protein C  does not turn off and keeps clotting  confirm with DNA test

Protein C/S •Autosomal dominant •DVT, PE •Protein C and S assay Indefinite anticoagulant
Deficiency •Vitamin K dependent anticoagulant proteins •purpura fulminans in newborns: red •plasma antigen levels
purpuric lesion at pressure points  eschar Necrosis: IV vit K, heparin,
•Protein C: risk warfarin induced necrosis protein C, FFP, Heparin

Anti-thrombin •Autosomal dominant •Venous thrombosis, DVT, PE •antithrombin III assay Thrombosis: high-dose IV
III deficiency Normal: inactivated thrombin, IIa, IXa, Xa •MC in deep veins and mesenteric veins heparin  oral

Antiphospholipi •Acquired hypercoaguable condition Cardiac: Libman-Sacks, endocarditis, Antibodies anticoagulation


d Syndrome •antibody (aPL) in plasmsa pulmonary HTN, thrombotic events
•necrosis of hip
HX: pregnancy complications •Livdeo reticularis, adrenal insufficiency
ONC
Hodgkins •Germinal B cells undergo •PAINLESS cervical/supraclavicular LAD •Staging: Ann Arbor Stage I-II: Combo Chemo
transformation *alcohol may cause pain + radiation
•2 peaks: 20s, 50s •lymph spread in contiguous fashion •CBC normal, elevated ESR
(cervical MC) •CT/PET for staging Stage III-IV: combo
Risks: EBV, smoking, •Mediastinal mass, fatigue, wt loss ABVD: Adiramycin,
immunosuppressed Biopsy: Reed Sternberg cells: large cells Bleomycin, Vinblastine,
•B SX: wt loss >10%, T >38C, sweats with bi/multi-lobed nuclei (owl-eye Dacarbazine
Types: *indicate advanced disease appearance) & inclusions
•nodular: MC, females MOPP: Mustine,
•mixed: EBV Exam: HSM, nephrotic syndrome, Oncovorin/Vincristine,
•lymphocyte rich: MC in males hyercalcemia PRocarbazine,
•lymphocyte poor: MC >60yo Prednisolone

Non-Hodgkins •Accumulate in lymph tissue •PAINLESS lymphadenopathy •viral serology Asymptomatic: no tx


(spleen, nodes, thymus) *non-contiguous •CXR  CT of chest
•malignant overgrowth of •Cough, dyspnea, edema Stage I: Chemo
lymphocytes; MC: B cells •Mediastinal mass, RLQ mass tissue biopsy: DIAGNOSTIC
•extranodal (GI MC) Intermediate/Aggressive:
Risks: increased age, radaiation, •bone marrow RCHOP: Rituximab,
fhx, immunosuppressed, infections Types: Cyclophasphamide,
(EBV, HIV, HHV-8), autoimmune •Diffuse large B-Cell: MC, aggressive Doxorubicin,
•Folicular: slow growing, hard to cure Hydrochloride,
•Lymphoblastic: TT “teen, T-cells” ONcovorin, Prednisolone
•Burkitt: BBBB: boy, B-cells, belly,
chromosome B *EBV
•Large Cells: B and T cell
ONC
ALL •MC malignancy in children •Fever, weakness, fatigue CBC: ANC <1000, WBC 5-100,000 Combination chemo
•Bone pain, pallor, anemia Smear: lymphoblasts (~25%) (anthracyclines,
•Leukemic BLASTS replace marrow •Petechia, purpura, infection Vincristine, steroids)
(immature cells) •Marrow FNA & biopsy DEFINTIVE
Exam: HSM, weight loss, lymphadenopathy, maintenance: 6-MP, MTX
abdominal pain

AM •MC acute leukemia in adults •Fatigue(MC), infection, nodules, HSM CBC: low WBC & plt, normocytic Combo chemo
L •accumulation of BLASTS •pantocytopenia normochromic anemia
Smear: auer rod Leukostasis:
Types: Leukostasis reaction: MC w/ AML & CML •leukapheresis
•acute promyelocytic (APL): MC associated •increased viscosity  plug Bone Marrow Biopsy: GOLD
with DIC, myeloperoxidase (+), auer rods microvasculature •>20% myeloblasts, auer rods
•acute megakaryoblastic: MC in children •SOB, hypoxic, HA, dizzy, vision change
<5yo and downs •WBC >100,000
•acute monocytic: infiltration in gums •tre
(gingival hyperplasia)
CLL •MC LEUKEMIA IN US! •Slow onset! Usually found incidentally •pantocytopenia, WBC > 20,000 Low stage: observe
•mature B cell malignancy •lymphadenopathy (MC!) •Isolated lymphocytes (>5,000)
•recurrent infections (PNA, HSV, HZV) *small, well-differentiated, normal High stage/SX:
Risks: old age, men appearing with “smudge cells” chemotherapy
•hepatosplenomegaly (Fludarabine, Rituximab,
Complications: *upper abdominal discomfort/fullness FLOW CYTOMETRY: CONFIRMS Cyclophasphamide,
•obstructive lymphadenopathy •anemia/thrombocytopenia Confirms abnormal B-lymphocytes Chlorambucil)
•transformation into aggressive large cell
lymphoma (Richter syndrome) BM Aspiration/Biopsy: small lymphocytes •stem cell transplant is
*not required curative

CM •uncontrolled production of •Fatigue! *often first symptom CBC: WBC 100-150,00 w/ left Chronic:
L maturegranulocytes •Low-grade fever, night sweats, wt loss (neutrophilia, basophilia, eosinophilia) Tyrosine kinase inhibitor
•Single specific genetic mutation •abdominal fullness, bone tenderness (Imatinib, Nilotinib,
(translocation) of gene 9:22 •pruritis, flushing Alk Phos: low Dasatinib) “-tinib”
•PHILADELPHIA on PCR •GI ulcers with elevated basophils due to
histamine Smear: Accelerated: blast cells, Accelerate or Blast:
First (Chronic): MC  mature cells promyelocytes TKI + multidrug chemo
Second (Accelerated): cytogenic PE: HSM, LAD *possible stem cell
Third (Terminal Blast): immature BM Aspiration/Biopsy: hypercellular with
increased granulocytes and progenitors
ID 4%

ID-Bacterial
Typhoid •Gram (-) Salmonella Typhi •headache, malaise, anorexia •culture •oral rehydration and electrolyte
(Enteric) Fever •MC in children, young adults •intractable fever, chills, abd pain replacement
*Invasive •”pea soup” green diarrhea, non-bloody
•Crosses intestinal epithelium through M ABX: Ciprofloxacin, Ofloxacin
cells  Peyer’s patches; may colonize in Exam: •Azithromycin, Ceftriaxone
gallbladder •fever with bradycardia
•rose spots (faint pain or salmon
Transmission: fecal-oral, food, water, hx macular rash from trunk to extremities)
travel to areas of poor sanitation •HSM, GI bleeding

Incubation: 5-21 days

Non-Typhoidal S. enteriditis, S. tymphimurium •nausea, vomiting, abdominal cramping •culture •oral rehydration and electrolyte
Salmonella •common cause of foodbourne illness •diarrhea “pea soup” brown-green replacement
*Invasive •abdominal cramping
Sources: •malaise, headache ABX: Ciprofloxacin, Ofloxacin
poultry, eggs, milk, reptiles (turtles)

Shigellosis •gram (-) rods: Shigella sonnei (MC) •lower abdominal pain, cramps •culture •oral rehydration and electrolyte
*Invasive S. flexneri, S. dysenteriae (enterotoxin) •high fever, tenesmus replacement
•explosive, watery diarrhea  mucoid •WBC >50,000
Transmission: fecal-oral, food, water and bloody diarrhea •avoid anti-motility drugs
Sigmoidoscopy:
Incubation: 1-7 days, Neuro: febrile seizure •punctate areas of ulceration Severe: ABX-Ciproofloxacin,
HIGHLY virulent Ceftriaxone, Azithromycin, Bactrim
Complications: reactice arthritis, HUS,
toxic megacolon

Spirochet
e
Lyme Borrelia burgdorferi Stage 1: early, localized *within 1 week Criteria: 1/2 •Doxycyline
Disease •MC tickborne illness •erythema migrans: bulls eye with central clear •developed erythema migrans
•MC in spring and summer •flu-like symptoms: HA, myalgia, fatigue •one late manifestation + one lab Alternative:
•MC in northeast & central US confirmation •Amoxicillin *pregnancy
Stage 2: early disseminated
Transmission: Ixodes tick (deer) •bacteremia, worsening flu-like sx, Serology: ELISA  western blot Late or Severe;
•cardiac: arrhythmias, heart block Use in patients if: •IV Ceftriaxone if AV block,
•neuro: aspetic meningitis, facial palsy (CN VII) -reside/travel in area syncope, dyspnea, chest pain,
-risk factor + symptoms CNS, meningitis
Stage 3: Late persistant *months-years after Not in:
•MSK (joint pain-arthritis), neuro, skin -pateints with erythema migran
-screening asymptomatic
Viral About Contagious Presentation Diagnostics/Treatment
Epstein-Barr •HHV 4 •malaise/fatigue, fever, sore throat, Heterophile Antibody: test of choice •Supportive: fluids, antipyretics
Virus (EBV, •tonsillar pharyngitis +/- exudates (Monospot) *IgM acute, IgG life •corticosteroids if airway obstruction
Mononucleosis) Transmission: •lymphadenopathy (posterior cervical) 
•Saliva, blood •splenomegaly Peripheral Smear: lymphocytosis >50%
• “kissing disease” •palatal petechial, maculopapular rash with 10% atypical lymphocytes AVOID TRAUMA & CONTACT
*Ampicillin increases rash SPORTS x3-4 WEEKS if HSM
•EBV antibodies

Viral
Influenza Transmission: respiratory droplets ABRUPT ONSET •rapid influenza nasal swab or Mild disease and healthy: supportive
(Viral) (sneezing, coughing, talking, breathing), •headache viral culture •Acetaminophen, fluids, rest
touching contaminated surfaces •fever, chills •RT-PCR (1-3d)
•malaise •NP viral culture (3-7d) Ostelamivir (Tamiflu) within 48 hours
3 types: A (most pathogenic), B & C •URI symptoms, pharyngitis MOA: neuramidase inhibitors A & B
*A has more severe outbreaks than B •PNA •hospitalized patients
•myalgia: legs, lumbosacral •high risk of complications (age,
Increased Risk: cardiovascular disease, pulmonary
•age >65 years old *MC seen in children Hemaglutinin: bind to epithelium disease, immunosuppression, chronic
•pregnancy (cellular infection) liver disease, hemoglobinopathies-sickle
•immunocompromised Complications: PNA, respiratory cells, thalassemia)
Neuraminidase: cleaves bond failure, death, meningitis,
holdings new virions (spread of myocarditis encephalitis, rhabdo, Prevention:
infection) kidney failure •influenza vaccine (inactive): ages 6
months and above, x2 if <9yo
-CI: high fever, GB w/n 6wk, <6mo
•Chemoprophylaxis in long-term facilities
where outbreaks occur

PHARM MOA Route Dosing


Neuramidase Attaches to the receptor sites to prevent •oral Tamiflu: Rapivab 600mg IV
Inhibitor binding to the host cell  •capsules 75mg po BID x 5d •infuse 15-30min, adults 18+
•Diskhaler  •all ages (prophylaxis only
3mo+), age and wt based Relenza: 2 inhalations BID x5d
•ages 7+ (5+ for prophylax)
Viral About
HIV Retrovirus: changes RNA into Acute seroconversion: •ELISA  western blot *offer to all HIV (+) pateints
(Viral) DNA via reverse transcriptase •flu-like illness •HIV RNA viral load >100K copies
•fever, fatigue, myalgia, sore throat -can be positive in window period Regimens:
Transmission: •nontender generalized lymphadenopathy •elevated LFTs, mild anemia, thrombocytopenia •NNRTI + 2 NRTIs
•sex •PI + 2 NRTIs
Suspected Early Infection: •INSTI + 2 NRTIs
•IVDU Opportunistic Infection: •combo antigen/antibody immunoassay +
•mother •oral and esophageal candidiasis HIV viral loading test (RT-PCR)
•blood contact •CMV •both (-) but high suspicion: repeat in 1-2wk
•PCP pneumonia •(-) immune, (+) viral: early HIV
AIDS: CD4 <200 •cryptosporidiosis •both (+): early/established infection
-confirm with a second test (repeat one)

Viral Drugs MOA Adverse EFfects


NRTIs Zidovudine Abacavir Inhbits viral replication by interfering with HIV •Zidovudine: bone marrow suppression
Lamivudine Emtricitabine viral RNA-depenent DNA polymerase •Abacavir: CI HLA B5701
Tenofovir Stavudine

NNRTIs Efavirenz Nevirapine Inhibits viral replication by interfering with •Rash


Delavirdine Rilpivirine HIV viral RNA-dependent DNA polymerase •Efavirenz: vivid dreams
Etravirdine

Protease Atazanavir Indinavir Inhibits HIV protease leading to production of •N/V/D


Inhibitor Darunavir Ritonavir noninfectious, immature HIV particles •Lipodystrophy, hyperlipidemia
s Nelfinavir Saquinavir

INTI Raltegravir Dolutegravir Orevents insertion of DNA copy into host •hyperlipdiemia, GI symptoms

Fusion Enfuvirtide Disrupts the virus from fusing with T-cells •hyperlipdiemia, GI symptoms
Inhibitor
s
CCR5 Maraviroc Blocks viral entry into WBC

HIV Prophylaxis:
CD4 <200: Pneumocystitis  Bactrim
CD4 <150: Histoplasmosis  Amphotericin B
CD4 <100: Toxoplasmosis  Bactrim
CD4 <100: Cryptococcus  Fluconazole
CD4 <50: MAC  Azithromycin or Clarithromycin
CD4 <50: CMV Retinitis  Valgangciclovir
NEURO
Bacterial Acute infections of the arachnoid Adults: •Blood cultures x2 + stain, C/S GOAL: TX WIHTIN 60min
Meningitis mater and subarachnoid space •Fever, HA, Nuchal, Meningeal •PROMPT LP first, UNLESS:
•N/V, photophobia, seizure -immunocompromised Standard for all (3mo-55yo) ALL 3
Adults: -history of CNS 1.Dexamethasone 0-20minutes
•Community Acquired increased ICP: -new onset seizure 2.Ceftriaxone (Cefotaxime <1mo)
•S. pneumo (children, adults >20) -papilledema, N/V, LOC change -papilledema 3.Vancomycin
•N. Meningitidis (10-19yo) -meningococcal rash -abnormal LOC
•GBS, Listeria Monoctogenes, H. flu -focal neurologic deficit PLUS ONE additional below:
Pediatrics: •Meet any above=CT first! •Acyclovir (HSV encephalitis)
•Healthcare Acquired: Staph aureus •Fever, lethargy, seizure, resp. distress *DO NOT DELAY ABX! •Doxycycline (tick season)
•Jaundice, N/V/D •Ampicillin (<3mo or >55yo)
Pediatrics: •Poor feeding/decreased appetite Lumbar Puncture: •Metronidazole (OM, sinus, mastoid)
•Neonatal: GBS, E. coli, Gram (-) •Restlessness/irritability •low glucose
•Bulging fontanel (infants) •high protein & WBC Duration of ABX:
•Children >1 month: S. pneumo, N. •cloudy, pressure 200-300 •S. pneumo 10-14d
meningitides, GBS, H. flu •N. mening & HiB 7 days
•S. aureus 14 days, L.mono 14-21 days

Rifampin or Cipro prophylaxis if:


•Exposure H. flu, S. pneumo, N.mening
•GBS (+) 35-37 weeks gestation
•Neurosurgery

Viral •MCC-Enterovirus Present similar to bacterial meningitis Work-Up: same as bacterial •Most cases self-limited (7-10d) & tx is
Meningitis but less severe supportive: fluids, rest, sx control
Risk factors: •HA, nuchal rigidity, Photophobia Lumbar Puncture
•Infants <1 month •Constitutional symptoms •CSF analysis, Gram stain and Specific treatments:
•Immunodeficient patient •mildly diminished LOC •PCR amplification •HIV  refer; HSV  Acyclovir
•Exposure to someone *NO seizures or focal neuro •clear, pressure <200, rest
nml/low Empiric ABX: old & immunocomp
Historical features:
•travel and exposure history CT scan Indeterminate diagnosis on CSF: 1 of 2
•TB exposure -not needed in uncomplicated 1. administer ABX after cultures
•ill contacts with similar sx/viruses indications: focal neuro, 2. observe w/ repeat LP in 6-24h
•vaccination history immunocompromised
•sexual history (HSV, HIV, syphilis) Prevention: IPV, MMR, VAR/Zoster
URGENT CARE 4%

Heat Illness Heat Edema Heat Syncope Heat Cramps


Wasp, •stinging results in Fire Ant Sting Labs: Wound Management:
Bee, Ants envenomation causing one of 3 •sterile pustule evolves of 6-24h •ONLY FOR SYSTEMIC and •do not delay tx of systemic reaction
presentation •may results in necrosis and scarring ANPHYLACTIC sx to care for wound
-localized reaction •directed at complications: •remove stinger w/ scraping technique
-systemic reactions Systemic/Toxic (nona-allergic) -CBC, CMP, caogs, CK •wash wound
-anaphylaxis •more common with >50 stings •ice and elevated
•N/V/D with irticarial lesions distant from Management: •tetanus update
Clinical Presentation: site of string Anaphylaxis
•localized bee sting: MC •symptoms usually subside within 48h •intubation if needed Disposition:
reactions •severe acses may lead to complications •Epinephrine 1:1000 (adults 0.3-0.5) •local: discharge home
-small pruritis, painful, -rhabdo, liver failure, hemolysis, •IV Methylprednisolone •systemic
erythematous, edematous thrombocytopenia, DIC •IV Diphenhydramien (Benadryl) -admit child, elderly, comorbid,
lesion at sthe sting site •IV Famotidine (Pepcid) 50+ stings, or prolonged reactions
-occasionalyl >5cm Anaphylactic Reactions: allergic rxn •Nebulized albuterol
•occurs within 6hr (most within 15 min) -otherwise can send home:
•itchy eyes, urticarial, cough, resp failure, Localized reactions only: observe for 6 hours in the ED to
CV collapse •Benadryl make sure not rebound sx, repeat
•oral pain control: NSAIDS, Tlenol labs before discharge, RX
•systemic: IV methylpred, Benadryl, EpiPen, f/u with allergist
famotidine
•complications: may require blood,
dialysis, and extensive hostpital care

Abortion Presentation Management


Scorpion •most scorpions produce •sting is painful without initial •most pateints can be managed •antivenin (anascorp) indicated in
localized reactions erythema/swelling symptomatically severe systemic toxicity
•exquisite pain with light percussion -oral or IV pain medication
•bark scorpion known as “tap sign” -Benzo for motor control
-only systemic toxic scorpion •neuromuscular excitation
-infants & child high risk -spasms, CN dysfunction, roving eye
-southwestern US movement, diplopia, dysphagia

Snake •large triable shaped head with •fang marks wit hpain, edema, •consult poison control Antivenin (CroFab)
Bites (Pit a heat sensitive depression “pit” hemorrhage and necrosis around bite •cardiac monitoring and IV access •approved for bites from rattlesnackes,
Vipers) between their eyes & extending distant from bite if severe -administer fluids and resp support copperheads and cottonmouths/water
-usually within 30 minutes but may be moccasins
•venom is cytotoxic delayed up to 12h Labs: CBC, CMP, coags, CK, urine •compare severity of envenomation to
-if no s/s after 12h: dry bite myoglobin, type and crossmatch SE of antivenin
•copperhead, rattlesnake -hypersensitvity reaction in 5-19%
Systemic: •Immobilize biten extremity -recurrent coagulopathy in 50%
•hemolysis, thrombocytopenia, -EMS may apply contrisction band •td upate
coagulopathy -not effective from EBM
•vomiting •serial (30 mintes) wound evaluation Disposition
•respiratory failure with CV and collape -measure affected limb above and •observe in ER for 8 hours, DC if no
below bite and mark border progression, ICU if severe
-assess sx of compartment syndrome

Physical Exam Diagnostics Gastric Decontamination


Poisoned •mental status, vital signs, pupillary •skin (full exposure): cyanosis, flushing, Abdominal X-ray: activated charcoal *BEST!
Patient examination help classify excitation or diaphoresis or dry, injury, ulcer, bullae… •suspected body packer •indicated if ingestion of toxic
depression •Bezoar formation: small stony substance within 1 hour prior to arrival
•Eyes concentration form in stomach or *can used after 1hr
Excitation: -mydriasis: anticholinergics, sympatho intestines (ASA, salicylates) if antichol or salicylates
•CNS stimulation, mydriasis -miosis: cholinergics, opioids •AC is not able to bind to metals,
•tachy, increased BP, RR, temperature -nystagmus: ethanol, phenytoin, CXR: pulmonary edema corosives, alcohols
•anticholinergics, sympathomimetic, ketamine, PCP •CI: cant protect air  aspirate
serotonin syndrome, hallucinogens -lacrimation: cholinergic Toxicology:
•UNNECESSARY if non- Gastric Lavage
Depression: •mucous membranes intentional ingestion, •removes non-absorbed toxins
•depressed mental status, miosis -hypersalivation: cholinergic asympatomatic, or have clinical •indication: <1hour, no antidote to
•low BP, RR, temperature -dryness: anticholinergics findings consistent with medical toxin, poor response to supportive care
•sedative-hynoptic agents, opiates, history •high risk of aspiration
cholinergic agents •heart •individual screen for lithium and
•lungs: bronchorrhea (cholinergic), wheeze digoxin may be needed Procedure
Exam •insert 36-40F orogastric tube
•vital signs •abdomen: Common Coigestatnts: •lie LLD w/ head of bed tilted down
•mental status -diminished: anticholinergic, opiate •APAP, ethanol, salicylate •200ml warm tap water in stomach
•general appearance: agitation, -enlarged bladder; anticholinergic *ALWAYS in unknown ingestion
confused -tender/rigid: ASA, anticholinergic Whole Bowel Irrigation
UA: calcium oxalate crystals with •indications: ingestion of chemicals
•neuro: ethylene glycol (antifreeze) poorly absorbed to charcoal (lithium,
-tone, tremor: cholinergic, serotonin iron, lead) and ingestion of drug-filled
-assess cogitation, CN, DTRs, strength, packets
coordination, gait •GoLYTELY to flush out
-administer NG tube 1-2L/ht
-continue until clear (2-5h)
CI: absent BS, ileus, obstruction

Poison Enhanced Elimination Decontamination/Contamination


TX Multi-Dose Activated Charcoal Urinary Alkalinization Hemodialysis/Hemoperfusion Decontamination: Inhaled
•ionizes acidotic toxins preventing •hemodialysis: more effective at •administer oxygen
indications: ingestion of toxic levels resorption across the renal tubule clearing highly protein-bound drugs •water aerosol inhalation  dilute irritants
of carbamazepine, dapsone, and lipid-soluble drugs •delayed upper airway obstruction or
phenobarbital, quinine, theophylline •indications: moderate-severe pulmonary edema
salicylate toxicity •hemoperfusion: clear water-soluble
CI: uprotected airway, absent BS low molecular weight Decontamination: Eyes
•caution in ingestions resulting in •cautions: hypokalemia •irrigate with plain water or NS
reduced GI motility •both require critical care setting and •assess pH after 2L irrigation
•Procedure: are expensive and invasive
-IV sodium bicarb +/- KCL  complications Contaminated Skin
Urinary Alkalinization -monitor K+ and bicarb q2-4h •avoid direct self-exposure
•maintain K+ between 4-4.5 •wash skin with water and dilute soap
-pH q15-20min with goal 7.5-8.5 •hydrofluoric acid burns are particularly
penetrating & corrosive10% Ca gluconate gel

Poison MOA and Examples Presentation Management


Opioid •heroin, morphine, methadone, “physiologically depressed” •ventilation
meperidine, hydrocodone, oxycodone •MC: CNS and respiratory depression, decreased BS, miosis •Naloxone
•hypothermia, bradycardia, orthostatic hypotension, bronchospasm,
pulmonary edema, seizures
Sympthato- Mimic the effects of endogenous •physiologic excitation •External cooling, sedation with benzos,
mimetic agonists of the sympathetic nervous •MC: psychomotor agitation, diaphoresis, HTN, hyperthermia, hydration
system (epi, norepi, dopamine) mydriasis, tachycardia
•seizures, rhabdomyolysis, MI •CI: Beta Blockers
EX: cocaine, caffeine, amphetamines,
cathinones (bath salts)
Cholinergic Blocks acetylcholinesterase, leading to MC: salivation, lacrimation, diaphoresis, bronchorrhea, •airway protection and ventilation
excessive Ach urination & defecation, N/V, muscle fasiculations, weakness •Atropine: blocks Ach receptors
•bradycardia/tachycardia, rales, miosis/mydriasis, seizures, •Pralidoxine: Antidote
EX: organophosphate insecticides, respiratory failure, paralysis •Diazepam: seizure, muscle fasiculations
carbamate insecticides

Anti- Block the acetylcholine muscanaric MC symptoms: •activated charcoal to decrease drug
cholinergic receptors  inhibits parsympathetic •hot as hades: hyperthermia absorption
nerve impulses •fast as a hare: tachycardia •address complications
•dry as a bone: no secretions or sweat -wide QRS: sodium bicarb
Examples: Scopolamine, Atropine, •mad as a hatter: AMS -agitation: benzos
Antihistamines, TCAs, Antiparkinson, •full as a tick: urinary retention -fluids: rabdo
Antispasmodics/ •blind as a bat: mydriasis
muscle relaxants •red as a beet: flushed skin supportive: treat hyperthermia with external
cooling methods
Disposition: •decreased/absent BS, dysrhythmias, rhabdo, hypotension/HTN
•discharge home if resolve in 6h •Physostimine: cholinesterase inhibitors
•admit with more significant symptoms increases concentration of Ach
or those receiving physostigimine -if conventional therapies fail

Sedative/ 3 classes: •MC: slurred sppech, lethargy, CNS depression, respiratory Supportive
Hyponotic •benzos depression, confusion •venilation/intubation, 2 large bore IV
•carisoprodol/alcohol.hypnotics •bradycardia, hypotension, hypothermia, bradypnea, hyporeflexia •IV fluids bolus for hypotension
•barbituates -dopamine or norepi if fails
•activated charcoal
Disposition •barbituates that don’t response
•admit if sx after 6 hours •benzos: confirmed benzo OD with
•consult psych for intentional OD respiratory depression
-Flumazenil *get toxicity back 1st

Serotonin •increased serotonergic activity •altered mental status, seizures, tremors/increased muscle tone •veniality support
Syndrome •hyperthermia, tachycardia, HTN, tachypnea •benzos for agitation, tremors, seizures
EX: MAOI, SSRI, Meperidine, •discontinue serotonergic drug
Dextromethorphan, TCAs, L-tryphtophan Labs: UA/CK, ABG *ADMIT ALL PATIENTS •Crypoheptadine if above fails
Hallucinogenic •LSD, PCP, muschroom, •disorientation, hallucination, anxiety, seizures, agitation, muscle •Benzo: agitation, hyperthermia, tachy, HTN
dextromethorphan, ketamine tension, N/V, HTN, tachycardia, tachypnea, hyperthermia, mydriasis •Nitropruss, phentolamine: refractory HTN
•aggressive IV fluids if rhabdo concern

Poison About Presentation Diagnostics Management


Salycilate EX: ASA, pepto bismol, Chronic salicylate level: >30 is toxic •ABCs
liniments, flavoring agents •fever, hyperventilation, AMS, volume -peak concentration may not •correct volume depletion
depletion, acidosis, hypokalemia occur for 4-6 hours
•ASA hydrolyzed to salicylic acid -repeat every 1-2hr until peak •GI decontamination: activated charcoal
 acid environment Acute then every 4-6 hours single dose
•<150mg: tinnitus, hearing loss,
•higher mortality with chronic dizziness, N/V •CMP and magnesium •reduce salicylate burden
•risk for bezoar formation -systemic/urinary alkalinization
•150-300: high RR, hyperpyrexia, •ABG: mixed acid/base disturbance with sodium bicarb
diaphoresis, ataxia, anxiety -alkalemia w/ resp alkalosis and -FIRST LINE in mod-severe
metabolic acidosis
•>300: AMS, seizure, heart/lung/renal,
shock •assess for coingestants Hemodialysis:
-APAP, toxicology severe not responding to above

•CXR and EKG for end-organ

•ABD image if bezoar suspected


-suspect is salicylate level
continue to rise despite tx with
gastric lavage or charcoal

Poison About Presentation Diagnostics Management


Tylenol Risks: Stage 1 (day 1 after ingestion) serum APAP level •Activated charcoal
•chronic alcohol, AIDS •anorexia, N/V, malaise, hypokalemia •assess levels in asymptomatic
•anticonvulstants and anti-TB patients too due to delay •Acetylcysteine: oral or IV
Stage 2 (days 2-3 after ingestion) •peak serum concentration 30- -prevents metabolite from
Suggested When: •improvement in stage 1 sx 120min after acute ingestion binding to hepatic cells (if within
6 years old and older: •RUQ abd pain, elevated LFTs and bilirubin, 8hr) & diminished necrosis
•ingests >10g (200mg/kg) over as prolonged PT •Rumach-Mathhew nomogram
single ingestion or over 24hr -determine clinical outcome •Extra-corporal excretion
•>6g/day (150mg/kg) x2 Stage 3 (Days 3-4 after ingestion) -only after one acute ingestion -patient with severe intoxication
consecutive days •fulminant liver failure, metabolic acidosis, between 4-24 hours post who present too late to remove drug
coagulopathy, renal failure, encephalopathy, -hepatic encephalopathy
Children <6 years old pancreatitis, recurrent GI symptoms
•200mg/kg as single ingestion or Disposition:
over 8hours Stage 4 (Day 5 and beyond) •admit all patients requires
•150mg/kg per day •survival: improvement & recovery acetylcysteine
x2consecutive days •continued deterioration to multi-organ failure •discharge after 4-6h of observation if
and death acetylcysteine is not needed
DERM About Clinical Presentation Diagnostics Managment
Thermal Causes: scalding, direct thermal, flame burns Descriptions: •lab evaluation: assess Management:
Burn *based on % of BSA complications •supplemental O2 & intubation if needed
Depth •rule of 9s •ABG, CBC, CK, CMP •vitals: pulse ox may be falsely high by CO
•old: 1st, 2nd, 3rd, 4th degree •lund and broder •UA, carboxyhemoglobin •assess & treat trauma, inhalation, CO
•new systemic: superficial partial thickness, •palmar method: palm=1% •IV opiates for pain control
deep partial thickness, full thickness Complications •urinary cath: measure I&Os
•inhalation injury
•1st degree (sunburn): •carbon monoxide IV Fluids:
erythema, skin blanches with pressure •bacterial super infection •IV LR: 2 bore needle in unburned area
•sepsis •Parkland formula to determine fluid
•2nd degree (partial): red and blistered •multiorgan failure *½ over first 8 hours, ½ over 16 hours
-adults: 4ml x wt (kg) x %BSA
•3rd degree (full): tough, lethargy, non-tender -children: 3ml x wt (kg) x %BSA

•4th degree: bone and muscle Minor: clean with soap and water
Large Bullae >2cm: drain, debride, 1% silver
Moderate/Severe: dry sterile sheet & admit

About Signs and Symptoms/PE Tests and Stages Diagnostics and Treatment
Shoulder •most common joint to dislocate •visible or palpable deformity X-Ray: scalpular Y & Axillary views Treatment:
Dislocation •MC is ANTERIOR •swell, ecchymosis, pain, decreased ROM •Anterior: head anterior & inferior •reduction & immobilization
•hold affected limb close w/ elbow flexed •Hill Sach Lesion: groove fracture
Risks: young males •numb & weakness (neck & muscle spasm) •Bankart lesion: glenoid rim fracture 1st time anterior dislocations:
•AP: “light bulb sign” appearance of humeral •reduced then immobilized
Most Common Causes Anterior: abduction and external rotation head  posterior in neutral rotation for 3
•sports: football, hockey, volleyball Posterior: adduction and internal rotation weeks followed by PT
•trauma: during MVA, falls Complications: *focus on supraspinatous
•seizures: POSTERIOR dislocation •axillary nerve (MC)  anterior
AC Joint •MCC: fall on tip of shoulder •pain over AC joint Type I: AC joint partially, CC intact Type I & II: non-surgical
Dislocation •scapula moves downward due to •pain w/ lifting arm (esp abduction) Type II: AC torn, CC intact, partial clavicle •ice, sling & analgesia
weight of arm  bump or bulge •deformity with type III – VI Type III: AC and CC complete, clavicle •full return within 4 weeks
above the shoulder •support arm in an adducted position separated from acromium
•clavicle prominence-esp with type II Type IV: clavicle posterior displaced Type III: nonsurgical
Type V: AC, CC, DT disruption
Type VI: inferior displacement or clavicle Type IV+: surgery

Arm About Presentation & Exam Diagnostics Treatment


Elbow •MC dislocation in children •extreme pain, swelling •AP and lateral x-ray Stable: REDUCE: steady, downward
Dislocation •>80% posterior •flexed elbow traction of forearm
•inability to bend elbow (extend) •posterior splint at 90 degrees flexion
Mehcanism: FOOSH •marked olecranon prominence •NSAIDS

•associated with ulnar collateral Physical Exam Unstale: ORIF


ligament disruption and fx of •extreme tenderness over elbow
radial head or medial epicondyle •deformity Refer: cant reduce, instability >3wk,
neurovascular injury

Radial Head •Subluxation of the radial head •child avoiding using the arm •X-rays are typically normal •Reduction
Subluxation due to sudden pulling force on -Usually not ordered unless - Flex elbow w/ forearm in supination
(Nursemaids) the child’s arm Physical Exam there is a hx of fall or trauma or the OR:
•Pain over the radial head & arm child continue not to use arm -Extension w/ forearm in pronation
Mechanism: lift, swing, pull slightly flexed

Medial •inflammation of the pronator •pain w/ wrist flexion & forearm •AP and lateral radiographs •activity modification, RICE. NSAIDS
Epicondyliti teres-flexor carpui radialis pronation •MRI is helpful in confirming •surgery if refractory (4-6 weeks)
s •tenderness over medial epicondyle *not necessary
“Golf Elbow” Causes: repetivite overuse, •pain worse with pulling activities
excessive wrist extension

Lateral •inflammation at tendon insertion •pain in the lateral elbow and forearm
Epicondyliti of extensor carpi radialis brevis with activities involving wrist extension
s muscle •more severe & occur at rest or with
“Tennis minimal activity (holding a cup)
Elbow”

About Clinical Presentation Diagnostics Treatment


Humeral Mechanisms: •pain, swelling, bruising, decreased ROM AP & axillary lateral views •minimally displaced < 1 cm
Fracture •FOOSH •arm held in adducted position •initial test -sling, analgesics, PT
•direct trauma
Physical Exam •2 part fx greater tuberosity > 0.5cm
Risks: young or elderly patient •if forearm & hand appear pale  r/o *proximal humerus/humeral head is -surgery
axillary artery injury common site for metastatic fx in breast
•the most common 2 part •RULE OUT RADIAL INJURY in CA Operative: ORIF for open, vascular or
fracture occurs at neck humeral hsaft pfracture nerve injury
Clavicle •most common bony injury •pain increases with shoulder ROM AP and 10 degree cephalic tilt views non-surgically: figure 8 splint / sling
Fracture Causes: FOOSH, sports, MVAs •swelling, tenderness, bruising, bulging confirm diagnosis
•grinding or crackling w/ ROM Proximal 1/3: ortho consult
Groups: •sagging of the arm downward and forward Complications:
I: midshaft (middle 1/3) (MC) •skin may appear tented •pneumothorax, hemothorax •surgical repair for open, shortened or
II: lateral (distal) third •CC ligmanet disruption fractures associated with neurovascular
III: proximal (medial) third •nerve injuries compromise

Arm About Presentation & Exam Diagnostics Treatment


Distal Mechanism: FOOSH •swelling, echhymosis, deformity AP and lateral xray: FAT PADS Stable, nondisplaced:
Humeral •pain around the elbow •lateral fat pad: bleeding in joint •long arm posterior splinting x10days
Fracture Patterns: •worsens with flexion of elbow •posterior fat pad: always pathologic
“Supracondylar •complex •anterior fat pads: nml “sail sign: Displaced fractures: ORIF
Humerus •communiuted Types:
Fracture” •intra-articular A: fracture is non-articular Refer: displaced, neurovascular,
B: fracture is partially articular small patients that fail to regain motion
part remains in continuity with shaft
C: fractures are articular and no Complcations: medial & brachial nerve
fragment remains in continuity *Y and T injury  Volkmann ischemia (claw)
•radial nerve injury

Radial Head Mechanism: FOOSH •Pain & swelling of the lateral aspect of AP and lateral Radiographs Type I and II: long arm 90 degrees
Fracture the elbow •FAT PADS: posterior or displaced
Types: •May be concomitant with an elbow anterior Type III: ORIF
Type I: Nondisplaced/minimal dislocation Type IV: closed reduction
Type II: Partial fractures with •Crepitus with passive forearm
displacement >2 mm rotation Referral
Type III: Comminuted fractures •Type II or greater
Type IV: fracture of radial head •Failure of conservative TX or
w/ dislocation of elbow joint persistent pain with limited ROM

Fracture •Easily fractured and occurs with •Pain worse on flexion AP & Lateral x-ray Nondisplaced: posterior long arm
Olecranon a direct blow to the elbow •May see ulnar nerve compression splint w/ in 90 degree flexion x4-6wk
•swelling of the entire elbow joint Non Displaced:
Mechanism: direct blow •Tenderness of Olecranon Process •Posterior long arm splint with elbow Displaced Fractures: ORIF
in 90o flexion for 4-6 weeks
•F/u xray in 7-10 Referral
•Protected ROM in ~2-3 weeks •displaced, open, malunion, neurovasc

Hand Exam and Diagnostics Treatment Bennett Fracture Rolando Fracture


Fracture of The most frequently occurring •Acute pain, tenderness, AP & lateral xray: forearm and wrist Stable: closed reduction
Distal fracture in adults swelling, and deformity *may want to due an xray of the elbow •sugar tong splint x2-3 weeks
Radius •pain worse with passive •Colles: dorsally displaced or angulated
(Colles and Mechanism: FOOSH motion •Smith: ventrally displaced or angulated Unstable/comminuted: ORIF
Smith)
Colles: distal fragment dorsal (back) Physical Exam Colles Complications:
•deformity and bruising •extensor pollicus longus tendon
Smith: distal fragment ventral (front) • “Silver Fork Deformity” rupture (MC)
-fx of ulnar styloid Process •malunion or nonunion, median nerve
•Garden Spade (smith fx)

colles smith

Lunate •lunate does not articulate with both •swelling, pain of wrist AP view: lunate appears triangular Emergent closed reduction and splint
Dislocation the capitate and the radius • “peice of pie” sign  ORIF *EMERGENCY
and Fracture Fracutre:
*near pinky Mechanism: high energy while wrist •most serious carpal fracture Lateral View: volar displacement and tilt Fracture: immobilization
is extended and ularly deviated due to avascular necrosis • “spilled tea cup” sign

Fractures of •metacarpal fracture of base of thumb •pain with thumb movement AP and lateral x-ray Thumb splica splint
Base of •tenderness, swelling over •Bennet: fragment articulating with trapezium
Thumb (1st) Mechanism: the CMC joint •Rolando: Y sign Bennet: immobilization
Bennet and •axial force to flexed thumb
Rolando Rolando: ORIF, esternal fication,
Bennet: 2 pieces, oblique, non- closed reduction w/ pinning
comminuted
Rolando: Y-shape intra-articular Rolando-

fracture; comminuted Bennet

Fracture of •MC carpal bone fracture •pain worse ulnar deviation Ulnar Deviation PA/olique view •thumb spica splint
Scaphoid •MC in young adult males •Marked tenderness of the •may ne normal for 2 weeks •referred to orthopedic surgery
(Navicular) anatomical snuffbox (radial *if there is snuffbox pain then treat likfe FX
*near thumb Mechanism: FOOSH surface of wrist)
•Decreased ROM & strength
Complications: nonunion, avascular
necrosis (radial artery

Boxers •Fracture of the 5th metacarpal neck •pain along 5th metacarpal •x-rays •ulner gutter splint 60 degree flexion
Fracture •MC in distal phalanx •swelling, ecchymosis •reductoin first if FX >25-30 degree
•distal fragment rotated angulation
Mechanism: direct trauma against a
closed wrist (boxers) >40 degree angulation: ORIF

Gamekeeper •tear of ulnar collateral ligament – •Pain and weakness of •clinical, xray •thumb splica splint and referral
(Skiers) instability at MCP joint pinch grasp (valgus stress)
Thumb •Pain, swelling, ecchymosis Complete rupture: surgery
Mechanism: forced abduction around thenar eminence

Pelvis Overview Clinical Presentation Diagnostics Adverse Outcomes and Treatment


Hip •occurs when femoral head is •severe pain in the hip/groin X-Ray: AP of pelvis, AP & lateral femur •EMERGENCY
Dislocation displaced from the acetabulum •unable to move LE •normal: femoral head and joint equal •reduction ASAP: closed reduction
•MC is posterior •other MSK injuries with it •posterior: head smaller & superior -rule out femoral head FX and intra-
•anterior: head larger, inferior, medial articular loose bodies before reducing
Cause: Physical Exam: •post-reduction xray/CT
•trauma (MVA, fall) •posterior: shortened, hip flexed, Complications: •neurovascular assessment
adduction, internal rotation •osteonecrosis is MC early complication
Mechanisms: - sciatic nerve palsies -may not appear for 2-3 years Post-Reduction:
•Posterior (MC): axial loading •sciatic nerve injury (posterior) •abduction exercises, use walking aid in the
on adducted femur •anterior: mild flexion, •femoral nerve injury (anterior) OPPOSITE hand until walk without limp
•Anterior: axial load on abduction, external rotation •DVT
abducted & externally rotated - femoral nerve palsy •bleed

Fracture •fracture of the pelvic ring and •pain in the groin, lateral hip, •AP xray of pelvis and oblique views A and B: conservative management
of the acetabulum or buttock with attempts at C/severe: traction and/or fixation (ORIF)
Pelvis weight bearing or inability Stable: involve one side
Mechanism: High energy: surgery
•high impact injuries (MVA) Low energy Unstable: disrupt ring at two sites as well as Low Energy: Stable, anlagesics
•low imapc injuries •pain with attempted ROM and involvement of the symphysis or SI structure
•fall straight leg raise, antalgic gait, Adverse Outcomes:
stance shortened •Grade A: stable, minimal displacement •GU injuries
Type: open book •Grade B: rotational unstable, vertical stable •distal neurologic injuries
High energy •Grade C: all unstable •chronic pain
•pain, swelling, deformity,
ecchymosis (perineal)

Femur Overview Presentation and Diagnostics Treatment


Fracture of •MC in elderly and osteoporosis pts •limb externally rotated, abducted, short xrays: AP of pelvis and cross-table •ORIF
Proximal •pain with attemting to rotate •MRI if xray normal •surgery ASAP, delay no
Femur Mechanism: •unable to SLR more than 48h
“hip fracture” •minor/indirect trauma elderly Proximal femur fractures can be
•high impact in younger patients Femoral Neck associated with thromboembolic •type of repaire depends on
•INTRASCAPULAR event, PNA, decubitus ulcers, UTIs location
3 main types: •disruption of blood supply causing •thromboembolic prophylaxis
•femoral neck: above trochanters nonunion FX or osteonecrosis
*associated with avascular necrosis Femoral Shaft:
•intertrochanteric: between greater and Intertrochanteric/Subtrochanteric: •surgical treatment (open
lesser trochanter EXTRASCALPULAR reduction internal fixation
•subtrochanteric: below trochanters •does NOT disrupt blood supply
•more extensive repair

Femoral Mechanisms: •pain, swelling Complications: •immediate ortho consult


Condyle •axial loading •inability to bear weight •peroneal nerve injury: foot drop and •ORIF
Fractures •direct blow to the femur decreased sensation in web spaces
•popliteal artery injury

Knee About Presentation Diagnostics Treatment


Tibia •lower leg comprised of tibia and fibula •swelling, pain, inability to bear weight X-ray: AP/lateral •urgent refer to ortho (1-2d)
(Pilon) FX •tibia and fibula usually fractured together •OPEN FX MOST COMMON Displaced: intramedullary nailing
•MC long bone fracture: tibial shaft emergent if: open, NV Nondisplaced: long leg cast x4-6wk
Compartment SX: increase and injury, compartment
unrelenting pain, tense swelling, pain syndrome, dislocation
with passive flexion/extension of toes
Fibula FX •fibula is non-weight bearing •deformity, shortening or angulation X-ray: AP/lateral Distal FX: short leg cast
•stabilizer of ankle against eversion *however, usually normal Proximal FX: long leg cast
•twisting injury or direct blow •tenderness, swelling, bruising
Displaced/angulate, spiral,
comminuted: urgent ortho referral

F/A About Presentation Diagnostics Treatment


Fracture of •due to trauma with a •swelling, pain on weight X-ray: AP, lateral, oblique of foot nondisplaced metatarsal neck and shaft:
Metatarsals twisting or rotational force, bearing -short leg cast or fracture brace
or blunt trauma (drop •ecchymosis and tenderness Ottawa Rules  Foot
something on foot) •navicular bone (midfoot) pain •multiple fx, displacement, angulation, avulsion:
•5th metatarsal pain -surgery w/ open or closed reduction & fixation
•inability to walk >4 steps

Stress •fracture due to overuse or •localized aching pain, X-rays: usually negative •rest, avoid high-impact activities
(March) high-impact activities swelling, tenderness •ice, splint
Fracture •worse with activity
•3rd metatarsal is the MC •loaclized bone tenderness

Lisfranc 1+ metatarsal bones are •midfoot pain, swelling “Fleck Sign” avulsion fracture of medial REFER TO ORTHO
Injury displaced from the tarus •trouble bearing weight cuneiform or second metatarsal
•tenderness of TMT joints -FX to proximal metatarsal

Jones •transverse fracture through •pain over 5th metatarsal and x-ray: transverse fracture involving the •short leg cast x6-8 weeks
Fracture diaphysis of 5th metatarsal lateral border of foot metaphyseal-diaphyseal junction
at the metaphyseal-
diaphyseal junction

Pseudojones •fracture through base •transverse abulsion fracture •walking cast x2-3 weeks
Fracture (tuberosity) of 5th metatarsal •ORIF id displaced
due to plantar flexion
Fractures of •involves proximal •pain, swelling, ecchymosis X-ray: AP radiographs •buddy taping the fractured toe to the adjacent toe
Phalanges phalanx and is a result of •limited ROM of toe •gauze pad placed between toes to absorb moisture
direct trauma *5th toe MC

MSK About Presentation Diagnostics Treatment


Ankle Includes: •ankle pain with weight bearing Weber Classification: •minimally displaced fractures may not
Fracture •lateral malleolus (distal fibula) •palpable gap A: below syndesmosis show on x-ray, if you are suspicious then
•medial malleolus (distal tibia) •tenderness over fractured site B: level of syndesmosis treat as fx
•posterior tibia (malleolus) C: above syndesmosis
•collateral ligament of talar dome Stable: short or long leg cast
stable: one side of joint Unstable/open: ORIF
unstable: both sides of ankle

Calcaneous/ Cause: severe trauma (MVA, fall) •severe pain, inability to bear X-ray: 3-view ankle xrays •posterior splint toe to upper calf
Talus FX •most involve articular surface and weight •CT •urgent referral
are unstable •swelling and tenderness •most require ORIF
compression of lumbar spine

Maisonneuv •spiral fracture of proximal third of •associated with distal medial •proximal x-rays
e Frature the fibula malleolar fracture or rupture of
deep deltoid ligament

Peds Fracture About Clinical Presentation Diagnostics Treatment


Greenstick •incomplete fracture •due to bone softness, there is a X-Ray: bowing •cast to prevent further movement are further
Fracture with angular deformity fracture on one side of the bone and a fracturing of the bone  stays on 4-6 weeks
and corticol disruption buckle on the other •Tylebol
•result of rotational force •fracture line does not extend
completely through bone width

Torus (Buckle) •Incomplete fracture of •distal metaphysis where the bone is • stable and do not require surgical
Fracture one cortex more spongy intervention
•peds patients •may be very subtle so multiple xray •can apply simple splinting for ~3-4weeks
•MC site: distal dorsal views are needed •NSAIDs/analgesia
radius

Salter-Harris •fracture that involves S: slipped  type I straight across •goal is for anatomic reduction
Fracture the epiphyseal growth A: above  type II •fractures heal rapidly usually within 4-6
plate (physis) L: lower  Type III weeks
TE: through everything Type IV
Slipped(I) Above(II)
•females 12-14yo, males R: rammed  Type V •Type I/II: closed reduction & cast
14-16yo •Type III/IV: open reduction & fix (surgery)

If the fracture is 7+ days post injury,


reduction should be AVOIDED in children
Lower (III) <13yo because of risk of growth plate re-
injury and growth arrest

Through everything(IV) Rammed (V)

GI
Intussusceptio Telescoping of an intestinal segment into adjoining distal TRIAD: Ultrasound: BEST INTIAL •Barium enema with
n intestinal lumen •vomiting •donut or target sign pneumatic air or
•MCC of obstruction first 2 years of life •abdominal pain  draw legs up hydrostatic (saline)
•6-12 months of age MC • bloody stool: “currant jelly” stools Abdominal x-ray: lack of gas
•fluid and electrolyte
Risks: idiopathic, meckels, enlarged lymph nodes, Exam: sausage shaped mass in RUQ Air or contrast enema: replacement  NG
tumors, FB, hamartomas Emptiness in RLQ (Dance’s sign) DIAGNOSTIC AND decompression
THERAPEUTIC
Hirschprung Congenital megacolon due to absence of ganglion cells •meconium ileus (failure of Contrast enema: transition zone Resection of affected
Disease  functional obstruction *MC in distal colon & rectum meconium to pass in >48h) bowel
•bilious vomiting Anorectal manometry: increased
Risks: male, downs, Chagas, MEN II •abdominal distention anal sphincter pressure and lack
•enterocolitis: vomit, diarrhea, toxic of relaxation
Pathophysiology:
•failure of complete neural crest  absence of enteric Rectal biopsy: DEFINITIVE
ganglion cells (auerbach plexus) *rectal suction biopsy

Meckels’ (Ileal) Persistent portion of embryonic vitelline duct (yolk •usually asymptomatic Meckel Scan: Surgical excision
Diverticulum stalk, omphalomesenteric duct) in the small intestine •painless rectal bleeding or •look for ectopic gastric tissue in
ulceration ileal area
Rule of 2s:
•2% population, 2% symptomatic Complications: Mesenteric arteriography or
•w/n 2 feet of ileocecal valve, 2in length •intussuception abdominal exploration
•2 tissues, 2 years old, 2x MC in males •volvus
•obstruction
Tissues: gastric (MC), pancreatic

GI
Paralytic Ileus Loss of peristalsis of the intestine •N/V obstipation (severe const.) Labs: CBC, CMP •Complete bowel rest
(STOP) without structural obstruction •Abdominal distention with tympany  IV fluids/TPN, NG tube
Abd X-ray: dilated loops of •Slowly advance diet *gas=good
Causes: surgery (abdominal and Exam:Diminished/absent BSs bowel with no transition zone •Activity
pelvic), peritonitis, meds(opiates), illness, •Remove drugs that reduce
infection, metabolic (low K, high Ca) intestinal motility

Volvulus •Torsion bowel  bowel obstruction •crampy abdominal pain, distention X-ray: “bent inner tube” •Endoscopic decompression
•N/V, constipation “coffeebean sign” (proctosignmoidoscopy) 
Location: Sigmoid (MC!), cecum •tympanitic abdomen  U shaped appearance of the elective surgery
•fever, tachycardia, peritonitis air-filled closed loop of colon, •IV Fluids
loss of hausta
Neonates: bilous vomiting, colicky pain
GI series & CT: “birds beak”

Splenic Spleen is MC organ injured with trauma •abdominal pain, hypotension, shock FAST abdominal exam Incomplete rupture:
Rupture or endovascular embolization
Laceration Causes: L sided rib fracture, blunt •Kehr sign: referred left shoulder pain due
abdominal trauma, infectious mono to irritation of diaphragm and phrenic nerve Complete: splenectomy

GI
Toxic Total/segmental non-obstructive colonic •colitis present for at least 1wk XRAY (best!): •Complete bowel rest
Megacolon dilatation (>6cm) + toxicity prior to onset •Transverse or R colon is dilated, •bowel decompression: NG tube
•Profound bloody diarrhea 6-15cm supine •fluid and electrolyte replacement
Causes: •abdominal pain, distention, N/V •broad ABX: Ceftriaxone +
•Complication of IBD (ulcerative colitis) •toxicity: fever, AMS, tachycardic, 3+ of following: Metronidazole
•infectious or ischemic colitis hypotension, dehydration •fever >38C
•volvulus •pulse >120
•diverticulitis PE: FAT BAT •neutrophil leukocytosis >10,500 •DC all antimotility agents (opiates,
•Fever, AMS, Toxic •anemia anticholinergics)
•BP low, abd pain, tachycardia
AND 1+ of following:
•hypotension
•altered mental status (AMS)
•dehydration
•electrolyte abnormalities

Chronic Ischemic bowel disaese due to mesenteric •chronic, dull abdominal pain that Angiography: DEFINITIVE Revascularization
Mesenteric atherosclerosis  decreased supply during is worse AFTER MEALS (Angioplasty with stenting or bypass)
Ischemia increased demand (eating) •anorexia (aversion to eating)
•weight loss
Risk: Atherosclerosis

Acute •Abrupt onset of small intestine •Crampy, abdominal pain Labs: leukocytosis, lactic •NPO, rest, fluids
Mesenteric hypoperfusion •Bloody diarrhea acidosis, increased hematocrit, •SURGERY-revascularization or
Ischemia *MC in superior mesenteric artery •N/V/D increased amylase resection
•Pain control, anti-emetics
Causes: emboli (A-Fib), thrombus, shock, •HALLMARK: pain out of DX:
cocain, vasopressors proportion •CT angiogram (initial)
•Arteriography: DEFINITIVE

Ischemia Decreased colonic perfusion  inflammation •LLQ pain, crampy CT Abdomen: 1st test •Restore perfusion
Colitis •Bloody diarrhea •”thumbprinting”  segmental •bowel rest
MCC: hypotension or atherosclerosis •abdominal tenderness bowel wall thickening •IV Fluids
*superior and inferior mesenteric arteries •observe for perforation
*MC splenic fixture & rectosigmoid junction Colonoscopy
•segmental ischemic changes in
Risks: elderly, DM, cardiac cath, MI, areas of low perfusion
sontipcation inducing medications

GI
Small Bowel Causes: “CAVO” •Leukocytosis on labs •NPO, IVF
Obstruction •post-surgical adhesions (MC) •crampy abdominal pain(mod/severe) •bowel decompression: NG tube
•incarcerated hernias, crohns, malignancy •abdominal distention Abdominal x-ray: multiple air •Surgery
(MCC of large bowel), intussusception •vomiting fluid levels in a “step-ladder”
•obstipation (flatulence) appearance, dilated loops
Types:
•closed v open: closed-lumen occluded at Exam: CT scan: transition zone from
two points •High-pitched BS  hypoactive (late) dialted loops with contrast to no
•partial v complete: complete have •visible peristalsis (early) contrast
severe obstipation
•distal v parietal: distal is more
distention and less vomiting

Appendicitis •Obstruction of the lumen of the appendix •dull periumbilical or epigastric pain  •CBC, UA, pregnancy •Consult surgeon
 inflammation and bacterial overgrowth RLQ •CT scan: STUDY OF CHOICE •Appendectomy
•MCC of acute abdomen in ages 12-18yo •anorexia, nausea *don’t need contrast •IVF, ABX  Zosyn
•vomiting after pain *consult first in children to
Causes: •SX increase over 24h determine if imaging needed
•fecalith and lymphoid hyperplasia
(MCC) PE:
•inflammation, malignancy, foreign body •rebound tenderness, guarding
•McBurneys point: RLQ pain
Rectocecal: abd and flank pain •Rovsing: RLQ pain w/ LLQ palpation
Pelvic: tender abd and rectal pain •Psoas: RLQ pain with hip
flexion/extension (SLR)
•Obturator: RLQ pain with internal/external
rotation of hip with flexed knee

Small Bowel 24-40% adenocarcinomas in the •Abdominal pain: intermittent & crampy Diagnosis usually delayed  •Surgery
Carcinoma duodenum •N/V, wt loss, jaundice poor outcomes •Chemo if (+) lymph nodes
•Anemia
Risks: hereditary, CF, Crohns, alcohol, •CT scan
sugar, red meat, salt-cured, smoked foods • (+) CEA •Wireless capsule endoscopy

PULM About Presentation Diagnostics Treatment


Pneumo spontaneous: •Sudden chest pain CLINICALLY Tension Pneumothorax:
-thorax •primary: no lung disease, atraumatic -pleuritic, unilateral, non-exertion •Needle aspiration (14-16G)  chest tube
-rupture of subpleural apical bleb •Dyspnea CXR: expiratory view upright -2nd ICS at midclavicular line
-tall, thin males 20-40yo •decreased peripheral markings •Oxygen 2-4L nasal cannula
•secondary: underlying lung disease Physical Exam: •companion lines (visceral pleural
•diminished breath sounds & TF line running parallel to ribs) Small Primary (<3cm):
traumatic: penetrating or blunt, •hyperresonance to percussion •deep sulcus sign •observation and oxygen; repeat CXR in 6hr
iatrogranic ((+) pressure, CPR) •mild tachypnea
•pneumothorax under tension Chest CT, bedside US: large or Large Primary (>3 cm):
tension (EMERGENT) -increased JVP unstable •needle or catheter aspiration v. chest tube
•air entering pleural sac -systemic hypotension
•MCC: cardiopulm, (+) pressure -pulsus paradoxus Stable Secondary: chest tube + hospital

PULM
Pulmonary Obstruction of pulmonary blood flow due to a Symptoms: EKG: tachycardia, S1Q3T3 Oxygen, IV Fluids if needed
Embolism blood clot (thromboembolism) •Dyspnea (sudden onset)
•Pleuritic chest pain CXR: Stable
Risks (Virchows Triad): •Hemoptysis (pulm infarction) •normal is MC Anticoagulation
•venous stasis: immbolization, sitting > 4 hours, •Cough, wheeze •atelectasis •Heparin or LMWH + Warfarin
surgery •Hamptoms Hump: wedge •Dabigatran, Rivaroxaban,
•intimal damage: trauma, infection, inflammation PE: shaped infiltrate due to infarction Apixaban, Edoxaban
•hypercoagulability: medications, protein C or S •tachypnea, tachycardia •Westmark Sign: avascular
deficiency, Factor V Leiden, antithrombin III •hypoxemia; rales markings distal to PE IVC Filters
deficiency, OCP use, pregnancy, malignancy •hypotension and syncope •if anticoagulation is CI or
•JVD & S3 and S4 sounds ABG: respiratory alkalosis and unsuccessful or RV dysfunction
•(+) homans: calf pain w/ hypoxemia  acidosis seen on echo
dorsiflex
D-dimer: if low suspicion Unstable:
PERC (PE rule out criteria) BP <90, acute RB dysfunction
•age <50 years old VQ Scan: •Thrombolytics
•HR <100 *pregnant, increased creatinine •Thromboembolectomy (if TPA
•O2 95% or greater •poor perfusion, good ventilation is contraindicated)
•no hemoptysis
•no estrogen use Helical (Spiral) CT: BEST
•no prior DVT or PE Pulmonary angiography: GOLD
•no unilateral leg swelling
•no surgery/trauma requiring Wells Score >6: high, 2-6: mod
hospitalization 4 weeks prior +3: DVT, alt dx not likely
+1.5: HR >100, VTE hx,
immobilization for 4 weeks
+1: malignancy, hemoptysis

PULM About Presentation Diagnostics Treatment


Acute MC form non-cardiogenic •rapid onset of profound CXR: •Non-invasive or mechanical ventilation
Respiratory pulmonary edema DYSPNEA within 12-48h af •bilateral pulm infiltrates -CPAP-full face ≥ 5 (mild disease)
Distress •SOB, tachypnea, intercostal •AIR BRONCHOGRAMS -PEEP ≥ 5 (mild, mod, or severe disease)
Pathophysiology: retractions, crackles •spares costophrenic angles •Treat underlying cause
•diffuse alveolar damage and •hypoxemia (no response to O2)
surfactant ABG: severe hypoxemia refractory PEEP ((+) end-expiratory pressure)
•increased permeability of the •multiple organ failure: kidney, to supplemental O2 -prevents alveolar collapse
capillary-alveolar barrier liver, CV, CNS •Mild: PaO2/FiO2 201-300 -give at lowest effective levels
•acute hypoxemic without •Mod: PaO2/FIO2 101-200 -does not improve mortality
hypercarbia HALLMARK: diffuse alveolar •Severe: PaO2/FIO2 <100 -risks: barotrauma, pneumothorax, auto-peep
damage
Risks: Absence of cardio pulmonary edema
•critically ill (gram - sepsis) •pulmonary wedge <18mmHg
•trauma, aspiration
FB Aspiration Common items: food (MC SUDDEN onset of choking, CXR: air trapping Rigid Broncoscopy to remove FB
peanuts), coins, toys, balloons cough, dyspnea, wheezing,
assymetric breath sounds Rigid bronchoscopy: MC on the R side (wider, vertical, shorter
MC age is 2 years old DEFINITIVE DIAGNOSIS main bronchus)

OBGYN About/Rsiks Presentation Diagnostics Management


Placental Separation of the placenta either partially or •Sudden onset of •DIAGNOSIS OF EXCLUSION •fluids and blood if needed
Abruption totally from its implantation site before delivery abdominal PAIN •CBC, CMP, PT/PTT/INR, blood type •emergenct OBGYN consult
•vaginal bleeding -possibly emergent C-section
Cause: hemorrhage into decidua basalis •uterine tenderness •transabdominal US  TVUS
Complications:
Risks: MCC bleeding in THIRD •May be associated with elevated AFP •hypovolemic shock
•materal HTN TRIMESTER •DIC
•trauma, uterine fibroids, lupus Speculum and digital pelvic exam CI •AKI (hypovolemia)
•increasing maternal age, past aborption Exam: unltil US r/o placenta previa •Couvelair uterus: extravasation of
•Preeclampsia, smoking, cocaine •tender, rigid hypertonic blood into uterine musculature &
uterus beneath serosa
•NO PELVIC EXAM -myometrium “blue-purple” tone

Placenta Placenta that is implanted either over or near •PAINLESS vaginal


Previa internal cervical os bleeding usually after the C-Section performed in complete,
second trimester (28 wks) major degrees, and fetal distress
Classifications:
•Placental previa: os covered partial or complete *uterine body is
•Low-lying: implantation in lower uterine segment remodeling to form the
•partial: partial coverage lower uterine segment 
•marginal: adjacent to internal os (<2cm away) internal os dilates 
bleeding occurs
Risks: maternal age, mult. gestations, C-section,
smoking, high MSAFP, preveious previa

You might also like